Sei sulla pagina 1di 206

Self-Assessment in Multiprofessional Critical Care A Comprehensive

Review
Seventh Edition

ANSWER SHEET
Instructions: Circle the correct answer.
Note: The response "E" appears on this answer sheet for all questions. If a question does not cite a choice "E" as an option, please disregard the choice "E" on this answer sheet.

PART 1: Neurologic Critical Care

1. A B C D E
2. A B C D E
3. A B C D E
4. A B C D E
5 A B C D E
6 A B C D E
7 A B C D E
8. A B C D E
9 A B C D E
10 A B C D E
11 A B C D E
12 A B C D E
13 A B C D E
14. A B C D E
15 A B C D E
16. A B C D E
17 A B C D E
18 A B C D E
19 A B C D E
20 A B C D E
21 A B C D E
22 A B C D E
23 A B C D E
24 A B C D E
25 A B C D E
26 A B C D E
27 A B C D E
29 A B C D E
30 A B C D E
31 A B C D E
32 A B C D E
33 A B C D E
34 A B C D E

PART 2: Cardiovascular Critical Care

1. A B C D E
2. A B C D E
3. A B C D E
4. A B C D E
5 A B C D E
6 A B C D E
7 A B C D E
8. A B C D E
9 A B C D E
10 A B C D E
11 A B C D E
12 A B C D E
13 A B C D E
14. A B C D E
15 A B C D E
16. A B C D E
17 A B C D E
18 A B C D E
19 A B C D E
20 A B C D E
21 A B C D E
22 A B C D E
23 A B C D E
24 A B C D E
25 A B C D E
26 A B C D E
27 A B C D E
29 A B C D E
30 A B C D E
31 A B C D E
32 A B C D E
33 A B C D E
34 A B C D E
35 A B C D E
36 A B C D E
37 A B C D E
38 A B C D E
39 A B C D E
40 A B C D E

PART 3: Respiratory Critical Care

1. A B C D E
2. A B C D E
3. A B C D E
4. A B C D E
5 A B C D E
6 A B C D E
7 A B C D E
8. A B C D E
9 A B C D E
10 A B C D E
11 A B C D E
12 A B C D E
13 A B C D E
14. A B C D E
15 A B C D E
16. A B C D E
17 A B C D E
18 A B C D E
19 A B C D E
20 A B C D E
21 A B C D E
22 A B C D E
23 A B C D E
24 A B C D E
25 A B C D E
26 A B C D E
27 A B C D E
29 A B C D E
30 A B C D E
31 A B C D E
32 A B C D E
33 A B C D E
34 A B C D E
PART 4: Critical Care Infectious Diseases

1. A B C D E
2. A B C D E
3. A B C D E
4. A B C D E
5 A B C D E
6 A B C D E
7 A B C D E
8. A B C D E
9 A B C D E
10 A B C D E
11 A B C D E
12 A B C D E
13 A B C D E
14. A B C D E
15 A B C D E
16. A B C D E
17 A B C D E
18 A B C D E
19 A B C D E
20 A B C D E
21 A B C D E
22 A B C D E
23 A B C D E
24 A B C D E
25 A B C D E
PART 5: Hepatic, Gastrointestinal, Hematologic and Oncologic Disease in the ICU

1. A B C D E
2. A B C D E
3. A B C D E
4. A B C D E
5 A B C D E
6 A B C D E
7 A B C D E
8. A B C D E
9 A B C D E
10 A B C D E
11 A B C D E
12 A B C D E
13 A B C D E
14. A B C D E
15 A B C D E
16. A B C D E
17 A B C D E
18 A B C D E
19 A B C D E
20 A B C D E
21 A B C D E
22 A B C D E
23 A B C D E
24 A B C D E
25 A B C D E
26 A B C D E
27 A B C D E
29 A B C D E
PART 6: Renal and Metabolic Disorders in the ICU

1. A B C D E
2. A B C D E
3. A B C D E
4. A B C D E
5 A B C D E
6 A B C D E
7 A B C D E
8. A B C D E
9 A B C D E
10 A B C D E
11 A B C D E
12 A B C D E
13 A B C D E
14. A B C D E
15 A B C D E
16. A B C D E
PART 7: Environmental and Toxicologic Injury

1. A B C D E
2. A B C D E
3. A B C D E
4. A B C D E
5 A B C D E
6 A B C D E
7 A B C D E
8. A B C D E
9 A B C D E
10 A B C D E
11 A B C D E
12 A B C D E
13 A B C D E
14. A B C D E
15 A B C D E
16. A B C D E
PART 8: Pharmacologic Issues in the ICU

1. A B C D E
2. A B C D E
3. A B C D E
4. A B C D E
5 A B C D E
6 A B C D E
7 A B C D E
8. A B C D E
9 A B C D E
10 A B C D E
11 A B C D E
12 A B C D E
13 A B C D E
14. A B C D E
15 A B C D E
16. A B C D E
17. A B C D E
PART 9: Surgical and Obstetrical Critical Care

1. A B C D E
2. A B C D E
3. A B C D E
4. A B C D E
5 A B C D E
6 A B C D E
7 A B C D E
8. A B C D E
9 A B C D E
10 A B C D E
11 A B C D E
12 A B C D E
13 A B C D E
14. A B C D E
15 A B C D E
16. A B C D E
17 A B C D E
18 A B C D E
19 A B C D E
20 A B C D E
21 A B C D E
22 A B C D E
23 A B C D E
24 A B C D E
25 A B C D E
PART 10: Administrative and Ethical Issues in the Critically Ill

1. A B C D E
2. A B C D E
3. A B C D E
4. A B C D E
5 A B C D E
6 A B C D E
7 A B C D E
8. A B C D E
9 A B C D E
10 A B C D E
11 A B C D E
12 A B C D E
13 A B C D E
14. A B C D E
15 A B C D E
Copyright 2011 Society of Critical Care Medicine,
exclusive of any U.S. Government material.
All rights reserved.

No part of this book may be reproduced in any manner or media, including but not limited to print or electronic format, without prior written permission
of the copyright holder.

The views expressed herein are those of the authors and do not necessarily reflect the views of the Society of Critical Care Medicine.

Use of trade names or names of commercial sources is for information only and does not imply endorsement by the Society of Critical Care Medicine.

This publication is intended to provide accurate information regarding the subject matter addressed herein. However, it is published with the
understanding that the Society of Critical Care Medicine is not engaged in the rendering of medical, legal, financial, accounting, or other professional
service and THE SOCIETY OF CRITICAL CARE MEDICINE HEREBY DISCLAIMS ANY AND ALL LIABILITY TO ALL THIRD PARTIES
ARISING OUT OF OR RELATED TO THE CONTENT OF THIS PUBLICATION. The information in this publication is subject to change at any time
without notice and should not be relied upon as a substitute for professional advice from an experienced, competent practitioner in the relevant field.
NEITHER THE SOCIETY OF CRITICAL CARE MEDICINE, NOR THE AUTHORS OF THE PUBLICATION, MAKE ANY GUARANTEES OR
WARRANTIES CONCERNING THE INFORMATION CONTAINED HEREIN AND NO PERSON OR ENTITY IS ENTITLED TO RELY ON ANY
STATEMENTS OR INFORMATION CONTAINED HEREIN. If expert assistance is required, please seek the services of an experienced, competent
professional in the relevant field. Accurate indications, adverse reactions, and dosage schedules for drugs may be provided in this text, but it is possible
that they may change. Readers must review current package indications and usage guidelines provided by the manufacturers of the agents mentioned.

Managing Editor: Matthew Ferris


Editorial Assistant: Amanda Cozza

Printed in the United States of America


First Printing, July 2011

Society of Critical Care Medicine


Headquarters
500 Midway Drive
Mount Prospect, IL 60056 USA
Phone +1 847 827-6869
Fax +1 847 827-6886
www.sccm.org

International Standard Book Number: 978-1-620750-308


CONTRIBUTORS

Sergio L. Zanotti-Cavazzoni, MD, FCCM


Editor
Director, Critical Care Medicine Fellowship
Assistant Professor of Medicine
Division of Critical Care Medicine
Cooper Medical School of Rowan University
Camden, New Jersey
No disclosures

Richard M. Pino, MD
Editor
Associate Professor
Clinical Vice Chairman
Anesthesia, Critical Care, & Pain Medicine
Massachusetts General Hospital
Harvard Medical School
Boston, Massachusetts
No disclosures

Fred Rincon, MD, MSc


Section Editor
Assistant Professor
Neurological Surgery/Neurology
Thomas Jefferson University
Philadelphia, Pennsylvania
No disclosures

Ram M. Subramanian, MD
Section Editor
Assistant Professor
Medicine and Surgery
Emory University
Atlanta, Georgia
No disclosures

Janice L. Zimmerman, MD
Section Editor
Professor of Clinical Medicine
Weill Cornell Medical College
Department of Medicine
The Methodist Hospital
Houston, Texas
No disclosures

Mary Jane Reed, MD, FCCM


Section Editor
Critical Care Medicine and General Surgery
Geisinger Medical Center
Danville, Pennsylvania
No disclosures

Gustavo Angaramo, MD
Assistant Professor in Anesthesiology and
Critical Care Medicine
University of Massachusetts Medical School
Worcester, Massachusetts
No disclosures

Martha Arellano, MD
Assistant Professor of Hematology/Oncology
Winship Cancer Institute of Emory
University
Atlanta, Georgia
No disclosures

Amado Alejandro Baez, MD, MPH, FCCP


Emergency Medicine/Critical Care
Chairman, Emergency Medicine
Co-Chair, Critical Care Education
CEDIMAT
Santo Domingo, Dominican Republic
No disclosures

Marie R. Baldisseri, MD, FCCM


Associate Professor, Critical Care Medicine
University of Pittsburgh Medical Center
Pittsburgh, Pennsylvania
No disclosures

Rodney Bell, MD
Professor of Neurology and Neurosurgery
Thomas Jefferson University
Philadelphia, Pennsylvania
Sanofi Aventis honoraria/speaking fees

Edward A. Bittner, MD, PhD


Director, Critical Care Fellowship
Assistant Professor, Harvard Medical School
Anesthesia, Critical Care, & Pain Medicine
Massachusetts General Hospital
Boston, Massachusetts
No disclosures

Mariane Charron, MD
Critical Care Fellow
Cardiovascular Disease and Critical Care Medicine
Cooper University Hospital
UMDNJ - Robert Wood Johnson Medical School
Camden, New Jersey
No disclosures

Jose L. Diaz-Gomez, MD
Staff Anesthesiologist Intensivist
Cardiothoracic Anesthesiology
Anesthesiology Institute
Cleveland Clinic
Cleveland, Ohio
No disclosures

Svetolik Djurkovic, MD
Attending Physician
Medicine, Critical Care
Inova Fairfax Hospital
Falls Church, Virginia
No disclosures

Ryan M. Ford, MD
Assistant Professor
Transplant Hepatology and Digestive Diseases
Emory Transplant Center
Emory University
Atlanta, Georgia
No disclosures

Vadim Gudzenko, MD
Assistant Clinical Professor
Anesthesiology
David Geffen School of Medicine of UCLA
Los Angeles, California
No disclosures

Daniel W. Johnson, MD
Instructor in Anesthesia
Staff Anesthesiologist and Intensivist
Department of Anesthesia, Critical Care, & Pain Medicine
Massachusetts General Hospital
Harvard Medical School
Boston, Massachusetts
No disclosures

Ghulam Khaleeq, MD
Pulmonary Critical Care
Geisinger Medical Center
Danville, Pennsylvania
No disclosures

Corry J. Kucik, MD, DMCC, FCCP


Assistant Professor
Anesthesiology
University of Southern California
Keck School of Medicine
Los Angeles, California
No disclosures

Asheesh Kumar, MD
Clinical Fellow in Critical Care Anesthesia
Department of Anesthesia, Critical Care, & Pain Medicine
Massachusetts General Hospital
Harvard Medical School
Boston, Massachusetts
No disclosures

Eugenio Lujan, MD
Anesthesia Residency Program Director
Assistant Professor of Anesthesiology, USUHS
Naval Medical Center San Diego
San Diego, California
No disclosures

G. Marshall Lyon, MD, MMSc


Assistant Professor of Medicine
Director of Transplant Infectious Disease
Division of Infectious Diseases
Emory University School of Medicine
Atlanta, Georgia
No disclosures

Victor J. Matos, MD
Centro Mdico Moderno
Los Prados, Santo Domingo, Dominican Republic
No disclosures

William McBride, MD
Department of Neurology, Cerebrovascular Division
Thomas Jefferson University Hospital
Philadelphia, Pennsylvania
No disclosures

Charles Medico, PharmD, BCPS


Clinical Coordinator
System Therapeutics
Geisinger Medical Center
Danville, Pennsylvania
No disclosures
Aneesh K. Mehta, MD
Assistant Professor of Medicine
Assistant Director of Transplant Infectious Disease
Division of Infectious Diseases
Emory University School of Medicine
Atlanta, Georgia
Bristol-Myers Squibb grant

Yatin Mehta, MD
Departments of Critical Care Medicine and Pulmonary Medicine
Geisinger Medical Center
Danville, Pennsylvania
No disclosures

Michael Moussouttas, MD
Assistant Professor
Department of Neurology
Thomas Jefferson Medical Center
Philadelphia, Pennsylvania
No disclosures

J. Scott Parrish, MD
Fellowship Program Director
Pulmonary and Critical Care Medicine
Naval Medical Center
San Diego, California
No disclosures

Bruce M. Potenza, MD
Director of Burn Surgery Professor of Surgery
Division of Trauma, Critical Care and Burns
University of California San Diego
San Diego, California
No disclosures

Nitin Puri, MD
Pulmonary Critical Care Fellow
Pulmonary Critical Care Medicine
Cooper University Hospital
UMDNJ - Robert Wood Johnson Medical School
Camden, New Jersey
No disclosures

Jean-Sebastien Rachoin, MD, FASN


Assistant Professor of Medicine
Division of Critical Care and of Hospital Medicine
Cooper University Hospital
UMDNJ - Robert Wood Johnson Medical School
Camden, New Jersey
No disclosures

Omar Rahman, MD
Director Adult ICU
Critical Care Medicine
Geisinger Medical Center
Danville, Pennsylvania
No disclosures

Anita J. Reddy, MD, FCCP


Pulmonary and Critical Care
Cleveland Clinic
Cleveland, Ohio
No disclosures

Juan C. Salgado, MD
Assistant Professor of Medicine
Division of Pulmonary, Critical Care, and Sleep Medicine
University of Florida
Gainesville, Florida
No disclosures

Shelly D. Timmons, MD, PhD, FACS, FAANS


Director of Neurotrauma
Department of Neurosurgery
Neurosciences Institute
Geisinger Health System
Danville, Pennsylvania
No disclosures

Denis M. Torres, MD
Associate Professor of Surgery
Critical Care Medicine
Geisinger Medical Center
Danville, Pennsylvania
No disclosures

Jay B. Varkey, MD
Assistant Professor of Medicine
Division of Infectious Diseases
Emory University School of Medicine
Atlanta, Georgia
No disclosures

Matthew Vibbert, MD, MSc


Assistant Professor
Neurology/Neurological Surgery
Thomas Jefferson University
Philadelphia, Pennsylvania
No disclosures

Patrick F. Walsh, DO, FACP, FCCP


Intensivist
Critical Care Medicine
Geisinger Medical Center
Danville, Pennsylvania
No disclosures

Kenneth Widom, MD
Associate
Trauma Surgery, General Surgery, and
Critical Care
Geisinger Medical Center
Danville, Pennsylvania
No disclosures

Steve Zanders, DO, FCCP


Intensivist, Assistant Professor of Medicine
Pulmonary/Critical Care, Allergy/Immunology
Wake Forest University Medical School and Hospital
Winston-Salem, North Carolina
No disclosures
TABLE OF CONTENTS

ITEMS

Part 1Neurologic Critical Care


Part 2Cardiovascular Critical Care
Part 3Respiratory Critical Care
Part 4Critical Care Infectious Diseases
Part 5Hepatic, Gastrointestinal, Hematologic, and Oncologic Disease in the ICU
Part 6Renal and Metabolic Disorders in the ICU
Part 7Environmental and Toxicologic Injury
Part 8Pharmacologic Issues in the ICU
Part 9Surgical and Obstetrical Critical Care
Part 10Administrative and Ethical Issues in the Critically Ill

RATIONALES

Part 1Neurologic Critical Care


Part 2Cardiovascular Critical Care
Part 3Respiratory Critical Care
Part 4Critical Care Infectious Diseases
Part 5Hepatic, Gastrointestinal, Hematologic, and Oncologic Disease in the ICU
Part 6Renal and Metabolic Disorders in the ICU
Part 7Environmental and Toxicologic Injury
Part 8Pharmacologic Issues in the ICU
Part 9Surgical and Obstetrical Critical Care
Part 10Administrative and Ethical Issues in the Critically Ill
PART 1: Neurologic Critical Care

Instructions: For each question, select the most correct answer.

1. A 50-year-old man is brought to the emergency department by paramedics after collapsing at a tennis club. A bystander provided CPR, and on
arrival to the scene the paramedics successfully cardioverted the patient from ventricular fibrillation. Patient is now unresponsive to stimuli and
receiving mechanical ventilation. His temperature is 37.4C (99.3F), HR is 110/min, BP is 135/78 mm Hg, and RR is 20/min.

Which of the following interventions is most likely to improve neurologic outcome in this patient?
A. Amiodarone
B. Normothermia
C. Primary seizure prophylaxis with phenytoin
D. Hypothermia (32C34C [89.6F93.2F])
E. Beta blockade

2. A 75-year-old woman with a past medical history of breast cancer presents to the emergency department after being found comatose. She was last
seen well yesterday by her daughter. On admission to the ICU, her BP is 147/88 mm Hg, HR is 108/min, and RR is 1420/min. She is intubated and
mechanically ventilated but not sedated. Her examination shows clear lung fields and normal findings of cardiac and abdominal examinations.
There is no evidence of pressure sores. Cranial nerve examination shows symmetric face, normal pupillary reaction, and normal oculocephalic
reflexes. Her motor responses are flexor bilaterally, and toes are upgoing. Urine toxicology results are normal and her WBC count is 11,600/L,
with 80% neutrophils and 5% bands. She has a creatinine level of 256 mg/dL, normal results on arterial blood gas testing and chemistry panel, and
a normal CT and CT angiography of the head and neck.

Which of the following is the most appropriate next step?


A. Cerebral MRI
B. Comprehensive blood hepatic profile
C. Blood cultures
D. Electroencephalogram
E. Random cortisol level

3. A 68-year-old man with significant history of hypertension, diabetes mellitus, and benign prostatic hypertrophy is admitted to the hospital for onset
of confusion, memory loss, and agitation for the last 4 weeks. In the hospital he becomes very agitated, requiring a 1-on-1 supervision, chemical
restraints, and oxygen through a Venturi mask and has to be transferred to the ICU. Neurological examination findings are significant for expressive
aphasia, normal cranial nerves, and a motor examination showing extrapyramidal rigidity and stimulus-induced myoclonus. Serology and blood
chemistry, and lumbar puncture results are normal. MRI is shown in the Figure.

Which of the following is the most likely diagnosis for this patients altered mental status?
A. Acute ischemic stroke
B. Spongiform encephalopathy
C. Status epilepticus
D. Hepatic encephalopathy
E. New-onset Alzheimer dementia

4. A 43-year-old woman is brought to the hospital for abnormal mental status during the past 6 hours. Initially, she felt dizzy and nauseated and fell
asleep. The husband then called the ambulance as she was not waking up. She received fluids, naloxone, thiamine, and dextrose en route to the
emergency department without improvement. Her temperature is 37C (98.5F), BP is 150/102 mm Hg, HR is 108/min, RR is 1420/min, and
oxygen saturation is 92% on 2 L per nasal cannula. The patient is lethargic with Glasgow Coma Scale score of 8 (eye response of 2, motor response
of 4, and verbal response of 2), pupils are midpoint and fixed, and oculocephalic reflexes are only present on vertical movement of the head. Her
reflexes are 3+ throughout and she has bilateral upgoing toes. Abnormal asynchronous fast tonic movements of the arms and legs are also noted.
Arterial blood gas studies show pH of 7.20, PaCO2 of 60 mm Hg, PaO2 of 62 mm Hg, and oxygen saturation of 90%. The patient is intubated and
mechanically ventilated. CT shows a hyperdense signal at the level of the basilar artery.

Which of the following interventions is most appropriate at this point?


A. Systemic thrombolytics
B. Immediate IV antibiotics
C. IV phenytoin
D. Local thrombolytics
E. Intracranial pressure monitor

5. A 65-year-old woman with an unknown medical history is admitted to the hospital through the emergency department with concerns of fevers,
shortness of breath, productive cough, and myalgias. Chest radiograph on admission shows a right lower lobe infiltrate and IV antibiotics are
started for the treatment of a community-acquired pneumonia. Laboratory studies show an elevated white blood cell count with a neutrophil
predominance, mild transaminitis, hypoalbuminemia, anemia, thrombocytopenia, and an elevated prothrombin time/partial thromboplastin time.
During the night, the patient is confused, agitated, and paranoid. The nurses note that she is making up vivid and elaborate stories to account for her
presence in the hospital. On examination, she is awake, alert with fluent speech, and moving all extremities equally. She does not cooperate with the
cranial nerve examination but shows limitation of eye movements.

Which of the following IV treatments is it most appropriate to give this patient next?
A. Antibiotics
B. Thiamine and dextrose
C. Naloxone
D. Haloperidol
E. Dextrose solution

6. A 25-year-old graduate student is transferred to the ICU from the student health service after presenting with changes in mental status and a seizure.
The patient had returned from a white-water rafting vacation with friends when he had onset of an acute febrile illness characterized by headache,
disorientation, and behavioral abnormalities. He was aphasic and appeared to have hallucinations before presentation. He had right arm motor
weakness and a focal seizure confined to that area. In the ICU, the patient progresses to unresponsiveness. There is no significant past medical
history, and the patient has not traveled outside of the United States in the past 10 years. Temperature is 39.3C (102.6F) rectally, RR is 18/min,
HR is 106/min, and BP is 110/70 mm Hg. On physical examination, the patient is unresponsive with a slightly rigid neck. His pupils are
midposition, reactive to light, and optic disks appear to be flat. Findings of heart, lung, and abdominal examinations are normal. Neurologic
examination reveals hyperreflexia in the right upper extremity with an upgoing toe on that side. Cerebrospinal fluid from a lumbar puncture reveals
an opening pressure of 35 cm H2O and WBC count of 200/L, predominantly monocytic; closing pressure is 5 cm H2O. Protein level is mildly
elevated, glucose level is normal, Gram stain is negative, and RBC count is 1,000/L.

Which of the following is the most appropriate intervention for this patient?
A. IV dexamethasone
B. Immediate neurosurgical evaluation
C. IV acyclovir
D. IV amphotericin B
E. Performing another lumbar puncture in 6 hours

7. A 27-year-old woman is admitted for lethargy and headache during the last 7 days. Her past medical history is significant for oral contraceptive use
and recent appendectomy. On physical examination, her temperature is 38.1C (100.6F), BP is 138/86 mm Hg, HR is 100/ min, and RR is 24/min.
Her neurological examination reveals a lethargic patient who arouses to voice but is oriented to self. Her cranial nerve examination findings are
normal except for papilledema. Motor examination findings are normal, with no spasticity and movement of all 4 extremities to simple commands.
Reflexes are 3+ throughout. No ataxia is revealed and she does not cooperate for the gait or sensory examination. CT shows petechial hemorrhages
over the left parietal convexity.

At this point, which of the following interventions is most appropriate?


A. Hyperventilation to achieve a PaCO2 of 2832 mm Hg
B. Starting IV vancomycin and cefepime
C. Starting IV dexamethasone
D. Infusing IV fluids and heparin
E. Starting IV nicardipine

8. A 27-year-old woman is found on the floor by her husband, with tonic-clonic movements and foaming from the mouth. Emergency medical services
administers IV lorazepam, with control of the seizure in the field. On arrival to the emergency department, she is intubated for airway protection
using etomidate and succinylcholine. On arrival to the ICU, temperature is 41.1C (106 F), BP is 90/48 mm Hg, HR is 128/min, and RR is 18/min.
Her arterial blood gas results show pH of 6.98, PaO2 of 60 mm Hg, PaCO2 of 120 mm Hg, and oxygen saturation of 99%.

The most appropriate next step in the management of this patient is to:
A. Administer IV bicarbonate.
B. Decrease the minute ventilation by lowering tidal volume.
C. Start dantrolene.
D. Start IV hydrocortisone.
E. Increase minute ventilation.

9. The lockedin syndrome is best characterized by which of the following clinical descriptions:

Arousal Awareness Motor Activity Electroencephalographic Findings


A. Present Present Quadriplegic, vertical eye movements preserved Normal
B. Present Partial Normal Diffuse slowing
C. Absent Absent Absent Flat
D. Absent Absent Nonpurposeful, posturing (flexor or extensor) Slow, delta, theta
E. Absent Partial Purposeful Diffuse slowing

Note: modified from Stevens RD, Bhardwaj A. Approach to the comatose patient. Crit Care Med 2006; 34:3141.

10. A 45-year-old, right-handed man has sudden onset of confusion. On examination he is alert, not cooperative, and unable to follow commands. On
direct examination, he cannot read or write. On further questioning he is unable to repeat, but keeps on saying mani-pani-mani. The motor
examination shows normal strength of the face, arm, and leg. He cannot cooperate with the sensory examination, but does not seem to be ataxic. His
gait and reflexes are normal.
The patient is most likely experiencing which of the following types of aphasia?
A. Broca aphasia
B. Wernicke aphasia
C. Transcortical motor aphasia
D. Conduction aphasia
E. Global aphasia

11. A 43-year-old man ingests approximately 20 grams of acetaminophen in a suicide attempt and is treated with N-acetylcysteine. In the hospital, he
develops acute liver failure, becoming stuporous within 2 days of admission. His ammonia level is 123 mg/dL. An electroencephalogram is
performed showing diffuse slowing but no ictal activity.

Apart from a liver transplant, which of the following interventions is most likely to prevent neurological deterioration?
A. Prophylactic antiepileptic drugs
B. IV corticosteroids
C. Hyperosmolar therapy for intracranial pressure control
D. Lactulose for correction of hyperammonemia

12. A 78-year-old patient with past medical history of hypertension, diabetes, and atrial fibrillation on anticoagulation is brought to the hospital after a
motor vehicle accident. CT of the head shows an intracranial hemorrhage and he is taken emergently for surgical evacuation. His postoperative
course is uneventful until urine output begins to increase (200300 mL/h) on day 3. He is still intubated and mechanically ventilated.

Central venous pressure is estimated at 3 mm Hg. BP is 85/55 mm Hg, HR is 108/min, RR is 24/min, and SpO2 is 98%. The rest of the examination
is unchanged. Serum sodium level is 136 mEq/L.

Half-normal saline (0.45% sodium) is initiated at 100 mL/h; 12 hours later the sodium level is 132 mEq/L. His urine output has been 250 mL/h for
the past 10 hours. Urine electrolytes show sodium concentration of 90 mEq/L.

Which of the following is the most likely diagnosis?


A. Cerebral salt wasting
B. Syndrome of inappropriate antidiuretic hormone
C. Central diabetes insipidus
D. Osmotic diuresis

13. A 72-year-old man was admitted to the ICU 2 weeks ago with a left basal ganglia hypertensive hemorrhage without intraventricular extension. The
patient has a medical history significant for chronic obstructive pulmonary disease, hypertension, and hyperlipidemia. He remains in the ICU
because of an ongoing need for mechanical ventilation. On examination he remains hemiparetic over his right side. Lower extremity ultrasonography
reveals a right common femoral deep venous thrombosis.

Which of the following interventions is most appropriate in this patient?


A. Inferior vena cava filter placement
B. Localized thrombolytics
C. IV heparin (goal activated partial thromboplastin time 1.52 times baseline)
D. Antiplatelet therapy
E. Warfarin (goal international normalized ratio 2.03.0)
14. A 78-year-old man presents to the emergency department with right hemiparesis and expressive aphasia. Onset of symptoms was witnessed by his
wife and occurred 4 hours ago. CT of the head shows no evidence of developing hypodensity or hyperdensity. Temperature is 36.7C (98F), HR is
94/min, BP is 170/88 mm Hg, and RR is 18/min. Initial laboratory results are within normal limits.

Which of the following is a contraindication to treatment with IV thrombolytics?


A. Hip replacement 6 months ago
B. Duration of symptoms
C. Blood pressure
D. History of diabetes and previous stroke
E. His age

15. A 62-year-old man is admitted to the hospital after sudden onset of right hemiparesis and aphasia 5 hours ago. Currently he is mute and is not
following commands. Temperature is 37.3C (99.1F), BP is 182/100 mm Hg, HR is 102/min, RR is 22/min, and oxygen saturation is 96% on
oxygen, 2 L/min via nasal cannula.

a. Cerebral blood flow

b. Cerebral blood volume

c. Mean transit time

Based on the results of the CT perfusion study shown in the Figure, which of the following interventions is most appropriate?
A. Systemic tPA
B. External ventricular drain
C. Decompressive hemicraniectomy
D. Brain tissue oxygen monitor
E. Local tPA

16. A 38-year-old woman with episodic migraine and tobacco use is brought to the emergency department for evaluation of persisting headache. She
reports that her symptoms started 3 days ago with sudden onset of nausea, vomiting, and what she describes as the worst headache of her life.
Because she knew it could be a migraine, she took her prescribed triptan dose. In the emergency department she receives IV fluids, opioid
analgesics, and antiemetics. Her pain is down to 4/10, and her neurological examination findings are unremarkable. CT findings of the head are
reported as normal.

The most appropriate next step in the management of this patient is:
A. MRI of the brain
B. Repeating the dose of triptan and administering oxygen, 2 L/min
C. Admitting her to the ICU for close monitoring
D. Lumbar puncture
E. Discharge home with follow-up with primary neurologist

17. A 52-year-old woman with a subarachnoid hemorrhage of Hunt and Hess grade 3 and Fisher class 3 was admitted to the ICU after endovascular
coiling of a left posterior communicating artery aneurysm. On index day 7, you are called to see the patient because she has stopped following
commands and has a new left facial droop and leg weakness. Temperature is 37.3C (99.1F), HR is 98/min, BP is 190/92 mm Hg (mean arterial
pressure of 123 mm Hg), RR is 20/min, and central venous pressure is 10 cm H2O. Her angiogram is shown in the Figure.

Which of the following is the most appropriate immediate intervention for this patient?
A. Decrease blood pressure.
B. Initiate heparin infusion.
C. Administer bolus of normal saline.
D. Administer local tPA.
E. Perform angioplasty.

18. A 64-year-old, hypertensive patient presents with a spontaneous intracerebral (intraparenchymal) hemorrhage.

Which of the following locations of intracerebral hemorrhage would be most likely to require surgical intervention (ie, craniotomy)?
A. Basal ganglia
B. Cerebellum
C. Internal capsule
D. Parietal lobe (unilateral)
E. Pons

19. A 40-year-old man is admitted to the ICU with progressive inability to walk during the preceding 24 hours. His BP is 140/80 mm Hg, pulse rate is
88/min, RR is 20/min, and temperature is 37.3C (99F). He is alert and oriented but reports cramping pain in his legs bilaterally with paresthesias.
Neurologic examination shows 2/5+ strength in the proximal and distal muscle groups of the lower extremities and absent ankle and knee deep
tendon reflexes. Rectal sphincter tone is normal. He has normal strength of his upper extremities and cranial nerves are normal.

Which of the following interventions is the most appropriate initial therapy to improve neurologic outcome with the least adverse effects?
A. Plasmapheresis and concomitant IV immunoglobulin
B. Plasmapheresis followed by IV immunoglobulin
C. IV immunoglobulin only
D. High-dose corticosteroids

20. A 34-year-old man is admitted to the ICU for diabetic ketoacidosis. The patient has a past medical history significant for tobacco and cocaine
abuse. He responds well to treatment and is improving to the point where transfer to the regular medical floor is considered. You are called to the
bedside by the nurse because the patient reports new-onset headache with new ptosis. On physical examination, his temperature is 36C (96.7F),
BP is 140/78 mm Hg, HR is 98/min, and RR is 18/min. He is alert, oriented, and cooperative. There is no evidence of aphasia or neglect. The right
eye is ptotic with hypotropia/exotropia. The right pupil is 5 mm and nonreactive and the left pupil is 3 mm and reactive. His face is not weak but
looks asymmetric because of the right eye ptosis. No papilledema is noted. The rest of the neurological examination findings are normal.

Which of the following is the most appropriate next step in this patients care?
A. Electroencephalography
B. Checking blood glucose level
C. Performing the pyridostigmine test
D. MR angiography of the brain
E. Noncontrast CT

21. An 18-year-old male is brought to the emergency department after a motor vehicle crash. He was intubated in the field with cervical spine in-line
immobilization and given IV fluids. On arrival to the emergency department, he was hypotensive and received 4 L of crystalloid fluids with marked
improvement in the hemodynamics. On arrival to the ICU, his Glasgow Coma Scale score remains 6 (verbal response, 1; motor response, 3; eye
response, 2).

For which of the following interventions is there the strongest evidence for reducing this patients mortality risk?
A. Avoidance of hypotension (systolic BP <90 mm Hg)
B. Decompressive hemicraniectomy
C. Hyperosmolar therapy
D. Induced hypothermia
E. Intracranial pressure monitoring

22. The patient whose brain image is shown in the Figure has suffered severe traumatic brain injury. He remains comatose.

Which of the following treatments can be eliminated from consideration?


A. Sedation and analgesia
B. Intracranial pressure monitoring
C. Head elevation to 30 degrees
D. Treatment of coagulopathy
E. IV corticosteroids
23. Which of the following intracranial pressure waveforms is most indicative of low cerebral compliance?

A.

B.

C.

D.

24. The only intervention proven to decrease the incidence of epilepsy after severe traumatic brain injury is:

A. Prophylaxis with phenytoin


B. Early decompressive hemicraniectomy and clot evacuation
C. Intracranial pressure monitoring with a goal <15 mm Hg
D. IV corticosteroids
E. Primary prevention

25. A 48-year-old man was admitted to the trauma ICU after multiorgan trauma. He had a fractured femur with open book hip fracture and severe
traumatic brain injury. The orthopedic surgeons have repaired his injuries and the neurosurgeons have inserted an intraparenchymal intracranial
pressure (ICP) monitor. Currently, he is receiving midazolam, 0.5 mg/h, and fentanyl, 25 g/h. His temperature is 37.5C (99.4F), BP is 150/104
mm Hg, HR is 112/min, and he is over-breathing the ventilator. His pupils are 3 mm bilaterally and reactive, and he withdraws symmetrically with
both arms. His head remains elevated at 30 degrees, but the ICP monitor is reading 35 cm H2O. Blood gas results show pH of 7.38, PaCO2 of 43 mm
Hg, PaO2 of 110 mm Hg, and oxygen saturation of 99% (0.4).

The next step to attempt to decrease his ICP is:


A. Immediately give a bolus of 20% mannitol solution, 0.251 g/kg over 5 minutes.
B. Immediately give a bolus of 23.4% hypertonic saline solution, 20 mL over 10 minutes.
C. Immediately give a bolus of fentanyl, 50 g, and increase sedation and analgesia.
D. Target PaCO2 to 2835 mm Hg.
E. Start mild induced hypothermia to 33C35C (91.4F94.9F).

26. A 57-year-old man is admitted to the ICU after a fall from a 6- foot ladder. An initial trauma survey reveals an isolated traumatic brain injury with
small, bifrontal contusions associated with cerebral edema, cortical subarachnoid hemorrhage, and intraventricular hemorrhage in the fourth
ventricle; he has moderate hydrocephalus. The patient was intubated in the field and has a Glasgow Coma Scale score of 6. Elevated intracranial
pressure (ICP) is suspected.

Which of the following interventions can lower ICP immediately?


A. 23.4% hypertonic saline, 30-mL bolus
B. Head elevation
C. External ventricular drain and cerebrospinal fluid diversion
D. Mannitol
E. Hyperventilation to keep PaCO2 within 2832 mm Hg for <30 minutes

27. The patient with the brain imaging shown in the Figure had severe traumatic brain injury due to a fall from the second floor of his home. On arrival
he was awake, conversant, and wanted to go home. Thirty minutes after arrival to the emergency department, he was found comatose with a 7-mm
unreactive pupil on the right and hemiplegia on the left.

The survival of this patient will depend on:


A. Burr holes
B. Emergent craniotomy
C. External ventricular drain
D. Corticosteroids
E. Starting 3% saline solution at 30 mL/h

28. A 28-year-old woman is admitted to the trauma ICU after falling from her horse. She was intubated in the field and received 6 L of crystalloid
fluids in the emergency department. Temperature is 38C (100.3F), BP is 68/42 mm Hg, HR is 41/min, RR is 16/min on the ventilator, and oxygen
saturation is 100% (1.0). Her neurologic examination reveals a Glasgow Coma Scale score of 7 (verbal response, 1; eye response, 3; motor
response, 3), pupils are reactive to light, and she has good gag reflex but no cough reflex. Her motor examination reveals flaccid quadriparesis and
absent reflexes in arms and legs. Primary and secondary surveys have excluded life-threatening injuries and a focused assessment with sonography
for trauma (FAST) ruled out abdominal hemorrhage. Spinal CT is shown in the Figure. Urinary output is 0.3 mL/kg/h during the last 2 hours.
Her shock state will likely respond to the following:
A. Phenylephrine, 510 g/kg/min
B. Blood transfusion
C. Neurosurgical decompression
D. Dopamine, 510 g/kg/min
E. IV methylprednisolone

29. A 72-year-old woman is involved in a car crash. She is confused immediately after the collision and is taken to the emergency department. On
arrival, she has recovered her baseline mental status. Head CT reveals a small, acute right frontal subdural hematoma without mass effect. A
neurosurgical consultant recommends conservative management and observation.

Which of the following anticonvulsant regimens will most effectively prevent posttraumatic seizure for this patient?
A. Phenytoin for 7 days
B. Phenytoin for 6 weeks, followed by outpatient EEG
C. Valproate for 1 month
D. Levetiracetam for 1 month

30. A 58-year-old, otherwise healthy man is brought to the emergency department after a witnessed cardiac arrest. After 26 minutes of CPR, he
recovered his pulse. The patient is admitted to the ICU for hypothermia protocol. After rewarming, he remains on high-dose vasopressors
(norepinephrine and vasopressin) to keep mean arterial pressure greater than 65 mm Hg. Seventy-two hours after his cardiac arrest, his neurological
examination findings are characterized by a Glasgow Coma Scale score of 3, and absent cranial nerve function (papillary, oculocephalic, cold-
caloric, and gag/cough responses are absent). During motor examination, a right hand twitch is noted as well as left flexion/rotation of the neck after
painful stimulation. The following brain perfusion study (cerebral scintigraphy with technetium Tc99m-hexametazime [HMPAO]) is performed
based on the equivocal findings of his motor examination.

When the family requests the intensivists expert opinion, it is most appropriate to tell them that:
A. The patient may not be brain dead and the findings of his motor examination suggest that medical therapy should continue
B. The patient is brain dead and there is no reversibility
C. The patient should have a cerebral angiography to confirm brain death
D. A trial of supramaximal elevation of mean arterial pressure may be helpful
E. Seizures are likely, so a trial of IV midazolam may be indicated

31. A 65-year-old man has suffered a devastating intracerebral hemorrhage. An extraventricular drain was inserted for hydrocephalus, and he remained
in the ICU for 12 hours. He is currently receiving norepinephrine at maximum doses to keep a mean arterial pressure greater than 65 mm Hg, and his
urine output has been 300 mL/h. On physical examination, his BP is 98/58 mm Hg, HR is 118/min, RR is 14/min on the ventilator, oxygen saturation
is 98% (0.6), and temperature is 36C (96.7F). His pupils are 5 mm and nonreactive, with no corneal reflex, no oculocephalic reflexes and no
cold-caloric responses. Gag and cough reflexes are absent, as are spontaneous respirations. During nail-bed pinching, he has a triple flexion
response of both lower extremities. The team repeats the examination within 6 hours and finds similar results, but at this time fasciculations are
apparent in the upper and lower extremities.

On the basis of his neurological examination findings, the team should:


A. Perform an apnea test
B. Continue ongoing therapy and have a family discussion
C. Declare him brain dead
D. Activate organ donation network
E. Rewarm the patient to 37C (98.5F) and repeat examination

32. A 32-year-old woman is admitted to the ICU for hypoxemic respiratory failure, septic shock, and acute respiratory distress syndrome secondary to
H1N1 flu. Because of severe untreatable hypoxemia, the team decides to bridge her to extracorporeal membrane oxygenation. Thirteen days after
admission, the patient remains critically ill with mild improvement of oxygenation. Her temperature is 36.7C (98F), BP is 120/78 mm Hg, HR is
118/min, RR is 18/min, and oxygen saturation is 92%. Without sedatives and paralytics, she appears comatose; Glasgow Coma Scale score is 3;
pupils are 5 mm and nonreactive; and no corneal, oculocephalic, cold-caloric, or gag/cough responses are seen. Spontaneous respirations are
absent. Motor examination is unrevealing. CT shows diffuse cerebral edema.

The next step in the management of this patient is to:


A. Declare her brain dead
B. Perform a modified apnea test
C. Perform a classic apnea test
D. Perform an MRI of the brain
E. Continue medical management as she is likely to survive

33. Which of the following precludes the diagnosis of brain death?


A. Absence of respiratory drive
B. Absence of cranial nerve responses
C. Body temperature of 34C (93.2F)
D. Known cause of coma
E. PaCO2 of 46 mm Hg

34. The persistent vegetative state is particularly characterized by:


A. Sleep and awake cycles
B. Organized EEG patterns
C. Absent N20 responses on somatosensory evoked potential testing
D. State of awareness
E. Purposeful motor function
PART 1: Neurologic Critical Care

ANSWERS:

1D; 2D; 3B; 4D; 5B; 6C; 7D; 8C; 9A; 10B; 11C; 12A; 13C; 14D; 15C; 16D; 17E; 18B; 19C; 20D; 21A; 22E; 23B; 24E;
25C; 26E; 27B; 28D; 29A; 30B; 31A; 32B; 33C; 34A

RATIONALE (1) Answer: D

The current advanced cardiac life support guidelines put an important emphasis on appropriate postresuscitation support, in an effort to improve
outcomes of patients with cardiac arrest. Increasing attention has been placed on the institution of measures that may improve long-term, neurologically
intact survival. Among the choices provided in the question, the correct answer is D, hypothermia to 32C34 C (89.6F93.2F) for 1224 hours.
Therapeutic hypothermia for patients without a hospital cardiac arrest who have a return of spontaneous circulation and present to the hospital
unresponsive has been studied in 2 recently published, randomized clinical trials. Both studies demonstrated that therapeutic hypothermia can improve
long-term neurologic outcomes and survival in patients with ventricular fibrillation in out-of-hospital cardiac arrest. Other important measures to
improve outcomes include the active treatment of hyperthermia when present, the aggressive treatment of seizures if present, and control of
hyperglycemia associated with critical illness. Amiodarone and beta-blockers have not been associated with improved neurologic outcomes in patients
with cardiac arrest; thus, options A and E are incorrect. Tight glycemic control has received a lot of attention in the past several years based on the
findings of a study that demonstrated improved survival in a surgical critical care population in which glucose levels were kept between 80 and 110
mg/dL. No studies have demonstrated that tight glycemic control improved neurologic outcomes following cardiac arrest; therefore, option B is
incorrect. However, it is recognized that hypoglycemia can be deleterious to neurologic recovery, and it is recommended that it be treated. Although
aggressive treatment of seizures when present is recommended by the advanced cardiac life support guidelines, there is no literature to support primary
seizure prophylaxis in these patients. Therefore, option C is also incorrect.

REFERENCES (1)

1. Peberdy MA, Callaway CW, Neumar RW, et al. Part 9: post-cardiac arrest care: 2010 American Heart Association Guidelines for Cardiopulmonary
Resuscitation and Emergency Cardiovascular Care. Circulation 2010;122:S768S786.
2. Hypothernia After Cardiac Arrest Study Group. Mild therapeutic hypothermia to improve the neurologic outcome after cardiac arrest. N Engl J Med
2002;346:549556.
3. Bernard SA, Gray TW, Buist MD, et al. Treatment of comatose survivors of out-of-hospital cardiac arrest with induced hypothermia. N Engl J Med.
2002;346:557563.
4. Polderman K. Application of therapeutic hypothermia in the intensive care unit. Intensive Care Med. 2004;30:757769.5.
5. Nunnally ME, Jaeschke R, Bellingan GJ, et al. Targeted temperature management in critical care: A report and recommendations from five
professional societies. Crit Care Med.2011;39:11131125.

RATIONALE (2) Answer: D

Patients who come to the emergency department in a coma can be categorized into a number of groups: metabolic disarray, overdoses and poisoning,
vascular disease, infection, and seizures. Of these groups, a commonly overlooked diagnosis is nonconvulsive status epilepticus. According to Towne et
al2, 8% of patients evaluated for coma in their study had nonconvulsive status epilepticus. The evaluation of a patient with coma should include a search
for metabolic abnormalities and drugs of abuse, an investigation of medications in the home, a search for infection including meningitis, and an
evaluation by both physical examination and imaging for vascular disease. If all of these tests show no reason for the coma, nonconvulsive status
epilepticus should be considered. The gold standard test is electroencephalography. Patients with previous stroke, brain tumors, and previous history of
seizures are at particular risk of continuous seizures.

REFERENCES (2)

1. Plum F, Posner JB. The diagnosis of stupor and coma.Third ed. Philadelphia:Davis; 1980.
2. Towne AR, Waterhouse EJ, Boggs JG, et al. Prevalence of nonconvulsive status epilepticus in comatose patients. Neurology. 2000; 54:340345.
3. Oddo M, Carrera E, Claassen J, et al. Continuous electroencephalography in the medical intensive care unit. Crit Care Med 2009;37:20512056.

RATIONALE (3) Answer: B

Creutzfeldt-Jakob disease (CJD), or spongiform encephalopathy, is an incurable and invariably fatal degenerative neurological condition. Symptoms of
CJD are rapidly progressive dementia, memory loss, personality changes, and hallucinations. Clinically, patients may have aphasia, dysarthria,
spasticity, and myoclonus. The diagnosis requires a high level of suspicion. Lumbar puncture is always normal, but a high 14:3:3 protein level may be
seen, though its specificity is very poor. Electroencephalogram may show diffuse slowing or triphasic waves. Fluid-attenuated inversion recovery
(FLAIR) or diffusion-weighted imaging (DWI) MRI may show hyperintensity in basal ganglia and cortex (MRI in this case shows DWI hyperintensity in
cortical areas of the insula and the parietal lobe of the left hemisphere). Apart from CJD, other entitites such as endovascular lymphomatosis, Hashimoto
thyroiditis, Whipple disease, and paraneoplastic syndromes may present with rapidly progressive dementias.

REFERENCES (3)

1. Rowland PL, Pedley T. Merritt's Neurology. 12th Edition. Philadelphia: Lippincott Williams & Wilkins; 2009.
2. Appleby BS, Lyketsos CG. Rapidly progressive dementias and the treatment of human prion diseases. Expert Opin Pharmacother. 2011;12:112

RATIONALE (4) Answer: D

The clinical syndrome localizes to the brainstem. This patient is comatose from a basilar artery thrombosis. CT shows a hyperdense sign in the basilar
artery suggesting a fresh clot. As she is past the 4.5-hour window, she is no longer a candidate for systemic thrombolytics (ie, IV tPA). Intra-arterial
thrombolysis is an option for treatment of selected patients who have major stroke of less than 6 hours' duration due to occlusions of the middle cerebral
artery and who are not otherwise candidates for IV rtPA. Posterior circulation occlusions, on the other hand, may be amenable to endovascular
management within up to 24 hours of symptom onset. However this treatment is supported by small studies and expert opinions. Alternatives for the
management of ischemic stroke beyond the usual time windows using endovascular approaches include concentric retrievers or fibrinolysis catheters,
and/or local intra-arterial thrombolysis (depending on individual assessment). However, these interventions have not been shown to improve outcomes
after stroke. Treatment requires the patient to be at an experienced stroke center with immediate access to cerebral angiography and qualified
interventionists.

REFERENCES (4)

1. Janjua N, Brisman JL. Endovascular treatment of acute ischaemic stroke. Lancet Neurology. 2007;6:10861093.
2. Adams HP Jr, del Zoppo G, Alberts MJ, et al. Guidelines for the early management of adults with ischemic stroke: a guideline from the American
Heart Association/American Stroke Association Stroke Council, Clinical Cardiology Council, Cardiovascular Radiology and Intervention Council,
and the Atherosclerotic Peripheral Vascular Disease and Quality of Care Outcomes in Research Interdisciplinary Working Groups: The American
Academy of Neurology affirms the value of this guideline as an educational tool for neurologists. Circulation. 2007;115:e478534.

RATIONALE (5) Answer: B

The patient presents with a community-acquired pneumonia and is found to be confused, confabulating, and without recent memories of her illness. Her
examination shows limited eye movements, and laboratory tests show evidence of malnutrition. With Korsakoff syndrome, patients typically present
with anterograde memory impairment. Wernicke encephalopathy is characterized by confusion, gait ataxia, and oculomotor abnormalities in an alcoholic
patient, but is now also appreciated to occur in any malnourished state including end-stage cancer, intractable vomiting after gastric reduction
procedures, hyperemesis gravidarum, prolonged hospital courses of IV fluids, and in chronic hemodialysis. On MRI imaging, there is mammillary body
shrinkage, and involvement of the medial thalamic and periaqueductal nuclei. Treatment consists of parenteral administration of thiamine, 100 mg/day
for several days, as the gastrointestinal mucosa absorbs poorly in the malnourished state. Glucose should not be administered before thiamine, to avoid
precipitating Wernicke or causing an early form of the disease to progress.

REFERENCES (5)

1. Ihara M, Ito T, Yanagihara C, et al. Wernicke's encephalopathy associated with hemodialysis: Report of 2 cases and review of the literature. Clin
Neurol Neurosurg. 1999;101:118121.
2. Kumar PD, Nartsupha C, West BC. Unilateral internuclear ophthalmoplegia and recovery with thiamine in Wernicke syndrome. Am J Med Sci. 2000;
320:278280.
3. Ogershok PR, Rahman A, Nestor S, et al. Wernicke encephalopathy in nonalcoholic patients. Am J Med Sci. 2002;323:107111.
4. Zubaran C, Fernandes JG, Rodnight R. Wernicke-Korsakoff syndrome. Postgrad Med J. 1997; 73:2731.

RATIONALE (6) Answer: C

This patient most likely has herpes simplex encephalitis based on the physical examination findings, the clinical presentation, and cerebrospinal fluid
analysis. Viral encephalitis begins with the acute onset of a febrile illness. Additional findings include fever, headache, disorientation, alterations in
behavior and speech, and neurologic findings that occasionally may be focal. These clinical findings distinguish a patient with encephalitis from a
patient with viral meningitis. Characteristically, patients with viral meningitis have nuchal rigidity, headache, and photophobia but lack focal neurologic
findings or significant changes in mental status. The lumbar puncture is characteristic in this case, revealing a cerebrospinal fluid pleocytosis with
monocytic predominance, a normal glucose level, and red blood cells suggesting herpes simplex encephalitis. Herpes simplex virus has a predilection
for the temporal lobe and electroencephalography may reveal focal findings. In addition, CT or MRI may reveal changes in the temporal lobe area. For
bacterial meningitis, Gram stain might be expected to have positive results. Although fungal meningitis would be a concern, this usually presents in a
subacute or chronic manner.

Viruses characteristically gain access to the central nervous system through either hematogenous or neuronal spread. It is felt that herpes simplex gains
access to the central nervous system via a neuronal pathway, with some suggestion that the olfactory tract may be the access of choice. Herpes simplex
virus is the most common cause of nonepidemic acute focal encephalitis in the United States. Infections with this virus may occur any time of the year
and are more common in patients younger than 20 and older than 50 years of age. Blood in the cerebrospinal fluid is characteristic of herpes simplex
encephalitis. A definitive diagnosis is established by brain biopsy, although this is usually reserved for patients for whom therapy with acyclovir fails.
Successful therapy depends upon early institution of acyclovir, based upon a high index of suspicion, along with clinical findings and cerebrospinal fluid
values.

REFERENCES (6)

1. Jackson AC. Acute viral infections. Curr Opin Neurol. 1995;8:170.


2. Lipton JD, Schafermeuer RW. Central nervous system infections: the usual and the unusual. Emerg Med Clinics North Am. 1995;13:417443.
3. Schmutzhard E. Viral infections of the CNS with special emphasis on herpes simplex infections. J Neuro.l 2001; 248:469-477.

RATIONALE (7) Answer: D

Acute thrombosis of intracranial venous draining systems may present as secondary hemorrhages from infarcted brain areas. The clinical presentation
depends generally on the venous anatomy, but the onset of headache, abnormal mental status, seizures, and papilledema suggests the diagnosis. Superior
sagittal sinus thrombosis is involved in about 70% of the cases; the transverse, straight, and sigmoid sinuses may also be involved. Acute sinus
thrombosis may be seen in patients with predisposing factors such as infection (otitis media), prothrombotic states (hormone-related, cancer, and
pregnancy), and inherited hypercoagulable states (activated protein C resistance by factor V Leiden mutation, protein C and S deficiency). Full
anticoagulation is indicated despite the presence of hemorrhagic infarction, accompanied by judicious IV fluid hydration, seizure management, and
measures to control intracranial pressure, such as head elevation and analgesia. For rapidly progressive cases, transvenous endovascular thrombolytic
therapy is a therapeutic option.

REFERENCES (7)

1. Einhaupl KM, Villringer A, Meister W, et al. Heparin treatment in sinus venous thrombosis. Lancet. 1991;338:597.
2. Stam J.Thrombosis of the cerebral veins and sinuses. N Engl J Med. 2005 Apr 28;352:17911798.
3. Sacco RL, Adams R, Albers G, et al. Guidelines for prevention of stroke in patients with ischemic stroke or transient ischemic attack: a statement for
healthcare professionals from the American Heart Association/American Stroke Association Council on Stroke: co-sponsored by the Council on
Cardiovascular Radiology and Intervention: the American Academy of Neurology affirms the value of this guideline. Circulation. 2006 Mar
14;113:e40949.

RATIONALE (8) Answer: C

The patient has malignant hyperthermia, a genetic condition with autosomal dominant inheritance and an incidence of 1 in every 15,000 episodes of
general anesthesia or exposure to succinylcholine. The clinical picture is characterized by a sudden rise of PaCO2 or end-tidal carbon dioxide, muscle
rigidity, hyperthermia, depressed consciousness, and autonomic instability, and could lead to myonecrosis, rhabdomyolysis, and acute renal failure. The
syndrome is explained by excessive calcium efflux from the sarcoplasmic reticulum in response to halogenated inhalational agents as a result of
uncoupling of oxidative phosphorylation with dramatically increased metabolic rate.
REFERENCES (8)

1. McAllen KJ, Schwartz DR. Adverse drug reactions resulting in hyperthermia in the intensive care unit. Crit Care Med. 2010 Jun;38(6 Suppl):S244
52.
2. Litman RS, Flood CD, Kaplan RF, et al. Postoperative malignant hyperthermia: an analysis of cases from the North American Malignant
Hyperthermia Registry. Anesthesiology. 2008;109: 825829.
3. Malignant Hyperthermia Association of the United States. http://www.mhaus.org/. Accessed June 2010.

RATIONALE (9) Answer: A

Locked-in syndrome (LIS) consists of quadriplegia and anarthria in the setting of preserved awareness and arousal. It not a consciousness disorder per
se but may be clinically confused with one given the limited expressive capability of these patients. It is associated with acute injury to the ventral pons,
just below the level of the third nerve nuclei, thus classically sparing vertical eye movements and blinking. More rostral lesions may induce a "total"
LIS, in which even eye and lid movement is lost, prohibiting all communication. Common etiologies of the LIS are pontine infarction, hemorrhage, and
trauma. The electroencephalogram is usually normal.

REFERENCES (9)

1. Stevens RD, Bhardwaj A. Approach to the comatose patient. Crit Care Med. 2006;34:3141.
2. Smith E, Delargy M. Locked-in syndrome. BMJ. 2005;330:406409.

RATIONALE (10) Answer: B

This patient has suffered a stroke of the inferior division of the dominant (left ) middle cerebral artery (MCA) and has Wernicke-type aphasia (receptive
or sensory). Apart from posterior cerebral artery and top-of-the-basilar strokes, sudden onset of Wernicke aphasia secondary to cerebral infarction is
most often caused by cardioembolic phenomena. The typical picture is that of a patient with abnormal language examination findings and inability to
read, write, or follow commands. Patients with inferior division strokes and Wernicke aphasia have normal motor examination findings but a
hemianopia may be missed if they are not thoroughly examined during direct confrontation. A "word salad" speech with neologisms or paraphasias is
part the typical picture. Patients with nondominant inferior division MCA strokes usually present with confusional states, which tend to be missed by
inexperienced examiners. Truncation of the upper division of the MCA will produce a language dysfunction characterized by expressive or motor
aphasia (Broca-type aphasia), where comprehension may be preserved. In the hyperacute phases of MCA occlusion, patients manifest global language
disruption, and in this state the abnormality is so profound that patients are globally aphasic with null comprehension or expression. In transcortical
motor aphasias, patients have no motor expression but retain the ability to repeat verbal commands. In transcortical sensory aphasias, patients have no
comprehension but retain the ability to follow verbal commands. Subcortical strokes (thalamus, caudate, arcuate fasciculus, and cingulate gyrus) are
often associated with transcortical type aphasias. Conduction aphasia, also called associative aphasia, is a relatively rare form of aphasia. An acquired
language disorder, it is characterized by intact auditory comprehension, fluent (yet paraphasic) speech production, but poor speech repetition.

Fluency Comprehension Repetition


Broca No Yes No
Wernicke Yes No No
Transcortical motor No Yes Yes
Transcortical sensory Yes No Yes
Global No No No
Conduction Yes Yes No

REFERENCES (10)

1. Mohr JP, Lazar RM, Marshal RP. Middle cerebral artery disease. In: Stroke:Pathophysiology, Diagnosis, and Management. Fourth Edition. New
York: Elsevier; 2004.
2. Pedersen PM, Vinter K, Olsen TS. Aphasia after stroke: type, severity and prognosis. The Copenhagen aphasia study. Cerebrovascular Diseases.
2004;17:3543.

RATIONALE (11) Answer: C

Acute liver failure (ALF) is a catastrophic multisystem illness and defined as the onset of encephalopathy and coagulopathy within 24 weeks of a
hepatic insult in a previously healthy person. Survival in ALF depends primarily on liver transplantation and the treatment of multiorgan dysfunction in
the ICU. Cerebral edema occurs in up to 75% of patients with grade III and IV hepatic encephalopathy and frequently progresses to intracranial
hypertension. Cerebral edema and herniation are the leading cause of death after ALF. Though no randomized prospective controlled trial has
determined the optimal management of ALF and intracranial hypertension, a protocol of cerebral perfusion pressure optimization (CPP = MAP ICP)
greater than 60 mm Hg, hyperosmolar therapy with mannitol and 3% saline solution, initial hyperventilation to keep PaCO2 at 3035 mm Hg, mild
induced hypothermia to 33C (91.4F), and/or pentobarbital was highly successful in controlling intracranial pressure and mitigating the risk of
neurologic deterioration.

REFERENCES (11)

1. Raschke RA, Curry SC, Rempe S, et al. Results of a protocol for the management of patients with fulminant liver failure. Crit Care Med
2008;36:2244 2248.2.
2. Stravitz RT, Kramer AH, Davern T, et al. Intensive care of patients with acute liver failure: Recommendations of the U.S. Acute Liver Failure Study
Group. Crit Care Med 2007;35:24982508.

RATIONALE (12) Answer: A

This represents a typical case of cerebral salt wasting syndrome. This controversial entity is typically encountered in the first 12 weeks after
neurosurgical procedures (as in the case above) but can also occur with infectious causes (meningitis) or intracerebral bleeding. An increase in brain
natriuretic peptide is thought to be the inciting factor that results in reduced renal reabsorption of solutes, diuresis, and thus volume depletion and
hypotension.

The finding of volume depletion is paramount for diagnosing cerebral salt wasting syndrome in order to distinguish it from the syndrome of
inappropriate antidiuretic hormone (SIADH), in which patients are euvolemic. The clinician should suspect cerebral salt wasting in the appropriate
clinical setting (after neurosurgery or with other intracranial disease) when facing hypovolemic hyponatremia with high urinary sodium levels indicating
an improper renal response to volume depletion.

In the current case, the patient has overt signs of volume depletion with hypotension, tachycardia, and low central venous pressure and thus does not
have SIADH (option B is wrong). Central diabetes insipidus and osmotic diuresis are associated with hypernatremia, so options C and D are incorrect.
Volume expansion (with normal saline) is the initial treatment for cerebral salt wasting. Salt tablets can be substituted later in the course of the disease,
which is usually transient and resolves within a month of onset.

REFERENCES (12)

1. Maesaka JK, Imbriano LJ, Ali NM, et al. Is it cerebral or renal salt wasting? Kidney Int 2009;76:934.
2. Palmer BF. Hyponatraemia in a neurosurgical patient: syndrome of inappropriate antidiuretic hormone secretion versus cerebral salt wasting.
Nephrol Dial Transplant 2000;15:262.
3. Harrigan MR. Cerebral salt wasting syndrome: a review. Neurosurgery. 1996;38:152.

RATIONALE (13) Answer: C

According to the guidelines for prevention of stroke in patients with ischemic stroke or transient ischemic attack,1 for patients who develop an
intracranial hemorrhage, subarachnoid hemorrhage, or subdural hematoma, all anticoagulants and antiplatelets should be discontinued during the acute
period for at least 1 to 2 weeks after the hemorrhage, and the anticoagulant effect should be reversed immediately with appropriate agents (ie, vitamin K,
fresh frozen plasma) (class III, level of evidence B). Hence, resumption of anticoagulation may be considered at or beyond 2 weeks from ictus. These
guidelines also suggest that IV heparin with partial thromboplastin time 1.5 to 2.0 times normal may be a safer option when early reinstitution of
anticoagulation is indicated. Furthermore, eighth edition American College of Chest Physicians guidelines2 state that for patients with objectively
confirmed deep venous thrombosis, short-term treatment with IV unfractionated heparin, subcutaneous low-molecular-weight heparin, monitored
subcutaneous unfractionated heparin, fixed-dose subcutaneous unfractionated heparin, or subcutaneous fondaparinux is recommended (grade 1A).

REFERENCES (13)

1. Sacco RL, Adams R, Albers G, et al. Guidelines for prevention of stroke in patients with ischemic stroke or transient ischemic attack. A statement
for healthcare professionals from the American Heart Association/American Stroke Association Council on Stroke: Co-Sponsored by the Council on
Cardiovascular Radiology and Intervention. Circulation. 2006;113:e409e449.
2. Kearon C, Kahn SR, Agnelli G, et al. Antithrombotic therapy for venous thromboembolic disease: American College of Chest Physicians evidence-
based clinical practice guidelines. Eighth edition. Chest. 2008;133(6 Suppl):454S545S. Erratum in Chest. 2008 Oct;134:892.
RATIONALE (14) Answer: D

Based upon emerging data, especially Hackeet al,2 the American Heart Association/American Stroke Association has espoused an expanded time
window for the use of IV tPA. Patients that meet appropriate criteria may now be treated between 3 and 4.5 hours after symptom onset. However,
several criteria in addition to those associated with the 3-hour window must be met. Additional exclusion criteria include age older than 80 years, taking
oral anticoagulants with an international normalized ratio (INR) 1.7 (thus, anyone receiving oral anticoagulation regardless of INR), a baseline
National Institutes of Health Stroke Scale score greater than 25, and history of stroke and diabetes.

REFERENCES (14)

1. Del Zoppo, Saver JL, Jauch EC, et al. Expansion of the time window for treatment of acute ischemic stroke with intravenous tissue plasminogen
activator: a science advisory from the American Heart Association/American Stroke Association. Stroke. 2009;40:29452948.
2. Hacke W, Kaste M, Bluhmki E, et al; ECASS Investigators. Thrombolysis with alteplase 3 to 4.5 hours after acute ischemic stroke. N Engl J Med.
2008;359:28392841.

RATIONALE (15) Answer: C

CT-assisted dynamic perfusion imaging (CTP) has evolved in recent years as an adjuvant for the assessment of acute ischemic stroke. The method
permits quantitative determination of cerebral blood flow (CBF), cerebral blood volume (CBV), and mean transit time (MTT). CBF is the most
important parameter (CBF = CBV/MTT). It indicates how much blood is flowing through the brain tissues in a specific period, and it is measured in
milliliters of blood per 100 grams of brain tissue per minute. Normal values for CBF are between 50 and 80 mL/100 g/min. Areas of the brain with high
energy requirements, such as the cortical surface or the basal ganglia, exhibit CBF values some 23 times higher than those for white matter.

CBV is defined as the percentage of blood vessels in a specific volume of tissue. Highly vascularized areas of the brain such as the basal ganglia or the
cortical surface therefore have a higher CBV than the less vascularized cerebral white matter. The CBV, however, is also a functional parameter and
changes if vessel size changes in the context of vascular autoregulation.

MTT is a direct measurement of delayed perfusion. There is a direct correlation between it and cerebral perfusion pressure. Even slight disturbances to
the blood supply can lead to the MTT's being prolonged. In clinical studies on strokes, the MTT has been a very sensitive indicator of disruption in
regional perfusion of the brain.

In this case, CBF is low in the left middle cerebral arterty (MCA) territory secondary to an M1-segment occlusion. The CBV is low and MTT is very
prolonged, indicating a stroke. This pattern suggests that the patient will likely develop malignant infarction, increased cerebral edema, and intracranial
hypertension, and may require a decompressive hemicraniectomy to survive. About 80%90% of patients with complete MCA strokes develop
malignant stroke. The only intervention shown to improve mortality and functional outcome after malignant MCA stroke is decompressive
hemicraniectomy. The patient is beyond the 4.5-hour window for IV tPA, and intracranial pressure or brain tissue oxygen monitors have not been shown
to improve outcomes after malignant MCA stroke. Intra-arterial tPA has been suggested for the management of MCA occlusions within 68 hrs of
symptom onset. The presence of low CBF in the presence of normal or mildly decreased CBV and increased MTT suggests an area at risk or penumbra.
Some interventionalists have used CTP to triage interventions such as mechanical thrombectomy or intra-arterial tPA.

REFERENCES (15)

1. Derdeyn CP, Videen TO, Yundt KD, et al. Variability of cerebral blood volume and oxygen extraction: stages of cerebral haemodynamic impairment
revisited. Brain. 2002;125:595607.
2. Marshall RS. The functional relevance of cerebral hemodynamics: why blood flow matters to the injured and recovering brain. Curr Opin Neurol.
2004;17:7059.
3. Hassan AE, Zacharatos H, Rodriguez GJ, et al. A comparison of Computed Tomography perfusion-guided and time-guided endovascular treatments
for patients with acute ischemic stroke. Stroke. 2010;41:16738.
4. Vahedi K, Hofmeijer J, Juettler E, et al. Early decompressive surgery in malignant infarction of the middle cerebral artery: a pooled analysis of three
randomised controlled trials. Lancet Neurol. 2007;6:215222.

RATIONALE (16) Answer: D

Worst headaches of life (WHOL) or thunderclap headaches should alert practitioners to the possibility of subarachnoid hemorrhage (SAH) from
aneurysmal rupture. Several other entitites such as cervicoarterial dissections or ruptured arteriovenous malformations may present as WHOL. Ruling
out these potentially life-threatening conditions is very important when assessing patients with WHOL or thunderclap headaches. A negative CT finding
does not exclude the diagnosis of SAH, and the sensitivity of CT decreases after the first 24 hours. An inexpensive, high-yield, and relatively safe
lumbar puncture is indicated in these cases. SAH with negative CT findings and positive lumbar puncture findings is otherwise known as a Fisher class I
SAH and carries a modest but significant chance of rebleeding and vasospasm. Though MRI is sensible in subacute SAH, sensitivity and specificity is
suboptimal in acute SAH. Initial misdiagnosis after SAH increases the probability of poor outcome.

REFERENCES (16)

1. Kowalski RG, Claassen J, Kreiter KT, et al. Initial misdiagnosis and outcome after subarachnoid hemorrhage. JAMA. 2004;291:8669.
2. Diringer MN. Management of aneurysmal subarachnoid hemorrhage. Crit Care Med 2009;37:43240.
3. Bederson, Joshua B, et al. Guidelines for the management of aneurysmal subarachnoid hemorrhage. A statement for healthcare professionals from a
special writing group of the Stroke Council, American Heart Association. Stroke.2009;40:994.

RATIONALE (17) Answer: E

The patient has developed cerebral vasospasm of the right anterior cerebral artery and the distal M1 segment of the middle cerebral artery, as seen on
cerebral angiogram. The outcome after subarachnoid hemorrhage (SAH) depends on several factors, including the severity of the initial event, medical
management, surgical variables, and the incidence of complications. Cerebral vasospasm occurs in 70% of patients with SAH, correlating with a
baseline mortality 1.53 times greater. The diagnosis of vasospasm may be suspected based on clinical examination findings and daily variations of
transcranial Doppler ultrasonography results, with cerebral angiography serving as the gold standard for its confirmation. Most often, vasospasm begins
on day 3 after SAH and reaches a maximum on days 68. Symptoms are related to the region of cerebral ischemia. If the vasospasm is severe enough
and remains untreated, cerebral infarction may occur.

One of the most important aspects of vasospasm is its failure to consistently respond to treatment. Although it is well recognized that the volume of
blood on CT is the most reliable prognostic factor for vasospasm, the precise pathophysiology of vasospasm is poorly understood. Angioplasty and
intra-arterial vasodilators, either alone or in combination, are the mainstay of therapy for vasospasm after SAH. Angioplasty commonly results in
excellent, and nearly always permanent, angiographic reversal of vasospasm. By stretching the artery, balloon dilatation leads to an immediate and
profound impairment in smooth muscle function, with in vitro and in vivo studies demonstrating both a functional and morphologic change in these
fibers. Observations indicate that earlier treatment with balloon angioplasty for patients with medically intractable vasospasm may result in better
clinical outcomes. Rosenwasser et al3 sought to define a time frame in which balloon angioplasty would be maximally effective and determined that
early treatment within 2 hours was associated with better angiographic outcomes and clinical improvement. Maintenance of normovolemia is key in the
management of SAH patients; though the prophylactic use of the so called "triple-H therapy" (hypervolemia, hypertension, hemodilution) is not indicated
as initial management, it should be reserved for those patients with confirmed vasospasm. The rationale of triple-H is that the maintenance of high
circulating blood volume will enhance cerebral blood flow in vasospasm.

REFERENCES (17)

1. Broderick JP, Brott TG, Duldner JE, et al. Initial and recurrent bleeding are the major causes of death following subarachnoid hemorrhage. Stroke.
1994;25:13427.
2. Wartenberg KE, Schmidt JM, Claassen J, et al. Impact of medical complications on outcome after subarachnoid hemorrhage. Crit Care Med.
2006;34:61723.
3. Rosenwasser RH, Armonda RA, Thomas JE, et al. Therapeutic modalities for the management of cerebral vasospasm: timing of endovascular
options. Neurosurgery. 1999;44:9759; discussion 97980.
4. Lennihan L, Mayer SA, Fink ME, et al. Effect of hypervolemic therapy on cerebral blood flow after subarachnoid hemorrhage: a randomized
controlled trial. Stroke. 2000;31:38391.
5. Komotar RJ, Schmidt JM, Starke RM, et al. Resuscitation and critical care of poor-grade subarachnoid hemorrhage. Neurosurgery. 2009;64:397
410; discussion 1011.
6. Diringer MN. Management of aneurysmal subarachnoid hemorrhage. Crit Care Med 2009;37:43240.
7. Bederson, Joshua B, et al. Guidelines for the management of aneurysmal subarachnoid hemorrhage. A statement for healthcare professionals from a
special writing group of the Stroke Council, American Heart Association. Stroke.2009;40:994.

RATIONALE (18) Answer: B

Although spontaneous intracerebral (intraparenchymal) hemorrhage (ICH) is more than twice as common as spontaneous subarachnoid hemorrhage,
there have been comparatively few randomized trials of ICH treatment. Furthermore, as treatment strategies worldwide have been widely divergent,
there has been no consensus on medical or surgical management of ICH. In 1999, a writing group from the Stroke Council of the American Heart
Association (AHA) reviewed the available data and published guidelines for ICH management.1

The most common cause of ICH remains hypertension. Hypertensive hemorrhages secondary to small-vessel disease most often occur in the putamen,
global pallidum, thalamus, internal capsule, deep periventricular white matter, pons, and cerebellum. Other causes of ICH include illicit drug abuse
(usually cocaine), vascular malformations or aneurysms, hemorrhage into cerebral infarcts or brain tumors, or complications of thrombolytic or
anticoagulant therapy.
Recommendations from the AHA scientific statement included the following:

1. The initial study of choice for suspected ICH is noncontrast CT of the head.
2. Angiography should be considered for surgical candidates without a clear cause of hemorrhage (ie, young, normotensive patients). However,
angiography is not necessary for older hypertensive patients who have a deep hemorrhage in the basal ganglia, thalamus, cerebellum, or brainstem if
CT does not suggest a structural lesion. MRI and MR angiography may be useful in diagnosing cavernous malformations and may obviate the need
for angiography in selected patients.
3. Compared with management of ischemic stroke, hypertension in ICH should be treated more aggressively (without inducing hypotension), in order to
reduce ongoing bleeding from small vessels.
4. Clinical deterioration is the most important indication for surgical intervention. Patients with a moderate or large lobar hemorrhage who are
clinically deteriorating should be considered for craniotomy. Surgical management should also be considered for patients with relatively small
cerebellar hemorrhages (~3 cm) who are clinically deteriorating or have brainstem compression and/or obstructive hydrocephalus. Nonsurgical
management is appropriate for patients with small hemorrhages, mild neurologic deficits, very poor prognosis, or Glasgow Coma Scale score 4.

A cerebellar hemorrhage is more likely to require surgical intervention (ie, craniotomy), because there is less room in the posterior fossa for expansion
of blood and brain edema. Brainstem compression could occur with relatively small hemorrhages.

REFERENCES (18)

1. Broderick JP, Adams HP, Barsan WB, et al. Guidelines for the management of spontaneous intracerebral hemorrhage: a statement for healthcare
professionals from a special writing group of the stroke council. Stroke. 1999;30:905915.
2. Rincon F, Mayer SA. Clinical review: critical care management of spontaneous intracerebral hemorrhage. Crit Care. 2008;12:237.

RATIONALE (19) Answer: C

This patient most likely has Guillain-Barr syndrome (GBS) as manifested by an ascending motor paralysis with loss of deep tendon reflexes. GBS is a
heterogenous condition with multiple variants including immune-mediated, demyelinating, and axonal polyneuropathies. About two-thirds of patients
have a history of respiratory tract or gastrointestinal infection, particularly Campylobacter jejuni, but organisms including cytomegalovirus, Epstein-
Barr virus, Haemophilus, Mycoplasma, enterovirus, hepatitis A or B, herpesvirus, or Chlamydia have also been implicated. The most common variant is
the acute inflammatory demyelinating polyneuropathy in 80%95% of the cases, followed by acute motor axonal neuropathy, particularly in Asia; acute
motor-sensory axonal neuropathy; and the Miller Fisher syndrome, characterized by areflexia, muscle weakness, and ophthalmoparesis or ataxia, among
others.

Diagnosis is based on the clinical presentation and supported by cerebrospinal fluid examination, which shows a typical cyto-protein dissociation.
Additionally, electromyography and nerve conduction studies provide some support but are usually normal in early stages. However, absent F waves in
the lower extremities are a supportive finding.

The treatment of choice is either plasmapheresis or IV immunoglobulin. The combination of the 2 treatments has not been found to improve outcome
more than a single intervention and exposes the patient to increased risks. Greater adverse effects are associated with plasmapheresis than with IV
immunoglobulin. Corticosteroids do not affect the neurologic outcome in GBS.

REFERENCES (19)

1. Hughes RAC, Cornblath DR. Guillain-Barr syndrome. Lancet. 2005;366:1653.


2. Hughes RAC, Rafael JC, Swan AV, et al. Intravenous immunoglobulin for Guillain-Barr syndrome (review). Cochrane Database Syst Rev. 2006,
Issue 1, Art. No. CD002063.
3. Rafael JC, Chevret S, Hughes RAC, et al. Plasma exchange for Guillain-Barr syndrome (review). Cochrane Database Syst Rev. 2002, Issue 2, Art.
No. CD0017989.
4. van der Mech FG, Schmitz PI. A randomized trial comparing intravenous immune globulin and plasma exchange in Guillain-Barr syndrome. N
Engl J Med. 1992;326:11231129.

RATIONALE (20) Answer: D

This patient has a neurological emergency. The "reddest" sign in emergency neurology is a third nerve palsy with external opthalmoplegia (non
pupillary sparing third nerve palsy), which implies the mechanical disruption of parasympathetic fibers from the Edinger-Westphal nucleus by an
aneurysm of the posterior communicating artery. The best option for this patient would be MR angiography of the brain. This would most likely identify
the aneurysm. Electroencephalography is not indicated as this presentation does not suggest an electrical problem. This patient has diabetes and is
recovering from diabetic ketoacidosis. However, blood glucose abnormalities (high or low) will not explain the clinical findings in this patient.
Pyridostigmine testing is unlikely to provide diagnostic yield as symptoms suggest a vascular syndrome rather than a subacute or chronic syndrome
usually associated with myasthenia gravis. Finally, noncontrast CT would only be helpful if there has been a subarachnoid hemorrhage. The headache in
this patient may indicate a sentinel hemorrhage from the aneurysm. However, even if this were the case, the CT may not be sensitive enough to identify
the bleed, and in the case of an unruptured aneurysm it would not identify the vascular malformation causing these symptoms.

REFERENCES (20)

1. Kowalski RG, Claassen J, Kreiter KT, et al. Initial misdiagnosis and outcome after subarachnoid hemorrhage. JAMA. 2004;291:8669.
2. Diringer MN. Management of aneurysmal subarachnoid hemorrhage. Crit Care Med 2009;37:43240.

RATIONALE (21) Answer: A

Both prehospital and in-hospital hypotension with systolic BP less than 90 mm Hg in adults have been associated with a doubling of mortality in severe
brain injury. The goal of fluid resuscitation in the prehospital and hospital settings is to optimize cardiac output, cerebral blood flow, brain tissue
perfusion (cerebral perfusion pressure is equal to mean arterial pressure minus intracranial pressure), and prevent secondary brain injury. No
randomized, prospective, controlled trial could be designed to test the hypothesis that optimizing blood pressure after traumatic brain injury (TBI)
improves outcomes on the basis of ethical principles. The landmark study by Cooper DJ et al2 demonstrated that a regimen of hypertonic saline was not
superior to conventional crystalloid therapy for resuscitation after TBI in hypotensive patients with Glasgow Coma Scale scores 3-8. Furthermore,
decompressive hemicraniectomy, hyperosmolar therapy, induced hypothermia, and protocols of intracranial pressure monitoring have not been
associated with improvements in mortality after TBI.

REFERENCES (21)

1. Cooper DJ, Rosenfeld JV, Murray L, et al. Decompressive craniectomy in diffuse traumatic brain injury. N Engl J Med. 2011;364:14931502.
2. Cooper DJ, Myles PS, McDermott FT, et al; Hypertonic Saline Study Investigators. Prehospital hypertonic saline resuscitation of patients with
hypotension and severe traumatic brain injury: a randomized controlled trial. JAMA. 2004; 291:13501357.
3. Clift on GL, Valadka A, Zygun D, et al. Very early hypothermia induction in patients with severe brain injury (the National Acute Brain Injury Study:
Hypothermia II): a randomised trial. Lancet Neurol. 2011; 10:1319.
4. Heller JE, Maas AI. Severe brain injury. In: Jallo J and Loft us CM, eds. Neurotrauma and Critical Care of the Brain. New York: Thieme Medical
Publishers; 2009.
5. The Brain Trauma Foundation. The American Association of Neurological Surgeons. The Joint Section on Neurotrauma and Critical Care.
Hypotension. J Neurotrauma. 2000;17:591595.

RATIONALE (22) Answer: E

After severe traumatic brain injury, corticosteroids are contraindicated. The recommendation is based on the results of a large multicenter study
(CRASH) that showed this intervention to be harmful with higher mortality, hyperglycemia, and infection rate in the treatment group. The patient will
benefit from sedation, analgesia and head elevation at 30 degrees for ICP control; in this patient, indications for ICP monitoring include the presence of
an abnormal CT scan, mass effect, and effacement of the cisterns. Recognition of and correction of coagulopathy with FFP, platelets, Vitamin K, and/or
desmopressin acetate is critical to decrease risk of contusion expansion.

REFERENCES (22)

1. The Brain Trauma Foundation. The American Association of Neurological Surgeons. The Joint Section on Neurotrauma and Critical Care.
Hypotension. J Neurotrauma. 2000;17:591595.
2. Heller JE, Maas AI. Severe brain injury. In: Jallo J, Loft us CM, eds. Neurotrauma and Critical Care of the Brain. New York: Thieme Medical
Publishers; 2009.
3. Edwards P, Arango M, Balica L, et al. Final results of MRC CRASH, a randomised placebo-controlled trial of intravenous corticosteroid in adults
with head injuryoutcomes at 6 months. Lancet. 2005;365:19571959.

RATIONALE (23) Answer: B

The intracranial pressure (ICP) waveform is a modified arterial pressure tracing with 3 main components (P1, P2, and P3). In a normal ICP waveform,
P1 should have the highest upstroke, P2 should be in between, and P3 should have the lowest upstroke (as in waveform A above). The first peak, or P1,
is known as the vascular ,or "percussive," wave and results from arterial pressure transmitted from the choroid plexus. The second peak, or P2, is
known as the "tidal" wave and its amplitude varies with brain compliance. A lower brain compliance (cerebral edema, hydrocephalus, etc) increases
P2. The third peak, or P3, represents the dicrotic notch and is caused by the pressure transmitted during diastole after aortic valve closure.
REFERENCES (23)

1. Frontera J, Janjua S, Mayer SA. Intracranial pressure. In: Frontera J, ed. Decision Making in Neurocritical Care. New York: Thieme; 2008.

RATIONALE (24) Answer: E

No intervention has been proven to prevent long-term epilepsy after severe traumatic brain injury (TBI). The best preventive strategy has been the
reduction of the incidence of TBI in all of its forms (primary prevention). Early seizures may be prevented with early use of antiepileptics with little risk
of medication side effects for up to 1 week after severe TBI. Prevention of early seizure activity after TBI may be beneficial because it may prevent
spikes in intracranial pressure.

REFERENCES (24)

1. Chang BS, Lowestein DH. Practice parameter: antiepileptic drug prophylaxis in severe traumatic brain injury: report of the Quality Standards
Subcommittee of The American Academy of Neurology. Neurology. 2003;60:1016.
2. The Brain Trauma Foundation. The American Association of Neurological Surgeons. The Joint Section on Neurotrauma and Critical Care. Role of
antiseizure prophylaxis following head injury. J Neurotrauma. 2000;17:549553.

RATIONALE (25) Answer: C

This patient is most likely in pain. Adequate pain control and sedation are fundamental steps in the management of traumatized patients. Pain, fever, and
increased central venous pressure have been associated with subtle but significant changes of the intracranial pressure. Hyperosmolar therapy with 20%
mannitol solution and 23.4 % hypertonic saline solution should be used in cases of imminent transtentorial herniation, but not routinely. Thus, options A
and B are incorrect. Hyperventilation to PaCO2 between 28 and 35 mm Hg may be used in selected cases when transtentorial herniation is evident and the
patient is receiving mechanical ventilation. Mild induced hypothermia (option E) should be considered as a rescue therapy for intracranial pressure
when other alternatives have failed.

REFERENCES (25)

1. The Brain Trauma Foundation. The American Association of Neurological Surgeons. The Joint Section on Neurotrauma and Critical Care.
Hypotension. J Neurotrauma. 2000;17:591595.
2. Koenig MA, Bryan M, Lewin JL 3rd, et al. Reversal of transtentorial herniation with hypertonic saline. Neurology. 2008 Mar 25;70:10239.
3. Mayer SA, Chong J. Critical Care management of increased intracranial pressure. J Int Crit Care Med. 2002;17:5567.

RATIONALE (26) Answer: E

Hyperventilation has an immediate and profound lowering effect on intracranial pressure (ICP). This effect, however, is not sustained, lasting between
30 minutes and 6 hours. Further, very aggressive hyperventilation to PaCO2 less than 25 mm Hg may induce cerebral ischemia. In the management of
increased ICP, hyperventilation should be used in the setting of impending herniation for short durations only; it should serve only as a bridge to more
definitive management.

REFERENCES (26)

1. Mayer SA, Chong J. Critical care management of increased intracranial pressure. J IntCrit Care Med. 2002;17:5567.

RATIONALE (27) Answer: B

Neurosurgical consultation is an important step in the management of severe traumatic brain injury (TBI). An epidural hematoma requires immediate
diagnosis; high index of suspicion; and emergent referral for craniotomy, evacuation, and hemostasis. The typical clinical scenario is that of a patient
with a history of TBI who has a transient loss of consciousness, recovers to baseline, and then becomes comatose. Burr holes (option A) are indicated
for the management of acute subdural hemorrhage and would not be indicated in this case. An external ventricular drain (option C) may be required after
craniotomy for ICP monitoring but may not be indicated as a first-line therapy because of the high risk of subsequent herniation. Corticosteroids (option
D) are contraindicated in severe TBI, and though sometimes required for management of ICP and cerebral edema, the benefit of hypertonic saline in
transtentorial herniation has been demonstrated with rapid changes in serum sodium concentration of more than 5 mEq/L or a sodium goal of more than
145 mEq/L without any risk of central pontine myelinolysis. A drip of 30 mL/h of 3% saline solution will probably not increase the serum sodium level
acutely. Therefore, option E is incorrect.

REFERENCES (27)

1. The Brain Trauma Foundation. The American Association of Neurological Surgeons. The Joint Section on Neurotrauma and Critical Care.
Hypotension. J Neurotrauma. 2000;17:591595.
2. Koenig M et al. Reversal of transtentorial herniation with hypertonic saline. Neurology. 2008 Mar 25;70:10239.

RATIONALE (28) Answer: D

The patient has suffered a type III fracture of C2 with possible transection of the spinal cord at the C1C2 level. The de-efferentation of sympathetic
pathways will cause a neurogenic shock syndrome evidenced by hypotension, bradycardia, and organ perfusion failure. Spinal shock refers to the
syndrome characterized by absence of spinal cord function in the setting of spinal cord injury (hyporeflexia, sphincter incontinence, and neurogenic
shock). In this case, a vasopressor with strong beta1-adrenergic activity is preferred, such as dopamine. A vasopressor with only alpha-adrenergic
activity, such as phenylephrine (option A) may aggravate the patient's bradycardia by unopposed baroreflex response. Blood transfusion (option B) may
not be indicated in the absence of hypovolemia, and though she may require corticosteroids and surgical decompression, they would not be associated
with rapid improvement of her hemodynamics. Thus, options C and E are incorrect.

REFERENCES (28)

1. American Association of Neurological Surgeons. Guidelines for the management of acute cervical spine and spinal cord injuries.

RATIONALE (29) Answer: A

Posttraumatic seizure is common following traumatic brain injury (TBI). Risk of seizure within the first 7 days after TBI can be effectively reduced with
prophylactic anticonvulsant medications. Incidence of late posttraumatic seizure following TBI is not reduced with the use of prophylactic
anticonvulsants. Further, long-term anticonvulsant use has been associated with worsened cognitive outcomes in brain-injured patients. Use of
prophylactic anticonvulsants for periods longer than 7 days following TBI is not recommended, regardless of the specific agent used.

REFERENCES (29)

1. Chang BS, Lowestein DH. Practice parameter: antiepileptic drug prophylaxis in severe traumatic brain injury: report of the Quality Standards
Subcommittee of The American Academy of Neurology. Neurology. 2003;60:1016.
2. The Brain Trauma Foundation. The American Association of Neurological Surgeons. The Joint Section on Neurotrauma and Critical Care.
Hypotension. J Neurotrauma. 2000;17:591595.
3. Naidech AM, Kreiter KT, Janjua N, et al. Phenytoin exposure is associated with functional and cognitive disability after subarachnoid hemorrhage.
Stroke. 2005;36:583587.

RATIONALE (30) Answer: B

The cerebral scintigraphy with technetium Tc-99m-hexametazime (HMPAO) shows absent blood flow into the cranial vault. The test has confirmed the
clinical suspicion of brain death. No further actions are required and the patient can be pronounced dead. Confirmatory testing is usually necessary when
equivocal findings in the neurological exam confound the interpretation of absence of neurological function.

REFERENCES (30)

1. Wijdicks EF, Varelas PN, Gronseth GS, et al. Evidence-based guideline update: determining brain death in adults: report of the Quality Standards
Subcommittee of the American Academy of Neurology. Neurology. 2010 Jun 8;74:19118.

RATIONALE (31) Answer: A

For some patients with diagnoses suggesting brain death, complex, nonbrain-mediated, spontaneous movements can falsely suggest retained brain
function. Additionally, ventilator autocycling may falsely suggest patient-initiated breathing. The patient cannot be declared brain dead until apnea is
confirmed. In this case, the team should proceed with an apnea test that will suffice as confirmation of absence of brain function.

REFERENCES (31)

1. Beckmann YY. Fasciculations in brain death. Critical Care Medicine. 2010 Dec;38:23772378.
2. Wijdicks EF, Varelas PN, Gronseth GS, et al. Evidence-based guideline update: determining brain death in adults: report of the Quality Standards
Subcommittee of the American Academy of Neurology. Neurology. 2010 Jun 8;74:19118.
3. [No authors listed.] Guidelines for the determination of death: report of the medical consultants on the diagnosis of death to the President's
commission for the study of ethical problems in medicine and biochemical and behavioral research. JAMA. 1981;246:2184 2186.

RATIONALE (32) Answer: B

The American Academy of Neurology (AAN) published a 1995 practice parameter to delineate the medical standards for the determination of brain
death. The guideline was reviewed in 2010. The parameter emphasized the 3 clinical findings necessary to confirm irreversible cessation of all
functions of the entire brain, including the brainstem: coma (with a known cause), absence of brainstem reflexes, and apnea. A classic apnea test is
necessary for the confirmation of brain death and cannot be easily performed in patients receiving extracorporeal membrane oxygenation (ECMO). A
modified apnea test can be performed in ECMO patients by mixing the oxygen mixture with carbon dioxide to achieve a PaCO2>20 mm Hg greater than
the baseline. If no respirations are seen, the patient can then be pronounced brain dead. An MRI (option D) will add little to the diagnosis of brain death,
but may help to understand the etiology of brain injury.

REFERENCES (32)

1. Muralidharan R, Mateen FJ, Shinohara RT, et al. The challenges with brain death determination in adult patients on extracorporeal membrane
oxygenation. Neurocrit Care. 2011 Jun;14:4236.
2. Wijdicks EF, Varelas PN, Gronseth GS, et al. Evidence-based guideline update: determining brain death in adults: report of the Quality Standards
Subcommittee of the American Academy of Neurology. Neurology. 2010 Jun 8;74:19118.

RATIONALE (33) Answer: C

The presence of hypothermia precludes the diagnosis of brain death; a core temperature greater than 36C (96.7F) should be achieved before diagnosis
of brain death. Hypercapnia does not preclude the diagnosis of brain death if it is not accompanied by a severe metabolic disarray. However, for the
apnea test, normocapnia (PaCO2 of 3545 mm Hg) should be established first.

REFERENCES (33)

1. Wijdicks EF, Varelas PN, Gronseth GS, et al. Evidence-based guideline update: determining brain death in adults: report of the Quality Standards
Subcommittee of the American Academy of Neurology. Neurology. 2010 Jun 8;74:19118.

RATIONALE (34) Answer: A

Coma and other states of impaired consciousness are signs of extensive dysfunction or injury involving the brainstem, diencephalon, or cerebral cortex
and are associated with a substantial risk of death and disability. Neurologic prognosis is determined by the underlying etiology and may be predicted
by the combination of clinical signs and electrophysiological tests. The persistence of sleep-awake cycles sets patients in a persistent vegetative state
(PVS) apart from patients in a coma, but patients in PVS do not have sensation or a state of awareness. The minimally conscious state is characterized
by a partial state of awareness and intermittently purposeful motor activity. Absent somatosensory evoked potential (SSEP) responses are not usually
seen in patients with PVS and absence of N20 response favors a severe disruption of ascending spinocortical circuits such as in cardiac arrest.

REFERENCES (34)

1. Stevens RD, Bhardwaj A. Approach to the comatose patient. Crit Care Med. 2006;34:3141.
2. Bernat JL. Chronic disorders of consciousness. Lancet. 2006;367:11811192.
PART 2: Cardiovascular Critical Care

Instructions: For each question, select the most correct answer.

1. A 28-year-old, lethargic woman is admitted to the ICU with the following data: temperature, 38.8C (101.8F); HR, 128/min; RR, 26/min; BP,
68/36 mm Hg; mean arterial pressure, 46 mm Hg; SpO2, 96% on oxygen, 2 L/min; cardiac output, 9 L/min; central venous pressure, 3 mm Hg;
systemic vascular resistance, 382 dynes/cm5; pulmonary artery pressure, 15/8 mm Hg; and mean pulmonary artery pressure, 10 mm Hg.

The hospital is currently experiencing a shortage of several vasoactive medications, limiting options for therapy.

The administration of which of the following agents is the most appropriate initial step in the management of this patient?
A. Phenylephrine
B. Milrinone
C. Dobutamine
D. Vasopressin

2. A 19-year-old male was intubated in the field following a motorcycle crash and transported to a nearby trauma center. He has been lethargic since
the accident but has maintained normal vital signs with minimal intervention throughout the 6-hour stay in the emergency department and ICU.
Painful stimulus elicits minimal movement of bilateral upper extremities and no movement in the lower extremities.

On arrival in the ICU, vital signs are as follows: temperature, 35.5C (95.8F); HR, 67/min; BP, 108/62 mm Hg; SpO2, 97% on volume-control
ventilation; tidal volume, 480 mL; RR, 14/min; positive end-expiratory pressure, 5 cm H2O; and FIO2, 0.5.

About 1 hour later, the registered nurse calls for help as the patient now has a temperature of 34.9 (94.8F); HR of 22/min; BP of 58/20 mm Hg,
and SpO2 of 97%.

Which of the following is the most likely cause of this patients clinical deterioration?
A. A large tear in the inferior vena cava
B. Blunt injury to the heart
C. Complete transection of spinal cord at the C6 level
D. Sepsis from aspiration pneumonia

3. An otherwise healthy 17-year-old boy who tripped and fell into a bonfire arrives at your trauma center after transfer from a small community
hospital. On arrival in the ICU, physical examination reveals temperature of 35C (94.9F); HR of 118/min; BP of 94/50 mm Hg on phenylephrine,
250 g/min; and SpO2 of 90% on 60% oxygen via face mask. Neurologic examination reveals a somnolent patient able to follow simple commands
and to withdraw from painful stimulus in all 4 extremities. He has regular tachycardia, with no murmurs. Lung examination shows coarse rhonchi in
all fields.

He has a partial- and full-thickness burn involving the entire abdomen and chest (anterior and posterior), both upper extremities, and the entire right
lower extremity. Spared skin includes the entire left lower extremity and the head and neck.

The physician who initiated the transfer gave the following information: The patients burn occurred 6 hours ago. He has two 18-gauge IVs and has
received 800 mL of 0.9% normal saline. More fluid was initially ordered, but the oxygen saturation was progressively worsening, so the team
decided to use phenylephrine for blood pressure support in lieu of crystalloid administration.

Which of the following is the best immediate intervention?


A. Placement of a central venous line for monitoring of central venous pressure and administration of fluid and drugs.
B. Placement of an arterial line for beat-to-beat measurement of blood pressure and serial blood gas sampling.
C. Endotracheal intubation, mechanical ventilation, and placement of a pulmonary artery catheter to guide careful administration of IV fluids.
D. Endotracheal intubation, mechanical ventilation and rapid IV administration of multiple liters of warmed isotonic crystalloid solution

4. An intubated and heavily sedated patient with heart failure arrives from the catheterization lab to the ICU after left-sided coronary angiography
(without intervention) and placement of a central venous catheter in the right internal jugular vein. Vital signs are as follows:

Temperature, 36.7C (98F); HR, 74/min in sinus rhythm; BP, 110/62 mm Hg on dobutamine infusion, 7 g/kg/min; SpO2, 97% on volume control
ventilation; tidal volume, 480 mL; frequency, 14/min; positive end-expiratory pressure, 5 cm H2O; FIO2, 50%. The vital signs 8 minutes after arrival
in the ICU are as follows:

Temperature, 36.7C (98F); HR, 124/min in sinus rhythm; BP, 72/30 mm Hg on dobutamine infusion, 7 g/kg/min; and SpO2, 82% on unchanged
ventilator settings. The ventilator alarm indicates high airway pressure.

Which of the following is the most important initial step in the management of this patient?
A. Increase the dobutamine infusion.
B. Add epinephrine infusion.
C. Perform emergent needle decompression of the chest.
D. Obtain immediate chest radiography.

5. An otherwise healthy 27-year-old woman is walking out of the hospital cafeteria when she falls to the ground. She is somnolent and unresponsive to
command, with noisy breathing and a barely palpable radial pulse. She is rapidly transported to the emergency department. Upon arrival, her
temperature is 37.2C (98.9F), HR is 132/min, BP is 58/34 mm Hg, RR is 28/min, and SpO2 is 92% on 80% FIO2 via nonrebreather mask. Physical
examination reveals an unconscious woman with flushed skin in respiratory distress. Lung examination reveals bilateral wheezing.

Which of the following is the most appropriate initial intervention?


A. Administration of nebulized albuterol
B. IV administration of normal saline, 2 L
C. IV administration of hydrocortisone
D. IV (or intramuscular) administration of epinephrine

6. Which of the following statements best characterizes the appropriate method of measuring central venous pressure (CVP)?
A. CVP should be measured from the base of the v wave.
B. CVP should be measured at end-expiration if mechanically ventilated and end-inspiration if spontaneously breathing.
C. CVP can be estimated by adding 5 cm H2O to the measured distension of the jugular veins relative to the sternal angle.
D. If the midpoint of the right atrium is used as reference level for CVP monitoring, measurements are only reliable when the patient is supine.

7. Which of the following factors alters the interpretation of the pulmonary artery occlusion pressure as an indicator of left ventricular end-diastolic
pressure (preload)?
A. Increased pulmonary vascular resistance
B. Change in pulmonary artery catheter position from West zone 1 to West zone 2
C. Mitral insufficiency
D. Changes in right ventricular compliance
8. Which of the following statements best represents current recommendations and the management of the rhythm shown in the Figure?

A. 2010 American Heart Association Advanced Cardiac Life Support Guidelines recommend adenosine for initial management of this arrhythmia.
B. Three stacked shocks should be administered if a monophasic defibrillator is used to treat this rhythm.
C. Fibrinolytic therapy during CPR for patients presenting with this rhythm and suspected/ confirmed coronary occlusion has not been shown to
improve outcome.
D. Amiodarone has been shown to be superior to lidocaine for increasing the rate of survival to hospital discharge in patients presenting this
rhythm.

9. A 60-year-old man is admitted to the ICU after cardiac surgery. Findings of physical examination and chest radiography are consistent with
pulmonary edema. A vasoactive drug is administered.

Parameters On admission After therapy


BP (mm Hg) 98/66 88/50
Mean right atrial pressure (mm Hg) 23 15
Pulmonary artery pressure (mm Hg) 48/26 40/20
Pulmonary artery occlusion pressure (mm Hg) 26 18
Cardiac index (L/min/m2) 1.6 2.8
Heart rate (per min) 110 119

Which of the following drugs was administered?


A. Nitroglycerin
B. Milrinone
C. Dopamine
D. Norepinephrine

10. A 54-year-old man is admitted to the emergency department with severe substernal chest pain and acute shortness of breath. The patient has a
history of type 2 diabetes mellitus and hypertension controlled with medications. Examination reveals diaphoresis and respiratory distress. HR is
120/min, BP is 95/50 mm Hg, and RR is 24/ min. Lung examination reveals bilateral crackles; heart examination is significant for regular
tachycardia, with normal S1 and S2. An S3 is present without murmurs or rubs. Chest radiography shows changes consistent with pulmonary edema.
Twelve-lead ECG is significant for T-wave inversions in V1V5. Patient is taken for emergent cardiac catheterization. The following hemodynamic
values are obtained:

BP 85/46 mm Hg
Right atrial pressure 12 mm Hg
Right ventricular pressure 34/10 mm Hg
Pulmonary artery pressure 34/22 mm Hg
Pulmonary artery pressure (mean) 25 mm Hg
Pulmonary artery occlusion pressure (PAOP) 20 mm Hg
Cardiac output 4 L/min
Cardiac index (CI) 1.5 L/min

Oxygen consumption is calculated at 180 mL/min/m2. Significant triple vessel disease with a lesion in the proximal left anterior descending
coronary artery is found. An intraaortic balloon pump is placed in preparation for an emergent coronary artery bypass graft.

Which of the following hemodynamic parameters would be most consistent with an effective intraaortic balloon counterpulsation?
BP (mm Hg) PAOP (mm Hg) CI (mm Hg) Oxygen Consumption (mL/min/m2)

A. 80/40 20 2.8 190


B. 85/46 16 2.8 200
C. 95/60 16 2.8 140
D. 90/60 22 2.2 180

11. A 50-year-old man with a past medical history significant for hypertension presents to the emergency department with severe substernal chest pain
and shortness of breath that started 30 minutes ago while eating at a nearby restaurant. On physical examination, the patient is diaphoretic. BP is
90/50 mm Hg, HR is 75/min, and RR is 28/min. Physical examination reveals irregular tachycardia with no murmurs and a positive S3. Lung
auscultation reveals crackles in both bases. The rest of his examination findings are unremarkable. An ECG is shown in the Figure.

Which of the following management options is most appropriate at this point?


A. Aspirin, IV nitroglycerin, low-molecular-weight heparin, and beta-blockers
B. Aspirin, low-molecular-weight heparin, beta-blockers, and glycoprotein IIb/IIIa inhibition
C. Aspirin and urgent catheterization
D. Aspirin, beta-blockers, and rtPA

12. A 55-year-old man presents to the emergency department with hypoxemia, hypotension, and no past medical history of cardiopulmonary disease. He
has type 2 diabetes mellitus and chronic renal insufficiency with a creatinine level of 2.2 mg/dL. Chest radiograph shows no infiltrates, and his lung
examination findings are normal. Blood pressure initially normalizes with a fluid bolus, but then decreases, requiring dopamine, 10 g/kg/min, to
maintain a systolic BP of 90100 mm Hg. Oxygenation is good with oxygen, 5 L via nasal cannula. An echocardiogram reveals a dilated right
atrium and right ventricle and no other abnormalities. A catheter inserted in the right side of the neck reveals a central venous pressure of 12 mm Hg
and an ScVO2 of 60%. CT and remote interpretation are immediately available. Leg ultrasonography, perfusion lung scanning, and angiography are
only available by call-in.

Which of the following is the most appropriate management of this patient's illness?
A. Dobutamine and thrombolytic therapy
B. Fluid bolus and thrombolytic therapy
C. Dobutamine and helical chest CT
D. Fluid bolus and helical chest CT

13. A 56-year-old woman being treated for unresectable lung cancer is admitted to the ICU with hypotension that is poorly responsive to fluid
resuscitation, leading to initiation of vasopressors (dopamine, 10 g/kg/min) to maintain adequate blood pressure. A pulmonary artery catheter
reveals equalization of pressures (right atrial, right ventricular diastolic, pulmonary artery occlusive pressure). Emergent echocardiogram reveals
right atrial and right ventricular collapse with systole. Cardiac index is 1.0 L/min/m2. A 2.5-cm circumferential pericardial effusion is present.
Emergent pericardial centesis removes approximately 60 mL of fluid. Blood pressure rises significantly, and the patient is weaned from dopamine.
Cardiac index increases. The patient is transferred to the cardiac catheterization laboratory. Mean right atrial pressure is 31 mm Hg and
intrapericardial pressure is 31 mm Hg. Two hundred milliliters of sanguinous fluid is removed from the pericardial space, and a pigtail catheter is
left for drainage. Mean right atrial pressure is reduced to 23 mm Hg, while the intrapericardial pressure decreases to 8 mm Hg. ECG confirms that
most of the fluid has been removed. Patient is now slightly hypertensive and not receiving vasoactive drugs.

In addition to hemodynamically significant pericardial effusion, which of the following conditions is likely to be present?
A. Primary pulmonary artery hypertension
B. Secondary pulmonary artery hypertension
C. Right ventricular infarction
D. Effusive-constrictive pericarditis

14. A patient with a known hypertrophic cardiomyopathy and dynamic left ventricular outflow tract obstruction is intubated for community-acquired
pneumonia. Urine output is minimal, with no response to a fluid bolus. Patient has BP of 118/74 mm Hg and HR of 120/min.

Which of the following therapies is most appropriate as part of the treatment?


A. Furosemide
B. Inotropic-dose dopamine
C. Beta blockade
D. Nitroglycerin

15. A 70-year-old man is admitted to the ICU after insertion of an intraaortic balloon pump (IABP) for severe, intractable angina while awaiting
coronary artery bypass surgery. His IABP is set at 1:1 and he is treated with heparin and nitroglycerin infusions. During the course of the evening,
although his chest pain has lessened, he now has abdominal pain of increasing severity.

In addition to checking the ECG for new changes, which of the following is the most appropriate next step in his care?
A. Increase the nitroglycerin infusion dose.
B. Obtain a chest radiograph.
C. Reduce the IABP support from 1:1 to 1:2.
D. Check his stool for occult blood.

16. A 65-year-old woman with ischemic cardiomyopathy is awaiting implantable cardioverter-defibrillator placement for nonsustained ventricular
tachycardia. An episode of stable ventricular tachycardia quickly regresses to ventricular fibrillation. The initial end-tidal carbon dioxide partial
pressure (PETCO2) is 10 mm Hg during the initial chest compressions. After epinephrine, 1 mg, and defibrillation, the PETCO2 increases to 35 mm Hg.

Which of the following best represents the significance of the PETCO2 readings for this patient?
A. The increase in PETCO2 indicates a return of spontaneous circulation.
B. Increased PETCO2 indicates chest compressions can be stopped.
C. Initial low PETCO2 values indicate inadequate chest compressions.
D. Low PETCO2 values should be confirmed with blood gas analysis.

17. Which of the following is a relative contraindication to intraaortic balloon pump therapy?
A. Aortic dissection
B. Severe peripheral vascular disease
C. Aortic valve insufficiency
D. Patent ductus arteriosus

18. A 40-year-old woman supported with a left ventricular assist device (LVAD) for nonischemic cardiomyopathy was admitted to the ICU with a trace
guaiac-positive lower gastrointestinal bleed. She was hemodynamically stable until she received sedation for a colonoscopy, when she became
hypotensive. Chest radiograph is shown in the Figure.

Which of the following is the most appropriate immediate intervention?


A. Increase the blood pressure with administration of a pressor.
B. Increase the LVAD pump speed to increase the cardiac output.
C. Decrease the LVAD pump speed to increase the filling time of the left ventricle.
D. Perform transfusion to maintain hemoglobin level of 8 g/dL or until the systolic blood pressure is normal.

19. Following an uneventful mitral valve repair for severe mitral regurgitation secondary to mitral valve prolapse, a 55-year-old woman is admitted to
the ICU. She is AAI paced via transthoracic wires at 80/min. The PR interval is 0.29 seconds and the underlying rhythm is sinus bradycardia at
40/min.

The pacemaker should be changed to:


A. DDD pacing, with PR interval of 0.2 seconds
B. VOO pacing at 90/min
C. AOO pacing at 90/min
D. DDD pacing, with PR interval of 0.29 seconds

20. A 60-year-old woman is admitted to the ICU following a 3-vessel aortocoronary bypass graft. She is paced at 96/min via transthoracic epicardial
wires. Without pacing or inotropic support, she was in normal sinus rhythm at 80/min with a cardiac index of 3.2.

Which of the following is the most appropriate rationale for pacing?


A. Prophylaxis for heart block
B. Further increasing cardiac index for optimum myocardial perfusion
C. Prevention of atrial fibrillation
D. Initial support of cardiac function while sedated and mechanically ventilated

21. During a transfusion of fresh frozen plasma to normalize the international normalized ratio of a 60-year-old man treated with warfarin prior to
repair of a hip fracture, he has flash pulmonary edema and is admitted to the ICU.

Which of the following statements should guide management of his cardiac dysfunction?
A. The diagnosis is left ventricular failure.
B. The diagnosis is biventricular failure.
C. The type of failure cannot be diagnosed based on his symptoms.
D. The treatment goal of systolic failure is reduction of symptoms.
22. Which of the following patients best meets criteria for severe sepsis?
A. A. 59-year-old man hospitalized for 2 days following laparoscopic cholecystectomy for acute cholecystitis who develops fever to 38.5C
(101.2F), WBC count of 14,000/L, and a urine culture positive for Escherichia coli
B. 41-year-old woman with diabetes and perineal pain, fever to 38.9C (102F), and a blood glucose level above 400 mg/dL
C. 70-year-old man admitted with pneumonia with HR of 108 /min, BP of 105/70 mm Hg, RR of 22/min, positive sputum culture for Klebsiella
pneumoniae, and urine output of 10 mL/h after normal saline, 3 L
D. 45-year-old man with gram-positive bacteremia and persistent hypotension despite 5 L of lactated Ringer solution and high-dose norepinephrine
infusion

23. A 73-year-old woman presents to the emergency department with increasing left lower quadrant abdominal pain. She has fever to 38.8C (101.8F),
irregular HR of about120/min, BP of 75/35 mm Hg, and RR of 24/min. On examination, she is obtunded, and has significant pain with guarding on
palpation of her left lower quadrant. Laboratory values include WBC count of 17,000/L; hemoglobin level of 13.1 g/dL; creatinine level of 1.7
mg/dL; glucose level of 255 mg/dL; lactate level of 5.6 mmol/L; and arterial blood gas results showing pH of 7.22, PaCO2 of 30 mm Hg, and PaO2 of
117 mm Hg. CT shows a large, rim-enhancing fluid collection adjacent to her sigmoid colon, and colonic thickening consistent with acute
diverticulitis. IV access is obtained, and lactated Ringer solution, 4 L, is administered. BP rises to 82/48 mm Hg, and the patient is started on
norepinephrine infusion. Broad-spectrum antibiotics are administered. Blood cultures are positive for Enteroccocus. An interventional radiology
placed drain reveals a significant amount of purulent material.

Which of the following data points is significant in making the diagnosis of septic shock?
A. Fever to 38.8C (101.8 F) and irregular HR of about 120/min
B. Positive blood cultures and purulent material from drain
C. Significant metabolic acidosis and signs of systemic hypoperfusion
D. BP of 82/48 mm Hg despite adequate fluid resuscitation

24. A 23-year-old man is brought to the emergency department in a cervical collar after a head-on motor vehicle collision. He was unrestrained and
used cocaine within the previous hour. He has tachycardia and diaphoresis and reports abdominal and back pain. His initial BP is 80/40 mm Hg,
with HR of 100/min. A central line is placed, and a 500-mL bolus of normal saline solution brings his BP to 160/80 mm Hg and his HR to 80/min.
However, within 15 minutes, his BP falls to 60/30 mm Hg with a central venous pressure of 9 mm Hg, his HR accelerates to 120/min, and he
becomes unresponsive.

What is the most likely explanation for his fall in blood pressure?
A. Splenic laceration
B. Myocardial infarction
C. Tension pneumothorax
D. Resolution of cocaine-induced hypertension

25. A 58-year-old migrant farm worker arrives at a rural hospital in California, having suffered a traumatic amputation of his left arm from a farm
machinery accident. Although a tourniquet is in place high in the axilla, it is uncertain how long ago the injury occurred or how much blood has been
lost. Despite some slow oozing from his amputation site, ongoing blood loss now seems to be controlled. HR is 90/min, BP is 110/90 mm Hg,
respirations are 22/min and shallow, and oral temperature is 34C (93.2). Although anxious, the patient seems coherent and answers questions
appropriately. Two peripheral IV catheters (16- and 18-gauge) are present in the uninjured arm. Baseline laboratory results include hemoglobin
level of 8 mg/ dL; platelet count of 158,000/L; arterial blood gas pH of 7.25, PaCO2 of 45 mm Hg, and PaO2 of 75 mm Hg; and lactate level of 5
mg/dL. The operating room will not be available for at least 45 minutes.

What are the most appropriate interventions that can be accomplished in the time prior to the operating room becoming available?
A. Type and cross for 4 units of packed red blood cells (PRBCs); give 4 units of fresh frozen plasma (FFP), 6 units of platelets, and sodium
bicarbonate;warm patient to 35C (94.9F); place central line.
B. Perform rapid-sequence intubation; administer sedation; place nasogastric tube and suction gastric contents; place central line; infuse 1,000 mL
of hetastarch solution.
C. Type and cross for 6 units of PRBCs; give 6 units of FFP and 6 units of platelets; place urinary catheter; warm patient to 36C (96.7F); infuse
normal saline solution, 2 L.
D. Perform rapid sequence intubation; administer sedation, sodium bicarbonate, and 2 units of type O noncross-matched PRBCs.

Questions 26 and 27 relate to the following vignette.

A 35-year-old, 70-kg (154-lb) police officer is brought to the emergency department after falling from a motorcycle during a high-speed chase.
His uniform was removed in the field, and it is evident that he has suffered significant abrasions. He is in a cervical collar and his helmet has
been removed. Large contusions are present over the left hemithorax and right abdomen, and his left femur has been placed in a traction splint.
Two 18-gauge peripheral IV lines have been inserted in bilateral antecubital fossae, and he has received normal saline solution, 1.5 L, en route
to the hospital. He is somnolent, and when roused, he is confused. HR is 120/min, BP is 97/74 mm Hg, RR is 32/min, and SpO2 is 93% on
nonrebreather mask.Paramedics estimate that he has lost at least 1 L of blood. His left hemithorax is dull to percussion.

26. By Advanced Trauma Life Support (ATLS) standards, what class of hemorrhage has this patient sustained?
A. Class I
B. Class II
C. Class III
D. Class IV

27. Which of the following interventions is the most appropriate next step in this patients care?
A. Left chest tube placement
B. Right subclavian central line placement
C. Radiographs of the chest (anterior/posterior) and left femur
D. Exploratory laparotomy

Questions 28 and 29 relate to the following vignette.

A 63-year-old man with type 2 diabetes mellitus is brought to the ICU after emergent decompression of a right temporal subdural hematoma
sustained after a fall of approximately 15 feet. He is intubated and ventilated with FIO2 of 60%, synchronized intermittent mandatory ventilation at
12/min, tidal volume of 500 mL, positive end-expiratory pressure (PEEP) of 5 cm H2O, and SpO2 of 100%. He is estimated to have lost
approximately 1 L of blood from a scalp laceration preoperatively, and lost at least another 1 L intraoperatively. He was given 8 units of packed
red blood cells (PRBCs), 6 units of fresh frozen plasma, and 6 units of platelets intraoperatively. He remains hypotensive (80/40 mm Hg) while
being supported with dopamine, 10 g/kg/min. HR is 100/min and central venous pressure is 18 mm Hg. The anesthesiologist found sufficient left-
sided filling volumes using transesophageal echocardiography (TEE) toward the end of the operation.

28. Which of the following TEE views would have best informed the attending anesthesiologist of the patients volume status?
A. Deep transgastric long-axis view
B. Transgastric basal short-axis view
C. Mid-esophageal bicaval view
D. Transgastric midpapillary short-axis view

29. Which of the following would be the most appropriate initial intervention to improve this patients hemodynamic status?
A. Discontinue PEEP.
B. Increase dopamine to 20 g/kg/min.
C. Give PRBCs, 2 units, and normal saline, 1 L.
D. Give calcium chloride, 2 g.
30. The hemodynamic profile shown in the table is from a 62-year-old man admitted to the ICU after coronary artery bypass grafting:

Entering ICU +30 min


Heart rate (/min) 90 120
Blood pressure (mm Hg) 125/75 80/30
Pulmonary artery diastolic pressure (mm Hg) 12 25
Pulmonary artery occlusion pressure (mm Hg) 10 25
Central venous pressure (mm Hg) 6 8

Which of the following is the most likely cause of the changes occurring after 30 minutes?
A. Anaphylactic reaction
B. Left ventricular ischemia
C. Pericardial tamponade
D. Pulmonary embolism

31. During evaluation of oliguria in a 62-year-old male following operative repair of an aortic aneurysm, large V waves are noted in a pulmonary
artery occlusion pressure tracing.

Which of the following conditions is most likely associated with the findings in this patient?
A. Pulmonary regurgitation
B. Mitral regurgitation
C. Aortic regurgitation
D. Coronary artery disease

32. In which of the following conditions would increasing preload, systemic vascular resistance, and heart rate result in improved hemodynamic
stability?
A. Aortic stenosis
B. Asymmetric septal hypertrophy(idiopathic hypertrophic subaortic stenosis)
C. Cardiac tamponade
D. Mitral regurgitation
E. Mitral stenosis

33. A 45-year-old woman with mitral valve stenosis is scheduled for elective mitral valve replacement. Two minutes after tracheal intubation, she
develops atrial fibrillation with a ventricular response rate of 150/min and a decrease in BP to 75/45 mmHg.

Which of the following is the most appropriate first step in management?


A. Increasing the concentration of anesthetic gas
B. Increasing the infusion rate of lactated Ringer solution
C. IV administration of digoxin
D. IV administration of propranolol
E. Electrical cardioversion
34. A 52-year-old woman with severe mitral stenosis is scheduled for mitral valve replacement.

Which of the following preoperative findings is most likely to be present?


A. Atrial fibrillation
B. Left ventricular hypertrophy
C. Elevated left ventricular end-diastolic pressure
D. Symptomatic improvement with systemic vasodilation

35. A 68-year-old man with stable angina treated with propranolol and nifedipine is scheduled for coronary artery bypass grafting. After induction with
fentanyl and tracheal intubation, HR decreases to 40/min and BP decreases to 70/40 mmHg. The ECG is shown in the Figure.

Which of the following is the most appropriate first step in management?


A. Infusion of epinephrine or dopamine
B. Administration of phenylephrine
C. Closed-chest CPR
D. Defibrillation
E. Synchronized cardioversion

36. A 73-year-old man is admitted with chest pain radiating to the interscapular region, HR of 90/min and BP of 190/80 mm Hg. Initial CT shows a
dissecting aneurysm of the descending aorta and medical management is advised.

Which of the following is the first line of treatment?


A. Nitroprusside infusion
B. Nitroglycerin infusion
C. Labetalol infusion
D. Beta-blockers

37. A 76-year-old man with hypertension, type 2 diabetes mellitus, hepatic insufficiency secondary to ethanol abuse, and chronic kidney disease is
admitted to the ICU with severe chest pain radiating to the back. His initial examination is remarkable for BP of 210/115 mm Hg, creatinine level of
2.7 mg/dL, international normalized ratio of 2.8, and total bilirubin level of 4.8 mg/dL. Radiologic studies reveal an acute dissection of the
descending thoracic aorta starting 4 cm below the left subclavian artery and extending into the infrarenal abdominal aorta with the left renal artery
supplied from the false lumen.

Which of the following medication approaches is optimal for the control of his blood pressure?
A. Esmolol followed by nicardipine
B. Esmolol followed by nitroprusside
C. Nitroprusside followed by metoprolol
D. Nitroprusside followed by nicardipine
38. A 70-year-old woman with hypertension and hypercholesterolemia is admitted to the ICU after a 5-hour onset of left hemiparesis and BP of
225/105 mm Hg on presentation. CT shows a loss of gray-white distinction in right frontal and parietal lobes consistent with ischemic stroke.

Which of the following treatments would provide optimal management?


A. Systemic thrombolysis
B. Nitroprusside to reduce BP to 190/90 mm Hg
C. Labetalol to reduce BP to 130/75 mm Hg
D. Labetalol to reduce BP to 190/90 mm Hg

39. A 29-year-old presents with severe preeclapmsia (BP of 190/120 mm Hg) at 30 weeks' gestation. Pertinent laboratory values are: hemoglobin, 10.1
g/dL; platelets, 87,000/L; aspartate aminotransferase, 115 U/L; alanine aminotransferase, 93 U/L; and alkaline phosphatase, 55 U/L. A magnesium
infusion is started with IV hydration and she is admitted to the ICU.

Which of the following is the best approach for further management of this patient?
A. IV hydralazine as needed while systolic BP is greater than 160 mm Hg
B. IV enalapril as needed while systolic BP is greater than 160 mm Hg
C. IV nicardipine infusion to keep systolic BP less than 160 mm Hg
D. Emergent cesarean section

40. A 72-year-old man has hypertension, ischemic cardiomyopathy with a left ventricular ejection fraction of 21%, and an implantable cardioverter-
defibrillator (ICD) for treatment of ventricular dysrhythmias. The ICD was inactivated prior to a mitral valve annuloplasty for severe mitral
regurgitation. Postoperatively, he was supported with norepinephrine and milrinone. Two hours after admission, he developed atrial fibrillation
with rapid ventricular response and an HR of 120130/min, resulting in decreased blood pressure to 70/41 mm Hg and decreased cardiac index
from 2.6 to 1.4 L/min/m2.

Which of the following is the best approach for further management of this patient?
A. Reactivating the ICD
B. Amiodarone infusion after initial bolus
C. External defibrillation
D. External synchronized cardioversion
PART 2: Cardiovascular Critical Care

ANSWERS:

1A; 2C; 3D; 4C; 5D; 6C; 7A; 8C; 9B; 10C; 11C; 12D; 13D; 14C; 15B; 16A; 17B; 18A; 19A; 20C; 21C; 22C; 23D; 24A;
25C; 26C; 27A; 28D; 29D; 30B; 31B; 32C; 33E; 34A; 35A; 36D; 37A; 38D; 39C; 40D

RATIONALE (1) Answer: A

The patient has distributive shock, as evidenced by hypotension, preserved cardiac output, and profoundly low systemic vascular resistance. The
primary derangement in this patient appears to be low vascular tone. As a direct-acting alpha-adrenergic agonist, phenylephrine works to restore
systemic vascular resistance and increase blood pressure. Milrinone is a phosphodiesterase inhibitor that increases cardiac contractility and causes
vasodilation. The patient's data indicate that cardiac contractility is robust and peripheral vasodilation is the primary problem. Administration of
milrinone in this case would probably be detrimental. Dobutamine is a synthetic sympathomimetic with predominantly beta1-adrenergic effects. It
typically causes an increase in cardiac contractility and a small decrease in systemic vascular resistance. The patient's data indicate that these effects
would be unlikely to be of benefit. Vasopressin binds to V1 receptors, causing contraction of vascular smooth muscle cells, leading to increased
systemic vascular resistance. While the patient does require an agent to increase vascular tone, vasopressin would be difficult to rapidly titrate as the
patient's status changes over time. While administration of vasopressin might be reasonable as an adjunct to a titratable infusion, vasopressin is inferior
to phenylephrine as the initial step in the management of this patient.

REFERENCES (1)

1. Beale RJ, Hollenberg SM, Vincent JL, et al. Vasopressor and inotropic support in septic shock: an evidence-based review. Crit Care Med. 2004;
32:S455465.
2. Morelli A, Ertmer C, Lange M, et al. Effects of short-term simultaneous infusion of dobutamine and terlipressin in patients with septic shock: the
DOBUPRESS study. Br J Anaesth. 2008;100:494503.
3. Petersen JW, Felker M. Inotropes in the management of acute heart failure. Crit Care Med. 2008;36:S10611.
4. Mutlu GM, Factor P. Role of vasopressin in the management of septic shock. Intensive Care Med. 2004;30:12761291.

RATIONALE (2) Answer: C

This patient is suffering from the acute onset of profound bradycardia and hypotension. While several anatomic and physiologic insults can produce this
combination, in the blunt trauma patient, neurogenic shock is a common cause of this picture. Keys to the correct answer in this question include the
history of high-velocity blunt trauma, the neurologic examination findings, and the timing of cardiovascular collapse. Neurogenic shock can manifest
itself hours after spinal cord injury. A large tear in the inferior vena cava would result in immediate and profound hypotension at the scene of the
accident, detectable long before hour 6 of an emergency department and ICU stay. Blunt trauma to the heart would probably produce clinical signs
within a short time after injury. Sepsis from aspiration pneumonia, while common in blunt trauma patients, would not typically produce shock within the
first 24 hours after the accident.

Neurogenic shock is a form of distributive shock and is characterized by the triad of hypotension, bradycardia, and hypothermia resulting from functional
sympathectomy and/or from loss of cardiac inotropy/chronotropy secondary to high spinal cord injury. Functional sympathectomy results in loss of
vascular tone and profound hypotension. It is more common in injuries at the midthorax and higher. In high cord injuries, loss of cardiac accelerator
function and unopposed parasympathetic tone contribute to bradycardia, which exacerbates impaired cardiac output and thus exacerbates shock.
Anticholinergics (eg, atropine) or beta-adrenergic agonists are useful in increasing the heart rate. Alpha-adrenergic agonists are useful in restoring
peripheral vascular tone, improving venous return, and thus increasing cardiac output.

REFERENCES (2)

1. Guly HR, Bouamra O, Lecky FE. The incidence of neurogenic shock in patients with isolated spinal cord injury in the emergency department.
Resuscitation. 2008;76:5762.
2. Bilello JF, Davis JW, Cunningham MA, et al. Cervical spinal cord injury and the need for cardiovascular intervention. Archives of Surgery.
2003;138:11279.

RATIONALE (3) Answer: D

Large burn injury causes a combination of hypovolemic and distributive shock, at times combined with direct myocardial depression (in the immediate
postburn phase). The predominant component of postburn shock is hypovolemia, largely the result of microvascular injury leading to pathologic leaking
of fluid from the intravascular compartment. In large burns (>20% total body surface area [TBSA]), this fluid leak results in the need for large-volume
fluid administration to avoid organ failure and death.

The patient in this vignette is critically ill and has multiple systems requiring intervention. The question asks what the "best immediate intervention" is.
Considering the degree and trajectory of hypoxia in this previously healthy 17-year-old, the mechanism and extent of injury, and the resuscitation that is
about to occur, the intensivist is obligated to secure the airway and initiate mechanical ventilator support immediately. Deferring necessary airway
management for the placement of lines which are not life-saving (arterial, central venous, pulmonary arterial) is a common and sometimes fatal error in
the care of severely burned and injured patients.

By the "rule of nines," this patient's burn can be estimated at approximately 72% of TBSA. Regardless of whether the intensivist uses a conservative
formula such as the modified Brooke formula (2 mL fluid per kg per percent TBSA in 24 hours) or the traditional Parkland formula (4 mL fluid per kg
per percent TBSA in 24 hours), this patient is clearly in need of a large volume of IV fluids.

While the placement of a central venous, arterial, or pulmonary arterial catheter might help ongoing decision making with regard to fluid resuscitation,
this patient's history and physical examination findings are sufficient to know that he is in need of immediate fluid bolus. Delaying fluid administration
for the placement of invasive monitors would be inappropriate in this case and might result in ongoing damage to underperfused tissues.

Other important considerations in this patient include his hypothermia, his existing (and soon to worsen) lung injury, possible inhalation injury, and the
likely restrictive effects of a whole-torso burn on respiratory mechanics.

REFERENCES (3)

1. White CE, Renz EM. Advances in surgical care: management of severe burn injury. Critical Care Medicine. 2008;36:S318-24.
2. Ipaktchi K, Arbabi S. Advances in burn critical care. Critical Care Medicine. 2006;34:S239S244
3. Wolf SE, Pruitt BA. Burn management. In: Irwin RS, Rippe JM, eds. Intensive Care Medicine. Sixth Edition. Philadelphia: Lippincott Williams &
Wilkins; 2008.

RATIONALE (4) Answer: C

This patient, who has recently undergone placement of a central venous catheter, is demonstrating several of the classic signs of tension pneumothorax:
rapidly progressive and profound hypotension, tachycardia, hypoxia, and inadequate ventilation. While this constellation of findings is common among
ICU patients, the proximity in time to the central line placement and the low tidal volumes with high airway pressures necessitate strong consideration of
tension pneumothorax on the differential. The rate of clinical deterioration requires immediate action to correct the problem. When tension
pneumothorax is being strongly considered in an ICU patient who is near cardiac arrest, emergent needle decompression of the affected hemi-thorax is
essential. As a heart failure patient, this patient would tolerate the stress of tension pneumothorax even more poorly than most. Increasing the dobutamine
infusion would have little to no effect in this case. The shock state induced by tension pneumothorax is caused by elevated intrathoracic pressure leading
to poor venous return to the heart, thus reducing cardiac output. Inotropics such as dobutamine are not the treatment of choice for any shock state related
to extremely compromised venous return. Epinephrine might improve the blood pressure temporarily, but until the underlying probleminadequate
venous returnis solved, the patient is at risk of progressing toward cardiac arrest. While many clinicians associate chest radiography with
pneumothorax, the decision to wait for chest radiography while a tension pneumothorax expands is a potentially fatal error. This patient's systolic BP
has dropped from 110 to 54 mm Hg and the SpO2 has dropped from 97% to 61% in a matter of minutes. The time that it takes for a radiograph to be
obtained and processed is excessive. Alternatively, intensivists' increased use of point-of-care ultrasonography for detection of pneumothorax has been
advocated as a fast and accurate alternative to radiography.

REFERENCES (4)

1. Vezzani A, Brusasco C, Palermo S, et al. Ultrasound localization of central vein catheter and detection of postprocedural pneumothorax: an
alternative to chest radiography. Critical Care Medicine. 2010;38:533538.
2. American College of Surgeons Committee on Trauma. Advanced Trauma Life Support for Doctors. Eighth Edition. Chicago: American College of
Surgeons; 2008.

RATIONALE (5) Answer: D

This woman's history and physical examination are most consistent with anaphylaxis, which is a form of distributive shock. Anaphylaxis involves the
exposure to a trigger in a susceptible person, which leads to mast cell activation. The subsequent release of chemical mediators such as histamine and
numerous cytokines results in a complex clinical picture that can include systemic vasodilation, hypotension, tachycardia, urticaria, brochospasm,
arrhythmias, coronary vasoconstriction, and myocardial depression.
Prompt administration of epinephrine is an essential therapy for patients with anaphylactic shock. The mechanisms of epinephrine beneficial to
anaphylaxis patients include alpha1-agonist effects (restoring vascular tone and blood pressure and reducing mucosal edema), beta1-agonist effects
(increasing the rate and force of cardiac contractions), beta2-agonist effects (bronchodilation), and stabilization of mast cells.

While all 4 answers describe potential desirable measures for a patient suffering from anaphylactic shock, the question asks for the most appropriate
initial intervention. In addition to these measures, the administration of antihistamine agents (both H1and H2 blockers) is advised.

The proximity of the patient to a recent meal implicates unrecognized ingestion of a food allergen as a likely cause of anaphylaxis in this case.
Accidental ingestion of peanuts or tree nuts is a common cause of anaphylaxis in the setting of food ingestion.

REFERENCES (5)

1. Ben-Shoshan M, Clarke AE. Anaphylaxis: past, present and future. Allergy. 2011;66:114.
2. Little F, Hollingsworth H. Anaphylaxis. In: Irwin RS, Rippe JM, eds. Intensive Care Medicine. Sixth Edition. Philadelphia: Lippincott Williams &
Wilkins; 2008.

RATIONALE (6) Answer: C

Proper positioning of the pressure transducer is critical for measurement of central venous pressure (CVP) because seemingly small errors in transducer
height amplify errors in measuring cardiac filling pressure. The commonly accepted reference point for CVP measurement is the midpoint of the right
atrium, which can be identified on physical examination at a vertical distance 5 cm below the sternal angle. This is true whether the patient is supine or
sitting at a 60-degree angle. Therefore, patients do not have to be supine when this reference level is used, and option D is incorrect. More commonly in
the ICU, the midthoracic position at the level of the fifth rib is used for reference. This reference is easier to teach but should only be used for
measurements in the supine position because this reference point changes in relation to the right atrium with changes in posture.

The recommended position for measurement of the CVP is the base of the c wave, because this corresponds to the pressure in the atrium before the onset
of systole and therefore is the best estimate of preload. Thus, option A is incorrect.

CVP can be estimated on physical examination by measuring the distension of the jugular veins relative to the sternal angle and then adding 5 cm H2O to
this measurement. Therefore, option C is correct.

CVP measurements are referenced to atmospheric pressure, but physiologically it is transmural pressure (the difference between intracardiac and
intrathoracic-extracardiac pressure) that determines ventricular preload. Increased intrathoracic or elevated intrapericardiac pressure (eg, cardiac
tamponade, high positive end-expiratory pressure, large pleural effusion) may reduce venous return while paradoxically increasing the CVP.
Unfortunately, pleural pressure is not easily measured in critically ill patients. At end-expiration the lungs recoil inward and the chest wall outward,
which gives a pressure around the heart of approximately 2 to 3 cm H2O, not extremely different from atmospheric pressure. Accordingly, vascular
pressures (including CVP) should be measured at end-expiration whether the patient is ventilated with positive pressure ventilation or breathing
spontaneously. (Option B is incorrect.)

REFERENCES (6)

1. Magder S. Central venous pressure: a useful but not so simple measurement. Crit Care Med. 2006;34:22242227.
2. Magder S. Invasive intravascular hemodynamic monitoring: technical issues. Crit Care Clin. 2007;23:401414.
3. Barbeito A, Mark JB. Arterial and central venous pressure monitoring. Anesthesiology Clin. 2006;24:717735.

RATIONALE (7) Answer: A

Pulmonary artery occlusion pressure (PAOP) is obtained by inflation of the balloon at the tip of the pulmonary artery catheter. After inflation of the
balloon, there is a continuous column of blood from the pulmonary artery to the left ventricle during diastole. Consequently, PAOP is considered an
approximation of left ventricular end-diastolic pressure (LVEDP). However, the assumption that PAOP always reflects LVEDP is not always valid.
When the tip of the catheter is in West zone 1 or 2, alveolar pressure interrupts the blood column. Consequently, the measured PAOP is higher than end-
diastolic pulmonary pressure and pulmonary venous pressure. Thus, for accurate measurement, the catheter tip must be in West zone 3 of the lung.

Other factors that can adversely alter interpretation of the relationship between PAOP and LVEDP include increases in pulmonary vascular resistance
(option A) (eg, chronic obstructive pulmonary disease, acidosis, pulmonary emboli), changes in intrathoracic pressure (eg, positive end-expiratory
pressure), changes in the intracardiac environment (eg, mitral stenosis where PAOP reflects increased left atrial pressure and not LVEDP), and changes
in ventricular compliance (eg, myocardial ischemia, myocardial hypertrophy or dilatation, aortic or pericardial disease).
REFERENCES (7)

1. PACEP Collaborative. Pulmonary Artery Catheter Education Project. http://www.PACEP.org. Released 2001, Reviewed 2004, 2007. Last accessed
1/22/10.
2. Robin E, Costecalde M, Lebuffe G et al. Clinical relevance of data from the pulmonary artery catheter. Crit Care. 2006;10:S3.

RATIONALE (8) Answer: C

The 2010 American Heart Association (AHA) Advanced Cardiac Life Support (ACLS) guidelines recommend adenosine as a safe and potentially
effective therapy in the initial management of stable, undifferentiated, regular, monomorphic, wide-complex tachycardia, not for ventricular fibrillation,
which the rhythm strip shows. Thus, option A is incorrect.

The AHA ACLS guidelines recommend 1 shock of 150 or 200 J (based on manufacturer guidelines) if using a biphasic defibrillator or 360 J if using a
monophasic defibrillator. This takes the place of the 3 stacked shocks of 200, 300, and 360 J that were recommended in previous ACLS guidelines
(option B). The 3-shock recommendation was based on the low first-shock efficacy of monophasic dampened waveforms and efforts to decrease
transthoracic impedance with delivery of shocks in rapid succession. Modern biphasic defibrillators have a high first-shock efficacy in termination of
ventricular fibrillation. In addition, 3 stacked shocks delay resumption of chest compressions.

Approximately 70% of adults who have an out-of-hospital cardiac arrest have underlying acute myocardial infarction or pulmonary embolism.
Therefore, it has been hypothesized that thrombolysis during CPR might improve survival. A recent double-blind, multicenter trial compared
thrombolysis (with tenecteplase) to placebo in adult patients with witnessed out-of-hospital cardiac arrest. The trial was terminated prematurely for
futility after enrolling a total of 1,050 patients. No significant differences were found between the thrombolysis and placebo groups in the primary end
point of 30-day survival or in the secondary end points of hospital admission, return of spontaneous circulation, 24-hour survival, survival to hospital
discharge, or neurologic outcome. There were more intracranial hemorrhages in the thrombolysis group.

There is no evidence that routine administration of either amiodarone or lidocaine during cardiac arrest increases the rate of survival to hospital
discharge. The Amiodarone in Out-of-Hospital Resuscitation of Refractory Sustained Ventricular Tachycardia (ARREST) trial compared an
amiodarone 300-mg bolus with placebo in patients with out-of-hospital cardiac arrest who had received 3 or more shocks, had no pulse, and had
ventricular fibrillation or ventricular tachycardia. More patients in the amiodarone group had admission to the hospital with successful resuscitation than
the placebo group although there were no significant differences in the rates of survival to hospital discharge between the amiodarone-treated group and
placebo. The Amiodarone versus Lidocaine in Prehospital Ventricular Fibrillation Evaluation (ALIVE) trial compared an amiodarone 5-mg/kg bolus
with a lidocaine 1.5-mg/kg bolus in patients with out-of-hospital ventricular fibrillation. More patients in the amiodarone group had successful survival
to hospital admission than the lidocaine group, but the study found no significant difference in the rates of hospital discharge between the amiodarone
and lidocaine groups. Therefore, option D is incorrect.

REFERENCES (8)

1. Bottiger BW, Arntz HR, Chamberlain DA, et al. Thrombolysis during resuscitation for out of hospital cardiac arrest. N Engl J Med. 2008;359:2651
2662.
2. Nolan JP, Soar J. Defibrillation in clinical practice. Curr Opin Crit Care. 2009;15:209215.
3. Neumar RW, Otto CW, Link MS, et al. Part 8: adult advanced cardiovascular life support: 2010 American Heart Association Guidelines for
Cardiopulmonary Resuscitation and Emergency Cardiovascular Care. Circulation. 2010;122:S72967.
4. White RD, Blackwell TH, Russell JK, et al. Transthoracic impedance does not affect defibrillation, resuscitation or survival in patients with out-of-
hosptial cardiac arrest treated with a nonescalating biphasic waveform defibrillator. Resuscitation. 2005;64:6369.

RATIONALE (9) Answer: B

Milrinone, a second-generation phosphodiesterase inhibitor, is a nonadrenergic inotrope. Inhibiting phosphodiesterase increases intracellular cyclic
adenosine monophosphate, resulting in positive inotropy and peripheral vasodilation. The drug may be utilized for decompensated heart failure, as well
as for weaning from cardiopulmonary bypass after cardiac surgery. Milrinone decreases both left ventricular afterload and pulmonary vascular
resistance. Cardiac output is increased.

In this patient, the vasodilatory properties of milrinone decreased systemic blood pressure. The cardiac index increased. Pulmonary artery occlusion
pressure decreased.

Nitroglycerin (option A) is a venous vasodilator that would decrease systemic blood pressure and right atrial pressure, but the cardiac index would not
be increased. Dopamine (option C) has both inotropic and chronotropic properties. Dopamine would increase the heart rate and cardiac index, but
would not lower the blood pressure. Norepinephrine (option D) would have inotropic effects, but the alpha-adrenergic effects would raise the systemic
blood pressure and increase systemic vascular resistance.
Despite the favorable hemodynamic profile for cases of decompensated congestive heart failure (CHF), a recent randomized controlled trial of
milrinone (versus placebo) in CHF failed to show a decrease in the number of days hospitalized for cardiovascular causes. There was no significant
difference in mortality. The authors concluded that IV milrinone should not be used routinely for cases of decompensated CHF.

REFERENCES (9)

1. Cuffe MS, Califf RM, Adams KF, et al. Short-term intravenous milrinone for acute exacerbation of chronic heart failure: a randomized controlled
trial. JAMA. 2002; 287:1541-1547.
2. Kumar A, Parrillo JE. Shock: classification, pathophysiology, and approach to management. In: Parrillo JE, Dellinger RP, eds. Critical Care
Medicine: Principles of Diagnosis and Management in the Adult. Third edition. St. Louis, MO: Mosby; 2008: 377422.

RATIONALE (10) Answer: C

The intraaortic balloon pump (IABP) is an important and established therapy for various serious cardiac conditions. The underlying principle of the
IABP is that inflating the balloon during early diastole will displace blood in the aorta, increasing the diastolic BP, and by this mechanism improve
coronary perfusion pressure; deflation of the balloon just before systole will decrease left ventricle afterload by producing a rapid decrease in aortic
pressure. This, in turn, will decrease myocardial oxygen demand and consumption. Hemodynamic effects that would be expected in this clinical scenario
include the following:

Increase in the diastolic arterial pressure, systolic arterial pressure, and mean arterial pressure.
Decrease of the pulmonary artery occlusion pressure;
Improved left ventricular performance and ejection fraction, with increased cardiac index.
Decreased myocardial oxygen demand that would result in decreased oxygen consumption.
Only the values in option C match all these changes, making it the best answer.

REFERENCES (10)

1. Goldstein DJ, Oz MC, Rose EA. Implantable left ventricular assist devices. N Engl J Med. 1998;339:1522.
2. Turi Z. Intra-aortic balloon counterpulsation. In: Parrillo JE, Dellinger RP, eds. Critical Care Medicine. Third edition. St. Louis: Mosby; 2008: 93
116

RATIONALE (11) Answer: C

This patient is having an acute ST-elevation anterior myocardial infarction. In addition, he presents signs of developing cardiogenic shock with
tachycardia, low blood pressure, and signs of a cardiogenic pulmonary edema. The SHOCK (Should We Emergently Revascularize Occluded
Coronaries for Cardiogenic Shock) randomized trial demonstrated that, in patients with acute myocardial infarction complicated by cardiogenic shock,
early mechanical revascularization reduced mortality as compared with initial medical stabilization followed by late or no revascularization. Based on
these findings, in their most recent guidelines the American College of Cardiology and the American Heart Association elevated early mechanical
revascularization for cardiogenic shock to a class I recommendation for patients younger than 75 years with ST elevation or a left bundle-branch block
acute myocardial infarction.

Although all the other options include treatments that are important for patients with acute myocardial infarction, some of them may be deleterious in
patients developing cardiogenic shock, and may have negative impacts on patients with hypertension. In hospitals without the capability for onsite
cardiac interventions, stabilization and transfer to an institution capable of performing revascularization is probably the best strategy with acute
myocardial infarction and cardiogenic shock.

The Danish Trial in Acute Myocardial Infarction (DANAMI) trial compared primary mechanical revascularization (angioplasty) with thrombolytic
therapy in patients with ST-elevation acute myocardial infarction. Patients randomized at sites without cardiac catheterization facilities were transferred
for revascularization. In this trial, the combined outcome of death, recurrent myocardial infarction, and stroke was improved in patients randomized to
the mechanical revascularization arm (whether it was done onsite or the patient was transferred to another facility).

REFERENCES (11)

1. Anderson HR, Nielson TP, Rasmussen K, et al. A comparison of coronary angioplasty with fibrinolytic N Engl J Med. 2003; 349:733742.
2. Babaev A, Frederick PD, Pasta DJ, et al. Trends in management and outcomes of patients with acute myocardial infarction complicated by
cardiogenic shock. JAMA. 2005; 294:448454.
3. Hochman JS, Sleeper LA, Webb JG, et al. SHOCK Investigators. Early revascularization and acute myocardial infarction complicated by
cardiogenic shock. N Engl J Med. 1999;341:625634
RATIONALE (12) Answer: D

This patient's pulmonary artery hypertension is probably due to a pulmonary embolism. He requires vasopressor levels of dopamine and has a central
venous pressure (CVP) of 12 mm Hg. It is generally accepted that fluid resuscitation effects are variable in patients with suspected pulmonary embolism,
depending upon the degree of pulmonary artery hypertension and current intravascular volume status. A CVP of 12 mm Hg, however, would be
considered inadequate, 2025 mm Hg being the optimal range. Thrombolytic therapy (options A and B) may on occasion be utilized in patients who are
at high risk for pulmonary embolisms but have only echocardiographic support for pulmonary embolism. In this patient, a definitive test would be
optimal management, as he is adequately oxygenated with supplemental oxygen, requires levels of dopamine that are not considered high, and has a
relative contraindication to thrombolytic therapy.

Since CT of the chest is the only test currently available, the choice is between dye exposure, which in a patient with chronic renal insufficiency carries
a significant risk, versus thrombolytic exposure, with its risk of intracranial hemorrhage. The risks associated with the use of the dye in a patient with
renal insufficiency, even if it results in the need for renal replacement therapy, when weighed against the risk of an intracranial hemorrhage, give the
edge to immediate definitive testing for the presence or absence of a pulmonary embolism. Delaying testing in this patient may allow his condition to
deteriorate. Given these considerations, urgent thrombolytic therapy would not be the most appropriate choice.

REFERENCES (12)

1. Carbone R, Bossone E, Bottino G, et al. Secondary pulmonary hypertension: diagnosis and management. Eur Rev Med Pharmacol Sci. 2005;9:331
342.
2. Dong B, Kirong Y, Liu G, et al. Thrombolytic therapy for pulmonary embolism. Cochrane Database Syst Rev. 2006;2:CD004437.
3. Goldhaber SZ. Thrombolysis in pulmonary embolism: a debatable indication. Thromb Haemost. 2001;86:444451.
4. Jardin F, Vieillard-Baron A. Monitoring of right-sided heart function. Curr Opin Crit Care. 2005;11:271279.
5. Meyer FJ, Schoene AM, Borst MM. Pathophysiological aspects of cardiopulmonary interaction. Clin Nephrol. 2003;60(suppl 1):S75S80.
6. Perrier A, Roy PM, Sanchez O, et al. Multidetector-row computed tomography in suspected pulmonary embolism. N Engl J Med. 2005;352:1760
1768.
7. Segal JB, Streiff MB, Hoffman LV, et al. Management of venous thromboembolism; a systematic review Ann Intern Med. 2007;146:211222.
8. Via G, Braschi A. Pathophysiology of severe pulmonary hypertension in the critically ill patient. Minerva Anestesiol. 2004;70:233237.

RATIONALE (13) Answer: D

Effusive-constrictive pericarditis is a pericardial syndrome characterized by tense pericardial effusion with a tamponade, in addition to a visceral
pericardial constriction. The diagnostic criterion is the failure of the right atrial pressure to fall by 50% or more, or to a level below 10 mm Hg after
intrapericardial pressure is normalized. Although the most frequent cause is idiopathic, followed by neoplastic, a higher percentage of patients with
radiation-induced pericarditis will also have an effusive-constrictive pericarditis. Following the drainage of the pericardial effusion, there is typically
some improvement in the cardiac index, although it may not be as dramatic as in this case. In some cases of constrictive pericarditis associated with
idiopathic acute exudative pericarditis, spontaneous resolution is possible. This is also called transient cardiac constriction. In other cases, surgery may
be required to relieve visceral constriction. Prior to pericardiocentesis, the right atrial transmural pressures are very low (less than 2 mm Hg), with a
pulsus paradoxus of 10 mm Hg or more. These are typical criteria for a cardiac tamponade and typically resolve with the drainage of the pericardial
effusion. However, constriction and elevated right atrial pressure remain. Radiotherapy is a likely risk factor for this patient given her medical history of
lung cancer not in remission and her previous therapy.

REFERENCES (13)

1. Sagrista-Sauleda J, Angel J, Sanchez A, et al. Effusive-constrictive pericarditis. N Engl J Med. 2004;350:469475.


2. Ling LH, Oh JK, Schaff HV, et al. Constrictive pericarditis in the modern era: evolving clinical spectrum and impact on outcome after
pericardiectomy. Circulation. 1999;100:13801386.
3. Sagrista-Sauleda J, Permanyer-Miralda G, Candell-Riera J, et al: Transient cardiac constriction: an unrecognized pattern of evolution in effusive
acute idiopathic pericarditis. Am J Cardiol. 1987; 59:961966.

RATIONALE (14) Answer: C

Therapy for hypertrophic cardiomyopathy is directed at the dynamic left ventricular (LV) outflow tract obstruction. The obstruction causes an increase in
LV systolic pressure which leads to a complex interplay of abnormalities that decrease cardiac output. In the intensive care setting, this condition often
deteriorates with volume depletion, and with the institution of inotropic agents. In that circumstance, the infusion of fluids and the discontinuation of
inotropic agents is the initial therapy. A beta-blocker should also be added; however, if hypotension is present, a vasoconstrictor such as phenylephrine
should be administered first. Acute onset of atrial fibrillation may result in severe hemodynamic compromise due to the loss of atrial contractions.
Prompt cardioversion should occur in this circumstance.
The first-line approach to the relief of symptoms is to block the effects of catecholamines that exacerbate the outflow tract obstruction, and to slow the
heart rate to enhance diastolic filling. Beta-blockers are generally the initial choice to accomplish these goals. Verapamil, the calcium channel blocker,
can also be used. Sudden death has been reported in patients with severe pulmonary hypertension and severe outflow obstruction who are given
verapamil. This drug should be given with caution in patients with this combination of findings. Nitroglycerin (option D) would decrease cardiac filling
and is problematic.

REFERENCES (14)

1. Braunwald E, Seidman CE, Sigwart U. Contemporary evaluation and management of hypertrophic cardiomyopathy. Circulation. 2002; 106:1312
1316.
2. Maron, BJ. Hypertrophic cardiomyopathy: a systematic review. JAMA. 2002; 287:1308 1320.
3. Nishumura RA. Hypertrophic obstructive cardiomyopathy. N Engl J Med. 2004; 350:13201327.
4. Semsarian C. Guidelines for the diagnosis and management of hypertrophic cardiomyopathy. Heart Lung Circ. 2007;16:16 18.
5. Sherrid MV. Pathophysiology and treatment of hypertrophic cardiomyopathy. Prog Cardiovas Dis. 2006;49:123151.
6. Spirito P, Seidman CE, McKenna WJ, et al. The management of hypertrophic cardiomyopathy. N Engl J Med. 1997;336:775 785. RA

RATIONALE (15) Answer: B

The major known complication of intraaortic balloon pump (IABP) therapy is noncardiac ischemia. Since the IABP is usually inserted via a femoral
artery, limb ischemia is the most common. Malposition of the IABP can result in occlusion of the celiac, superior mesenteric, or renal arteries. After
comparison of position of the IABP in chest radiographs and CTs, visualization of the tip at the level of the carina can be used as confirmation of the
correct position. The tip of the IABP catheter is usually seen. However, the balloon itself will not be visible unless the radiography is performed during
balloon insufflation. Because this patient's angina has resolved, there is no indication for increase of the nitroglycerin (option A). Reduction of the IABP
support might increase the angina and is not an appropriate intervention at this time, so option C is incorrect. It is likely that occult blood in the stool
would be secondary to anticoagulation with heparin and nondiagnostic, so option D is also incorrect.

REFERENCES (15)

1. Kim JT, Lee JR, Kim JK, et al. The carina as a useful radiographic landmark for positioning the intraaortic balloon pump. Anesth Analg.
2007;205:735738.
2. Rastan AJ, Tillmann E, Subramanian S, et al. Visceral arterial compromise during intra-aortic balloon counterpulsation therapy. Circulation. 2010;
122:S92S99.

RATIONALE (16) Answer: A

The most recent guidelines for Advanced Cardiac Life Support (ACLS) from the American Heart Association include the use of continuous waveform
capnography. Studies have demonstrated high sensitivity and specificity of waveform capnography. During cardiac arrest, PETCO2 less than or equal to 10
mm Hg indicates that the cardiac output is inadequate to achieve a return of spontaneous circulation. Although there is no established value that
optimizes the chance of return of spontaneous circulation, a marked increase in PETCO2 would indicate sufficient perfusion. Chest compression should
continue for an additional 5 cycles after a return to spontaneous circulation indicated by PETCO2. Monitoring PETCO2 has the potential to guide the
adequacy of CPR.

REFERENCES (16)

1. Ahrens T, Schallom L, Bettorf K, et al. End-tidal carbon dioxide measurements as a prognostic indicator of outcome in cardiac arrest. Am J Crit
Care. 2001;10:391398.
2. Neumar RW, Otto CW, Link MS, et al. Part 8. Adult advanced cardiovascular life support: 2010. American Heart Association guidelines for
cardiopulmonary resuscitation and emergency cardiovascular care. Circulation. 2010;122:S729 S767.
3. Pokorna M, Necas E, Kratochvil J,et al. A sudden increase in partial pressure end-tidal carbon dioxide (P(ET)CO(2)) at the moment of return of
spontaneous circulation. J Emerg Med. 2009; 38:614621.
4. Silvestri S, Ralls GA, Krauss B, et al. The effectiveness of out-of-hospital use of continuous end-tidal carbon dioxide monitoring on the rate of
unrecognized misplaced intubation within a regional emergency medical services system. Ann Emerg Med. 2005;45:497503.

RATIONALE (17) Answer: B


The intraaortic balloon pump (IABP) is the most used mechanical device for cardiac support. The traditional thought is that coronary artery blood flow
is augmented during inflation of the IABP during isovolemic diastole while afterload and wall tension are decreased when the balloon is deflated during
systole. The latter are likely the primary mechanisms of action. The IABP is absolutely contraindicated in patients with aortic dissection (because of the
possibility of insertion in a false lumen), aortic valve insufficiency (because it is ineffective and worsens regurgitation), or a patent ductus arteriosus
(because of increased left-to-right shunt). Severe vascular disease, although a comorbidity, does not exclude percutaneous IABP placement in the
brachial or external iliac arteries or surgically in the descending thoracic aorta.

REFERENCES (17)

1. Lee MS, Makkar RR. Percutaneous left ventricular support devices. Cardiol Clin. 2006;24:265275.
2. Pfeiffer S, Frisch P, Weyand M, et al. The use of preoperative intra-aortic balloon pump in open heart surgery. J Cardiovasc Surg (Torino). 2005;
46:5560.
3. Trost JC, Hillis LD. Intra-aortic balloon counterpulsation. Am J Cardiol. 2006; 97:13911398.
4. Turi ZG. Intra-aortic balloon counterpulsation. In: Parrillo JE, Dellinger RP. Critical Care Medicine: Principles of Diagnosis and Management in the
Adult. Philadelphia: Mosby; 2008: 93115.

RATIONALE (18) Answer: A

Ventricular assist devices are inserted in patients with advanced heart failure. Most often these are left ventricular assist devices (LVAD) to improve
patient outcome. These have been used as a bridge to cardiac transplantation or as "destination therapy" to maintain some quality of life if a transplant is
not possible. Adjustment of an LVAD can be technically challenging and should be performed by qualified individuals. After the initial surgical
placement, it is often performed with monitoring via transesophageal echocardiogram. The LVAD functions by moving blood from the apex of the left
ventricle to the aorta. Increasing or decreasing the pump speed may actually decrease the cardiac output by altering ventricular filling and decreasing the
output to the aorta. Thus, options B and C are incorrect. Since LVAD patients receive anticoagulation, trace-positive gastrointestinal bleeds are
common, but still need to be worked up to determine severity. In this patient, there is no evidence for significant anemia. Therefore, option D is
incorrect. Since the hypotension occurred with the administration of sedation, it is likely that it was secondary to a reduction in vascular tone. Increasing
the blood pressure with pressor administration (option A) would be appropriate until a definitive diagnosis for the hypotension is identified.

REFERENCES (18)

1. Cowger J, Romano MA, Stulak J, et al. Left ventricular assist device management in patients chronically supported for advanced heart failure. Curr
Opin Cardiol. 2011;26:149154.
2. Leff JD, Shore-Lesserson L. Left ventricular assist devices: an evolving state of the art. Semin Cardiothorac Vasc Anesth. 2010;14:2123.
3. Miller LW, Pagani FD, Russell SD, et al. Use of a continuous-flow device in patients awaiting heart transplantation. N Engl J Med. 2007;357:885
896.
4. Nicolosi AC, Pagel PS. Perioperative considerations in the patient with left ventricular assist device. Anesthesiology. 2003; 98:565570.

RATIONALE (19) Answer: A

Atrioventricular (AV) conduction defects are present in approximately 25% of patients after cardiac surgery and are most common after aortic and
mitral valve surgeries. The nomenclature for pacemakers is chamber paced (atrial [A], ventricular [V], atrial and ventricular [D]), chamber sensed, and
function (inhibited [I], not effected [O], and triggers or inhibits [D]). VOO is not indicated since the patient has atrial activity, so option B is incorrect
When the PR interval exhibits a prolonged first-degree AV block, faster pacing with AOO does not achieve adequate conduction since the AV node
remains refractory when the next beat occurs and creates an iatrogenic second-degree block. Therefore, option C is incorrect. DDD pacing with a
shorter PR interval will markedly improve hemodynamics by eliminating the first-degree AV block. Thus, option A is correct.

REFERENCES (19)

1. Bernstein AD, Daubert J-C, Fletcher RD, et al. The revised NASPE/BPEG generic code for antibradycardia, adaptive-rate, and multisite pacing.
North American Society of Pacing and Electrophysiology Group. Pacing Clin Electrophysiol. 2002;25:260264.
2. Reade MC. Temporary epicardial pacing after cardiac surgery: a practical review. Part 1: General considerations in the management of epicardial
pacing. Anaesthesia. 2007; 62:264271.
3. Reade MC. Temporary epicardial pacing after cardiac surgery: a practical review. Part 2: Selection of epicardial pacing modes and troubleshooting.
Anaesthesia. 2007;62:364373

RATIONALE (20) Answer: C


Atrial overdrive pacing has been shown to be beneficial for the prevention of atrial fibrillation. Postoperative atrial fibrillation occurs in 30%40% of
patients in the second postoperative day after cardiac surgery. The pathogenesis is multifactorial and related to pericarditis, supraventricular premature
contractions, electrolyte disturbances, and sympathetic tone. Atrial overdrive pacing for the first 24 hours after cardiac surgery has been shown to
markedly decrease the incidence of postoperative atrial fibrillation. Therefore, option C is correct. Heart block secondary to edema or surgical
disruption of the conduction system is seen immediately in the operating room and in the ICU, so option A is incorrect. The cardiac index of 3.2 without
inotropics would not require augmentation whether or not the patient was supported with mechanical ventilation. Therefore options B and D are
incorrect.

REFERENCES (20)

1. Blommaert D, Gonzalez M, Mucumbitsi J, et al. Effective prevention of atrial fibrillation by continuous atrial overdrive pacing after coronary artery
bypass surgery. J Am Coll Cardiol. 2000;35:14111415.
2. Carlson MD, Ip J, Messenger J, et al. A new pacemaker algorithm for the treatment of atrial fibrillation. Results of the atrial dynamic overdrive
pacing trial (ADOPT). J Am Coll Cardiol. 2003;42:627633.
3. Daubert JC, Mabo P. Atrial pacing for the prevention of postoperative atrial fibrillation: how and where to pace? J Am Coll Cardiol.
2000;35:14231427.

RATIONALE (21) Answer: C

The diagnosis of systolic and diastolic heart failure cannot be made based on history and physical examination. Echocardiography is needed. Both
classifications of heart failure could give this patient pulmonary edema with a worse prognosis for diastolic heart failure. The primary goal of treatment
for diastolic dysfunction is to alleviate symptoms with diuretics, beta-blockers, and inhibitors of the renin-angiotensin system.

REFERENCES (21)

1. Leite-Moreira AF. Current perspectives in diastolic dysfunction and diastolic heart failure. Heart. 2006;92:712718.
2. Owan TE, Hodge DO, Herges RM, et al. Trends in prevalence and outcome of heart failure with preserved ejection fraction. N Engl J Med. 2006;
355:251259.
3. Yancy CW, Lopatin M, Stevenson LW, et al. Clinical presentation, management, and in-hospital outcomes of patients admitted with acute
decompensated heart failure with preserved systolic function. J Am Coll Cardiol. 2006;47:7684.

RATIONALE (22) Answer: C

A 2001 Consensus Statement by the Society of Critical Care Medicine, European Society of Intensive Care Medicine, American College of Chest
Physicians, American Thoracic Society, and Surgical Infection Society revised the definition of sepsis to include infection with clinical or laboratory
evidence of inflammation. The definition of severe sepsis remains unchanged as sepsis with an organ dysfunction or evidence of organ hypoperfusion.
Tissue hypoperfusion can occur in the absence of hypotension and can be present for several hours before clinical evidence is manifest. Clinical
recognition of organ dysfunction includes areas of mottled skin, capillary refill greater than 3 seconds, urine output less than 0.5 mL/kg for at least 1 hour
or need for renal replacement therapy, lactate level less than 2 mmol/L, abrupt change in mental status or abnormal EEG findings, platelet count less than
100,000/L, acute lung injury/acute respiratory distress syndrome, and cardiac dysfunction. Septic shock refers to refractory hypotension despite
adequate volume resuscitation, often necessitating the use of vasopressors.

REFERENCES (22)

1. American College of Chest Physicians/Society of Critical Care Medicine Consensus Conference. Definitions for sepsis and organ failure and
guidelines for the use of innovative therapies in sepsis. Crit Care Med. 1992;20:864874.
2. Annane D, Bellissant E, Cavaillon JM. Septic Shock. Lancet. 2005;365:6378.
3. Balk, RA. Severe sepsis and septic shock: definitions, epidemiology, and clinical manifestations. Crit Care Clin. 2000;16:179192.
4. Levy MM, Fink MP, Marshall JC, et al. 2001 SCCM/EISCM/ACCP/ATS/SIS International Sepsis Definitions Conference. Crit Care Med.
2003;31:12501256.

RATIONALE (23) Answer: D

Septic shock is defined as the presence of severe sepsis and systemic mean arterial pressure less than 60 mm Hg (or <80 mm Hg with previous
hypertension) after 4060 mL/kg of saline, 2030 mL/kg of starch, and/or a pulmonary capillary wedge pressure between 12 and 20 mm Hg. The need
for vasopressors in the setting of adequate fluid resuscitation also defines septic shock. Refractory septic shock refers to the need for high-dose
vasopressors, such as dopamine >15 g/kg/min, or norepinephrine or epinephrine >0.25 g/kg/min to maintain mean arterial pressure greater than 60
mm Hg (or >80 mm Hg with previous hypertension). The presence of hypotension in the setting of aggressive fluid resuscitation in a patient with
infection is the hallmark of septic shock. Signs and symptoms of systemic hypoperfusion may occur several hours before onset of shock, or may occur
even in the presence of normal blood pressure parameters. Multiple-Organ Dysfunction Syndrome (MODS) refers to the decline in organ function to the
point that homeostasis cannot be maintained without intervention, as outlined in the 1991 American College of Chest Physicians/Society of Critical Care
Medicine consensus statement.1 Primary MODS occurs as a result of a well-defined insult in which organ dysfunction occurs early and can be directly
attributed to the insult itself (eg, renal failure due to rhabdomyolysis). Secondary MODS is organ failure not in direct response to the insult itself, but as
a consequence of a host response (eg, acute respiratory distress syndrome in patients with pancreatitis). In the context of the definitions of sepsis and
systemic inflammatory response syndrome, MODS represents the more severe end of the spectrum of severity of illness characterized by systemic
inflammatory response syndrome or sepsis. There are no universally accepted criteria for MODS, and there is considerable overlap with severe sepsis.
Marshall6 critically evaluated the definitions of MODS adopted in the clinical literature and provided a rationale for the physiologic descriptors
commonly used to define this syndrome, focusing on markers of dysfunction for 6 organ systems: PaO2/FIO2, serum creatinine level, Glasgow Coma Scale
score, platelet count, serum bilirubin level, and pressure-adjusted heart rate (heart rate multiplied by the ratio of central

REFERENCES (23)

1. American College of Chest Physicians/Society of Critical Care Medicine Consensus Conference. Definitions for sepsis and organ failure and
guidelines for the use of innovative therapies in sepsis. Crit Care Med. 1992;20:864874.
2. Annane D, Bellissant E, Cavaillon JM. Septic Shock. Lancet. 2005;365:6378.
3. Balk RA. Severe sepsis and septic shock: definitions, epidemiology, and clinical manifestations. Crit Care Clin. 2000;16:179192.
4. Dellinger RP, Levy MM, Carlet JM, et al. Surviving Sepsis Campaign: international guidelines for management of severe sepsis and septic shock:
2008. Crit Care Med. 2008;36:296327.
5. Levy MM, Fink MP, Marshall JC, et al. 2001 SCCM/EISCM/ACCP/ATS/SIS International Sepsis Definitions Conference. Crit Care Med.
2003;31:12501256.
6. Marshall JC, Cook DL, Christou NV, et al. Multiple Organ Dysfunction Score: a reliable descriptor of a complex clinical outcome. Crit Care Med.
1995;23:16381652.

RATIONALE (24) Answer: A

This patient displays the classic "transient responder" profile of ongoing blood loss. Given his mechanism of injury, an occult intra-abdominal source is
likely. While myocardial ischemia is possible even in young people without coronary artery disease while under the influence of cocaine, his
hypertension appears not to rise to a level consistent with this presentation. Further, with the placement of a central line and the ability to monitor central
filling pressures, a normal value tends to rule out both cardiogenic and obstructive shock. A decrease in blood pressure would be expected with the
resolution of cocaine-induced hypertension (option D), but this would probably be accompanied by a resolution of tachycardia as well. The
reemergence of hypotension and tachycardia simultaneously is most suspicious for occult hemorrhage.

REFERENCES (24)

1. Committee on Trauma, American College of Surgeons. Advanced Trauma Life Support Program for Doctors. Chicago: American College of
Surgeons; 2008.
2. Dutton RP. Management of traumatic shock. In: Prough DS, Fleisher L, eds. Problems in Anesthesia: Trauma Care. London: Lippincott Williams and
Wilkins; 2002.
3. Mullins RJ. Management of shock. In: Mattox KL, Feliciano DV, Moore EE, eds. Trauma. Fourth edition. New York: McGraw-Hill; 2000.

RATIONALE (25) Answer: C

Delays in definitive treatment are common, particularly in mass casualty situations or austere locations, and constitute opportunities for preoperative
optimization of the patient's status. This patient is showing classic symptoms of a class II hemorrhage, appearing anxious, with shallow tachypnea and a
narrowed pulse pressure. Further, the patient is hypothermic, acidotic, and possibly trending toward coagulopathy (the "lethal triad").

While several of the individual therapies listed may be beneficial, the only combination that coherently serves the patient's best interests is option C.
The patient is oriented and able to protect his airway; therefore, rapid-sequence intubation, maintenance of sedation/anesthesia, and ventilator
management (options B and D) are most appropriately deferred to the operating room. Since this patient's narrowed pulse pressure serves as a
compensatory mechanism against shock, induction of anesthesia remote from definitive treatment may cause a sudden loss of peripheral vascular
resistance and assumes the unnecessary risk of cardiovascular collapse, another argument for deferring until the operating room. Placement of a
nasogastric tube and removal of stomach contents (option B) would not be a high priority in this patient, especially once a definitive airway has been
placed.

Hyperventilation and sodium bicarbonate (options A and D) are of only transient value in the treatment of acidosis, and given the patient's increased
lactate, he is more in need of fluid resuscitation. His 2 large-bore IV lines are probably adequate for resuscitation, making central line placement
(options A and B) a low priority and possibly an unnecessary risk. Hydroxyethyl starches (option B) have been associated with a dose-dependent
decrease in factor VIII and von Willebrand factor activity and may effect platelet function and prothrombin time, both of which could be deleterious in a
patient nearing coagulopathy. While the patient will probably need blood and factor replacement soon, he seems sufficiently stable to await type and
cross of an appropriate number of blood products; therefore, option D is incorrect.

Warming this patient is of critical importance. Hypothermia contributes significantly to morbid myocardial events, impairs platelet function and blood
loss, increases the risk of wound infections, and lengthens hospital stay. A forced-air warmer should be placed as soon as possible and exposed areas
should be covered. If blood products are begun in the emergency department, they should be delivered through a fluid warmer.

An increase in the patient's urinary output will indicate the effectiveness of ongoing fluid resuscitation. Ideally, a urinary catheter will be placed
preoperatively, particularly if any significant delay is anticipated; if not, one should be inserted soon after induction and intubation.

REFERENCES (25)

1. Shoemaker WC, Peitzman AB, Bellamy R, et al. Resuscitation from severe hemorrhage. Crit Care Med. 1996;24:S1223.
2. Scalea TM, Henry SM. Assessment and initial management in the trauma patient. Probl Anesth. 2001;13:271-8.
3. Sessler DI. New surgical thermal management guidelines. Lancet. 2009;374:104950.

RATIONALE (26) Answer: C

This patient exhibits the symptoms of a class III hemorrhage, suggesting a loss of 30%40% of his blood volume: confusion, pulse rate greater than
120/min, decreased blood pressure, decreased pulse pressure, and RR of 3040/min. Estimates of blood loss are often underestimated, as 30%40% of
the blood volume of a patient of this size would be expected to be roughly 1.52 L.

RATIONALE (27) Answer: A

He is unstable and in class III hemorrhagic shock. A cause for this shock is evident, in the form of a left hemothorax. After securing his airway, this
condition should be treated with a left chest tube, possibly with progression to a left thoracotomy to control bleeding. If available, sterile collection and
autotransfusion of the patient's shed blood should be considered.

With two large-bore IV lines, the patient likely has sufficient vascular access for resuscitation. Were a central line to be placed after chest tube
placement, a left-sided approach would be prudent, as the risk of line-associated pneumothorax would be obviated and the unaffected right lung would
not be placed at risk. Thus, option B is incorrect.

Obtaining a chest radiograph (option C) would be unnecessary in a clinically diagnosed hemothorax; this study should be deferred until the patient is
stable in order to confirm appropriate placement of both the endotracheal tube and the chest tube.

While an exploratory laparotomy (option D) may be necessary should the patient continue to manifest signs of ongoing occult blood loss, it should only
be undertaken after the readily obvious sources of loss, namely the left hemothorax and left femur fracture, have been addressed.

REFERENCES (27)

1. Committee on Trauma, American College of Surgeons. Advanced Trauma Life Support Program for Doctors. Chicago: American College of
Surgeons; 2008.
2. Dutton RP. Pathophysiology of traumatic shock. In: Smith CE, Rosenberg AD, Grande CM, ed. Massive Transfusion and Control of Hemorrhage in
the Trauma Patient; 2001:7-10.
3. Freischlag JA. Intraoperative blood salvage in vascular surgeryworth the effort? Crit Care. 2004;8(Suppl 2):S53S56.

RATIONALE (28) Answer: D

The transgastric midpapillary short-axis view will show the cross-sectional chamber size (and therefore the volume status) of the left ventricle, and is
readily applied for globally assessing both volume and left ventricular (LV) function.

While the deep transgastric long-axis view (option A) focuses on the LV, it is difficult to use for volume assessment, as the probe may not be centered in
the LV chamber. This view is most often used for interrogation of the aortic valve and the left ventricular outflow tract. While close to and in the same
orientation as the midpapillary view, the transgastric basal short-axis view focuses on the cross-sectional view of the mitral valve at the base (most
superior aspect) of the LV. Very little information about the LV volume status can be gained from this view in isolation from other views. Thus, option B
is incorrect. The mid-esophageal bicaval view (option C) focuses on the right atrium, the superior vena cava, the inferior vena cava, and the interatrial
septum. While inspecting the relative fullness of the venae cavae can give some indication as to volume status, the LV will not be present in this view.

RATIONALE (29) Answer: D

The citrate in banked blood products chelates divalent cations such as calcium and magnesium. After a massive transfusion, hypocalcemia may decrease
cardiac inotropy and blood pressure. As the percentage of total calcium made up by ionized calcium is small, as there is a large safety margin prior to
the dangers of hypercalcemia setting in, and as awaiting return of an ionized calcium level from the lab may be time consuming, empiric replacement of
calcium should be considered in this patient.

While decreasing positive end-expiratory pressure (PEEP) (option A) may increase preload in some cases, this patient's preload, as assessed both by
central venous pressure and transesophageal echocardiography, is more than adequate to overcome the modest level of PEEP currently employed.

Dopamine dosages of 510 g/kg/min should have the greatest effect on beta1-adrenergic receptors, increasing cardiac inotropy. Increasing this infusion
to 20 g/kg/min (option B) would shift dopamine's profile to favor alpha1-adrenergic effects, producing vasoconstriction and increased peripheral
vascular resistance. While measured blood pressure may increase, this increase to forward flow may actually decrease cardiac output even further.

In the absence of ongoing blood loss, anemia, or hypovolemia, additional blood or fluid resuscitation would not be indicated in this patient. Further
banked blood may even exacerbate this patient's hypocalcemia. Therefore, option C is incorrect.

REFERENCES (29)

1. Dzik WH, Kirkley S. Citrate toxicity in massive transfusion. Transfus Med Rev. 1988;2:7694.
2. Pinsky MR. Assessment of indices of preload and volume responsiveness. Curr Opin Crit Care. 2005;11:2359.
3. Rivers E, Nguyen B, Havstad S, et al. Early goal-directed therapy in the treatment of severe sepsis and septic shock. N Engl J Med. 2001;345:1368
77.
4. Vignon P. Hemodynamic assessment of critically ill patients using echocardiography Doppler. Curr Opin Crit Care. 2005;11:22734.

RATIONALE (30) Answer: B

This patient has developed acute left ventricular ischemia immediately after coronary artery bypass grafting with a significant elevation of left-sided
filling pressures. Marked elevation in the pulmonary artery diastolic pressure (PADP) but not the pulmonary artery occlusion pressure (PAOP) suggests
the presence of pulmonary hypertension. Marked elevations in the central venous pressure, PADP, and PAOP, with equalization of diastolic filling
pressures, is consistent with cardiac tamponade. Either an anaphylactic reaction or a septic shock will be characterized by lower filling pressures in the
face of hypotension and tachycardia.

REFERENCES (30)

1. Hensley FA, Martin DE, Gravlee GP. A Practical Approach to Cardiac Anesthesia. Third Edition. Philadelphia: Lippincott Williams & Wilkins;
2003: 251268.
2. Baumgartner F. On call in cardiothoracic surgery. ICU management. 1997; 9: 135140.

RATIONALE (31) Answer: B

In the central venous pressure tracing, a normal V wave indicates venous return. However, giant V waves can be seen with tricuspid regurgitation or
mitral regurgitation due to papillary muscle ischemia on the pulmonary artery occlusion pressure trace.

Eventually, coronary artery disease could provoke ischemia and then acute mitral valve regurgitation.

REFERENCES (31)

1. Barash Paul, Cullen B, Stoelting R. Monitoring the anesthetized patient. Clinical Anesthesia. 2001;25: 675679.
2. Snyder RW, Glamann DB, Lange RA, et al. Predictive value of prominent pulmonary arterial wedge V waves in assessing the presence and severity
of mitral regurgitation. American Journal of Cardiology. 1994;73:56870.

RATIONALE (32) Answer: C


Increasing preload, systemic vascular resistance (SVR), and heart rate will best maintain hemodynamic stability in patients with pericardial tamponade;
the ultimate treatment will be the evacuation of the effusion. In patients with aortic stenosis, increasing preload and systemic vascular resistance while
maintaining sinus rhythm will be recommended. Mitral stenosis should be approached by increasing left ventricular preload, keeping sinus rhythm, and
maintaining SVR at normal values. In patients with mitral regurgitation, avoiding bradycardia, carefully increasing left ventricular preload, and keeping
SVR low to allow forward blood flow are key. Patients with idiopathic hypertrophic subaortic stenosis benefit from preload and afterload augmentation
and from slow heart rates.

REFERENCES (32)

1. Khandaker MH, Espinosa RE, Nishimura RA, et al. Pericardial disease: diagnosis and management. Mayo Clin Proc. 2010;85:57293.
2. Edmunds LH Jr. Cardiac Surgery in the Adult. New York: McGraw-Hill; 1997:13031317.

RATIONALE (33) Answer: E

The lack of tolerance to rapid atrial fibrillation could be explained as a twofold problem: First, the patient is being induced into general anesthesia
which per se could have caused some transient hypotension. Second, the sudden loss of atrial power could cause hemodynamic instability. Electrical
cardioversion should be the first line of treatment to help reestablish normal hemodynamics. Deepening the anesthetic could only cause more
hypotension. A rapid bolus of fluid could be helpful, but would be transient. Beta-blockers and or digoxin have shown some benefit by controlling the
rate, but they lack cardioverter properties.

REFERENCES (33)

1. Estafanous FG Barash PG, Reves JG. Cardiac Anesthesia: Principles and Clinical Practice. Philadelphia: Williams & Wilkins; 2001:557.
2. Fuster V, Ryden LE, et al. ACC/AHA/ESC 2006 Guidelines for the Management of Patients with Atrial Fibrillation: a report of the Amerjican
College of Cardiology/American Association Task Force on Practice Guidelies and the European Society of Cardiology Committee for Practice
Guidelines. Circulation. 2007; 116:e138.

RATIONALE (34) Answer: A

Atrial fibrillation is a common feature of mitral stenosis because of significant dilation of the left atrium. The left ventricle is chronically underloaded
and contractility is usually preserved. Afterload should be maintained, and large increases should be avoided because of possible left ventricular
dysfunction.

REFERENCES (34)

1. Chandrashekhar Y, Westaby S, Narula J. Mitral stenosis. Lancet. 2009; 374:127183.


2. Miller R. Anesthesia for cardiac surgery procedures. Miller's Anesthesia. Sixth Edition. New York: Elsevier/Churchill Livingstone; 2005:1959
1962.

RATIONALE (35) Answer: A

This patient has a Mobitz type II atrioventricular block. The advanced cardiac life support (ACLS) bradycardia algorithm should be followed; if
transcutaneous pacing is not available or fails to improve this condition, atropine should be considered. If there is still no response, an infusion of
epinephrine or dopamine should be started. Phenylephrine could certainly be considered; however, it could lead to more bradycardia by causing a vagal
reflex. CPR should become part of the treatment if pulse and blood pressure cannot be detected.

REFERENCES (35)

1. Advanced Cardiovascular Life Support Provider Manual. American Heart Association 2010.
2. Wogan JM, Lowenstein SR, Gordon GS. Second-degree atrioventricular block: Mobitz type II. Journal of Emergency Medicine. 1993;11:4754.

RATIONALE (36) Answer: D

If the patient is stable, the BP should be lowered to 90120 mm Hg, first by using beta-blockers, then considering vasodilators. Starting vasodilators
first could increase shear forces and dP/dt, potentially expanding the dissection.

REFERENCES (36)

1. Elefteriades JA, Lovoulos CJ, Coady MA, et al. Management of descending aortic dissection. Annals of Thoracic Surgery. 1999;67: 20022005.
2. Hensley F, Martin D, Gravlee G. A Practical Approach to Cardiac Anesthesia. Third Edition. Philadelphia: Lippincott Williams & Wilkins;
2003:618643.

RATIONALE (37) Answer: A

This patient has type B aortic dissection. The cornerstone of management is long-term blood pressure and heart rate control. In the acute setting, aortic
dissection presents as a hypertensive emergency that requires aggressive blood pressure management. The propagation of the dissection primarily
depends on the mean arterial pressure and shearing force created by the high velocity of left ventricular contraction (aortic pulse dP/dt). Reducing the
rate of rise of the aortic pulse by decreasing the force of the left ventricular contractions retards the progression of the dissection and decreases the risk
of aortic rupture. Beta-adrenergic blockade with peripheral vasodilatation is the basis of modern management of aortic dissection. Beta-blocker therapy
should be started before a vasodilator to limit reflex catecholamine release secondary to the direct vasodilatation that will worsen the dissection.
Esmolol, with its quick onset, short duration of action and easy titratability is the ideal beta-blocker for this circumstance. If heart rate and blood
pressure are not controlled with esmolol alone, the addition of a vasodilator will be beneficial. Nicardipine is a calcium-channel blocker with a high
vascular selectivity, fast onset, and relatively short half-life that produces primarily arterial vasodilatation. Nitroprusside has been traditionally used as
an arterial vasodilator, but cyanide toxicity may be problematic for patients with liver and kidney dysfunction.

REFERENCES (37)

1. Estrera AL, Miller CC III, Safi HJ, et al. Outcomes of medical management of acute type B aortic dissection. Circulation. 2006;114,I384I389.
2. Khan IA, Nair CK. Clinical, diagnostic, and management perspectives of aortic dissection. Chest. 2002;122:311328.
3. Robin ED, McCauley R, Nitroprusside-related cyanide poisoning: time (long past due) for urgent, effective interventions. Chest. 1992;102:1842
1845.

RATIONALE (38) Answer: D

Increased blood pressure in patients with acute ischemic stroke is the mechanism to maintain perfusion of affected cerebral tissue. A reduction of blood
pressure to normotensive values has the potential for further cereberal ischemia. Systemic thrombolysis is a treatment option if given within 3 hours of
the onset of stroke. The American Stroke Association recommends withholding antihypertensive treatment unless other noncerebral organ damage is
present, the patient is a candidate for thrombolytic therapy, or systolic BP is greater than 220 mm Hg and/or diastolic BP is greater than 120 mm Hg. The
treatment goal is a 15%20% reduction in blood pressure within 24 hours.

Nitroprusside causes cerebral vasodilation and increases intracerebral pressure. It can produce a "cerebral steal" syndrome as a result of increasing
vasodilation more in systemic arterioles than in the cerebral vascular bed. Labetalol or nicardipine are better choices for management of severe
hypertension in the setting of acute ischemic stroke.

REFERENCES (38)

1. Adams HP Jr, Adams RJ, Brott T, et al Guidelines for the early management of patients with ischemic stroke: a scientific statement from the Stroke
Council of the American Stroke Association. Stroke. 2003;34:10561083.
2. Semplicini A, Maresca A, Boscolo G, et al. Hypertension in acute ischemic stroke: a compensatory mechanism or an additional damaging factor?
Arch Intern Med. 2003;163:211216.
3. Griswold WR, Reznik V, Mendoza SA. Nitroprusside-induced intracranial hypertension. JAMA. 1981;246:267980.
4. Immink RV, van den Born BJ, van Montfrans GA, et al. Cerebral hemodynamics during treatment with sodium nitroprusside versus labetalol in
malignant hypertension. Hypertension. 2008; 52:23640.

RATIONALE (39) Answer: C

Major steps in the management of patients with severe preeclampsia are hydration and magnesium sulfate for seizure prophylaxis and blood pressure
control. Treatment of severe hypertension in these settings is primarily directed toward reducing the risks of intracerebral hemorrhage (systolic BP >160
mm Hg is an important factor in stroke with preeclampsia) and cardiac failure while maintaining uteroplacental blood flow. The American College of
Obstetricians and Gynecologists recommends treating severe hypertension during pregnancy with a goal to reduce systolic BP to 140160 mm Hg and/or
reduce diastolic BP to 90105 mm Hg. Hydralazine was traditionally used in pregnancy-induced hypertension and preeclampsia, but it has a delayed
onset and has been linked to a higher incidence of maternal hypotension, placental abruption, and emergent cesarean sections. Angiotensin-converting
enzyme inhibitors are contraindicated in pregnancy. Delivery is the ultimate treatment for preeclampsia when the fetus is at a more advanced gestational
age.

REFERENCES (39)

1. Gifford RW, August PA, Cunningham G. Report of the National High Blood Pressure Education Program Working Group on High Blood Pressure in
Pregnancy. Am J Obstet Gynecol. 2000;183:S1S22.
2. Martin JN Jr, Thigpen BD, Moore RC, et al. Stroke and severe preeclampsia and eclampsia: a paradigm shift focusing on systolic blood pressure.
Obstet Gynecol. 2005;105:246254.
3. Magee LA, Cham C, Waterman EJ, et al. Hydralazine for treatment of severe hypertension in pregnancy: meta-analysis. BMJ. 2003;327:955960.
4. Elatrous S, Nouira S, Ouanes BL, et al. Short-term treatment of severe hypertension of pregnancy: prospective comparison of nicardipine and
labetalol. Intensive Care Med. 2002;28:12811286.

RATIONALE (40) Answer: C

Emergent synchronized cardioversion is the treatment for new-onset atrial fibrillation with hemodynamic instability. Unlike defibrillation that is
performed irrespectively of the cardiac cycle, synchronized cardioversion is produced by synchronizing discharge with the R wave. Timing the
discharge prevents the delivery of a high-energy shock within a vulnerable period of the cardiac cycle (T wave), which might result in ventricular
fibrillation. Higher success rates are achieved when direct current cardioversion is performed earlier, from onset of atrial fibrillation, with delivery of a
higher energy, biphasic shock.

The main purpose of the implantable cardioverter-defibrillator (ICD) in this patient is the potential treatment of a lethal ventricular dysrhythmia due to
his cardiomyopathy. The ICD is programmed to detect ventricular tachycardia and perform pacing or cardioversion according to the algorithm. Modern
ICDs have special algorithms to distinguish between ventricular and supraventricular dysrhythmias or sinus tachycardia in order to prevent misfire of
the device. External cardioversion will not damage modern implantable pacemakers and ICDs. If several attempts of external cardioversion are
unsuccessful, then the patient should be given amiodarone to facilitate external cardioversion. Beta-blockade may result in further hypotension, may not
be efficacious, and may not have an immediate effect.

REFERENCES (40)

1. Fuster V, Ryden LE, Cannom DS, et al. ACC/AHA/ESC 2006 guidelines for the management of patients with atrial fibrillationexecutive summary.
Circulation. 2006; 114:700752.
2. Lown B, Amarasingham R, Neuman J. New method for terminating cardiac arrhythmias: use of synchronized capacitor discharge. JAMA.
1962;182:54855.
3. ACC/AHA/HRS 2008 guidelines for device-based therapy of cardiac rhythm abnormalities: a report of the American College of
Cardiology/American Heart Association Task Force on Practice Guidelines (Writing Committee to Revise the ACC/AHA/NASPE 2002 Guideline
Update for Implantation of Cardiac Pacemakers and Antiarrhythmia Devices): developed in collaboration with the American Association for
Thoracic Surgery and Society of Thoracic Surgeons. Circulation. 2008 May 27; 117:e350e408.
4. Di Marco JP. Implantable cardioverter-defibrillators. N Engl J Med. 2003 Nov 6;349:183647.
5. Manegold JC, Israel CW, Ehrlich JR, et al. External cardioversion of atrial fibrillation in patients with implanted pacemaker or cardioverter-
defibrillator systems: a randomized comparison of monophasic and biphasic shock energy application. Eur Heart J. 2007 Jul;28:17318.
PART 3: Respiratory Critical Care

Instructions: For each question, select the most correct answer.

1. Following a motor vehicle collision, a 20-year-old man is admitted to the ICU with injuries including a fracture of the left humerus and a possible
closed head injury. Glasgow Coma Scale score is 11 and he is in a cervical collar. Within the first hour, his neurologic status deteriorates to a
Glasgow Coma Scale score of 7, and he requires endotracheal intubation for airway protection. To accomplish this intubation, which of the
following sequences provides the optimum conditions?

A. No sedation, cervical collar in place, fiberoptic intubation


B. No sedation, cervical collar in place, direct laryngoscopy
C. Rapid-sequence induction with propofol and succinylcholine, in-line manual stabilization, cervical collar off, direct laryngoscopy
D. Rapid-sequence induction with etomidate and vecuronium, in-line manual stabilization, cervical collar in place, direct laryngoscopy
E. Propofol, in-line manual stabilization, fiberoptic intubation

2. Following a modified radical neck dissection, a 72-year-old man is admitted to the ICU for routine progressive care. Past medical history includes
hypertension treated with lisinopril. Shortly after admission, the nurse notices that his tongue appears to be larger than upon admission. When you
arrive, the nurse informs you that the tongue has now doubled in size since you were called to evaluate him. The patient has become increasingly
anxious. Which of the following is the most appropriate treatment at this time?

A. Nebulized epinephrine
B. IV diphenydramine, 50 mg, and IV dexamethasone, 12 mg
C. Observation
D. Immediate intubation by an anesthesiologist with a surgeon present
E. Elevation of the head of the bed to 30

3. A thoracoabdominal esophagectomy is planned for a 46-year-old man with an otherwise unremarkable past medical history. Unfortunately, the
laparotomy reveals diffuse metastatic disease and the procedure is aborted. He is brought to the ICU because he is still chemically paralyzed. Prior
to extubation, which of the following would confirm return of muscle strength?

A. A return of the train-of-four to 4 equal, strong twitches


B. Inspiratory force of 20 cm H2O
C. 600-mL tidal volume on pressure support ventilation at 16 cm H2O
D. Ability to follow commands
E. RR greater than 18/min

4. Which of the following patients is the best candidate for noninvasive ventilation?

A. 55-year-old man with ST-elevation myocardial infarction, pulmonary edema; BP of 90/64 mm Hg, HR of 110/min, RR of 25/min, pulse
oximetry of 90% on nonrebreather mask, arterial blood gas (ABG) pH of 7.28, PaCO2 of 45 mm Hg, and PaO2 of 58 mm Hg
B. 68-year-old man receiving home oxygen for chronic obstructive pulmonary disease that is in exacerbation with mild lethargy, BP of 140/92 mm
Hg, HR of 110/min, RR of 30/min, pulse oximetry of 96% on nonrebreather mask, ABG pH of 7.20, PaCO2 of 70 mm Hg, and PaO2 of 120 mm Hg
C. 28-year-old woman with an exacerbation of asthma, BP of 130/88 mm Hg, HR of 118/ min, RR of 20/min, pulse oximetry of 97% on 2 L/min of
oxygen via nasal cannula, ABG pH of 7.45, PaCO2 of 35 mm Hg, and PaO2 of 120 mm Hg
D. 59-year-old man with bronchiectasis, worsening shortness of breath, significant ongoing hemoptysis, BP of 118/68 mm Hg, HR of 105/min, RR
of 32/min, pulse oximetry of 92% on nonrebreather mask, ABG pH of 7.42, PaCO2 of 30 mm Hg, and PaO2 of 65 mm Hg
E. 62-year-old woman with coronary artery bypass surgery followed by extubation immediately postoperatively and respiratory distress 8 hours
later, BP of 108/70 mm Hg, HR of 72/min, RR of 25/min, pulse oximetry of 94% on nonrebreather mask, ABG pH of 7.37, PaCO2 of 34 mm Hg,
and PaO2 of 76 mm Hg

5. A 62-year-old woman is admitted to the ICU with an exacerbation of acute chronic obstructive pulmonary disease. An arterial blood gas study
showing pH of 7.15, PaCO2 of 78 mm Hg, PaO2 of 68 mm Hg, and oxygen saturation of 96% is obtained during administration of supplemental
oxygen. Noninvasive ventilation is initiated with expiratory pressure of 4 cm H2O, inspiratory pressure of 10 cm H2O, and FIO2 of 50%.

An hour later, the patient's BP has dropped from 140/85 to 110/60 mm Hg. On examination, the patient seems more lethargic. SaO2 is now 89%. An
ECG shows T-wave inversions in the anterior leads. Which of the following is the best next step in this patient's management?

A. Measure arterial blood gases


B. Increase oxygen to maintain saturation at 92% or higher
C. Orally intubate the patient and provide mechanical ventilator support
D. Increase the expiratory pressure setting by 5 cm H2O
E. Increase the inspiratory pressure setting by 3 cm H2O

6. Which of the following patient data are utilized in the calculation of the appropriate tidal volume for a patient with acute respiratory distress
syndrome (ARDS) in accordance with ARDSNet recommendations?

A. Body mass index


B. Weight
C. Height
D. Age

7. A 64-year-old woman weighing approximately 60 kg (132 lbs), is admitted to the ICU from the emergency department following intubation and
initiation of mechanical ventilation. No past medical history is available. Chest radiograph reveals a right lower lobe infiltrate consistent with
pneumonia. Ventilator settings are FIO2 of 0.5, assist-control volume ventilation, rate of 16/min (total assisted rate of 20/min), tidal volume of 600
mL, square inspiratory waveform, peak inspiratory flow rate of 80 L/min, and positive end-expiratory pressure of 5 cm H2O. The patient exhibits
dyssychronous breathing. Pulse oximetry oxyhemoglobin saturation is 98%. Flow-over-time waveform is shown (see Figure below). Which of the
following is most likely to be effective in improving the dyssynchronous breathing?

A. Decreasing peak inspiratory flow rate


B. Switching to decelerating inspiratory flow waveform
C. Increasing set ventilator rate to 20/min
D. Positioning patient with left side down
E. Increasing set positive end-expiratory pressure

8. A 28-year-old man weighing 80 kg (176 lbs) is admitted to the ICU following intubation and initiation of mechanical ventilation for community-
acquired pneumonia associated with severe hypoxemia. His breathing pattern is dyssynchronous. Ventilation settings on assist-control, volume-
cycled ventilation are FIO2 of 0.6, rate of 14/min (total rate assisted is 25/min), tidal volume of 700 mL, positive end-expiratory pressure (PEEP) of
10 cm H2O, and peak inspiratory flow rate of 60 L/min, with a square inspiratory flow waveform. Pulse oximetry reveals an oxyhemoglobin
saturation of 96%. Pressure-over-time waveform is shown in the Figure. Which of the following is most likely to relieve the patient's
dyssynchronous breathing?
A. Increasing peak inspiratory flow rate
B. Switching to a decelerating inspiratory flow waveform
C. Increasing ventilator rate
D. Increasing PEEP
E. Decreasing PEEP

9. A 25-year-old, asthmatic woman undergoes emergent laparotomy for a perforated appendix. Postoperatively, she develops profound dyspnea and
acute respiratory failure. She is intubated with a rapid-sequence induction. Mechanical ventilation is initiated in an assist-control mode at 16/min,
tidal volume of 550 mL, positive end-expiratory pressure of 0 cm H2O, and FIO2 of 1.0. In the postanesthesia recovery room, the patient is deeply
sedated, with RR of 16/min on a set rate of 16/min with the ventilator. The flow graphic waveform shown in the Figure is displayed on the
ventilator. Which of the following would be most effective in correcting this problem?

A. Decrease the inspiratory flow rate.


B. Increase tidal volume.
C. Decrease respiratory rate.
D. Increase positive end-expiratory pressure.

10. A 58-year-old patient is intubated with an exacerbation of his chronic obstructive pulmonary disorder. There is no pneumonia present, but
secretions are thick and yellow. Bronchodilator and steroid therapies are initiated. On day 4 of mechanical ventilation, the patient suffers
respiratory distress followed by cardiac arrest. Respiratory support data over the 5 days leading up to the arrest are provided below:

Day 1 Day 2 Day 3 Day 4


FIO2 0.4 0.4 0.4 0.4
Positive end 5 5 5 5
expiratory pressure, cm
H2O
Oxygen 9295 9196 9094 9195
saturation, %
Peak 2225 2528 3035 4245
inspiratory
pressure, cm
H2O
Inspiratory 1518 1519 1618 1518
plateau
pressure, cm
H2O
RR per min 1820 2024 2226 2432

Which of the following is the most likely reason for the respiratory arrest?

A. Flash pulmonary edema


B. Endotracheal tube obstruction
C. Tension pneumothorax
D. Pulmonary embolus

11. A 50-year-old woman intubated 7 days for pneumonia and severe sepsis is being evaluated for weaning from mechanical ventilation. Current
ventilator settings include volume-assist mode with a tidal volume of 450 mL, rate of 12/min, FIO2 of 0.35, and positive end-expiratory pressure of 5
cm H20. She has stable vital signs off sedative or vasoactive drips and is awake and able to follow simple commands. Her hand grip is weak
bilaterally, lungs are clear, and heart rate is regular with no murmurs or gallop. She is placed on continuous positive airway pressure for 3 minutes,
during which time her RR is 25/min with an average tidal volume of 350 mL, and a negative inspiratory force measured at 30 cm H20. An arterial
blood gas study is pending.

Which of the following is the most appropriate course of action regarding this patient's weaning from mechanical ventilation?

A. Patient should not be extubated as she is likely to fail extubation.


B. Patient should be placed on pressure support ventilation of 5 cm H2O for 120 minutes.
C. Patient should be extubated at this point without further action.
D. Patient should be extubated and placed on noninvasive ventilation.
E. Patient should be extubated based on arterial blood gas results.

12. A 75 year-old man with hypertension, chronic obstructive pulmonary disease (COPD), and type 2 diabetes was admitted to the ICU with pneumonia
and has been on mechanical ventilation for 12 days. He has completed a course of antibiotics and has also received aggressive treatment for COPD
with albuterol, ipratropium, and steroids. Current ventilator settings are assist-control mode with tidal volume of 400 mL, rate of 14/min, FIO2 of
0.30, and positive end-expiratory pressure of 5 cm H2O. His oxygen saturation is 90%92% and his peak airway pressure is 12 cm H2O. On these
settings, HR is 88/min, BP is 128/ 72 mm Hg, and RR is 20/min. On examination, he is awake and able to follow commands. He has generalized
decreased muscle strength (upper extremities 4+/5 and lower extremities 4/5). Lungs are clear to auscultation (with prolonged expiratory phase),
and cardiac examination findings are unremarkable. Spontaneous breathing trials have failed on previous days but the patient is now more awake
and cooperative. He is placed on pressure support of +5 cm H2O and after 40 minutes seems to develop increased distress and anxiety. HR is
108/min, BP is 140/86 mm Hg, and RR is 30/min. Arterial blood gases show a pH of 7.35, PaCO2 of 40 mm Hg, PaO2 of 58 mm Hg, and an SaO2 of
88%. Blood work sent at this time shows normal electrolyte level and a serum protein level of 7 mg/dL (serum protein level drawn earlier in the
day was 5.5 mg/dL). An ECG done during the weaning trial shows sinus tachycardia with no ST-segment changes.

Which of the following is the most likely cause of this patient's failure to wean from the ventilator?
A. Acute delirium
B. Increased airway resistance
C. Cardiac dysfunction
D. Inspiratory muscle fatigue
E. Endocrine dysfunction

13. A 46-year-old woman is admitted to the ICU from the floor after developing respiratory distress with increased work of breathing and increased
oxygen requirement. The patient has history of systemic lupus erythematosus (SLE) and diabetes. She was admitted 2 days ago with fever and
dyspnea. On admission to the hospital, significant laboratory data included: WBC count of 12,000/L, hemoglobin level of 11 g/dL, platelet count
of 110,000/L, and creatinine level of 1.3 mg/dL. Initial chest radiograph showed a possible consolidation versus atelectasis at the right lung base.
The patient was admitted for further workup and started on antibiotics for possible infection. Temperature is 37.3C (99.1F), HR is 120/min, RR
is 32/ min, and BP is 155/89 mm Hg. Arterial blood gas findings obtained prior to intubation show a pH of 7.32, PaO2 of 58 mmHg, PaCO2 of 48 mm
Hg, and hemoglobin level of 7.4 g/dL. Chest radiograph is shown in the Figure 13.

Figure 13
Which of the following interventions would be most appropriate as initial therapy for this patient?
A. Trimethoprim/sulfamethoxazole
B. Furosemide
C. Methylprednisolone
D. Factor VIIa
E. Rituximab

14. A 28-year old man presents to the emergency department with worsening shortness of breath and a cough productive of blood-tinged sputum and
lasting 2 days. He denies fever, chills, or chest pain but admits to shortness of breath while jogging on the day of presentation. He denies injection
drug use or tuberculosis exposure. Patient smokes 1 pack of cigarettes a day. Patient has no past medical problems and recently had a normal
physical examination with normal blood findings at work. Physical examination reveals an anxious young male in respiratory distress; temperature
is 37.3C (99F), RR is 28/min, HR is 110/ min, BP is 138/70 mm Hg, and SpO2 is 87%. There are no petechiae or ecchymoses; cardiac
examination shows tachycardia with normal S1 and S2, and no murmurs or gallops. Lung auscultation reveals crackles over both bases; abdomen is
soft, nontender, and nondistended, with no organomegaly. Chest radiograph reveals bilateral patchy alveolar infiltrates. Additional testing reveals
hemoglobin level of 10 g/dL, WBC count of 10,800/L with normal differential, platelet count of 450,000/L, and serum creatinine level of 3.5
mg/dL. Urinalysis shows 10 RBCs, RBC casts, proteinuria, and no bacteria.

The patient develops worsening respiratory distress with hypoxemia and is intubated prior to transfer to the ICU.

Which of the following diagnostic tests is most likely to influence the therapeutic approach to this patient's underlying condition?
A. Bronchoscopy with bronchoalveolar lavage
B. CT angiogram of the chest
C. Antineutrophil cytoplasmic autoantibody titer
D. Antibasement membrane antibody titer
E. Bone marrow biopsy

15. A 21-year-old man is transferred to the ICU with an exacerbation of his asthma. He was recently admitted to the hospital after a motor vehicle
collision that resulted in cervical spine instability that required immobility using a halo-vest. From reviewing his chart, it is clear that he frequently
visits the emergency department for acute respiratory distress. During the past 6 months, he has been treated for 5 acute asthma exacerbations, and
once required mechanical ventilation. His asthma appears to be refractory since he has poor response to bronchodilators and steroids; he has
persistent wheezing and increased work of breathing.

On examination, the patient is anxious, with tachypnea and audible wheezing. In the ICU initially, temperature is 37.5C (99.4F); HR is 110/min;
RR is 25/min; BP is 134/78 mm Hg; and SpO2 is 100% on nonrebreather mask. Arterial blood gas results show pH of 7.30, PaCO2 of 50 mm Hg, and
PaO2 of 90 mm Hg.

Because of his cervical spine immobility, the anesthesiologist decides to intubate the patient awake with fiberoptics.

Which of the following findings is most likely observed during the intubation?
A. Bilateral true vocal cord fixation
B. Unilateral vocal cord paralysis
C. True vocal cord abduction on inspiration, and partial adduction during expiration
D. True vocal cord adduction during inspiration
16. A 76-year-old man presents to the emergency department with worsening shortness of breath. Medical history is significant for chronic obstructive
pulmonary disease on home oxygen and pepticulcer disease with a recent episode of upper gastrointestinal bleeding. In the emergency department,
his respiratory status quickly deteriorates and he requires intubation. The patient is admitted to the ICU with a diagnosis of respiratory failure
secondary to community-acquired pneumonia. After 2 weeks of mechanical ventilation, a tracheostomy is performed because the patient is not
making progress towards weaning from mechanical ventilation. He is transferred to a chronic ventilatory unit in the hospital for weaning. Twelve
days after his tracheostomy, his secretions have become bloody. Evaluation reveals stable hemodynamics with no change in hemoglobin or
hematocrit. Respiratory and nursing staff are asked to limit suctioning through the tracheostomy to avoid traumatic bleeding. Two days later, the
patient has massive bleeding through his tracheostomy. Despite rapid and aggressive measures to control the hemorrhage, the patient quickly
exsanguinates and dies.

Which of the following is the most likely diagnosis?

A. Pulmonary embolism with lung infarction


B. Trauma from overaggressive suctioning
C. Trachea-innominate artery fistula
D. Pulmonary alveolar hemorrhage
E. Upper gastrointestinal hemorrhage

17. A 60 year-old man presents to the emergency department with acute-onset shortness of breath and chest pain. The patient has a recent diagnosis of
colon cancer and had a colectomy 8 days ago. He has a medical history of hypertension and type 2 diabetes mellitus. On physical examination, he is
alert with mild respiratory discomfort, temperature of 37.9C (100.1F), HR of 117/min, RR of 22/min, and BP of 135/84 mm Hg. Patient weighs
120 kg (264 lbs) and his body mass index is 32. Oxygen saturation on room air is 90% and is 96% with 3 L/min of supplemental oxygen. Lung
auscultation findings are normal, and cardiac examination reveals regular tachycardia with normal S1 and S2 and no murmurs or gallops. Abdomen
is soft and nontender with a clean surgical scar. ECG shows sinus tachycardia with no signs of cardiac ischemia. Chest radiograph is shown in the
Figure. Serum creatinine concentration is 2.5 mg/dL (was 1.2 mg/dL before surgery).

Which of the following is the most appropriate initial management?


A. CT angiography prior to initiating treatment
B. IV unfractioned heparin, 80-U/kg-bolus followed by 18-U/kg/h IV (based on ideal body weight)
C. Subcutaneous enoxaparin, 120 mg/kg every 12 hours
D. IV argatroban, 2 mg/kg/min
E. IV unfractioned heparin, 9,600-U bolus followed by 2,100 U/h

18. A 48-year-old woman presents to the emergency department with acute-onset shortness of breath and chest pain. She has a history of breast cancer
and is currently undergoing chemotherapy. On examination, temperature is 37.2C (98.9F), HR is 120/min, RR is 22/min, and BP is 110/68 mm
Hg. CT angiography shows bilateral filling defects consistent with pulmonary embolism. A bedside echocardiogram reveals a dilated right
ventricle with decreased contractility (free-wall hypokinesis) and paradoxical movement of the interventricular septum. The intensivist is consulted
to determine if ICU care is warranted.
Which of the following management decisions should be instituted based on the findings on the echocardiogram?
A. Lower extremity ultrasonography before further therapeutic decisions
B. Systemic thrombolytic therapy
C. Admission to the ICU and treatment with unfractionated heparin
D. No ICU admission, but inferior vena cava filter placement

19. Which of the following statements should guide the utilization of low-molecular-weight heparins (LMWHs) in the prophylaxis and treatment of
pulmonary embolism (PE) in critically ill patients?
A. LMWH is contraindicated in critically ill patients with renal failure.
B. LMWH is superior to unfractioned heparin in treating PE.
C. LMWH can be utilized in critically ill patients suspected of having developed heparin induced thrombocytopenia.
D. Subcutaneous LMWH administration in surgical critically ill patients can result in lower plasma drug levels.
E. LMWHs are not indicated for critically ill patients with PE.

20. A 52-year-old man is admitted to the ICU from the emergency department with a diagnosis of acute pulmonary embolism. The patient is intubated
and receiving unfractioned heparin, 1,000 U/h as a continuous infusion on arrival to the ICU. Initial examination reveals a mildly sedated patient
with a temperature of 37.9C (100.1F), HR of 110/min, RR of 20/min, and BP of 135/82 mm Hg. Shortly after arrival, you are called to the
bedside due to hypotension. The patient's HR is 125/min and BP is 88/54 mm Hg. Findings of focused ultrasonography done at the bedside are
shown in the Figure.

Which of the following therapeutic interventions is most appropriate?

A. Hold heparin and perform pericardiocentesis.


B. Administer bolus of 0.9% normal saline.
C. Initiate norepinephrine infusion.
D. Initiate systemic thrombolyitics.

21. A 35-year-old woman is transferred intubated to your ICU from an outside hospital with respiratory failure. The patient presented to the outside
hospital yesterday with a 3-day history of high fevers, myalgia, and increased shortness of breath. She has no significant past medical history. She
was initially treated for a possible bacterial pneumonia and for influenza but developed progressive hypoxemia with increased oxygen
requirements.

On arrival to the ICU, she is sedated with propofol. On physical examination, her temperature is 38.1C (100.5F), HR is 108/min, BP is 128/78
mm Hg, and RR is 26/min. The patient weighs 100 kg (220 lbs), height is 157 cm (5 ft 2 in), and body mass index is 41. She responds to stimulation.
Lung exam has coarse breath sounds bilaterally, cardiac examination reveals tachycardia with regular rate and normal heart sounds, and the
abdomen is soft and nontender. Chest radiograph from the outside hospital is shown in the Figure.
Which of the following are the most appropriate initial ventilator settings for this patient?

A. Pressure-controlled ventilation with tidal volume of 600 mL; RR, 18/min; FIO2, 100%; positive end-expiratory pressure (PEEP), 12 cm H2O
B. Assist-control with tidal volume of 400 mL; RR, 20/min, FIO2, 100%; PEEP, 8 cm H2O
C. High-frequency oscillatory ventilation with frequency of 6 Hz; amplitude, 70 cm H2O; peak airway pressure, 30 cm H2O; FIO2, 100%;
inspiratory time, 33%; bias flow, 40 L/min
D. Airway pressure release ventilation, with Thigh of 5 seconds; Tlow, 0.6 seconds; Phigh, 30 cm H2O; Plow, 0 cm H2O; FIO2 100%

22. A 44-year-old woman is admitted to the ICU from the emergency department, where she presented with severe acute pancreatitis. She required
intubation and seems to have developed acute respiratory distress syndrome (ARDS). On physical examination, she is intubated and sedated.
Examination of the lungs reveals diffuse crackles and rhonchi, cardiac examination reveals regular tachycardia with normal heart sounds, and her
abdomen is very tender and has decreased bowel sounds. Chest radiograph shows diffuse bilateral infiltrates. Initially, the patient had a good
oxygen saturation on a FIO2 of 50% and positive end-expiratory pressure of 8 cm H2O. Now on arrival to the ICU her oxygen saturation has dropped
and her FIO2 has been increased to 100%. A pressure-volume curve is shown in the Figure.

Please select the point on this pressure-volume curve that would be most appropriate as the initial target to set PEEP in this patient.

A. Point A
B. Point B
C. Point C
D. Point D

23. A patient with acute respiratory failure is in the ICU on mechanical ventilation. The graph of airway pressure over time for this patient's current
mechanical ventilation settings is shown below (figure 23).
Figure 23. PAW = Airway pressure in cm H2O

Which of the following changes would be most appropriate to correct a high carbon dioxide level?
A. Increase Thigh
B. Increase frequency by 12 Hz
C. Increase Tlow
D. Increase amplitude (Power) by 510 cm H2O
E. Decrease Phigh

24. 61-year-old man with a history of myasthenia gravis is admitted to the ICU for progressive dyspnea and weakness. The patient's wife reports that he
stopped taking his medications several days ago. On admission to the ICU, the patient has shallow breaths and difficulty speaking. Lung examination
findings are normal, with clear breath sounds on both sides. A chest radiograph shows no infiltrates. Bedside respiratory testing shows a vital
capacity of 1.6 L (40% of the predicted value) and a maximum negative inspiratory force of 15 cm H2O. Treatment with pyridostigmine and
steroids is initiated. During the next day, his overall weakness remains unchanged. During morning rounds, you notice that he is having difficulty
with secretions and the nurse reports that he has been coughing when given water with his pills.

Which of the following courses of action is most appropriate for the management of this patient at this point?
A. Order the patient to be kept fasting and administer all medications intravenously.
B. Continue to monitor vital capacity and intubate if it drops below 1.0 L.
C. Initiate continuous positive airway pressure via face mask.
D. Initiate bilevel positive airway pressure ventilation via nasal mask.
E. Orally intubate the patient and initiate mechanical ventilation.

25. A 52-year-old man is evaluated in the emergency department for a 3-day history of increased shortness of breath with cough productive of yellow
sputum. The patient has a history of chronic obstructive pulmonary disease (on home oxygen) and continues to smoke 2 packs of cigarettes a day.

On examination, his temperature is 38.1C (100.5F), HR is 110/min, BP is 132/84 mm Hg, and RR is 28/min. The patient has a prolonged
expiratory phase on auscultation, with crackles in the left lower lung field and intercostal retractions during respiration. His cardiac examination
findings are significant for tachycardia with normal heart sounds and no murmurs or gallop. Patient is awake and neurologic examination is
nonfocal. WBC count is 18,000/L with 10% bands. On 2 L/min of oxygen via nasal cannula, arterial blood gas (ABG) results show pH of 7.26,
PaO2 of 58 mm Hg, and PaCO2 of 65 mm Hg. Therapy with IV methyprednisolone, nebulized albuterol/ipratropium, and IV levofloxacin is initiated.
The patient is started on noninvasive positive-pressure ventilation with an FIO2 of 70%, inspiratory airway pressure (IPAP) of 10 cm H2O, and an
expiratory airway pressure (EPAP) of 5 cm H2O.

The patient is admitted to the ICU. Approximately 2 hours after arrival, the patient is more lethargic, although he seems to respond to stimulation.
Temperature is 37.3C (99F), HR is 128/min, BP is 88/54 mm Hg, and RR is 32/min. ECG shows what seems to be multifocal tachycardia. Repeat
ABG results show pH of 7.24, PaO2 of 60 mm Hg, and PaCO2 of 68 mm Hg. The physician orders a bolus of IV fluids.

Which of the following is the most appropriate next step in this patient's management?
A. Increase IPAP to 14 cm H2O.
B. Increase EPAP to 8 cm H2O.
C. Intubate and begin mechanical ventilation.
D. Increase IPAP and EPAP by 2 cm H2O.
E. Continue current settings and recheck ABG in 3060 minutes.

26. A 29-year-old woman with a history of asthma presents to the emergency department in respiratory distress with a 2-day history of increasing
shortness of breath after a febrile illness. The patient states that, despite using her routine albuterol and inhaled steroids, her symptoms have
worsened. Her temperature is 37.3C (99F), HR is 110/min, RR is 34/min, and BP is 130/78 mm Hg. The patient weighs 60 kg (132 lbs).
Auscultation of the chest reveals diffuse wheezing with decreased breath sounds. Chest radiograph shows hyperinflated lungs with no evidence of
infiltrates. An arterial blood gas (ABG) study on 4 L/min of oxygen shows pH of 7.38, PaO2 of 79 mm Hg, and PaCO2 of 45 mm Hg. She is started on
continuous albuterol nebulizer and given IV methylprednisolone. Approximately 1 hour later, she is more lethargic, and after a repeat ABG shows
pH of 7.31, PaO2 of 70 mm Hg, and PaCO2 of 52 mm Hg, she is intubated.

On arrival to the ICU, the patient is heavily sedated and paralyzed. Ventilator settings are assist-control mode with tidal volume of 600 mL, RR of
16/min, FIO2 of 60%, and positive end-expiratory pressure (PEEP) of 5 cm H2O. With these settings, the peak airway pressure is 55 cm H2O and an
ABG shows pH of 7.37, PaO2 of 90 mm Hg, and PaCO2 of 46 mm Hg. Her vital signs are stable. Treatment with inhaled albuterol/ipratropium and IV
steroids is continued. One hour later, when an ABG shows pH of 7.33, PaO2 of 70 mm Hg, and PaCO2 of 50 mm Hg, the physician on call increases
the RR to 24/min. An ABG done after the ventilator changes shows pH of 7.32, PaO2 of 70 mm Hg, and PaCO2 of 46 mm Hg, but the patient becomes
hypotensive and tachycardic (BP is 72/48 mm Hg and HR is 138/min). Physical examination reveals a midline trachea and bilateral hyper-
resonance of the chest.

Which of the following interventions is most appropriate as the next step in this patient's management?
A. Administer a bolus of IV fluids.
B. Perform bilateral needle decompression of the chest.
C. Decrease respiratory rate on ventilator.
D. Disconnect patient from ventilator.

27. A 28-year-old woman with no previous medical history presents to the emergency department with a 2-day history of lower right abdominal pain
that has progressively increased and today has been associated with nausea and vomiting. A right ovarian torsion is diagnosed. The patient
undergoes a right salpingo-oophorectomy under general anesthesia that is uneventful. Approximately 1 hour after admission to the postoperative
care unit, the patient reports increased shortness of breath and coughs frothy, blood-tinged sputum. She is started on supplemental oxygen, 4 L/min,
to keep her SaO2 above 90%. On examination, the patient has bilateral crackles on auscultation and mild tachycardia. A portable chest radiography
reveals a normal cardiac silhouette with diffuse bilateral infiltrates.

Which of the following interventions is most appropriate for this patient's condition?
A. Supplemental oxygen and supportive care
B. Levofloxacin and metronidazole
C. Methylprednisolone
D. Furosemide

28. A 25-year-old man with asthma that is managed with albuterol and inhaled steroids is admitted to the hospital after a 3-day history of increasing
shortness of breath, coughing, and wheezing. He has had multiple asthma exacerbations during the past year and has taken a short course of systemic
corticosteroids. He admits to poor adherence with his asthma regimen when he is feeling well. Initial assessment in the emergency department
reveals severe bronchospasm and tachypnea. After aggressive treatment with albuterol/ipratropium, magnesium, and corticosteroids, the patient
shows improvement and is admitted to the medical floors. He is continued on IV corticosteroids, albuterol/ipratropium inhalers, and levofloxacin.
Several hours after admission, he starts having increased respiratory distress unresponsive to bronchodilators and is transferred to the ICU, where
he is intubated and started on mechanical ventilation. Because of refractory bronchospasm, he requires sedation and neuromuscular paralysis. On
day 3 in the ICU, he develops new fevers. Chest radiography shows diffuse alveolar infiltrates. Nondirected bronchial aspirates are obtained and
sent for Gram stain and culture. Antibiotic coverage is expanded to vancomycin and cefepime. On day 4, the lab reports that the Gram stain from the
sputum has gram-positive, dichotomously branching hyphae. Clinically, the patient has persistent fevers and increased oxygen requirements.

Which of the following is most appropriate for the management of this patient based on the available information?
A. Change vancomycin to linezolid.
B. Initiate fluconazole.
C. Initiate voriconazole.
D. Initiate trimethoprim/sulfamethoxazole.
E. Keep current antibiotic regimen pending cultures.
29. You are part of the critical care rapid response team called to the general medical floor to evaluate a 41-year-old woman with acute shortness of
breath. The patient has a history of sickle cell disease and was admitted 4 days ago with upper extremity and flank pain reminiscent of previous
vaso-occlusive sickle cell crisis. Temperature is 38.4C (101F), HR is 110/min, RR is 38/min, BP is 165/ 92 mm Hg, and oxygen saturation is
74%. On examination, the patient is in distress from labored breathing. Lung auscultation reveals diffuse crackles with expiratory wheezes, and an
emergent portable chest radiograph shows new diffuse, bilateral pulmonary infiltrates. Bedside focused ultrasonography of the heart reveals a
dilated right ventricle with decreased right ventricular contractility, normal left ventricular contractility, and no pericardial effusions or regional
wall motion abnormalities. The patient is intubated after becoming lethargic and transferred to the ICU.

Which of the following interventions is most likely to produce rapid improvement in the patient's oxygenation?
A. IV heparin infusion
B. Broad-spectrum antibiotics
C. Blood transfusion
D. Corticosteroids
E. Furosemide

30. You have completed placement of a left subclavian central venous catheter under ultrasound guidance in a mechanically ventilated patient. You plan
to perform ultrasonography of the lung to determine if there is a pneumothorax after your procedure. Which of the following ultrasonography
findings would be most useful in evaluation of the lung for a potential pneumothorax?

A. Identification of the lung point


B. Absence of pleural sliding
C. Presence of A-lines
D. Absence of B-lines

31. A 32-year-old woman is admitted to the ICU with severe respiratory failure due to H1N1 influenza with superimposed bacterial pneumonia. She is
placed on high-frequency oscillatory ventilation because of refractory hypoxemia on conventional ventilation. Her current settings are as follows:
FIO2, 60%; frequency, 6 Hz; amplitude, 68 cm H2O; airway pressure, 29 cm H2O; inspiratory time, 33%; and bias flow, 35 L/min. Arterial blood gas
results show a pH of 7.13, PaO2 of 70mm Hg, and PaCO2 of 77 mmHg.

Which of the following interventions would be most appropriate to address the increased PaCO2 ?
A. Decrease frequency to 5 Hz
B. Increase frequency to 7 Hz
C. Decrease power by 5 cm H2O
D. Increase mean airway pressure to 34 cm H2O

32. A 24-year-old man develops severe respiratory failure with acute respiratory distress syndrome after a near-drowning episode. Because of
refractory hypoxemia requiring high positive end-expiratory pressure (PEEP) and FIO2 of 100% on pressure-control ventilation, he is placed on
high-frequency oscillatory ventilation (HFOV) as salvage therapy. The patient tolerates HFOV well for the first 24 hours. Oxygenation improves
and the current FIO2 is down to 60%. However, during the past hour there has been an increase in the amplitude from 70 to 85 cm H2O in order to
maintain a good chest wiggle going to the mid-thighs.

Which of the following interventions is most likely to improve the increased amplitude requirements?
A. Bilateral chest tubes
B. Suctioning of respiratory secretions
C. Increased level of sedation
D. Deflation of endotracheal cuff
E. Disconnecting the patient from the ventilator
33. A 44-year-old man admitted to the ICU with severe acute respiratory distress syndrome develops refractory hypoxemia that is difficult to correct
with conventional mechanical ventilation. Salvage therapy with high-frequency oscillatory ventilation (HFOV) is initiated.

Which of the following parameters should be increased in order to improve oxygenation and wean down FIO2 in this patient on HFOV?
A. Amplitude
B. Frequency
C. Power
D. Bias flow

34. You are consulted regarding a 37-year-old man who presents to the emergency department with increased dyspnea. As part of your initial
evaluation, you perform a bedside ultrasonography of his right chest (see Figure).

Which of the following can be diagnosed from this ultrasonogram?

A. Pneumothorax
B. Chest mass with diaphragm invagination
C. Pleural effusion
D. Pericardial effusion
E. Diaphragmatic paralysis

35. A 60-year-old, active man with known idiopathic pulmonary fibrosis (IPF) is admitted to the hospital with rapidly increasing shortness of breath.
Although his IPF has been stable over the last 2-and-a-half years, he now has had increasing dyspnea during the past 2 weeks, with chest CT
showing the rapid progression of his disease. During his hospital stay, the patient has a bronchoscopy with no growth of organisms on his
broncheoalveolar lavage. CT angiogram shows no evidence of pulmonary embolism and echocardiogram shows normal ejection fraction with no
valvular abnormalities. The patient has a mildly elevated serum brain natriuretic peptide (BNP) level.

Two days after admission, the patient is somnolent and hypoxic. His family produces his living will, which says the patient would not like artificial
life support if no easily reversible cause of respiratory failure is behind his clinical deterioration or if he could not get a lung transplant.

Which of the following is most appropriate at this point?


A. Intubate the patient and treat him for pulmonary edema because of his elevated BNP level.
B. Intubate the patient and transfer him to a tertiary care facility that offers lung transplant.
C. Offer the patient and his family hospice because of his acute exacerbation of idiopathic pulmonary fibrosis with known high mortality.
D. Intubate the patient and treat him with high-dose steroids.
E. Start the patient on noninvasive ventilation and antibiotics for possible subclinical infection.
PART 3: Respiratory Critical Care

ANSWERS:

1C; 2D; 3A; 4B; 5C; 6C; 7E; 8A; 9C; 10B; 11B; 12C; 13C; 14D; 15D; 16C; 17E; 18C; 19D; 20B; 21B; 22A; 23C; 24E;
25C; 26D; 27A; 28C; 29C; 30A; 31A; 32B; 33D; 34C; 35C

RATIONALE (1) Answer: C

The goals during the intubation of this patient are to prevent pulmonary aspiration, maintain cervical spine stability, and minimize the risk of hypercapnia
and hypoxemia. Because the patient was admitted after a trauma, a full stomach must be assumed. Fiberoptic-guided intubation requires time to
anesthetize the posterior pharynx with local anesthesia and will also blunt any airway reflexes in response to ventilation. Mask ventilation is not
indicated in a patient with a full stomach unless it is used as a rescue for a failed intubation. Although the patient's neurologic status has declined, it is
unlikely that direct intubation without the use of sedation would be possible.

Cervical collars often prevent the use of a jaw-thrust maneuver that can aid in the visualization of the vocal cords. With in-line manual stabilization, the
stability of the cervical spine can be maintained so that the cervical collar can be removed, facilitating the endotracheal intubation.

REFERENCES (1)

1. Forster N, Engelhard K. Managing elevated intracranial pressure. Curr Opin Anaesthesiol 2004;17:371376.
2. Jaber S, Amraoui J, Lefrant JY, et al. Clinical practice and risk factors for immediate complications of endotracheal intubation in the intensive care
unit: a prospective, multiple-center study. Crit Care Med 2006;34:23552361.
3. Practice guidelines for management of the difficult airway: an updated report by the American Society of Anesthesiologists Anesth 2003;98:1269
1277.
4. Rosen P, Sloane C, Ban KM, et al. Difficult airway management. Intern Emerg Med 2006;1:139147.
5. Shearer V. Modern airway management for the trauma patient. Curr Opin Anaesthesiol 2000; 13:135139.
6. Strawn JR, Keck PE, Caroff SN. Neuroleptic malignant syndrome. Am J Psychiatry 2007;164:870876.
7. Walls RM. Management of the difficult airway in the trauma patient. Emerg Med Clin NA 1998; 16:4561.

RATIONALE (2) Answer: D

Angioedema that can cause a rapid and progressive airway compromise can occur with the use of angiotensin-converting enzyme (ACE) inhibitors. ACE
inhibitors cause 25%39% of nonhereditary angioedemas. It has been suggested that the incidence of angioedema with lisinopril is greater than with the
other ACE inhibitors. The exact pathophysiologic mechanism is unknown, but it may involve inhibition of kinase activity with the accumulation of tissue
mediators. Up to 20% of patients may present with acute dyspnea, dysphagia, dysphonia, and stridor. As many as 22% of patients with angioedema
induced by ACE inhibitors require airway intervention, which may include cricothyrotomy if attempts at direct visualization of the larynx fail. Once the
airway is secured, the angioedema is self-limiting and resolves. There is no evidence that pharmacologic intervention effectively treats this type of
angioedema.

REFERENCES (2)

1. Agah R, Bandi V, Guntupalli II. Angioedema: the role of ACE inhibitors and factors associated with poor clinical outcome. Intensive Care Med
1997;23:793796.
2. Chin AG, Newkirk KA, Davidson BJ, et al. Angiotensinconverting enzyme inhibitor-induced angioedema: a multicenter review and an algorithm for
airway management. Ann Otol Rhinol Laryngol 2001;110:834840.
3. Sarkar P, Nicholson G, Hall G. Brief review: angiotensin converting enzyme inhibitors and angioedema: anesthetic implications. Can J Anesth
2006;53:9941003.

RATIONALE (3) Answer: A

Residual neuromuscular blockade is a common problem. All of the above choices should be considered when determining if a patient is suitable for
extubation. The only direct measurement of muscle strength is the assessment of twitch strength after motor nerve stimulation. The most common method
to accomplish this is train-of-four stimulation of the ulnar nerve at the wrist. With train-of-four stimulation, 4 stimuli at 2 Hz are repeated every 1012
seconds. A comparison of the strength of the first twitch (T1) of the adductor pollicis brevis muscle is made 2 seconds after the fourth twitch (T4). When
the strength of the first twitch is reduced to 90%, 80%, and 75% of the maximum twitch height in the absence of neuromuscular blockade, 1, 2, or 3
twitches will be elicited. At a T4 to T1 ratio of 0.75 (3 twitches), patients will manifest a 5-second head lift, achieve a vital capacity of 1520 mL/kg,
have an effective cough, and be able to generate an inspiratory force of 25 cm H2O. However, upper esophageal sphincter tone is reduced at T4 to T1
ratios less than 90%, with the risk of aspiration.

An inspiratory force of 20 cm H2O (option B) may be inadequate and is not a firm indicator for sufficient reversal of a neuromuscular blockade to
maintain a patent airway. An adequate tidal volume on pressure support ventilation of 16 cm H2O may be markedly reduced at lower levels of
ventilatory support. Thus, option C is incorrect. Patients may follow commands and have respiratory rates within normal ranges without complete return
of muscle function after neuromuscular blockade, so options D and E are also incorrect.

REFERENCES (3)

1. Ali HH. Criteria of adequate clinical recovery from neuromuscular block. Anesthesiology 2003;98:12781280.
2. Eriksson LI. Evidence-based practice and neuromuscular monitoring: it's time for routine quantitative assessment. Anesthesiology 2003;98:1037
1039.
3. Eriksson LI, Sundman E, Olsson R, et al. Functional assessment of the pharynx at rest and during swallowing in partially paralyzed humans:
simultaneous videomanometry and mechanomyography of awake human volunteers. Anesthesiology 1997;87:10351043.
4. Kopman A, Yee P, Neuman G. Relationship of the train-of-four fade ratio to clinical signs and symptoms of residual paralysis in awake volunteers.
Anesthesiology 1997;86:765761.
5. Pino RM. Residual neuromuscular blockade: a persistent clinical problem. Intl Anesth Clin 2006;44:7790.

RATIONALE (4) Answer: B

It is important to know the indications as well as the contraindications for noninvasive positive-pressure ventilation (NPPV), and the respiratory
conditions in which NPPV offers benefit. Patients with chonic obstructive pulmonary disease (COPD) exacerbations respond well to NPPV, with
decreased rates of intubation and improved mortality. Although depressed mental status and inability to cooperate are typical contraindications to
NPPV, the patient described in option B has only mild lethargy, which may be related to the increased carbon dioxide, partly from over-oxygenation
induced increase in dead space.

Recent studies also suggest that NPPV can be utilized effectively in selected patients with a depressed level of consciousness. Other contraindications to
NPPV are hemodynamic instability, as illustrated by the 55-year-old man with ST-elevation myocardial infarction and pulmonary edema (option A).
This patient is in cardiogenic shock with a metabolic acidosis and should be immediately intubated.

The inability to control secretions, vomiting, and hemoptysis (as in option D) preclude use of NPPV because of aspiration risk. NPPV is unlikely to
benefit the patient with mild disease, as in the asthmatic patient in option C. In asthma, NPPV should be reserved for patients with moderate to severe
asthma who fail to respond to initial therapy and who do not qualify for immediate intubation. Several studies have shown that NPPV does not prevent
reintubation in patients with postextubation respiratory failure (option E).

REFERENCES (4)

1. Caples SM, Gay PC. Noninvasive ventilation in the intensive care unit: a concise review. Crit Care Med 2005;33:26512658.
2. Diaz GG, Alcaraz AC, Talavera JCP, et al. Noninvasive positive-pressure ventilation to treat hypercapnic coma secondary to respiratory failure.
Chest 2005;127:952960.
3. Estaban A, Frutos-Vivar F, Ferguson N, et al. Noninvasive positive-pressure ventilation for respiratory failure after extubation. N Engl J Med
2004;350:24522460.
4. Keenan S, Powers C, McCormack DG, et al. Noninvasive positive-pressure ventilation for postextubation respiratory distress. JAMA
2002;287:32383244.
5. Liesching T, Kwok H, Hill NS. Acute applications of noninvasive positive pressure ventilation. Chest 2003;124:699713.
6. Scala R, Naldi M, Archinucci I, et al. Noninvasive positive pressure ventilation in patients with acute exacerbations of COPD and varying levels of
consciousness. Chest 2005;128:16571666.

RATIONALE (5) Answer: C

The patient in this question was initially placed on noninvasive positive pressure ventilation (NPPV) for treatment of an acute chronic obstructive
pulmonary disease (COPD) exacerbation. After initiation with appropriate settings, the patient develops hemodynamic changes that include a drop in
blood pressure, ECG changes suggesting ischemia, and a worsening clinical status as evidenced by increased lethargy and increased hypoxemia.
Although the institution of NPPV is often beneficial for patients with COPD exacerbations, it is important to recognize when to proceed with
endotracheal intubation and provision of conventional mechanical ventilator support. Studies have shown that delaying needed intubation can add to
patient morbidity and mortality.
Contraindications and factors that may indicate a high risk of failure with NPPV include the following: respiratory arrest, severe acid-base
abnormalities, medical instability of the patient (hemodynamic instability, uncontrolled gastrointestinal bleed, acute myocardial ischemia, uncontrolled
arrhythmias), inability to protect airway, excessive secretions, uncooperativeness or agitation, and recent upper airway or upper gastrointestinal surgery.

This patient shows clinical deterioration, with evidence of hemodynamic instability and myocardial ischemia. Therefore, he is not a candidate to
continue on NPPV and should be intubated immediately. In addition, predictors of failures of NPPV in patients with acute exacerbation of COPD include
RR greater than 35/min, Acute Physiology and Chronic Health Evaluation II (APACHE II) score greater than 29, and Glasgow Coma Scale score less
than 11.

REFERENCES (5)

1. British Thoracic Society Standards of Care Committee. Non-invasive ventilation in acute respiratory failure. Thorax 2002;57:192211.
2. Confalonieri M, Garuti G, Cattaruzza MS, et al. A chart of failure risk for noninvasive ventilation in patients with COPD exacerbation. Eur Respir J
2005;25:348355.
3. Keenan SP, Sinuff T, Cook DJ, et al. Which patients with acute exacerbation of chronic obstructive pulmonary disease benefit from noninvasive
positive-pressure ventilation? A systematic review of the literature. Ann Intern Med 2003;138:861870.
4. Lightowler JV, Wedzicha JA, Elliott MW, et al. Non-invasive positive pressure ventilation to treat respiratory failure resulting from exacerbations
of chronic obstructive pulmonary disease: Cochrane systematic review and metaanalysis. BMJ.2003;326:185.

RATIONALE (6) Answer: C

The Acute Respiratory Distress Syndrome (ARDS) Network clinical trial of low tidal volumes demonstrated significant improvements in mortality
when low-volume ventilation was utilized in patients with ARDS.1 In that study, patients were given a tidal volume of 6 mL/kg of predicted body weight
(PBW). The formulas for PBW are given below:

Male = 50 + 2.3 [height (inches) 60] or 50 + 0.91 [height (cm) 152.4]


Female = 45.5 + 2.3 [height (inches) 60] or 45.5 + 0.91 [height (cm) 152.4]

It is important to utilize the PBW when calculating tidal volume for patients with ARDS, because patients who are volume overloaded or who have a
high body mass index may have a much higher actual body weight than a PBW. In those cases, the use of an actual weight may provide deleterious tidal
volumes to the patient. In order to calculate the PBW, one must know the patient's height.

REFERENCES (6)

1. Acute Respiratory Distress Syndrome Network. Ventilation with lower tidal volumes compared with traditional tidal volumes for acute lung injury
and the acute respiratory distress syndrome. N Engl J Med 2002;342:13011308.
2. Kam E, Eslick G, James A, et al. Acute respiratory distress syndrome (ARDS) and low tidal volume ventilation: the debate about weight. Intensive
Care Med 2004;30:1502.

RATIONALE (7) Answer: E

This flow-over-time waveform demonstrates the presence of auto positive end-expiratory pressure (auto-PEEP), signified by significant expiratory flow
still occurring at end expiration (see Figure below, arrow 2). In addition, it demonstrates a dramatic drop in flow rate that occurs near the end of
expiration which is not associated with inspiratory triggering (see Figure, arrow 1). This occurs because the patient has made a significant inspiratory
effort, creating a negative intrapleural pressure and therefore decelerating the expiratory flow rate. The patient's effort, however, does not trigger the
next inspiration and subsequently subsides, followed by an unassisted ventilator triggering based on time interval from last inspiration.

Since the patient must first overcome the auto-PEEP before encountering the normal triggering mechanisms, the patient's work of breathing may be off set
to some degree by increasing the set ventilator PEEP, because the degree of inspiration effort to trigger is decreased as the difference between set PEEP
and total PEEP decreases. This occurs because the ventilator now triggers from the set PEEP level (plus some change in pressure or flow) instead of
from atmospheric pressure. Decreasing the peak inspiratory flow rate (option A) in a patient on volume ventilation would have no effect on this cause of
dyssynchrony, but in fact would worsen the auto-PEEP by prolonging inspiration. The inspiratory flow waveform is already set as decelerating flow, so
option B is incorrect. Switching to a square inspiratory flow waveform would, however, improve auto-PEEP, as it would shorten the inspiratory time.
Increasing the set ventilator rate to 20/min (option C) would also have no effect in this patient, as she is already breathing at a rate of 20/min. If the rate
was increased higher than 20/min, it would exacerbate the auto-PEEP by further decreasing expiratory time. Positioning the patient left side down would
have no predictable effect on this clinical problem, so option D is incorrect.
REFERENCES (7)

1. Aslanian P, El Atrous S, Isabey D, et al. Effects of flow triggering on breathing effort. Am J Respir Crit Care Med 1998;157:135143.
2. Chao DC, Scheinhorn DJ, Stearn-Hassenpflug M. Patient-ventilator trigger asynchrony in prolonged mechanical ventilation. Chest 1997;112:1592
1599.
3. Hess DR, Medoff BD. Mechanical ventilation of the patient with chronic obstructive pulmonary disease. Respir Care Clin North Am 1998;4:439
473.
4. Valta P, Corbeil C, Lavoie A, et al. Detection of expiratory flow limitation during mechanical ventilation. Am J Respir Crit Care 1994;150:1311
1317.

RATIONALE (8) Answer: A

The pressure-over-time waveform shown for this patient is typical for volume ventilation with rising pressure throughout inspiration. The waveform,
however, exhibits a decay in pressure after the onset of inspiration (see Figure, arrow A). This drop in pressure is related to significant inspiratory
effort occurring after inspiration is triggered. This typically occurs when a patient is not satisfied with the inspiratory flow rate setting.

This is particularly problematic in young, healthy individuals with good respiratory muscle strength who are in respiratory distress. Increasing the peak
inspiratory flow rate (PIFR) should accommodate the patient's desires and decrease dyssynchrony (see Figure, arrow B). Note that in addition to
smoothing out the inspiratory flow waveform, the peak inspiration pressure also rises with the higher rate. However, tidal volume and inspiratory
plateau pressure do not change.

Switching to a decelerating inspiratory flow waveform (option B) will likely worsen dyssynchrony, as the PIFR will steadily decrease after onset of
inspiration. Increasing the set ventilator rate (option C), unless it is increased higher then 30/min (current assisted rate), would have no effect on any
parameters, and if increased above 30/min, it would have no effect on dyssnychrony. Although this patient's positive end-expiratory pressure might need
to be increased, it is not a reason for the dyssynchrony, and with a FIO2 of 0.6 and oxyhemoglobin saturation of 96%, this is not a critical issue.
Therefore, option D is incorrect. Decreasing the positive end-expiratory pressure to less than 10 cm H2O with an FIO2 of 0.6 is probably not an
appropriate intervention. Thus, option E is incorrect.

REFERENCES (8)

1. Jubran A. Advances in respiratory monitoring during mechanical ventilation. Chest 1999;116:14161425.


2. Tobin MJ, Jubran A, Laghi F. Patient-ventilator interaction. Am J Respir Crit Care Med 2001;163:10591063.
3. Rau JL. Inspiratory flow patterns: The "shape" of ventilation. Respir Care 1993;38:132140.

RATIONALE (9) Answer: C

The arrow on the flow-time diagram shown in the Figure below indicates intrinsic positive end-expiratory pressure (or auto-PEEP).

Intrinsic PEEP is present when the end-expiratory lung volume represents a volume at which air would normally continue to escape if there was no
impediment to expiration. When the expiratory time is insufficient to allow full exhalation of a ventilator breath, expiratory flow is still occurring when
the next ventilator breath is delivered (arrow). Since the expiratory flow does not reach zero, there is also a pressure gradient at end-expiration.
Therefore, PEEP exists even when it is not set on the ventilator (or in excess of what is set on the ventilator). The presence or absence of intrinsic PEEP
may be confirmed by checking graphic flow waveforms on the ventilator.

Since severe bronchospasm increases the necessary expiratory time, patients with either status asthmaticus (as in this case scenario), or severe chronic
obstructive pulmonary disease exacerbation are at risk for intrinsic PEEP.

The patient in this case scenario should be ventilated with a strategy to limit intrinsic PEEP by decreasing inspiratory time relative to expiratory time.
This is best accomplished by decreasing the respiratory rate, giving the patient more time to exhale between breaths. Decreasing the tidal volume will
also result in a small improvement in intrinsic PEEP, but to a much lesser degree than decreasing the rate.

With volume-cycled ventilation, the inspiratory time is also dependent on the peak inspiratory flow rate, which determines how fast a breath will be
delivered. The choice of inspiratory flow waveform also influences the inspiratory to expiratory ratio. When providing volume-cycled ventilation in the
presence of a bronchospasm-induced intrinsic PEEP, the peak inspiratory flow rate should be set at 80100 L/min, with a square waveform in order to
limit the inspiratory time, leaving more time in the respiratory cycle for expiration.

However, it must be emphasized that the inspiratory to expiratory ratio should not be the primary goal in managing intrinsic PEEP. Rather, the chief
determinant of intrinsic PEEP is the absolute time for expiration of each breath. That is why decreasing the respiratory rate is the critical intervention.

REFERENCES (9)

1. Pepe PE, Marini JJ. Occult positive end-expiratory pressure in mechanically ventilated patients with airflow obstruction: the auto-PEEP effect. Am
Rev Respir Dis 1982;126:166170.
2. Stather DR, Stewart TE: Clinical review: Mechanical ventilation in severe asthma. Crit Care 2005;9:581587.

RATIONALE (10) Answer: B

This patient has an endotracheal tube obstruction. The key to the identification of this problem relates to an understanding of the components of peak
inspiratory pressure and inspiratory plateau pressure. Peak inspiratory pressure is created by the pressure required to overcome resistance in the
endotracheal tube and airways, as well as the elastance of the lung parenchyma, pleural space, and chest wall.

Inspiratory plateau pressure is pressure generated to overcome the elastance of the lung parenchyma, pleural space, and chest wall and is not related to
endotracheal tube and airways resistance. In this patient, it is noted that peak inspiratory pressure is steadily rising between day 1 and day 4, whereas
inspiratory plateau pressure remains constant. This is compatible with increasing endotracheal tube obstruction, in this case due to inspissated mucus.

Flash pulmonary edema and tension pneumothorax (options A and C, respectively) would be associated with increases in both peak inspiratory pressure
and inspiratory plateau pressure. Pulmonary embolus (option D) might not produce any changes in inspiratory pressure, but if it produced significant
parenchymal lung problems, it would be associated with the same pattern as flash pulmonary edema or tension pneumothorax.

REFERENCES (10)

1. Boque MC, Gualis B, Sandiumenge A, et al. Endotracheal tube intraluminal diameter narrowing after mechanical ventilation: use of acoustic
reflectometry. Intensive Care Med 2004;30:22042209.
2. Dieter RA Jr. Endotracheal tube obstruction, recognition, and management. Mil Med 1990;155:A11.
3. Guttmann J, Eberhard L, Haberthur C, et al. Detection of endotracheal tube obstruction by analysis of the expiratory flow signal. Intensive Care Med
1998;24:11631172.
4. Kawati R, Lattuada M, Sjostrand U, et al. Peak airway pressure increase is a late warning sign of partial endotracheal tube obstruction whereas
change in expiratory flow is an early warning sign. Anesth Analog 2005;100:889893.
5. Sprung J, Bourke DL, Harrison C, et al. Endotracheal tube and tracheobronchial obstruction as causes of hypoventilation with high inspiratory
pressures. Chest 1994;105:550552.
6. Stoen R, Smith-Erichesen N. Airway obstruction associated with an endotracheal tube. Intensive C are Med 1987;13:295296.

RATIONALE (11) Answer: B

Mechanical ventilation is an essential therapeutic intervention in patients with acute respiratory failure. However, there are several complications
associated with prolonged mechanical ventilation. Therefore, liberating patients from mechanical ventilation as their acute illness resolves is an
important challenge for intensivists. Our understanding of weaning from mechanical ventilation has evolved thanks to several clinical trials. Today it is
apparent that when assessing for readiness of weaning, clinicians should use liberal oxygenation criteria. Furthermore, the routine use of weaning
criteria (predictors) and arterial blood gas results is usually not required. In patients who appear to be ready for weaning, the rapid shallow breathing
index (RSBI = respiratory frequency tidal volume) is one weaning predictor that can quickly assess a patient's readiness. When done on continuous
positive airway pressure, it can identify patients who are unlikely to do well (RSBI >105). If it is less than 105, it does not predict success but suggests
that a patient should proceed to a spontaneous breathing trial (SBT). Based on results from clinical studies, SBTs are best done on pressure support
ventilation of 5 cm H2O or via T-piece and should last from 30 to 120 minutes. One study found that the median time to weaning trial failure in patients
with chronic obstructive pulmonary disease on mechanical ventilation for 15 or more days was 120 minutes. Based on this finding, it may be appropriate
in some patients to extend the SBT to 120 minutes. The patient in this case appears to be ready for an SBT. Based on the provided information, the best
option is to proceed with SBT on pressure support ventilation of 5 cm H2O for 120 minutes. If the patient tolerates this trial, she should be extubated.

REFERENCES (11)

1. Epstein, SK. Weaning from ventilatory support. Curr Opin Crit Care. 2009;15:3643.
2. Boles JM, BionJ, Connors A, et al. Weaning from mechanical ventilation. Eur Respi J. 2007;29:10331056.
3. Girard TD, Kress JP, Fuchs BD, et al. Efficacy and safety of a paired sedation and ventilator weaning protocol for mechanically ventilated patients
in intensive care (awakening and breathing controlled trial): a randomised controlled trial. Lancet. 2008;371:126134.

RATIONALE (12) Answer: C

About 20%30% of patients are difficult to wean from invasive mechanical ventilation. The pathophysiology leading to weaning failure is complex and
several causes contribute to difficult weaning. In this particular case, cardiac dysfunction is the most likely cause of the patient's failure to wean. The
transition from mechanical ventilation to spontaneous breathing can impose a significant strain on the cardiovascular system. Changes in intrathoracic
pressures during this transition produce changes in cardiac preload and afterload. In addition, the increased oxygen consumption of the respiratory
muscles can contribute to the increased strain on cardiovascular performance. In patients with chronic obstructive pulmonary disease, weaning from
mechanical ventilation can be associated with decreased left ventricular ejection fraction (probably due to increased afterload), and reduced myocardial
compliance. Elevated end-diastolic left ventricular pressures can produce bronchial wall edema and increase the work of breathing. Clinically, the
combined increase in arterial pressure and heart rate during unsuccessful weaning is quite suggestive of weaning failure from cardiac dysfunction.
Patients with cardiac dysfunction during weaning often have decreased mixed venous oxygen saturation (SVO2). A normal SVO2 with no changes during a
failed spontaneous weaning trial almost rules out cardiac dysfunction as the cause for the failure. Patients with cardiac dysfunction during weaning can
also show increased brain natriuretic peptide levels in blood. Signs of weaning-induced hemoconcentration, such as increased protein blood levels, are
highly suggestive of hydrostatic pulmonary edema. The ECG in this case did not show significant changes. However, ST changes and T-wave inversions
during a failed weaning trial suggest cardiac dysfunction/ischemia. This patient has various clinical signs suggesting weaning failure from cardiac
origin. Further evaluation can be done with echocardiography and/or pulmonary artery catheter during weaning trials. In some instances, evaluation for
possible underlying cardiac is chemia may be warranted. Treatment of cardiac dysfunction may include afterload reduction, inotropes, and in some
cases revascularization.

The patient shows no clinical evidence of acute delirium (option A is incorrect). Inspiratory muscle fatigue is extremely unlikely in a patient who fails a
weaning trial yet has a normal PaCO2 as in this case (option D is incorrect). Finally there are no clear clinical signs suggesting increased airway
resistance or endocrine dysfunction. However, in the absence of a strong suspicion for an alternative cause, the clinician should have a low threshold to
work up potential endocrine causes of weaning failure (hypothyroidism and adrenal insufficiency).

REFERENCES (12)

1. Heunks LM, van der Hoeven JG. Clinical review: the ABC of weaning failure--a structured approach. Crit Care. 2010;14(6):245.
2. Zapata L, Vera P, Roglan A, et al. B-type natriuretic peptides for prediction and diagnosis of weaning failure from cardiac origin. Intensive Care
Med. 2011 Mar;37:477485.
3. Teboul JL, Monnet X, Richard C. Weaning failure of cardiac origin: recent advances. Crit Care. 2010;14:211.

RATIONALE (13) Answer: C

This patient with known systemic lupus erythematosus (SLE) has developed a clinical picture most consistent with acute diffuse alveolar hemorrhage
(DAH). DAH is characterized by the clinical constellation of hemoptysis (not always present), anemia, diffuse radiographic pulmonary infiltrates, and
hypoxemic respiratory failure. DAH associated with SLE most often presents with a histological pattern of pulmonary capillaritis. In up to 30% of
patients, hemoptysis is absent. Diagnosis can be established with bronchoscopy, after sequential bronchoalveolar lavage reveals increasing red blood
cell counts or the presence of hemosiderin-laden macrophages. Methylprednisolone is recommended for the initial treatment of DAH related to SLE.
Pulse-dose steroids (methylprednisolone, 1 g in divided doses for 3 days) can be used to treat DAH or acute lupus pneumonitis (also a consideration in
this case). In patients not responding to steroids, the addition of cyclophosphamide or other treatments such as azathioprine or IV immunoglobulin should
be considered. In cases refractory to steroids, plasmapheresis has been used with limited success. Although patients with immunosupression could be at
risk for Pneumocystis carinii pneumonia, this does not seem a likely diagnosis in this patient; hence trimethoprim/sulfamethoxazole (option A) is
incorrect. The picture is not consistent with pulmonary edema, making furosemide incorrect (option B). Factor VIIa (option D) has been proposed as a
possible rescue therapy in patients with severe DAH due to microscopic polyangiitis with unremitting respiratory failure. However, it is not considered
a first-line therapy in patients with DAH and SLE. Finally, rituximab (a monoclonal antibody) has been utilized in patients with antineutrophil
cytoplasmic autoantibodyassociated vasculitis that is refractory to standard therapy. This patient was treated with methylprednisolone, 1 g/day for 3
days, with good clinical response (see figure 13b, radiograph taken 2 days after initiating steroids).

Figure 13b

REFERENCES (13)

1. Lara AR, Schwartz MI. Diffuse alveolar hemorrhage. CHEST 2010;137:11641171.


2. Betensley AD, Yankaskas JR. Factor VIIa for alveolar hemorrhage in microscopic polyangiitis. Am J Respir Crit Care Med. 2002;166:12911292.
3. Hoshi K, Matsuda M, Ishikawa M, et al. Successful treatment of fulminant pulmonary hemorrhage associated with systemic lupus erythematosus. Clin
Rheumatol. 2004;23:252255.
4. Stasi R, Stipa E, Del Poeta G, et al. Long-term observation of patients with antineutrophil cytoplasmic antibody-associated vasculitis treated with
rituximab. Rheumatology (Oxford). 2006;45:14321436.

RATIONALE (14) Answer: D

This patient's clinical picture is consistent with diffuse alveolar hemorrhage (DAH) and acute renal failure. In a previously healthy young male, this
presentation of an acute pulmonary-renal syndrome is highly suggestive of Goodpasture syndrome (antiglomerular basement membrane disease).
Goodpasture syndrome is a classic pulmonary-renal syndrome with high mortality if untreated. The disease only affects the lungs and the kidneys. DAH
is a common manifestation, especially in cigarette smokers. Patients with Goodpasture tend to be males in their 20s who also smoke. More than 90% of
patients with Goodpasture syndrome have circulating antibasement membrane. A positive titer for the antibasement membrane antibody would confirm
the diagnosis in this patient and would be most helpful in dictating therapy (option D is correct). Although a bronchoscopy with bronchoalveolar lavage
would help establish the diagnosis of alveolar hemorrhage, it would not provide a specific underlying diagnosis linked to therapy. Antineutrophil
cytoplasmic autoantibody titer (option C) is helpful in differentiating DAH due to vasculitis such as Wegener granulomatosis from microscopic
polyangiitis. However, the clinical presentation of this patient is not consistent with those diseases and clearly fits the profile of Goodpasture syndrome.
There is a small percentage of patients with Goodpasture syndrome in whom the antibasement membrane antibody titer may be negative. In these
patients, a renal or lung biopsy with immunofluorescence revealing linear antibody deposition within the alveolar or glomerular basement membrane
confirms the diagnosis. A bone marrow biopsy (option E) would not be helpful in the diagnosis of Goodpasture syndrome. Patients with Goodpasture
syndrome should be treated with immunosuppression (cyclophosphamide and steroids) and plasmapheresis. Early renal replacement therapy should also
be considered. With early dialysis, plasma exchange and immunosuppression, mortality decreases significantly.

REFERENCES (14)

1. Lara AR, Schwartz MI. Diffuse alveolar hemorrhage. CHEST 2010;137:11641171.


2. Lazor R, Bigay-Game L, Cottin V, et al; Groupe d'Etudes et de Recherche sur les Maladies Orphelines Pulmonaires (GERMOP): Swiss Group for
Intenstitial and Orphan Lung Diseases (SIOLD). Alveolar hemorrhage in anti-basement membrane antibody disease: a series of 28 cases. Medicine
(Baltimore). 2007;86:181193.
3. Peto P, Salama AD. Update on antiglomerular basement membrane disease. Current Opinion in Rheumatology 2011;23:3237.

RATIONALE (15) Answer: D

This patient most likely has vocal cord motion disorder or vocal cord dysfunction. This is a syndrome that commonly presents as asthma. During the
respiratory cycle, the vocal cords normally partially abduct with inhalation and partially adduct with exhalation. This physiological movement allows
the unimpeded movement of air inward to the lungs and outward to the atmosphere, while maintaining alveolar patency. In paradoxical vocal cord
motion disorder (PVCM) or vocal cord dysfunction, the vocal folds will adduct during inhalation and/or exhalation. This typically leads to episodic or
recurrent episodes of dyspnea, wheezing, and/or stridor. Because of these manifestations, PVCM is often confused with asthma and patients receive
steroid treatment without symptom relief. Rarely, this can lead to intubation and mechanical ventilation for "refractory asthma." The gold standard for
diagnosing PVCM is laryngoscopy.

REFERENCES (15)

1. Diamond E, Kane C, Dugan G. Presentation and evaluation of vocal cord dysfunction. Chest. 2000;118:199S.
2. Mathers-Schmidt BA. Paradoxical vocal fold motion: a tutorial on a complex disorder and the speech-language pathologist's role. American Journal
of Speech-Language Pathology. 2001;10:111125.
3. Ibrahim WH, Gheriani HA, Almohamed AA, et al. Paradoxical vocal cord motion disorder: past, present and future. Postgraduate Medical Journal.
2007;83:164172.

RATIONALE (16) Answer: C

Bleeding is one of the most important complications associated with percutaneous and surgical tracheostomies. Early bleeding, within days of the
procedure, is more likely caused by trauma during the procedure itself and usually resolves with minimal intervention aimed at correcting any existing
coagulopathy and proper local hemostasis. Bleeding can also result from more serious complications such as an arteriotracheal fistula. There are 2
different mechanisms by which a tracheostomy can lead to an arteriotracheal fistula. The first is erosion of the curved part of the tracheostomy through
the artery beneath it; the second is erosion by the tip or the cuff of the tracheostomy through the anterior wall of the trachea. Arteriotracheal fistulas
(tracheo-innominate artery fistula, tracheo-brachiocephalic artery fistula) are a rare but fatal complication, occurring in 0.7% of tracheostomies.
Mortality is attributed to acute hemorrhage, which classically occurs 24 weeks after the procedure. A so-called "sentinel bleed," a milder bleeding
episode that occurs prior to the acute hemorrhage, has also been described. It is important for clinicians to recognize the potential for devastating
bleeding complications in patients with tracheostomy. If bleeding is profuse and suspected to be from an arteriotracheal fistula, the only chance for
survival is immediate occlusion of the bleeding site (usually done by placing a finger directly on the bleeding site inside the trachea) enroute to the
operating room for emergent surgical repair.

REFERENCES (16)

1. Chapter 13 - Tracheal fistula to brachiocephalic artery. In: Grillo HC. Surgery of the Trachea and Bronchi. Hamilton, Ontario: B.C. Decker Editions;
2004.
2. Scalise P, Prunk SR, Healy D et al. The incidence of tracheoarterial fistula in patients with chronic thoracostomy tubes; a retrospective study of 544
patients in a long-term care facility. Chest. 2005;128:39063909.
3. Kapural L Sprung J, Gluncic I, et al. Tracheo-innominate artery fistula after tracheostomy. Anesth Analg. 1999;88:77780.

RATIONALE (17) Answer: E

The patient's clinical presentation is most consistent with acute pulmonary embolism (PE). The patient has significant risk factors for PE (recent surgical
intervention/hospitalization and cancer). Ideally, thromboembolic disease should be confirmed prior to initiating treatment. However, in situations with
a high clinical suspicion, it is appropriate to initiate anticoagulation if there are no clear contraindications pending a definitive diagnosis. CT
angiography is commonly the initial diagnostic test when pulmonary embolism is suspected. In this case, the increased creatinine would preclude
performing a CT angiography (choice A is incorrect). Low-molecular-weight heparin (LMWH) and unfractioned heparin are the first-line agents for
anticoagulation. Long-term anticoagulation with LMWH is preferred in cancer patients with thromboembolic disease. With renal failure, the dose of
enoxaparin should be reduced to 1 mg/kg every 24 hrs, so option C is incorrect. Argatroban is a direct thrombin inhibitor that is utilized in patients with
heparin-induced thrombocytopenia (HIT). There are no clinical data documenting efficacy of thrombin inhibitors as first-line therapy for PE and they
should not be utilized outside of the context of HIT (option D is incorrect). For this particular patient, the best option would be to initiate anticoagulation
with unfractioned heparin. Treatment with unfractioned heparin should be based on the patient's actual weight, not his ideal weight, so option B is
incorrect. The correct answer is option E, starting unfractionated heparin with a bolus of 80 U/kg (patient's weight of 120 kg = 9600-U bolus) followed
by 18 U/kg/h (patient's weight of 120 kg = approximately 2100 U/h). The dosing should be titrated to activated partial thromboplastin time or heparin
levels.

REFERENCES (17)

1. Carlbom DJ, Davidson BL. Pulmonary embolism in the critically ill. Chest. 2007;132:313324.
2. Goldhaber SZ. Advanced treatment strategies for acute pulmonary embolism, including thrombolysis and embolectomy. J Thromb Haemost. 2009
Jul;7 Suppl 1:322327.
RATIONALE (18) Answer: C

The results on the echocardiogram identify this patient as having a higher risk for complications and mortality. Echocardiography has an important role
in the management of patients with pulmonary embolism (PE). Patients with documented PE and echocardiographic findings consistent with right
ventricular strain/failure (right ventricular [RV] dilation, RV decreased contractility, abnormal movement of the interventricular septum into the left
ventricle) have a higher risk of death. Early identification of these patients can aid clinicians in triage decisions and prognostication. Studies have
shown that when patients with PE have echocardiographic signs of RV dysfunction, short-term, all-cause mortality and PE-related mortality increase
compared to patients without evidence of RV dysfunction (11% versus 0.5%). Triage to the ICU of high-risk patients with PE and RV dysfunction makes
sense if one considers the increased risk of death and potential decompensation. Although some have proposed that RV dysfunction could be a potential
indication for the use of systemic thrombolytics, there is no clinical trial data to support this practice (option B is incorrect). Similarly, findings of RV
dysfunction at present are not utilized in the decision process of placing inferior vena cava filters in patients with PE (option D is incorrect). Finally,
option A is incorrect because the decision to initiate anticoagulation is based on clinical suspicion and the findings of CT angiography. In the presence
of PE, echocardiographic findings of RV dysfunction are prognostic. However, RV dilation is not diagnostic of PE and can also occur in patients with
chronic pulmonary hypertension (cor pulmonale) from other causes.

REFERENCES (18)

1. Carlbom DJ, Davidson BL. Pulmonary embolism in the critically ill. Chest 2007;132:313324.
2. ten Wolde M, Shne M, Quack E, et al. Prognostic value of echocardiographically assessed right ventricular dysfunction in patients with pulmonary
embolism. Arch Intern Med 2004; 164:16851689.

RATIONALE (19) Answer: D

Proper anticoagulation is the basis for treatment in patients diagnosed with pulmonary embolism. Traditionally unfractioned heparin (UF) as a
continuous IV infusion has been utilized in critically ill patients with PE. Low-molecular-weight heparins (LMWHs) are an alternative to UF. In clinical
trials LMWHs have been shown to be non inferior to UH for treatment of deep vein thrombosis and PE. However, these studies did not include large
numbers of critically ill patients. Moreover, uncertainty exists regarding the subcutaneous dosing route for critically ill patients with PE. LMWHs are
administered via the subcutaneous route. Subcutaneous LMWH leads to lower plasma drug levels, measured as anti-factor Xa activity, in surgical
critically ill patients with or without shock when compared to non-ICU patients despite receiving the same dosing regimen (option D is correct). LMWH
have a lower incidence of causing heparin induced thrombocytopenia (HIT) than unfractioned heparin (UH). However, patients treated with UF with
probable or confirmed HIT should not be changed to LMWH (option C is incorrect). Both UF and LMWH should be avoided in these circumstances. If
there is a need for anticoagulation, patients with HIT should be changed to a thrombin inhibitor (argatroban or lepirudin). LMWH can be utilized in
patients with renal failure. In patients with a creatinine clearance < 30 ml the dose and frequency should be adjusted (option A is incorrect).

REFERENCES (19)

1. Goldhaber SZ. Advanced treatment strategies for acute pulmonary embolism, including thrombolysis and embolectomy. J Thromb Haemost. Jul
2009;7 Suppl 1:322-327.
2. Carlbom DJ, Davidson BL. Pulmonary Embolism in the Critically Ill. CHEST 2007; 132:313-324.
3. Drffler-Melly J, de Jonge E, de Pont AC, et al. Bioavailability of subcutaneous low-molecular-weight heparin to patients on vasopressors. Lancet.
2002;359:849850.

RATIONALE (20) Answer: B

In a patient with diagnosed pulmonary embolism, development of hypotension heralds a higher risk of death. The bedside ultrasonography reveals a
dilated right ventricle; as can be seen on the apical 4-chamber view provided, the right ventricle is almost the same size as the left ventricle. Although it
is not possible to determine by echocardiography if existing right ventricular dysfunction is acute or chronic, in the setting of this case it is most likely
acute and due to the pulmonary embolism. Treatment of hypotension in the context of pulmonary embolism should take into consideration the underlying
pathophysiology. A large clot burden will produce an acute rise in pulmonary vasculature pressures, which result in acute right ventricular strain
(dilation of the right ventricle on echocardiography with decreased contractility). As the right ventricular cardiac output drops, so will blood flow to the
left ventricle. The under-filled left ventricle will also drop its cardiac output, resulting in arterial hypotension with a reflex increase in systemic
vascular resistance. The first attempt to break this cycle shoud be the judicious administration of IV fluids with the aim of increasing venous return to the
right ventricle and increasing its cardiac output. In cases where fluids do not improve blood pressure quickly, vasopressor support may be needed.
Although there are no clinical studies evaluating vasopressors in pulmonary embolism, some experts recommend norepinephrine as the agent of choice
(based on animal data). Because the principal problem is related to the clot burden in the pulmonary vasculature, current guidelines recommend the use
of systemic thrombolytics in patients with pulmonary embolism and shock. Thrombolytic therapy can lead to rapid improvement of hemodynamic
alterations seen with massive pulmonary embolism. In this case, the first step would be the administration of volume. If the patient does not respond,
support with vasopressors and administration of systemic thrombolytics would be warranted. Option A is incorrect because there is no evidence of
pericardial effusion in this case.

REFERENCES (20)

1. Carlbom DJ, Davidson BL. Pulmonary embolism in the critically ill. Chest 2007;132:313324.
2. Todd JL, Tapson VF. Thrombolytic therapy for acute pulmonary embolism. Chest 2009; 135:13211329.
3. Goldhaber SZ, Haire WD, Feldstein ML, et al. Alteplase versus heparin in acute pulmonary embolism: randomised trial assessing right-ventricular
function and pulmonary perfusion. Lancet. Feb 27 1993;341:507511.
4. Kucher N, Rossi E, De Rosa M, et al. Massive pulmonary embolism. Circulation. Jan 31 2006;113:577582.
5. Goldhaber SZ. Advanced treatment strategies for acute pulmonary embolism, including thrombolysis and embolectomy. J Thromb Haemost. Jul
2009;7 Suppl 1:322327.

RATIONALE (21) Answer: B

This patient's presentation is most consistent with acute respiratory distress syndrome (ARDS) due to pneumonia (bacterial and/or viral). The
cornerstone of mechanical ventilation for patients with ARDS or acute lung injury is the prevention of ventilator-induced lung injury through a protective
lung strategy. The landmark study by the ARDSNet group1 demonstrated that low tidal volume (6 mL/ kg of predicted body weight) with limitation of
plateau airway pressures to less than 30 cm H2O was associated with significant improvement in patient mortality. Based on this important clinical trial
and many other studies, current recommendations for patients with ARDS use a low tidal volume calculated from the patient's predicted body weight. It
is essential for clinicians to understand that lung size does not increase proportionally with patient's weight. Average lung size is determined by the
patient's height and sex. Therefore, when starting mechanical ventilation for ARDS, one must use a predicted body weight and not the actual body
weight. Especially in patients with high body mass indexes, the use of actual weight can lead to large and harmful tidal volumes. Therefore, option A is
incorrect because the tidal volume is based on actual weight. Finally, high-frequency oscillatory ventilation and airway pressure release ventilation are
nonconventional modes of ventilation. To date there are no clinical studies showing that either one is superior to conventional ventilation with low tidal
volume. Therefore, they are reserved for salvage therapy in cases where conventional ventilation with adequate lung protective settings has failed. They
would not be utilized as the initial settings in a case such as the one presented in this question; therefore, options C and D are incorrect.

REFERENCES (21)

1. Acute Respiratory Distress Syndrome Network. Ventilation with lower tidal volumes compared with traditional tidal volumes for acute lung injury
and the acute respiratory distress syndrome. N Engl J Med 2002;342:13011308.
2. Kam E, Eslick G, James A, et al. Acute respiratory distress syndrome (ARDS) and low tidal volume ventilation: the debate about weight. Intensive
Care Med 2004;30:1502.
3. Gattinoni L, Caironi P. Refining ventilatory treatment for acute lung injury and acute respiratory distress syndrome. JAMA. Feb 13 2008;299:691
693.
4. Marini JJ, Gattinoni L. Ventilatory management of acute respiratory distress syndrome: a consensus of two. Crit Care Med. Jan 2004;32:250255.

RATIONALE (22) Answer: A

Point A corresponds to what is referred to as the lower inflection point on the inspiratory limb of the pressure-volume curve. Positive end-expiratory
pressure (PEEP) and FIO2 are the principal tools on the ventilator that can be adjusted to improve oxygenation. When PEEP produces alveolar
recruitment, shunt is decreased and PaO2 improves. Furthermore, when set above the lower inflection point, PEEP will prevent the collapse of alveoli at
the end of expiration. By decreasing the opening and closing of alveoli, PEEP can provide protection to the lung against increased injury from shearing
forces (atelectrauma). This concept is best understood by evaluating the pressure-volume curve. However, in practice, setting the best PEEP may be
more difficult. Currently, clinicians set PEEP based on tables utilized in clinical trials by ARDSNet (www.ardsnet.org). Optimal PEEP can also be set
at the bedside by increasing PEEP in 2- to 3-cm H2O increments and finding the level that provides the best arterial oxygen saturation (88% and 95%)
with the lowest possible FIO2. Many experts believe that higher levels of PEEP are best for patients with ARDS. The rationale behind this process is
based on understanding the advantages of setting PEEP above the lower inflection point and the ability to decrease FIO2 with higher PEEP. Three clinical
trials tested modest versus high levels of PEEP in ARDS and failed to show conclusive benefits in patient survival. These studies did report improved
PaO2 to FIO2 ratio in patients treated with higher PEEP. Two of these studies also reported decreased rates of refractory hypoxemia, lower rate of death
with refractory hypoxemia, and decreased use of rescue therapies in the patients treated with higher PEEP. A PEEP setting of 815 cm H2O is probably
a safe starting point for most patients with ARDS. There is still no evidence as to the best way to select PEEP in patients with ARDS. For the time being
clinicians must evaluate the benefits and risks of high PEEP in each individual patient.

Point B on the pressure-volume curve corresponds to the upper deflection point. Ventilation beyond this point would likely cause overdistension of
alveoli and lead to volutrauma. Points C and D do not have any particular significance.

REFERENCES (22)
1. Esan A, Hess DR, Raoof S, et al. Severe hypoxemic respiratory failure: part one ventilatory strategies. Chest. May;137:12031216.
2. Meade MO, Cook DJ, Guyatt GH, et al. Ventilation strategy using low tidal volumes, recruitment maneuvers, and high positive end-expiratory
pressure for acute lung injury and acute respiratory distress syndrome: a randomized controlled trial. JAMA. 2008 Feb 13;299:637645.
3. Mercat A, Richard JC, Vielle B, et al. Positive end-expiratory pressure setting in adults with acute lung injury and acute respiratory distress
syndrome: a randomized controlled trial. JAMA. 2008 Feb 13;299:646655.
4. Brower RG, Lanken PN, MacIntyre N, et al. Higher versus lower positive end-expiratory pressures in patients with the acute respiratory distress
syndrome. N Engl J Med. 2004 Jul 22;351:327336.
5. Gattinoni L, Caironi P. Refining ventilatory treatment for acute lung injury and acute respiratory distress syndrome. JAMA. 2008 Feb 13;299:691
693.

RATIONALE (23) Answer: C

This patient is being mechanically ventilated with airway pressure release ventilation (APRV). APRV is a newer nonconventional mode of ventilation
that has been proposed as an alternative for an open-lung mechanical ventilation strategy. APRV is thought to provide the benefits of a near-permanent
recruitment maneuver with the ability to augment ventilation in patients with poor lung compliance. APRV can be thought of as continuous positive
airway pressure with regular brief releases in pressure. APRV is a time-cycled, pressure-limited mode of mechanical ventilation that cycles between a
high pressure (determined by Phigh and Thigh) and a shorter lower pressure or release pressure (determined by Plow and Tlow). Oxygenation depends on the
FIO2, the set high airway pressure (set by Phigh), and the time this high pressure-phase will last (determined by Thigh). The release phase determines
ventilation (carbon dioxide dump) and is determined by the difference between Phigh and Plow (Plow is usually set at 0), the time spent in the low-pressure
phase (determined by Tlow), and spontaneous breathing by the patient throughout the whole respiratory cycle.

REFERENCES (23)

1. Esan A, Hess DR, Raoof S, et al. Severe hypoxemic respiratory failure: part 1--ventilatory strategies. Chest 2010;137:12031216.
2. Habashi NM. Other approaches to open-lung ventilation: airway pressure release ventilation. Crit Care Med. Mar 2005;33:S228S240.
3. Rose L, Hawkins M. Airway pressure release ventilation and biphasic positive airway pressure: a systematic review of definitional criteria.
Intensive Care Med. Oct 2008;34:17661773.
4. Stawicki SP, Goyal M, Sarani B. High-frequency oscillatory ventilation (HFOV) and airway pressure release ventilation (APRV): a practical guide.
J Intensive Care Med. 2009 Jul-Aug;24:215229.

RATIONALE (24) Answer: E

The main cause of mortality in neuromuscular diseases such as myasthenia gravis is respiratory failure and pneumonia. Close monitoring of respiratory
muscle strength with serial measurements of vital capacity and maximum negative inspiratory force can help identify impending respiratory failure.
Usually, rapidly declining values or a vital capacity less than 1 L indicate the need for mechanical respiratory support. Noninvasive modalities are
definitely helpful in treating patients with neuromuscular diseaseinduced respiratory failure. Normally bilevel positive airway pressure is required.
However, when there is evidence of inability to protect the airway as in our patient, respiratory support with endotracheal intubation or tracheostomy is
required. Intubation for mechanical ventilation may also be required if the patient does not respond to noninvasive positive-pressure ventilation.

REFERENCES (24)

1. Kumar, V, Kaminski HJ. Treatment of myasthenia gravis. Curr Neurol Neurosci Rep. 2011;11:8996.
2. Dhand UK. Clinical approach to the weak patient in the intensive care unit. Respir Care. 2006;51:102440.
3. Wu JY, Kuo PH, Fan PC,et al. The role of non-invasive ventilation and factors predicting extubation outcome in myasthenic crisis. Neurocrit Care.
2009;10:3542.
4. Seneviratne J, Mandrekar J, Wijdicks EF, et al. Noninvasive ventilation in myasthenic crisis. Arch Neurol. 2008;65:548.

RATIONALE (25) Answer: C

This patient presented to the emergency department with an exacerbation of chronic obstructive pulmonary disease (COPD) triggered by a community-
acquired pneumonia. Noninvasive positive-pressure ventilation (NPPV) is a valid therapeutic option in acute COPD exacerbations. In patients with
COPD who are able to cooperate and protect their airway and are hemodynamically stable, a trial of NPPV is certainly warranted. In patients with acute
COPD exacerbations, NPPV can often prevent intubation and mechanical ventilation. One could argue that in this patient with pneumonia and productive
cough, NPPV may not be the best option. Hence, one of the most important aspects of utilizing NPPV is understanding when to transition to more
aggressive respiratory support with intubation and conventional mechanical ventilation. To this effect, what many experts have called the "1-hour check"
is extremely important. If patients are not steadily improving after 12 hours of effective NPPV, the clinician should strongly consider moving to
intubation and mechanical ventilation. In this patient, not only do the arterial blood gas results fail to show any substantial improvement but more
importantly his clinical condition has deteriorated. The patient is more lethargic and has developed hypotension and tachycardia with multifocal
tachycardia on ECG. At this point, adjusting the NPPV settings is unlikely to lead to significant clinical improvement as it seems that the patient is
developing severe sepsis and perhaps shock from his pneumonia. At this point, the best option is to intubate the patient and control the airway before the
patient becomes more unstable. Contraindications to the use of NPPV include hemodynamic instability (shock, uncontrolled cardiac ischemia, or
arrhythmia), upper gastrointestinal bleeding, agitated/uncooperative patient, inability to protect airway, swallowing impairment, excessive secretions,
and recent upper airway or gastrointestinal surgery.

REFERENCES (25)

1. Nava S, Hill N. Non-invasive ventilation in acute respiratory failure. Lancet 2009; 374:25059.
2. Decramer M, Nici L, Nardini S, et al. Targeting the COPD exacerbation. Respiratory Medicine. 2008;102(Suppl 1):S315.
3. Roberts CM, Stone RA, Buckingham RJ, et al. Acidosis, non-invasive ventilation and mortality in hospitalized COPD exacerbations.Thorax.
2011;66:438.
4. Elliot MW. Non-invasive ventilation: established and expanding roles. Clin Med. 2011;11:1503.

RATIONALE (26) Answer: D

This patient with a severe asthma exacerbation has likely developed dynamic hyperinflation of the lungs (auto-positive end-expiratory pressure) leading
to increased intrathoracic pressure and hypotension (from decreased cardiac output). Although tension pneumothorax is certainly a consideration in a
mechanically ventilated patient with severe asthma, dynamic hyperinflation is more likely on the basis of the course of events in this case. In patients
with severe airway obstruction, exhalation time is prolonged. It may be so prolonged that all the tidal volume may not be exhaled in the allotted time
and, as a new breath is delivered, the patient stacks breaths with progressive increase in lung volume at the end of expiration (functional residual
capacity [FRC]). Since asthmatics usually start with a higher baseline FRC, if dynamic hyperinflation occurs, FRC may be pushed close to total lung
capacity and can produce severe hemodynamic instability. The hemodynamic profile of dynamic hyperinflation is similar to that seen with tension
pneumothorax or acute cardiac failure (increased central venous and pulmonary wedge pressures, decreased cardiac output, and hypotension). The
mistake in the management of this case was to continue to decrease exhalation time (increasing RR) in order to correct the high carbon dioxide level.
Mild to moderate elevations in carbon dioxide are usually well tolerated by patients on mechanical ventilation (permissive hypercapnia). In a patient
with asthma on mechanical ventilation, special attention should be directed to airway pressures and preventing dynamic hyperinflation. If she is
disconnected from the ventilator, the patient will have enough time to exhale the accumulated tidal volume; this should result in improved
hemodynamics. In this case, the pressure immediately recovered and the heart rate decreased. When the patient was placed back on the ventilator, both
the tidal volume (set to high initially) and the rate were decreased. Decreasing the rate, although eventually needed, would not take precedence over
disconnecting the patient from the ventilator in this case because of the severe hemodynamic compromise and the need to act emergently.

REFERENCES (26)

1. Mannam P, Siegel MD. Analytic review: management of life- threatening asthma in adults. J Intens Care Med. 2010;25:315.
2. Lazarus SC. Clinical practice. Emergency treatment of asthma. N Engl J Med. 2010;368:755764.
3. Mughal MM, Minai OA, Culver DA, et al. Auto-positive end-expiratory pressure: Mechanisms and treatment. Cleveland Clin J Med. 2005;72:801
809.

RATIONALE (27) Answer: A

This patient has most likely developed negative-pressure pulmonary edema (NPPE). NPPE may occur in the acute setting in association with
laryngospasm and other upper-airway obstructive processes such as epiglottitis, near drowning, and strangulation. It is not a frequent complication of
general anesthesia but can occur, especially in young, healthy adults after surgery. NPPE has also been described in chronic conditions such as
obstructive sleep apnea, nasopharyngeal mass, and tonsillar enlargement. Clinical findings of NPPE include cough, dyspnea, tachypnea, tachycardia,
hypoxemia, frothy secretions, and in rare instances, frank pulmonary hemorrhage. NPPE is usually triggered by laryngospasm causing inspiration against
a closed glottis, inducing highly negative intrathoracic pressures with a concomitant increase in venous return. The consequence of this phenomenon is
increased pulmonary blood flow with increased hydrostatic pressure leading to noncardiogenic pulmonary edema. This is usually a self-limited
phenomenon and resolves in 24 hours with appropriate supportive care. There is no evidence suggesting that corticosteroids or diuretics will accelerate
recovery. The clinical symptoms and findings of NPPE are nonspecific and should lead the clinician to a broad differential diagnosis. However, when
these symptoms occur in healthy young adults in the perioperative period as in this case, NPPE should be highly suspected.

REFERENCES (27)

1. Fremont RD, Kallet RH, Mathay M, et al. Case for hydrostatic mechanisms. Post obstructive pulmonary edema: a case of hydrostatic mechanisms.
Chest 2007;131:17421746.
2. Koh MS, Hsu AA, EngP. Negative pressure pulmonary edema in the medical intensive care unit. Intensive Care Med 2003;29:16011604.
3. Papaioannou V, Terzi I, Dragoumanis C, et al. Negative-pressure acute trancheobronchial hemorrhage and pulmonary edema. J Anesthesisa
2009;23:417420.

RATIONALE (28) Answer: C

This patient was admitted with severe status asthmaticus and now is clinically deteriorating. The changes in his chest radiograph and fevers suggest a
new acquired pulmonary process. The findings on the sputum are highly suggestive of Aspergillus fumigatus. The potential for invasive pulmonary
aspergillosis (IPA) complicating acute severe asthma should be considered. The risk of developing IPA is highest among patients with neutropenia,
hematological malignancy, or following bone marrow or solid organ transplant. However, the combination of respiratory disease (asthma, chronic
obstructive pulmonary disease) and corticosteroids also increases the risk of developing IPA. Current guidelines recommend early initiation of
antifungal therapy in critically ill patients with suspected IPA while diagnostic evaluation is conducted. Early empiric treatment is warranted
considering the high mortality of IPA in critically ill patients. For primary treatment of IPA, voriconazole is recommended as first-line agent. In
seriously ill patients, the parenteral formulation is recommended. There is no indication to change vancomycin to linezolid. Fluconozale is not
recommended for the treatment of Aspergillus/ IPA. There is no evidence suggesting Pneumocystis carinii pneumonia (PCP), so adding trimethoprim/
sulfamethoxazole is not warranted. Finally, waiting for confirmation of cultures would delay treatment and probably have a negative impact on the
patient's outcome considering the severity of the current clinical situation. Further diagnostic tests with cultures obtained from bronchoalveolar lavage,
serological testing for Aspergillus (antibody titers), and CT imaging can help establish the diagnosis. Patients may acquire Aspergillus species either in
the ICU or prior to entering it. Patients with asthma are commonly colonized by Aspergillus fumigatus. Fifty-two percent of critically ill patients with
Aspergillus in respiratory tract samples show no evidence of invasive disease and are thought to be colonized, whereas the remaining 48% have IPA.

REFERENCES (28)

1. Walsh TJ, Anaissie EJ, Denning DW, et al. Treatment of aspergillosis: clinical practice guidelines of the Infectious Diseases Society of America.
Clinical Infectious Diseases 2008;46:32760.
2. Herbrecht R, Denning DW, Patterson TF, et al. Invasive Fungal Infections Group of the European Organisation for Research and Treatment of Cancer
and the Global Aspergillus Study Group: Voriconazole versus amphoterecin B for primary therapy of invasive aspergillosis. N Eng J Med
2002;347:408415.
3. Garnacho-Montero J, Amaya-Villar R. A validated clinical approach for the management of aspergillosis in critically ill patients: ready, steady, go!
Crit Care 2006;10:132.
4. Meersseman W, Vandecasteele SJ, Wilmer A, et al. Invasive aspergillosis in critically ill patients without malignancy. Am J Respir Crit Care Med
2004;170:621625.

RATIONALE (29) Answer: C

The patient's clinical presentation is most consistent with acute chest syndrome (ACS). ACS is a serious complication of sickle cell disease that is
defined by the presence of a new pulmonary infiltrate on chest radiograph with chest pain, tachypnea, tachycardia, dyspnea, fever, hypoxia, crackles,
and/or wheezes. The most common causes of ACS are fat emboli from the bone marrow and infections. It is proposed that in ACS the injured lung is
unable to reoxygenate sickled red blood cells. This leads to activation of the endothelium with aggregation of red blood cells in the pulmonary
microvasculature, resulting in increased lung inflammation and potential lung infarcts. The mainstay of treatment in ACS is hydration, oxygen therapy,
and analgesia. Because infections are common triggers of ACS, antibiotic coverage is recommended. Multiple studies have shown that blood
transfusions can rapidly improve oxygenation and have a positive impact on the course of ACS. The role of exchange transfusions in ACS is not as clear.
However, they should be considered in critically ill patients with deteriorating cardiopulmonary status because they may improve outcomes. The
enlarged right ventricle in this patient is due to chronic pulmonary hypertension associated with her sickle cell disease. There is no indication that the
patient has pulmonary embolism, hence heparin is unlikely to help. Although antibiotics are indicated in ACS, they are unlikely to produce rapid
improvement in oxygenation. Corticosteroids and furosemide have not been shown to be effective in the treatment of ACS.

REFERENCES (29)

1. Gladwin MT, Vichinsky E. Pulmonary complication of sickle cell disease. N Engl J Med 2008;359:225465.
2. Marti-Carvajal AJ, Conterno LO, Knight-Madden JM. Antibiotics for treating acute chest syndrome in people with sickle cell disease. Cochrane
Database Syst Rev. 2007;(2):CD006110.
3. Mekontso Dessap A, Leon R, Habibi A, et al. Pulmonary hypertension and cor pulmonale during severe acute chest syndrome in sickle cell disease.
Am J Respir Crit Care Med. 2008 Mar 15;177:646653.
4. Turner JM, Kaplan JB, Cohen HW, et al. Exchange versus simple transfusion for acute chest syndrome in sickle cell anemia adults. Transfusion.
2009 May;49:863868.
RATIONALE (30) Answer: A

The use of bedside ultrasonography as a point-of-care diagnostic tool is rapidly gaining acceptance in the ICU. Within the growing number of
applications for critical care ultrasonography, evaluation of the lung is a relatively novel one. Normally, ultrasound is not transmitted through anatomical
structures filled with gas, and the lung parenchyma is not visible beyond the pleura. The injured lung produces characteristic artifacts resulting from the
abnormal gas-tissue interface. Recent studies have described these artifacts and their utility in evaluating several pathological conditions in the lung.

One of the first structures one can identify with ultrasonography is the pleura. In normal conditions, a hyperechoic sliding line (moving forward and back
with ventilation) is seen below the rib line and is called the pleural line. The presence of pleural sliding signifies a normal pleural space (no
pneumothorax) for the region of lung being directly visualized but does not rule out a partial pneumothorax elsewhere. If there is air between the parietal
and visceral pleurae, as occurs in a pneumothorax, there will be no sliding of the pleural line. However, this finding is not specific for pneumothorax as
it can occur in other situations such as presence of chest tubes, pleural adherences, bullous emphysema, and advanced chronic obstructive pulmonary
disease.

A-lines are artificial horizontal lines seen below the pleural line. A-lines will be present if there is a pneumothorax, but they will be motionless since
there is no pleural sliding. B-lines are artificial vertical lines seen in up to a third of patients with normal lungs. B-lines also move with the pleural line
and efface A-lines. Because B-lines are not always seen, their absence is not particularly useful in diagnosing a possible pneumothorax. However, the
presence of B-lines can rule out a pneumothorax in the portion of lung being evaluated.

Finally, probably the most valuable sign in the diagnosis of a pneumothorax is identification of the lung point. The lung point is the point where the
normal lung pattern (pleural sliding and/or the presence of B-lines) replaces the pneumothorax pattern (absent lung sliding and horizontal A-lines).
Utilization of M-mode can facilitate detection of the lung point. Studies have shown that, in trained hands, ultrasonography has better sensitivity and
specificity for diagnosing pneumothorax than conventional chest radiography (86% and 97% versus 28% and 100%, respectively). In addition, lung
ultrasonography was shown to be significantly faster than chest radiography.

REFERENCES (30)

1. Bouhemad B, Zhang M, Lu Q, et al. Clinical review: bedside lung ultrasound in critical care practice. Crit Care. 2007;11:205.
2. Murphy M, Nagdev A, Sisson C. Lack of lung sliding on ultrasound does not always indicate a pneumothorax. Resuscitation. May 2008;77:270.
3. Soldati G, Sher S. Bedside lung ultrasound in critical care practice. Minerva Anestesiol. Sep 2009;75:509517.

RATIONALE (31) Answer: A

High-frequency oscillatory ventilation (HFOV) has been proposed as an alternative mode for delivering low-tidal-volume, controlled-pressure
ventilation in patients with severe acute respiratory distress syndrome. HFOV, commonly utilized in neonatology and only recently recognized as
potentially useful in adults, is based on the principles of the open-lung ventilation and aims to improve oxygenation by keeping the lung uniformly
inflated for an extended period of time. In HFOV, the lung is kept open by a set mean airway pressure (oxygenation). High frequency oscillations
(180/min) produce inspiration and expiration with small volumes around this mean airway pressure (ventilation). Theoretically, this will improve
recruitment (oxygenation) and minimize ventilator-induced lung injury by limiting volutrauma and atelectrauma. In HFOV, alveolar ventilation (carbon
dioxide removal) is controlled by the power of oscillation, which determines amplitude, and by the frequency of oscillations measured in hertz. The
speed of oscillation is determined by the frequency. One hertz (Hz) is equal to 1 breath per second or 60 breaths per minute. A frequency of 6 Hz
therefore is equal to 360 breaths per minute. In HFOV, because of the fixed inspiratory to expiratory ratio, as frequency increases, the time allotted for
the piston to move air is decreased. In other words, changes in frequency are inversely proportional to tidal volume. With HFOV, in order to increase
alveolar ventilation (increase tidal volume) and remove more carbon dioxide, the frequency must be decreased. This is usually done by decreasing
frequency in 1-Hz decrements to a minimum of 3 Hz. Alveolar ventilation (carbon dioxide removal) may also be increased by increasing the power to
increase amplitude. Of the possible answers, only option A (decreasing frequency to 5 Hz) would result in improved carbon dioxide removal. If
adjustments in frequency and amplitude fail to correct increased carbon dioxide, one last option is to deflate the endotracheal cuff and produce an air
leak. This needs to be accompanied by an increase in mean airway pressure (by 5 cm H2O) and is usually the least preferred method of correcting high
carbon dioxide levels. Mean airway pressure determines oxygenation, and increasing it will not affect carbon dioxide level.

REFERENCES (31)

1. Esan A, Hess DR, Raoof S, et al. Severe hypoxemic respiratory failure: part ventilatory strategies. Chest 2010;137:12031216.
2. Stawicki SP, Goyal M, Sarani B. High-frequency oscillatory ventilation (HFOV) and airway pressure release ventilation (APRV): a practical guide.
J Intensive Care Med. 2009 Jul-Aug;24:215229.
3. Chan KP, Stewart TE, Mehta S. High-frequency oscillatory ventilation for adult patients with ARDS. Chest 2007;131:19071916.
4. Fessler HE, Hager DN, Brower RG. Feasibility of very high-frequency ventilation in adults with acute respiratory distress syndrome. Crit Care Med
2008;36:10431048.
RATIONALE (32) Answer: B

High-frequency oscillatory ventilation (HFOV) is a unique mode of ventilation that requires the critical care team to monitor patients with unique
potential complications in mind. The amplitude is set by increasing power until the patient has a clear chest wiggle that travels down to the mid-thighs.
Because the HFOV ventilator is poorly alarmed, it is essential to monitor the patient closely for potential problems. Clinical assessment and evaluation
of the patient's chest wiggle are needed to recognize potential problems such as endotracheal tube (ET) obstruction, tension pneumothorax, or pulmonary
overdistension. Observation of the patient for equal and continuous chest wiggle should be performed at the initiation of HFOV and regularly thereafter.
If the patient's chest wiggle abruptly changes or decreases, potential causes to be considered include ET obstruction (ie, mucus plug) and tension
pneumothorax (especially if the chest wiggle becomes asymmetric). Increases in amplitude (by 10 cm H2O or more) reflect changes in pulmonary
dynamics. The most common cause of changes in pulmonary dynamics leading to increased amplitude is airway secretions. Situations like the one
described in this case usually indicate the need for aggressive suctioning of respiratory secretions. It is important to emphasize that suctioning during
HFOV must be done with a closed circuit because disconnecting the patient will lead to derecruitment and likely hypoxemia. When patients are on
HFOV, every effort must be made to avoid disconnection from the oscillator in order to prevent derecruitment. There is no indication in this case of a
potential pneumothorax; therefore, the chest tubes are not indicated. Increasing sedation would not be correct based on the available information,
although it is important to remember that patients on HFOV require heavy sedation and often neuromuscular blockade. Deflation of the ET cuff is
sometimes utilized to increase carbon dioxide removal.

REFERENCES (32)

1. Esan A, Hess DR, Raoof S, et al. Severe hypoxemic respiratory failure: part ventilatory strategies. Chest 2010;137:12031216.
2. Stawicki SP, Goyal M, Sarani B. High-frequency oscillatory ventilation (HFOV) and airway pressure release ventilation (APRV): a practical guide.
J Intensive Care Med. 2009 Jul-Aug;24:215229.
3. Chan KP, Stewart TE, Mehta S. High-frequency oscillatory ventilation for adult patients with ARDS. Chest 2007;131:19071916.
4. Fessler HE, Hager DN, Brower RG. Feasibility of very high-frequency ventilation in adults with acute respiratory distress syndrome. Crit Care Med
2008;36:10431048.

RATIONALE (33) Answer: D

The principal determinants of oxygenation during high-frequency oscillatory ventilation (HFOV) are the mean airway pressure and FIO2. Oxygenation can
be improved by increasing mean airway pressure, FIO2, or by increasing the percent of inspiratory time (usually set a 33%). Bias flow is the rate at
which gas flows through the ventilator circuit. An increase in bias flow will increase mean airway pressure, thereby increasing oxygenation. Generally,
it is accepted that, as a starting point on HFOV, the bias flow should be set around 40 L/min. The maximum bias flow possible on current HFOV
machines is 60 L/min. It is important to recognize that even a maximal flow rate on HFOV is insufficient to support spontaneous breathing efforts. For
this reason, patients should be deeply sedated (and often paralyzed) during HFOV.

REFERENCES (33)

1. Esan A, Hess DR, Raoof S, et al. Severe hypoxemic respiratory failure: part ventilatory strategies. Chest 2010;137:12031216.
2. Stawicki SP, Goyal M, Sarani B. High-frequency oscillatory ventilation (HFOV) and airway pressure release ventilation (APRV): a practical guide.
J Intensive Care Med. 2009 Jul-Aug;24:215229.
3. Chan KP, Stewart TE, Mehta S. High-frequency oscillatory ventilation for adult patients with ARDS. Chest 2007;131:19071916.
4. Fessler HE, Hager DN, Brower RG. Feasibility of very high-frequency ventilation in adults with acute respiratory distress syndrome. Crit Care Med
2008;36:10431048.

RATIONALE (34) Answer: C

Ultrasonography is increasingly being used as a diagnostic modality in the critically ill. The patient has a pleural effusion as pleural fluid appears
hypoechoic (black) on ultrasonogram. The pleural fluid is demonstrated in the Figure by the star. The patient's liver is the salt-and-pepper structure
demonstrated by the square. The patient's diaphragm is demonstrated by the straight arrow and is above the patient's liver. The curved arrow represents
atelectatic lung due to the patient's pleural effusion. This still image from the ultrasonography does not explain the patient's respiratory distress. The
pleural effusion in this picture does not appear large. However, on only 1 view, it is difficult to fully appreciate the extent of this effusion; more views
are needed to complete the examination. There is no evidence of a pneumothorax, which would be demonstrated by absence of lung sliding in the proper
clinical context, or of a mass in the chest or a pericardial effusion. Diaphragmatic paralysis can be identified in a live ultrasonography clip that shows
respiratory motion, but it would not be a diagnosis made with a still ultrasonography image such as the one presented in this case. A normal
ultrasonogram of the patient's right thorax would demonstrate the patient's liver, diaphragm, and horizontal reflections from the patient's chest wall know
as A-lines secondary to the piezoelectric effect induced by ultrasonography.
REFERENCES (34)

1. Joyner CR, Herman RJ, Reid JM, et al. Reflected ultrasound in the detection and localization of pleural effusion. JAMA. 1967;200:399402.
2. Lichentstein DA, Meziere GA. Relevance of lung ultrasound in the diagnosis of acute respiratory failure: the Blue Protocol. Chest. 2008;134:117
125.
3. Eisen LA, Leung S, Galagher AE, et al. Barriers to ultrasound training in critical care medicine fellowships: a survey of program directors. Crit
Care Med 2010;38:19781983.

RATIONALE (35) Answer: C

Patients with idiopathic pulmonary fibrosis from the time of diagnosis have a median survival of approximately 3.5 years. Acute exacerbations are
associated with a mortality of 60%70% over 36 months. As was done with this patient, it is important to rule out reversible causes of respiratory
distress in idiopathic pulmonary fibrosis patients with acute deterioration. This was done with the bronchoscopy, which ruled out pulmonary infection;
the normal ejection fraction on the echocardiogram, which ruled out congestive heart failure; and the CT, which ruled out pulmonary embolism and other
lung disease such as pneumothorax. The patient's mildly elevated brain natriuretic peptide level would not explain his rapid deterioration. Neither
steroids nor any other pharmaceutical therapy have proven benefit in idiopathic pulmonary fibrosis. Noninvasive ventilation is not indicated for
somnolent patients with idiopathic pulmonary fibrosis, and the patient has no indication for antibiotics. Idiopathic pulmonary fibrosis is now the leading
indication for pulmonary transplantation in the United States. Early after diagnosis, patients should be referred to transplant centers to be evaluated for
possible transplant. Exacerbations of the disease that occurred in this patient can lead to rapid deterioration. Without prior evaluation, it is unlikely the
patient would receive a lung transplant because acutely ventilated idiopathic pulmonary fibrosis patients have high mortality even after transplant.
However, in lung transplant recipients, those with the worst long-term prognosis now have 1-year survival of 83% and 5-year survival of 54%
according to a scientific registry of transplant recipients.

REFERENCES (35)

1. Kreider M, Hadjiliadis D, Kotloff K. Candidate selection, time of listing and choice of procedure for lung transplantation. Clin Chest Med
2011;32:199211.
2. Agarwal R, Jindal SK. Acute exacerbation of idiopathic pulmonary fibrosis: a systematic review. Eur J Intern Med 2008;19:227235.
3. Harari S, Caminati A. Update on diffuse parenchymal lung disease. Eur Respir Rev 2010;19:97108.
4. Yusen RD, Shearon TH, Qian Y, et al. Lung transplantation in the United States, 1999-2008. Am J Transplant 2010;10(Part 2):10471068.
5. Mallick S. Outcome of patients with idiopathic pulmonary fibrosis (IPF) ventilated in intensive care unit. Respir Med 2008;10:13551359.
PART 4: Critical Care Infectious Diseases

Instructions: For each question, select the most correct answer.

1. A 44-year-old woman with an allergy to sulfa drugs is admitted to the ICU following a motor vehicle accident. She is intubated and mechanically
ventilated. On hospital day 5, she develops fever, purulent sputum, and a left lower lobe infiltrate. Gram stain from bronchoalveolar lavage (BAL)
shows many leukocytes and gram-negative bacilli. Culture from BAL and blood grows Enterobacter cloacae sensitive to third-generation
cephalosporins, piperacillin, and trimethoprim/sulfamethoxazole. She is started on IV ceftriaxone.

Seven days later, she continues to have intermittent fevers and produce purulent sputum; the left lower lobe infiltrate is unchanged. Repeat BAL and
blood cultures grow E cloacae that is now sensitive to trimethoprim/sulfamethoxazole and resistant to all beta-lactams except meropenem.

Which of the following is the most appropriate next step?


A. Continue ceftriaxone to complete a 14-day course and repeat cultures on day 14 to ensure she has cleared her bacteremia
B. Discontinue ceftriaxone and start piperacillin/tazobactam plus amikacin for synergy.
C. Discontinue ceftriaxone, desensitize to trimethoprim/sulfamethoxazole, and start trimethoprim/sulfamethoxazole to complete a 2-week course.
D. Discontinue ceftriaxone and start meropenem to complete a 2-week course.

2. A 37-year-old man seen 1 week ago in urgent care clinic for an influenza-like illness is now admitted to the ICU with severe community-acquired
pneumonia. Chest CT demonstrates bilateral cavitary infiltrates and a loculated right lower lobe effusion. Empirical therapy for methicillin-resistant
Staphylococcus aureus (MRSA) is started pending sputum and blood culture results.

Which of the following statements should guide antibiotic management of this patients suspected MRSA pneumonia?
A. Vancomycin trough concentrations of 1520 g/mL are most appropriate.
B. Linezolid is associated with lower mortality rates than vancomycin.
C. If the vancomycin minimum inhibitory concentration is greater than 2 g/mL, daptomycin should be used.
D. Empirical coverage for MRSA should be continued even if cultures do not grow the organism.

3. A 69-year-old woman is transferred to the ICU from an outside hospital. Two weeks ago, she underwent uncomplicated partial colectomy for
diverticulitis. One week ago, she was admitted to the outside hospital for fevers and abdominal pain. Blood cultures were positive for vancomycin-
resistant Enterococcus faecium (VRE) and a highly-resistant strain of Klebsiella pneumoniae. She was started on tigecycline but developed
respiratory failure and was intubated and mechanically ventilated 5 days ago. Chest radiography shows bilateral multifocal opacities consistent
with bilateral pneumonia and acute respiratory distress syndrome.

Which of the following statements about tigecycline should guide this patients treatment?
A. Tigecycline has minimal activity against anaerobic bacteria.
B. Most strains of Pseudomonas are sensitive to tigecycline.
C. Compared to other antibiotics, tigecycline is associated with an increased risk of death, especially in patients with ventilator-associated
pneumonia.
D. Tigecycline has poor activity against carbapenemase-producing, gram-negative bacteria.

4. A 74-year-old woman is admitted to the ICU for fever to 38.5C (101.2F), altered mental status, abdominal pain, and hypotension. She resides in a
nursing home and presented to the emergency department after several days of frequent, loose, watery bowel movements and abdominal cramping.
Her worsening shock necessitates endotracheal intubation, aggressive IV fluid resuscitation, and vasoactive agent support.
At admission, WBC count is 26,000/L (12% bands), serum creatinine level is 1.9 mg/dL (baseline 0.7 mg/dL), and serum lactate level is 27.9
mg/dL.

Clostridium difficile toxin assay detects toxin A and B.

Which of the following is the most appropriate next step in the menagement of this patient?
A. Oral vancomycin
B. Emergent colectomy
C. Diagnostic colonoscopy
D. CT of the abdomen
E. IV metronidazole

5. A 27-year-old man with a history of IV drug use is admitted to the ICU for fevers, altered mental status, and hypotension. Blood cultures at the time
of admission are growing methicillin-resistant Staphylococcus aureus (MRSA).

Which of the following statements is most appropriate to guide the management of his MRSA bacteremia?
A. Gentamicin should routinely be added to vancomycin.
B. Rifampin should routinely be added to vancomycin.
C. Echocardiographic evaluation for endocarditis is not recommended unless repeat blood culture results are positive for MRSA.
D. Appropriate initial antibiotics include vancomycin or daptomycin.

6. A 65-year-old woman presents to the emergency department with fever and altered mental status. She has a history of atrial fibrillation treated with
amiodarone and is severely allergic to penicillin. Chest radiography shows a questionable retrocardiac opacity. Blood and urine cultures are
obtained. She is started on moxifloxacin and admitted to the floor for community-acquired pneumonia. Overnight, she becomes increasingly
delirious and is transferred to the ICU for hypotension.

Which of the following statements is correct regarding moxifloxacin therapy?


A. Moxifloxacin concentrates optimally in the urine and can be used for suspected urinary tract infections.
B. Moxifloxacin should generally be avoided in patients with prolonged QT intervals.
C. Compared to ciprofloxacin, moxifloxacin has greater in vitro activity against Pseudomonas aeruginosa.
D. In contrast to ciprofloxacin, moxifloxacin demonstrates activity against vancomycinresistant Enterococcus.

7. A 75-year-old man with diabetes and end-stage renal disease on hemodialysis is admitted to the medical ICU for fever. Four weeks before
admission, he was started on linezolid for a diabetic foot infection.

Besides thrombocytopenia, which of the following side effects is associated with long durations of therapy with linezolid?
A. Guillain-Barr syndrome
B. Hyperglycemia due to pancreatic dysfunction
C. Lactic acidosis
D. Hypertriglyceridemia
E. Fanconi syndrome

8. A 52-year old man is admitted to the ICU for sepsis and multisystem organ failure and requires a urinary catheter for accurate monitoring of urine
output.

Which of the following strategies helps to prevent catheter-associated urinary tract infections?
A. Sterile equipment and aseptic technique for catheter insertion
B. Routine cleaning of the urethral meatus with antiseptic solution after catheter insertion
C. Keeping the collecting bag above the level of the bladder to promote reflux of urine into the bladder and not emptying it routinely
D. Bladder irrigation and instilling antiseptic or antimicrobial agents into the collecting bag

9. A 76-year-old woman is admitted to the ICU from a nursing home for health careassociated pneumonia. She has a history of infection with
multidrug-resistant organisms.

Which of the following statements should guide the use of colistin in this patients treatment?
A. Colistin is nephrotoxic but not neurotoxic.
B. Colistin is neurotoxic but not nephrotoxic.
C. Colistin retains activity against most strains of multidrug-resistant, gram-negative infections.
D. Unlike tobramycin, colistin cannot be nebulized for inhaled use.

10. A 46-year-old woman with a recent bilateral lung transplant for cystic fibrosis is transferred into the ICU with dyspnea, hypoxia, and fever. She is
receiving immunosuppression with mycophenolate, tacrolimus, and prednisone. Yesterday, she underwent bronchoscopy with transbronchial
biopsy, which is reported today as showing acute-angle branching hyphae. After a brief conversation with the infectious diseases consultant, the
patient is started on IV voriconazole. Doses of which of the following of her medications should be reduced?

A. Mycophenolate
B. Voriconazole
C. Tacrolimus
D. Prednisone
E. Propofol

11. A 63-year-old man was admitted to the ICU 4 days ago after undergoing a partial colectomy for a perforated diverticulum. He has been febrile since
postoperative day 1 despite treatment with vancomycin and piperacillin/tazobactam. This afternoon he developed hypotension requiring blood
pressure support. Which antimicrobial medication should be added empirically to his regimen?

A. Amphotericin B
B. Liposomal amphotericin B
C. Caspofungin
D. Fluconazole

12. A 34-year-old woman with recently diagnosed AIDS is admitted to the ICU with fever, hypoxia, respiratory failure, and pancytopenia. Six weeks
ago she was started on highly active antiretroviral therapy with tenofovir, emtricitabine, and ritonavir-boosted lopinavir. Her chest radiograph
shows bilateral infiltrates with an interstitial pattern. The evaluation of her pancytopenia included a bone marrow biopsy, which showed trilineage
hypoplasia and macrophages with multiple intracellular yeast forms consistent with histoplasmosis. Which of the following is the preferred initial
treatment?

A. Lipid preparation of amphotericin B


B. IV fluconazole
C. IV voriconazole
D. Caspofungin
E. Posaconazole

13. A 58-year-old man with a kidney transplant is admitted with confusion. In the emergency department, he was intubated for airway protection and
underwent a lumbar puncture with an opening pressure of 32 cm H2O. Analysis of cerebrospinal fluid showed 12 RBCs per L, 83 WBCs per L,
glucose level of 38 mg/dL, and protein level of 181 mg/dL. Gram stain showed polymorphonuclear cells but no organisms, and results of India ink
capsule stain were negative. Which of the following is the most appropriate initial regimen of antimicrobials?

A. Ceftriaxone alone
B. Ampicillin and ceftriaxone
C. Vancomycin and ceftriaxone only
D. Vancomycin, ceftriaxone, and amphotericin B

14. A 23-year-old man with acute myelogenous leukemia recently underwent induction chemotherapy. He has had neutropenia for 18 days with fever the
last 5 days. Yesterday, he reported headache, and a sinus CT showed opacification of his left maxillary and ethmoid sinuses that was of concern for
possible fungal sinusitis. He is admitted to the ICU after endoscopic evaluation of his sinuses, during which several areas of necrotic mucosa were
debrided; frozen pathologic sections showed a few hyphae invading blood vessels. Which of the following is the most appropriate empiric therapy
for this infection?

A. Fluconazole
B. Micafungin
C. Liposomal amphotericin B
D. Voriconazole alone
E. Voriconazole and caspofungin

15. A 63-year-old woman with a past medical history of type 2 diabetes mellitus and hypertension is admitted to the ICU with a diagnosis of
community-acquired pneumonia. Intially she is hypotensive and hyperglycemic. However, her blood pressure normalizes with a bolus of normal
saline. Unfortunately, her respiratory status declines, requiring intubation and mechanical ventilation. Shortly after intubation, bronchoalveolar
lavage is performed, with Gram stain showing many white blood cells, many gram-positive cocci in pairs, and few yeasts. The most appropriate
initial therapy for this patient is:

A. Vancomycin and piperacillin/tazobactam


B. Ceftriaxone and azithromycin
C. Ceftriaxone and fluconazole
D. Levofloxacin and fluconazole

16. A 57-year-old man was admitted to the ICU 5 days ago with an ST-elevation myocardial infarction. Yesterday he developed a fever and cultures of
blood, sputum, and urine were sent. Chest radiography showed a new right lower lobe infiltrate. Results of urinalysis were entirely with in normal
limits. The patient was started on piperacillin/tazobactam for the pneumonia and is afebrile today. The microbiology laboratory reports that more
than 100,000 colony-forming units per millilter of yeast are growing from the urine culture. The most appropriate next course of action is:

A. No intervention
B. Oral fluconazole
C. IV caspofungin
D. Amphotericin B bladder washes
E. IV fluconazole
17. A previously health y, 47-year-old man is admitted to the ICU after decompensation following a diagnostic bronchoscopy earlier today. He has a 6-
week history of a right middle lobe infiltrate that has not responded to a course of amoxicillin/clavulanate or levofloxacin. The patient is an avid
hunter and spent 2 weeks deer hunting in middle Tennessee before the development of the pneumonia. The most likely diagnosis is:

A. Coccidioidomycosis
B. Aspergillosis
C. Mucormycosis
D. Blastomycosis

18. A 54-year-old man underwent a single lung transplant 14 days ago for idiopathic pulmonary fibrosis. He had a difficult operation and primary graft
dysfunction and has had a prolonged ventilatory course. He has been on vancomycin and meropenem for an aspiration pneumonia in the donor, and
he has been maintained on voriconazole and trimethoprim/ sulfamethoxazole as routine prophylaxis of fungi and Pneumocystis. His lung has
improved to the point of initiating ventilator weaning. However, he is not waking up despite being off midazolam for 6 hours. An MRI of the brain
shows no evidence of ischemia or hemorrhage. Which of the following is the most likely explanation of his neurologic status?

A. He suffered a major cerebral ischemic event during lung transplantation.


B. Trimethoprim/sulfamethoxazole inhibits the clearance of midazolam.
C. Voriconazole inhibits the clearance of midazolam.
D. Voriconazole has major psychiatric side effects and the patient is in a fugue state.

19. A 70-year-old man with benign prostatic hypertrophy was an unrestrained passenger in a rollover motor vehicle crash. He is brought to a trauma
center in shock and respiratory failure. He undergoes an emergent exploratory laparotomy and splenectomy and requires multiple thoracostomy
tubes to manage bilateral pneumothoraxes. His ICU course is complicated by an abdominal evisceration, Enterobacter line sepsis, and ventilator-
associated pneumonias with Pseudomonas and Acinetobacter. After stabilizing from these conditions, he develops bright red blood per rectum,
diarrhea, leukopenia, and fever. A flexible sigmoidoscopy is performed and reveals multiple oozing ulcerations in the colon with the pathology
shown in the Figure.

Which of the following is the most appropriate antimicrobial medication to add?

A. IV ceftriaxone for disseminated pneumococcal disease


B. IV acyclovir for herpes simplex virus colitis
C. Vancomycin per nasogastric tube plus IV metronidazole for Clostridium difficile colitis
D. IV ganciclovir for cytomegalovirus colitis
E. IV metronidazole for Entamoeba histolytica colitis
20. A 51-year-old woman recently underwent an allogeneic bone marrow transplant for refractory multiple myeloma, with successful engraftment. She
is readmitted with suspected graft-versus-host disease (GVHD) because of recently developed shortness of breath, increased liver enzymes, and
conjunctivitis. She is also given IV levofloxacin for a suspected urinary tract infection because of blood and white blood cells on urine microscopy.
She is given high-dose steroids and thymoglobulin for GVHD. Twenty-four hours later, she develops respiratory distress, fulminant hepatitis, and
bloody urine and is transferred to the ICU. Chest radiographs show progressive bilateral infiltrates in a pattern consistent with acute respiratory
distress syndrome.

The most likely etiology of this course is:


A. Cytomegalovirus
B. Adenovirus
C. Legionella species
D. Parainfluenza 3
E. Toxicity from chemotherapy

21. A 35-year-old man with HIV diagnosed 4 years ago is admitted with fever, weight loss, and night sweats. A diagnosis of lymphoma is made, and he
is treated with rituximab plus cyclophosphamide, doxorubicin, vincristine, and prednisone (R-CHOP). A few days later, he is transferred to the ICU
for neutropenia and hypotension. He is treated with vancomycin and meropenem but remains hypotensive. After 48 hours, caspofungin is added, but
his blood pressure remains low. Adrenal insufficiency is diagnosed and he is started on stress-dose steroids. Seventy-two hours later, he develops
a burning, painful rash with vesicles on his back, left chest, and left upper abdomen.

Which of the following steps is most appropriate?


A. Substitute IV linezolid for vancomycin.
B. Begin IV acyclovir.
C. Begin valacyclovir.
D. Substitute amphotericin B for caspofungin.

22. A 26-year-old graduate student in chiropterology returned 2 days ago from a 2-week field study of eating habits of fruit bats in the Democratic
Republic of the Congo and Uganda. During her flight back, she started to feel fever, malaise, and nausea. Shortly after arriving in the US, she
developed diarrhea and vomiting. A local emergency department, suspecting food poisoning from the fish she ate on her flight, prescribed her
ciprofloxacin and metronidazole. The next day she returned with continued symptoms, rigors, and abdominal pain, at which time a maculopapular
rash was observed. She progressed to delirium, hypotension, and abrupt prostration. She was intubated for airway protection. Soon afterward, she
developed bloody pulmonary secretions, conjunctival hemorrhages, and bloody diarrhea.

Which of the following interventions is most likely to benefit the patient?


A. Ribavirin
B. Acyclovir
C. Oseltamivir
D. Aggressive fluid repletion
E. Replacement of coagulation factors

23. A 65-year-old, male resident of an assisted living center has asthma, hypertension, and coronary artery disease. He is transferred to an acute care
facility with acute respiratory distress and fever in mid January after spending the holidays with children and grandchildren. Unable to maintain his
oxygenation, he is intubated and placed on mechanical ventilation. His chest radiograph shows bilateral infiltrates. Results of a respiratory viral
panel are positive for influenza A. There have been sporadic reports of oseltamivir-resistant strains in the current season.
Which of the following is the best therapeutic option for this patient?
A. Oseltamivir, enteral
B. Zanamivir, inhaled
C. IV amantadine
D. IV immunologlobulins
E. IV foscarnet

24. A 44-year-old woman with weakness and difficulty in breathing presents to the emergency department reporting fever, abdominal pain, and a
pruritic maculopapular rash on her back and chest for the past 24 hours. She was diagnosed 10 years ago with HIV, which had been controlled with
lamivudine/zidovudine and lopinavir/ritonavir for the past 3 years. Approximately 2 weeks ago, her physician added abacavir to her regimen to
achieve an undetectable viral load. She also has diastolic dysfunction, hypertension, and hypothyroidism. She only takes lisinopril in addition to
antiretrovirals. She is found to be hypotensive, tachycardic, and tachypneic. Radiography shows an interstitial pneumonitis, but oxygen saturation is
95% on room air.

Which of the following is the most likely cause of this condition?


A. Congestive heart failure
B. Acute renal failure due to antiretrovirals
C. Hypersensitivity reaction due to an antiretroviral
D. Pneumocystis pneumonia
E. Acute HIV syndrome

25. A 22-year-old man with HIV and a history of epilepsy is admitted to an ICU with status epilepticus. He is intubated for airway protection and his
seizures are quickly controlled. His CD4 cell count is 485/L and his HIV viral load is undetectable. He receives tenofovir, emtricitabine, and
efavirenz for HIV. Findings of CT and MRI of the brain are negative.

Which of the following is the most appropriate management of this patients illness?
A. Stop his antiretroviral medications until he is back to baseline.
B. Reduce the dosages of his antiretroviral medications in case the seizures were induced by lactic acidosis.
C. Change his antiretroviral medications to include an unboosted protease inhibitor.
D. Continue his current antiretroviral regimen.
E. Perform a lumbar puncture to evaluate for encephalitis.
PART 4: Critical Care Infectious Diseases

ANSWERS:

1D; 2A; 3C; 4B; 5D; 6B; 7C; 8A; 9C; 10C; 11C; 12A; 13D; 14C; 15B; 16A; 17D; 18C; 19D; 20B; 21B; 22E; 23A; 24C;
25D

RATIONALE (1) Answer: D

Enterobacter cloacae possesses an inducible ampC chromosomal beta-lactamase. In Enterobacter infections, mutants can produce high levels of beta-
lactamase. These mutant strains can emerge during therapy with a third-generation cephalosporin such as ceftriaxone and result in treatment failure, even
if the isolates appear susceptible on initial testing. For serious infections caused by Enterobacter, carbapenems are first-line therapy.

REFERENCE (1)

1. Chow JW, Fine MJ, Shlaes DM, et al. Enterobacter bacteremia: Clinical features and emergence of antibiotic resistance during therapy. Ann Intern
Med. 1991;115:585590.

RATIONALE (2) Answer: A

Vancomycin generally has poor penetration into pulmonary tissue and lung epithelial lining fluid. The pharmacodynamic parameter that best predicts
efficacy of vancomycin is the ratio of the area under the curve (AUC) to the minimum inhibitory concentration (MIC) [AUC/MIC]. In Staphylococcus
aureus lower respiratory tract infections, an AUC/MIC greater than 400 is associated with improved clinical response and microbiologic eradication.
On the basis of currently available data, vancomycin trough concentrations of 1520 g/mL are needed to achieve this target.

In prospective studies, linezolid and vancomycin were associated with comparable cure rates. Thus, option B is incorrect. Daptomycin is inactivated by
pulmonary surfactant and should not be used in patients with pneumonia, so option C is also incorrect. Although empirical therapy for methicillin-
resistant S aureus (MRSA) should be considered in patients with severe community-acquired pneumonia and necrotizing or cavitary infiltrates,
empirical coverage for MRSA should be discontinued if sputum or blood cultures do not grow the organism. Therefore, option D is incorrect.

REFERENCE (2)

1. Liu C, Bayer A, Cosgrove SE, et al. Clinical practice guidelines by the Infectious Diseases Society of America for the treatment of methicillin-
resistant Staphylococcus aureus infections in adults and children. Clin Infect Dis. 2011;52:28592.

RATIONALE (3) Answer: C

Tigecycline has broad-spectrum activity against gram-positive, gram-negative, and anaerobic organisms. It also has in vitro activity against methicillin-
resistant Staphylococcus aureus, vancomycin-resistant enterococci, and many highly resistant gram-negative bacteria including strains that produce
carbapenemases; however, clinical data is limited. Tigecycline generally has poor activity against most strains of Pseudomonas. On July 6, 2010, the
U.S. Food and Drug Administration (FDA) updated the Warnings and Precautions and Adverse Reactions sections of the tigecycline drug label to
include information regarding increased mortality risk (seen most clearly in patients treated for hospital-acquired pneumonia, especially ventilator-
associated pneumonia).

REFERENCE (3)

1. Tygacil [package insert]. Philadelphia, PA: Wyeth Pharmaceuticals; 2005. Updated July 2010.

RATIONALE (4) Answer: B

Colectomy may be a lifesaving intervention for persons who are severely ill from Clostridium difficile infection (CDI), as illustrated in this case;
therefore, option B is correct. Studies indicate that in the setting of CDI, clinical deterioration characterized by cardiopulmonary compromise ,
peritoneal signs, or toxic megacolon should lead to urgent consideration of surgical intervention. Furthermore, conservative medical management with
antibiotic therapy (options A and E) or radiographic evaluation (option D) in the setting of severe CDI has been associated with higher mortality
compared with surgical intervention. Endoscopic evaluation (option C) is not indicated given the documentation of a positive stool assay result for C
difficile, and in fact, increases the risk of bowel perforation.

REFERENCES (4)

1. Cohen SH, Gerding DN, Johnson S, et al. Clinical practice guidelines for Clostridium difficile infection in adults: 2010 update by the Society for
Healthcare Epidemiology of America (SHEA) and the infectious diseases society of America (IDSA). Infect Control Hosp Epidemiol. 2010;31:431
455.
2. Hall JF, Berger D. Outcome of colectomy for Clostridium difficile colitis: a plea for early surgical management. Am J Surg. 2008;196: 384388.

RATIONALE (5) Answer: D

Vancomycin or daptomycin are reasonable choices for MRSA bacteremia. Echocardiography generally should be performed in all adult patients with
MRSA bacteremia to evaluate for endocarditis, so option C is incorrect. Routine addition of gentamicin and/or rifampin is not recommended for MRSA
bacteremia or native valve endocarditis, so options A and B are incorrect.

REFERENCES (5)

1. Liu C, Bayer A, Cosgrove SE, et al. Clinical practice guidelines by the Infectious Diseases Society of America for the treatment of methicillin-
resistant Staphylococcus aureus infections in adults and children. Clin Infect Dis. 2011;52:28592.

RATIONALE (6) Answer: B

Unlike ciprofloxacin and levofloxacin, moxifloxacin does not concentrate in the urine and should not be used for suspected urinary tract infections
(therefore, option A is incorrect). All of the fluoroquinolones can potentially cause prolongation of the QT interval (option B). Compared to
moxifloxacin, ciprofloxacin generally has better in vitro activity against most strains of Pseudomonas (therefore, option C is incorrect). None of the
fluoroquinolones demonstrates activity against vancomycin-resistant Enterococcus, so option D is incorrect.

REFERENCES (6)

1. Bolon MK. The newer fluoroquinolones. Infect Dis Clin NorThAm 2009;23:10271051.

RATIONALE (7) Answer: C

Although rare, long durations of linezolid treatment are associated with an increased risk of lactic acidosis. Additional potential risks after prolonged
linezolid therapy include optic and peripheral neuropathy, serotonin syndrome (especially in the setting of concominant selective serotonin reuptake
inhibitor therapy), and myelosuppression, including thrombocytopenia. Linezolid has not been associated with Guillain-Barr syndrome (option A),
hyperglycemia (option B), hypertriglyceridemia (option D) or Fanconi syndrome (option E).

REFERENCES (7)

1. Narita M, Tsuji BT, Yu VL. Linezolid-associated peripheral and optic neuropathy, lactic acidosis, and serotonin syndrome. Pharmacotherapy
2007;27:118997.

RATIONALE (8) Answer: A

To prevent catheter-associated urinary tract infections, sterile equipment and aseptic technique should be used for insertion (option A). Routine cleaning
of the urethral meatus with antibiotic solution after catheter insertion is not recommended and may actually increase the risk of bacteruria, so option B is
incorrect. The collecting bag should remain below the level of the bladder to prevent reflux of urine into the bladder and should be emptied routinely;
thus, option C is incorrect. Procedures such as bladder irrigation and instilling antiseptic or antimicrobial agents into the collecting bag have not been
shown to provide benefit and should not be performed. Therefore, option D is incorrect.

REFERENCES (8)
1. Shuman EK, Chenowith CE. Recognition and prevention of health care-associated urinary tract infections in the intensive care unit. Crit Care Med.
2010;38:S373S379.

RATIONALE (9) Answer: C

Colisitin is generally the treatment option of last resort for extremely multidrug-resistant, gram-negative infections. Thus, option C is correct. Colistin
can potentially cause both nephrotoxicity and neurotoxicity, so options A and B are incorrect. Colistin can be nebulized for inhaled use, so option D is
incorrect. However, premixing colistimethate into an aqueous solution and storing it for longer than 24 hours results in increased concentrations of
colistin in solution, increasing the potential for lung toxicity.

REFERENCES (9)

1. Lim LM, Ly N, Anderson D et al. Resurgence of colistin: a review of resistance, toxicity, pharmacodynamics, and dosing. Pharmacotherapy.
2010;30:12791291.

RATIONALE (10) Answer: C

Many antimicrobials have significant drug interactions, especially those which affect hepatic metabolism. Many medications are cleared through the
cytochrome P-450 enzymes. Some antimicrobials inhibit cytochrome P-450, especially cytochrome 3A4. The antifungal class of triazoles, or "azoles,"
are well known inhibitors of cytochrome 3A4. Consequently, any medication which is cleared through 3A4 will then accumulate resulting in higher
serum levels. Among the medications listed, tacrolimus is cleared through 3A4. Typically, when tacrolimus is co-administered with voriconazole, the
same serum levels can be achieved by reducing the dose of tacrolimus to one-half or one-third the previous dose. The other medications listed are
cleared renally or through other degradation pathways.

Antibiotics in the rifamycin class (rifampin, rifabutin, rifapentene, etc) are inducers of the cytochrome P-450 enzymes and rifampin is the most potent.
Therefore, when rifamycins are coadministered with medications metabolized through cytochrome P-450 , the result is lower drug levels of the
coadministered medication. If rifabutin had been started instead of voriconazole, then the dose of tacrolimus would have needed to be increased to
maintain therapeutic levels. Rifampin is such a potent inducer of 3A4 that it is difficult to maintain therapeutic levels of tacrolimus when the 2 are
coadministered.

REFERENCES (10)

1. Pai MP, Allen S. Voriconazole inhibition of tacrolimus metabolism. Clin Infect Dis. 2003;36:108991.

RATIONALE (11) Answer: C

Abdominal surgery is a risk factor for the development of candidiasis/candidemia. Since the patient has not responded to appropriately broad
antibacterial therapy, it is reasonable to suspect that he may have a Candida infection; it may be localized to the peritoneum or systemic in the
bloodstream. Candida are commensals of the human gastrointestinal tract. Thus, they are frequently encountered in gastrointestinal-associated infections.
In 2009, the Infectious Diseases Society of America released updated treatment guidelines for candidiasis. These guidelines recommend that an
antifungal from the echinocandin class be used to treat candidiasis in moderately to severely ill patients. The guidelines do not distinguish between the 3
FDA-approved echinocandins. Since caspofungin is the only listed echinocandin, it is the most correct choice (option C). The echinocandins are
fungicidal for Candida species. While fluconazole (option D) has been used empirically for years to treat candidiasis, it is fungistatic. Therefore, the
recommendation is to use an echinocandin in critically ill patients. Amphotericin B products are also fungicidal for Candida species. However, all
amphotericin Bcontaining medications have increased toxicity over the echinocandins. Because of this excess toxicity, amphotericin B should be used
as a second-line agent in patients who have had an adverse or allergic response to echinocandins. Therefore, options A and B are incorrect.

REFERENCES (11)

1. Pappas et al. Clinical Practice Guidelines for the Management of Candidiasis: 2009 Update by the Infectious Diseases Society of America. Clin
Infect Dis 2009;48:50335.

RATIONALE (12) Answer: A

Amphotericin B, whether a lipid preparation or the deoxycholate preparation, is the treatment of choice for disseminated histoplasmosis (option A).
Histoplasma capsulatum, the causative agent of histoplasmosis, is endemic in the Mississippi and Ohio River valleys. Patients with
immunocompromised immune systems, ie, AIDS, solid organ transplant recipients, or recipients of tumor necrosis factor antagonists, are at increased
risk for developing disseminated disease. When Histoplasma disseminates, it often goes to the bone marrow causing pancytopenia. The liver and spleen
are other organs commonly affected. Pathologic specimens may show "foamy" macrophages, or frequently granulomas, in less severely
immunocompromised patients.

Fluconazole (option B) is less effective against Histoplasma; therefore, it should not be used in the treatment of disseminated histoplasmosis. While
voriconazole and posaconazole (options C and E) have in vitro activity against Histoplasma, they have not been studied in controlled clinical trials.
Therefore, they cannot be recommended for routine use. Itraconazole does have activity against Histoplasma and it is the preferred oral option for less
severe disease. However, itraconazole has significant drug interactions with ritonavir and coadministration is contraindicated. The echinocandins,
including caspofungin (option D), have no activity against Histoplasma.

REFERENCES (12)

1. Wheat LJ, Freifeld AG, Kleiman MB, et al. Clinical practice guidelines for the management of patients with histoplasmosis: 2007 update by the
Infectious Diseases Society of America. Clin Infect Dis 2007;45:80725.

RATIONALE (13) Answer: D

This patient has meningitis. In an immunocompromised patient, such as a kidney transplant recipient, common and uncommon causes must be suspected.
Thus, the etiology could be usual organisms like Streptococcus pneumonia, or more unusual organisms like Listeria monocytogenes or Cryptococcus
neoformans. Typically, pneumococcus meningitis produces more inflammation than the 83 WBCs seen in this case. However, solid organ transplant
recipients often have a blunted inflammatory response. This is especially true of those patients receiving higher doses of steroids. Even so,
pneumococcus is usually seen readily on Gram stain of the cerebrospinal fluid. Ceftriaxone is excellent treatment for pneumonoccus, but there are some
isolates which exhibit high-level resistance, in which case vancomycin should be used.

Listeria typically causes a mononuclear pleocytosis in the cerebrospinal fluid. Thus, it is a less likely cause of meningitis in this patient. However,
medications active against Listeria should be used until the cultures have adequately incubated. Ceftriaxone does not treat Listeria. Ampicillin and
vancomycin are both active against Listeria.

Cryptococcus is frequently seen in transplant-recipient meningitis. The elevated opening pressure, high protein level, and moderate pleocytosis are
typical features of a cryptococcal meningitis. The India ink test is less sensitive than latex agglutination for the cryptoccocal antigen. Therefore, false
negatives are possible, especially if there is a low organism burden. Of the choices listed, only vancomycin plus ceftriaxone plus amphotericin B
(option D) is broad enough antimicrobial coverage for all these possibilities.

REFERENCES (13)

1. Davis JA, Horn DL, Marr KA, et al. Central nervous system involvement in cryptococcal infection in individuals after solid organ transplantation or
with AIDS. Tranpl Infect Dis 2009;11:432437.

RATIONALE (14) Answer: C

This patient clearly has invasive fungal sinusitis. The most common causes of invasive fungal sinusitis are aspergillosis and mucormycosis. both
diseases are characterized by necrosis, hyphae, and blood vessel invasion. Only culture or special pathologic techniques can distinguish between the 2.
Of the agents listed, only the liposomal amphotericin B (option C) has activity in both diseases. Fluconazole (option A) does not treat either
aspergillosis or mucormycosis. Micafungin, voriconazole, and caspofungin (options B, D, and E) have activity against aspergillosis but not against
mucormycosis.

REFERENCES (14)

1. Epstein VA, Kern RC. Invasive fungal sinusitis and complications of rhinosinusitis. Otolaryngol Clin North Am. 2008 Jun;41:497524.

RATIONALE (15) Answer: B

Community-acquired pneumonia (CAP) guidelines recommend coverage for both typical (Streptococcus pneumoniae, Haemophilus influenza) and
atypical (Mycoplasma, Chlamydia) organisms. However, organisms such as methicillin-resistant Staphylococcus aureus and Pseudomonas are not
usually encountered in CAP. Therefore, ceftriaxone plus azithromycin is a reasonable initial regimen until the causative agent has grown in culture.
Levofloxacin and other fluoroquinolones provide good coverage for both typical and atypical organisms seen in CAP. Vancomycin plus
piperacillin/tazobactam (option A) would be a good regimen for health careassociated pneumonia. However, it wouldn't cover atypical organisms
sometimes seen in CAP. Based on the Gram stain, this patient likely has an S pneumoniae pneumonia. The yeasts seen on the Gram stain are likely
colonizers, ie, Candida. Candida species are almost never a cause of pneumonia unless the patient has a prolonged neutropenia or embolic disease from
right-sided Candida endocarditis. Therefore, the 2 choices including fluconazole (options C and D) would not be correct.

REFERENCES (15)

1. Mandell LA et al. Infectious Diseases Society of America/American thoracic Society consensus guidelines on the management of community-
acquired pneumonia in adults. Clin Infect Dis 2007;44:S2772.

RATIONALE (16) Answer: A

The fever in this patient is caused by the pneumonia in the right lower lobe. The yeast in the urine represents asymptomatic candiduria as the urinalysis
did not show any pyuria. Research has shown that asymptomatic candiduria has the same response rate whether the patient receives no treatment,
systemic treatment with fluconazole (options B and E), or amphotericin B bladder washes (option D). There is no indication for systemic echinocandin
therapy (option C). Therefore, the most recent guidelines on candidiasis suggest no treatment for asymptomatic candiduria (option A).

REFERENCES (16)

1. Pappas PG, Kauffman CA, Andes D, et al. Clinical practice guidelines for the management of candidiasis: 2009 update by the Infectious Diseases
Society of America. Clin Infect Dis 2009;48:50335.

RATIONALE (17) Answer: D

This question requires understanding of basic fungal epidemiology. The correct answer is blastomycosis (option D). Blastomyces dermatitidis is a
fungus found on decaying vegetation and occurs in the southeastern US and the Mississippi and Ohio river valleys. Most individuals who become
infected have hunting or hiking as an avocational activity. Outbreaks have been observed surrounding earth-moving activities such as construction. Most
patients develop a pneumonia, and the diagnosis of blastomycosis is suspected or confirmed after failure to respond to repeated courses of antibacterial
medications. The diagnosis of blastomycosis can be made by isolating the organism in culture from appropriate specimens (sputum or bronchoalveolar
lavage in the scenario), or by detection of a fungal antigen in the urine similar to that for histoplasmosis. Coccidioidomycosis (option A) occurs only in
the desert southwest, and aspergillosis and mucormycosis (options B and C) occur only in immunocompromised patients.

REFERENCES (17)

1. Chapman SW, Dismukes WE, Proia LA, et al. Clinical practice guidelines for the management of blastomycosis: 2008 update by the Infectious
Diseases Society of America. Clin Infect Dis 2008;46:180112.

RATIONALE (18) Answer: C

The correct answer is option Cvoriconazole inhibits the clearance of midazolam. Midazolam is cleared through the cytochrome P-450 system, and
voriconazole greatly inhibits the activity of these enzymes. Thus, when coadministered, midazolam accumulates and has delayed clearance. Therefore,
the sedative effect of midazolam is prolonged. While a stroke (option A) could potentially be the cause of this patient's depressed neurologic status, the
normal brain MRI finding makes this diagnosis less likely.

Trimethoprim/sulfamethoxazole (option B) has no effect on the clearance of midazolam and is rarely if ever implicated as a central nervous system
depressant. Voriconazole does have major psychiatric side effects, but a fugue state (option D) has not been reported. The most common psychiatric side
effect is visual hallucinations. Typically, the patient is aware of the hallucinations and is quite disturbed by the adverse event. Very rarely, psychotic
breaks are associated with the visual hallucinations of voriconazole.

REFERENCES (18)

1. Saari TI, Laine K, Leino K, et al. Effect of voriconazole on the pharmacokinetics and pharmacodynamics of intravenous and oral midazolam. Clin
Pharmacol Ther. 2006;79:362370.
RATIONALE (19) Answer: D

Ganciclovir (option D) is the correct therapeutic choice for this patient. The diagnosis is cytomegalovirus (CMV) colitis, based on pathology showing
viral inclusions and on the clinical presentation. CMV infections occur, though rarely, in nonimmunocompromised patients. Latent CMV can reactivate
and cause end-organ disease in situations with high levels of tumor necrosis factor alpha, lipopolysaccharide, or exogenous catecholamines. Therefore,
it is not uncommon for CMV replication to be detected in patients in the ICU.

While the patient is at risk for infection from encapsulated organisms because of the splenectomy, the scenario is not consistent with pneumococcal
disease, so option A is incorrect. Herpes simplex virus colitis is extremely rare and unlikely in this situation. Thus, option B is incorrect. This patient is
at high risk for developing Clostridium difficileassociated colitis, but the pathology is consistent with viral colitis, not pseudomembranous colitis.
Therefore, option C is incorrect. While bloody diarrhea can be a presentation of amoebic colitis, this patient has no known risk factors, so option E is
also incorrect.

REFERENCES (19)

1. Kalil AC. A silent killer: cytomegalovirus infection in the nonimmunocompromised critically ill patient. Crit Care Med. 2008;36:32613264.

RATIONALE (20) Answer: B

Adenoviral disease is the most likely process in this patient. Although conjunctivitis, respiratory distress, hepatitis, and hemorrhagic cystitis all may be
caused by graft-versus-host-disease (GVHD), they are unlikely to occur simultaneously with this process. Worsening of symptoms with GVHD-focused
immunosuppression points toward an infectious etiology. While cytomegalovirus (option A) and Legionella (option C) may cause many of these
symptoms, hemorrhagic cystitis is unlikely. In this patient population, parainfluenza infections can be severe but are usually limited to pulmonary
disease, so option D is unlikely. Direct toxicity from chemotherapy (option E) should always be considered, but given the spectrum of symptoms, it is
also unlikely in this patient. There is no established therapy for adenoviral disease, but both cidofovir and ribavirin have been used, as well as IV
immunoglobulin.

REFERENCES (20)

1. Kim Y-J, Boeckh M, Englund JA. Community respiratory virus infections in immunocompromised patients: hematopoietic stem cell and solid organ
transplant recipients, and individuals with human immunodeficiency virus infection. Semin Respir Crit Care Med. 2007;28:222242.

RATIONALE (21) Answer: B

This patient has multidermatomal varicella zoster (shingles) that requires acyclovir treatment. Valacyclovir is an enteral antiviral and would not be
indicated for the initial treatment of this infection in an immunocompromised patient. There is no reason to substitute linezolid for vancomycin or
amphotericin B for the echinocandin caspofungin.

REFERENCES (21)

1. Siegel JD, Rhinehart E, Jackson M, et al. 2007 Guideline for Isolation Precautions: Preventing Transmission of Infectious Agents in Healthcare
Settings. http://www.cdc.gov/ncidod/dhqp/pdf/ isolation2007.pdf.

RATIONALE (22) Answer: E

The diagnosis is Ebola; therefore, the correct therapy is replacement of coagulation factors (option E). The bloody secretions are a clue to viral
hemorrhagic fever; the exposure to fruit bats in central Africa makes Ebola virus the most likely etiology. Treatment for viral hemorrhagic fever
infection is primarily supportive. There are no effective agents against Ebola, so treatment is primarily supportive care. However, with Ebola
infections, aggressive volume resuscitation (option D) has been linked to poor pulmonary outcomes. Careful monitoring of coagulation factors and
appropriate replacements have been shown to support the patient while the immune system builds a response to the virus.

REFERENCES (22)

1. Pigott DC. Hemorrhagic fever viruses. Crit Care Clin. 2005;21:765783.


RATIONALE (23) Answer: A

Despite the reported sporadic resistance, oseltamivir is the best option for this patient. Inhaled zanamivir (option B) should have good activity against
influenza but is relatively contraindicated in patients with asthma and cannot be effectively aerosolized for mechanical ventilation. Most circulating
strains of influenza are now amantadine resistant, ruling out option C. No data exists to support the efficacy of IV immunoglobulin (option D) or
foscarnet (option E) in this setting. Currently, oseltamivir is only available in oral formulation. IV oseltamivir and IV peramivir (another influenza
neuraminidase inhibitor) are in trials, but may be available for compassionate use.

REFERENCES (23)

1. Smith JR, Ariano RE, Toovey S. The use of antiviral agents for the management of severe influenza. Crit Care Med. 2010;38(4 Suppl):e43e51.

RATIONALE (24) Answer: C

This patient has abacavir hypersensitivity syndrome or reaction. While many antiretrovirals cause hypersensitivity, the reaction to abacavir is well
described and can be severe. It is associated with the HLA-B57 haplotype. It can occur from 1 week to 2 months after starting abacavir and usually
involves fever, rash, nausea, vomiting, and abdominal pain. Rarely, the clinical course can worsen, with associated hypotension, acute interstitial
pneumonitis, and cardiopulmonary failure.

REFERENCES (24)

1. Chaponda M, Pirmohamed M. Hypersensitivity reactions to HIV therapy. Br J Clin Pharmacol. 2011;71:65971.


2. Hughes CA, Foisy MM, Dewhurst N, et al. Abacavir hypersensitivity reaction: an update. Ann Pharmacother. 2008;42:38796.

RATIONALE (25) Answer: A

The correct management is to continue the current antiretroviral medications. None of these medications potentiates seizure. Unless there is a high
suspicion of direct toxicity or change in renal function, the antiretroviral medications do not need to be adjusted.

REFERENCES (25)

1. Morris A, Masur H, Huang L. Current issues in critical care of the human immunodeficiency virus-infected patient. Crit Care Med. 2006;34:429.
PART 5: Hepatic, Gastrointestinal, Hematologic and Oncologic Disease in
the ICU

Instructions: For each question, select the most correct answer.

1. What are the risk factors that have the strongest correlation with stress-related bleeding in critically ill patients?
A. Mechanical ventilation and glucocorticoids
B. Coagulopathy and sepsis
C. Mechanical ventilation and coagulopathy
D. Sepsis and renal failure
E. Mechanical ventilation and hypotension

2. A 62-year-old man with history of chronic obstructive pulmonary disorder was admitted to the ICU with respiratory failure. The patient has been on
mechanical ventilation for 7 days. He is currently receiving antibiotics for pneumonia. He has received intermittent doses of benzodiazepines and
narcotics for sedation and analgesia. Over the past 2 days, he has developed a progressive abdominal distension with poor tolerance of enteral
feedings secondary to high gastric residuals.

On examination, you notice a distended abdomen with no evidence of surgical scars. There is a significant tympany on percussion and diffuse
tenderness on palpation, with no rebound tenderness, guarding, rigidity, or evidence of ascites. On auscultation, bowel sounds are decreased on all
quadrants. An abdominal radiograph (flat plate) is obtained and shows dilatation of the cecum (12 cm) with no other significant findings.

Which of the following management steps is most appropriate?


A. Surgical intervention
B. Central venous alimentation for 2 weeks
C. Neostigmine
D. Atropine
E. Colonoscopy

3. In which of the following clinical scenarios is the placement of a large-bore nasogastric tube most indicated?
A. Suspected partial versus complete bowel obstruction
B. Acute pancreatitis
C. To assist with diagnosis of gastrointestinal bleeding
D. Refractory nausea and vomiting
E. Suspected esophageal perforation

4. A generally healthy, 64-year-old man presents to the emergency department with acute hematemesis and hypotension. Due to an ankle sprain, he has
been taking ibuprofen, 800 mg every 8 hours for the past 2 weeks. After initial resuscitation and initiation of an IV proton pump inhibitor, the patient
is stabilized and admitted to the ICU. An esophagogastroduodenoscopy (EGD) performed within 24 hours reveals a 1.5-cm gastric ulcer with a
visible vessel and adherent clot on the lesser curve of the stomach. This is treated with epinephrine and heater probe cautery. Two days later, the
patient develops acute hypotension, tachycardia, a drop in hematocrit, and melena.

After resuscitation and stabilization, which of the following is the best next step in management?
A. Urgent surgical intervention
B. Immediate CT of abdomen and pelvis
C. IV octreotide
D. Repeat EGD
E. Angiography

5. A 45-year-old, otherwise healthy man is brought by ambulance to the emergency department because of a massive upper gastrointestinal tract bleed.
Initial laboratory results show hemoglobin level of 3 g/dL, hematocrit of 9%, platelet count of 200,000/L, total bilirubin level of 1.4 mg/dL, and
international normalized ratio (INR) of 1.2. He is resuscitated and admitted to the ICU. An esophagogastroduodenoscopy (EGD) reveals a large
duodenal ulcer with active bleeding. He undergoes endoscopic therapy and is placed on a proton pump inhibitor drip. Over the course of the first 48
hours, the patient requires 12 units of packed red blood cells. On hospital day 3, the patients laboratory results reveal a platelet count of
90,000/L, total bilirubin level of 2.4 mg/dL (indirect bilirubin level of 1.8 mg/dL) and an INR of 2.2. His complete blood count is stable, and
testing for immunoglobulin G antibodies to Helicobacter pylori returns positive results.

Which of the following is the most likely cause of the patients thrombocytopenia?
A. Idiopathic thrombocytopenic purpura
B. Disseminated intravascular coagulation
C. Heparin-induced thrombocytopenia
D. Dilutional effect from massive transfusion of red blood cells
E. Hepatic failure from proton pump inhibitor therapy

6. A 69-year-old woman with a history of congestive heart failure and chronic atrial fibrillation comes to the emergency department because of the
abrupt onset of right lower quadrant abdominal pain and hematochezia. Current medications are digoxin and furosemide. On physical examination,
temperature is 37.9C (100.2F), pulse rate is 100/min with irregular rhythm, and BP is 140/80 mm Hg. Moderate right lower quadrant abdominal
tenderness is noted on palpation. No masses are evident. Rectal examination discloses bloody stool. Hemoglobin level is 13.5 g/dL. Leukocyte
count is 13,500/L. After a colon preparation, a colonoscopy is performed, which reveals mucosal erythema, edema, and friability involving the
cecum, ascending colon, and proximal transverse colon only. Examination of a stool specimen shows many leukocytes, but no ova or parasites.
Results of stool cultures are pending. The patient clinically improves, and her abdominal pain is gradually improving.

Which of the following is most appropriate to recommend at this time?


A. Supportive care and observation
B. Corticosteroid therapy
C. IV antibiotics
D. Mesenteric angiography
E. Surgical consultation

7. A 50-year-old woman with alcoholic cirrhosis and a model for end-stage liver disease (MELD) score of 20, is admitted to the ICU for management
of unexplained hypoxemia. Her arterial blood gas results on room air show pH of 7.43, PaCO2 of 36 mm Hg, and PaO2 of 55 mm Hg. Subjectively,
she describes dyspnea when she is moved from a supine to an upright position. Her diagnostic evaluation so far has included a normal chest
radiograph and a low-probability ventilation-perfusion scan.

The next diagnostic test for the evaluation of her hypoxemia should be:
A. Transthoracic echocardiography with a bubble study
B. IV contrast-enhanced chest CT
C. Dobutamine stress echocardiography
D. Cardiac MRI
E. Bronchoscopy

8. A 45-year-old man with hepatitis C cirrhosis, a model for end-stage liver disease (MELD) score of 29, and prior complications of ascites and
spontaneous bacterial peritonitis presents to the ICU with fever and encephalopathy. Due to worsening encephalopathy, endotracheal intubation is
required for airway protection.

The most appropriate management of hepatic encephalopathy in this patient includes:


A. Placement of an intracranial pressure monitor
B. IV mannitol therapy
C. Enteral rifaximin therapy
D. IV hypertonic saline
E. Initiation of hypothermia

9. A 45-year-old man with alcoholic cirrhosis and a previously documented hepatic hydrothorax presents with worsening lethargy and hypotension.
Initially, his temperature is 39.8C (103.6F), pulse rate is 110/ min, BP is 80/50 mm Hg, and RR is 24/min. His examination reveals decreased
breath sounds in the right lung base, marked scleral icterus, asterixis, and no ascites. Blood and urine cultures are obtained. A chest radiograph
demonstrates a moderate-sized right pleural effusion.

Additional infectious evaluation at this stage should include:


A. MRI with gadolinium of the brain
B. Thoracentesis with pleural fluid analysis
C. Lumbar puncture
D. Chest CT with contrast
E. Bronchoscopy with bronchoalveolar lavage

10. A 22-year-old woman presents with acute liver failure due to acetaminophen toxicity. Her vital signs are stable. On examination, grade 2
encephalopathy is found, and the INR is 7.0. No signs of active bleeding are noted, and serum hemoglobin level is 13 g/dL.

The most appropriate next step in the management of her severe coagulopathy is:
A. Administration of fresh frozen plasma
B. Administration of recombinant factor VIIa
C. Plasmapheresis
D. Monitoring without intervention
E. Administration of factor IX complex

11. A 36-year-old woman undergoes a percutaneous liver biopsy to assess for autoimmune hepatitis. Four days later, she presents to the emergency
department with hypotension, tachycardia, melena, and a significant drop in hematocrit, with no associated abdominal pain. After resuscitation,
noncontrast abdominal CT findings are negative for an intraperitoneal bleed. Esophagogastroduodenoscopy reveals copious blood coming from the
ampulla of Vater.

Which of the following is the best next step in management?


A. Observation with blood transfusions as needed
B. Octreotide infusion
C. Angiography with possible embolization
D. Endoscopic retrograde cholangiopancreatography with stent placement
E. Surgical consultation
12. A 62-year-old man with hepatitis C cirrhosis is transferred from an outside hospital with suspected septic shock. Following endotracheal intubation
for worsening shock, aggressive resuscitation with crystalloids and vasoactive agent support is initiated, resulting in stabilization of the mean
arterial pressure above 65 mm Hg. Blood and urine cultures are sent, and broad-spectrum antibacterial therapy is initiated. Twelve hours after
aggressive resuscitation, the patient demonstrates a worsening clinical status characterized by worsening hypotension, abdominal distension, high
ventilator peak pressures, and oliguria. Chest and abdominal radiographs are unremarkable. A transthoracic echocardiogram demonstrates
underfilling of the right heart and collapse of the inferior vena cava. Abdominal ultrasonography reveals hepatomegaly and a large amount of
ascites.

What is the best next step in management?


A. Abdominal MRI with and without contrast
B. Addition of antifungal therapy
C. Repeat echocardiography
D. Measurement of bladder pressure and paracentesis
E. Nephrology consultation for hemodialysis

13. A 60-year-old man with Child-Pugh stage C cirrhosis secondary to hepatitis C and alcohol abuse presents with confusion and is diagnosed with
hepatic encephalopathy. Treatment with lactulose is initiated.

On admission, the serum creatinine level is found to be 1.5 mg/dL, and over the next few days increases to 3.5 mg/dL despite daily administration of
albumin, placement of Foley catheter, and avoidance of all potentially nephrotoxic drugs.

On physical examination, the patient has signs of chronic liver failure, and his BP is 98/55 mm Hg, with HR of 70/min. He is confused.

Laboratory results are as follows: sodium, 137 mEq/L; potassium, 4.9 mEq/L; chloride, 92 mEq/L; bicarbonate, 17 mEq/L; blood urea nitrogen, 45
mg/dL; creatinine, 3.5 mg/ dL; aspartate aminotransferase, 55 U/L; alanine aminotransferase, 68 U/L; international normalized ratio, 4.0; WBCs,
1,900/L; hemoglobin, 8.9 g/dL; and platelets, 56,000/L. Urine sodium level is less than 10 mg/dL.

Which of the following is the best next step in the management of this patient?
A. Octreotide and midodrine
B. Transjugular intrahepatic portosystemic shunt
C. Ribavirin and interferon treatment
D. Dopamine

14. A 20-year-old with hemophilia A sustained significant chest wall trauma in a motor vehicle accident. After thoracostomy for a right pneumothorax,
he has 100 mL/h of bloody output from the chest tube. He has received transfusions multiple times in the past and is known to have inhibitors to
factor VIII.

Which of the following should be administered?


A. Recombinant factor VIII
B. Cryoprecipitate
C. Factor IX concentrate
D. Recombinant factor VIIa
E. Fresh frozen plasma

15. A 50-year-old patient with chronic kidney disease is admitted to the ICU at 11 pm with altered mental status for 2 days, fever, tachycardia, and
hypoxemia. A chest radiograph reveals a right lower lobe infiltrate. Prolonged bleeding from venipuncture sites and bleeding from the nasal mucosa
after passage of a nasogastric tube is also noted. Platelet count is 100,000/L, prothrombin time is 11.5 seconds, partial thromboplastin time is 35
seconds, blood urea nitrogen level is 150 mg/dL, and creatinine level is 10 mg/dL.
Which of the following treatments is most likely to result in rapid improvement in the bleeding diathesis?
A. Platelet transfusion
B. Hemodialysis for uremia
C. Conjugated estrogens
D. Cryoprecipitate
E. Desmopressin acetate

16. A 55-year-old man with coronary artery disease and hypertension is transferred from the telemetry unit to the ICU after developing an altered
mental status and fever. The patient underwent percutaneous coronary angioplasty 1 month ago with a stent placement and was recently admitted to
the hospital for evaluation of chest pains. On physical examination, temperature is 39.9C (103.8F), HR is 115/min, RR is 25/min and BP is
120/90 mm Hg. There is no jugular vein distension, no S3, lungs are clear on auscultation, abdominal examination findings are unremarkable, and
extremities are cool and mottled. Neurologic examination reveals a confused patient who is able to move all extremities spontaneously and opens
his eyes to voice.

Laboratory data include the following: hemoglobin, 8.1 g/dL; WBCs, 10,000/L; platelets, 4,000/L; partial thromboplastin time, 25 seconds;
international normalized ratio, 1.0; prothrombin time, 12 seconds; sodium, 139 mEq/L; potassium, 4.5 mEq/L; chloride, 102 mEq/L; bicarbonate, 22
mEq/L; and serum creatinine, 2.2 mg/dL. A peripheral blood smear is shown in the Figure.

Which of the following is most appropriate as initial treatment for this patient?
A. Argatroban
B. Dexamethasone
C. Plasmapheresis
D. Platelet transfusion
E. IV immunoglobulin

17. A 40-year-old woman is admitted to the hospital with probable immune thrombocytopenic purpura. She has some petechiae on her lower
extremities and ecchymosis in areas of mild trauma. Her platelet count is 5,000/L and prothrombin time and partial thromboplastin time are
normal. She is started on prednisone, 80 mg/day. On day 2 in the hospital, she slips and falls, sustaining a femoral neck fracture. Surgical repair is
required in the next several days.

Which of the following is the most appropriate intervention to increase the platelet count prior to surgery?

A. Platelet transfusion
B. Splenectomy
C. IV immunoglobulin
D. Rituximab
E. Danazol
18. A 65-year-old woman is admitted to the ICU after coronary bypass surgery for acute myocardial infarction with cardiogenic shock. The
postoperative course is complicated by acute renal insufficiency, and on postoperative day 5 the patient develops acute dyspnea with hypoxemia. A
diagnosis of left lower extremity deep venous thrombosis is made by lower extremity ultrasonography, with probable pulmonary embolism. The
patient is started on IV unfractionated heparin (weight-based nomogram) to achieve a partial thromboplastin time (PTT) 1.5 to 2 times the normal
value.

Laboratory studies at this time reveal normal electrolyte levels, a blood urea nitrogen level of 45 mg/dL, creatinine level of 2.8 mg/dL, WBC count
of 9,000/L, hemoglobin level of 10.1 g/dL, platelet count of 250,000/L, and normal prothrombin time, international normalized ratio, and partial
thromboplastin time. After 2 days of therapeutic anticoagulation with unfractionated heparin, the patient reports right lower extremity pain.
Examination of the right lower extremity reveals a cold painful extremity with loss of distal pulses. Laboratory studies reveal a blood urea nitrogen
level of 50 mg/dL, creatinine level of 2.8 mg/dL, platelet count of 90,000/L, international normalized ratio of 1.2, and PTT of 50 seconds.

Which of the following therapeutic options is most appropriate at this time?


A. Increase unfractionated heparin to achieve a higher PTT.
B. Discontinue unfractionated heparin and start enoxaparin.
C. Discontinue unfractionated heparin and start warfarin.
D. Discontinue unfractionated heparin and start argatroban.
E. Discontinue anticoagulation.

19. A 22-year-old, otherwise healthy man suffered blunt abdominal trauma in a motor vehicle collision. After exploratory laparotomy with a
splenectomy, he required transfusion of 4 units of packed red blood cells. Postoperatively, he had diffuse oozing at the site of the incision. His
laboratory studies preoperatively revealed a platelet count of 210,000/L, prothrombin time of 12.8 seconds, and partial thromboplastin time of 48
seconds.

Which of the following is the most likely etiology of this patients bleeding?
A. Disseminated intravascular coagulation
B. Hypothermia
C. Von Willebrand disease
D. Salicylate use
E. Citrate toxicity

20. A patient with septic shock secondary to pneumonia is admitted and treated in the ICU. On hospital day 3, his blood pressure is stable, and he is
being weaned from norepinephrine. Hemoglobin is 8.5 g/dL, platelets are 35,000/L, prothrombin time is 18 seconds, and partial thromboplastin
time is 40 seconds.

Which of the following best characterizes the blood products that should be transfused at this time?
A. Red blood cells, platelets, and fresh-frozen plasma
B. Red blood cells and platelets only
C. Red blood cells only
D. Platelets only
E. No blood products

21. A 68-year-old man with a 100-pack-year smoking history is admitted to the ICU with hypotension. He has a 2-month history of weight loss and
fatigue and a 3-day history of anorexia and polyuria. On admission, his temperature is 37C (98.6F), BP is 90/50 mm Hg, HR is 114/min, and RR
is 18/min. The patient is lethargic but has no focal neurologic deficits. Initial laboratory results are as follows: sodium, 148 mEq/L; potassium, 4.2
mEq/L; chloride, 110 mEq/L; carbon dioxide, 28 mEq/L; blood urea nitrogen, 35 mg/dL; creatinine, 2.1 mg/ dL; and glucose, 120 mg/dL. ECG
reveals sinus tachycardia and shortened QT interval. The admission chest radiograph is shown in the Figure.
In addition to IV normal saline solution, which of the following treatments will most likely benefit this patient?
A. Methylprednisolone
B. Magnesium sulfate
C. Pamidronate
D. Sodium phosphate
E. Mannitol

22. A 28-year-old man with hemoglobin SC disease is admitted to the hospital for leg and chest pain secondary to a vaso-occlusive crisis. On
admission, chest radiography shows mild cardiomegaly but no infiltrates, and pulse oximetry on room air is 97%. He is treated with IV hydration
and analgesics.

On hospital day 3, the patient develops a fever of 38C (100.3F) and shortness of breath. He continues to report sternal pain. Oxygen saturation on
room air is 85% and rales are noted in the right lower lobe. Chest radiography confirms infiltrates in the right and left lower lobe. His hemoglobin
at this time is 9.2 g/dL, with WBC count of 20,000/L (unchanged from admission). The patient is transferred to the ICU for further care.

Which of the following interventions is most appropriate for this patient?


A. Exchange transfusion
B. Simple transfusion
C. Administration of IV dexamethasone
D. Bronchoalveolar lavage to exclude infection
E. Administration of inhaled nitric oxide

23. A 55-year-old woman with a history of breast cancer is brought to the emergency department with back pain of several weeks duration, urinary
retention, constipation, and difficulty walking since waking up several hours before presentation. On examination, she has decreased sensation
below the navel and decreased strength in both lower extremities. Spinal MRI shows a solitary mass compressing the cord at T9.

Which of the following treatments is most likely to result in recovery of ambulation in this patient?
A. Chemotherapy alone
B. Radiation therapy
C. Surgical decompression plus radiation
D. Dexamethasone alone
E. Chemotherapy and corticosteroids
24. A 40-year-old, previously healthy man is brought to the emergency department with sudden onset of severe headache. He appears somnolent. Head
CT without contrast reveals a subarachnoid hemorrhage. Laboratory work-up reveals a WBC count of 10,000/L with frequent immature cells,
platelet count of 45,000/L, prothrombin time (PT) of 15 seconds, and partial thromboplastin time (PTT) of 50 seconds. He is admitted to the
neurological ICU for management.

Which of the following tests would be most helpful in determining the cause of bleeding and guiding immediate therapy?
A. Fibrinogen and D-dimer tests
B. PT and PTT mixing studies
C. Platelet function assay
D. Bone marrow biopsy
E. Review of the peripheral blood smear

25. An 18-year-old man is transferred from the hematology floor to the medical ICU because of a potassium level of 6 mEq/L and ECG changes on the
third day after initiation of chemotherapy for Burkitt lymphoma. Other laboratory results include a serum creatinine level of 3 mg/dL, phosphorus
level of 7 mg/dL, calcium level of 6.5 mg/dL, and uric acid level of 16 mg/dL.

In addition to IV fluids and management of hyperkalemia and hyperphosphatemia, which of the following interventions is most appropriate?
A. Allopurinol
B. Immediate hemodialysis
C. Urine alkalization
D. Furosemide
E. Rasburicase

26. A 35-year-old woman presents to the emergency department with fever and generalized weakness 10 days after starting her third cycle of
consolidation chemotherapy for acute myeloid leukemia. She denies cough, mouth sores, and exposure to illness. She has hypotension, tachycardia,
and clear lungs. Her central venous line is clean and nontender to palpation. WBC count is 300/L, hemoglobin level is 9 g/dL, and platelet count is
20,000/L. After blood cultures are sent, she is admitted to the medical ICU.

Which of the following management steps is most appropriate for this patient?
A. Vancomycin alone
B. Amphotericin
C. Fluconazole
D. Waiting on culture results to guide antibiotic choice
E. Ceftazidime and vancomycin

27. A 45-year-old man is brought by his wife because of a 2-week history of dyspnea, fatigue, and fever. On physical examination, he is pale and has
gum hypertrophy. BP is 100/70 mm Hg, pulse rate is 95/min, temperature is 37.3C (99F), and pulse oximetry is 87% on room air. Laboratory
work-up reveals a WBC count of 300,000/L with 95% blasts, platelet count of 25,000/L, and hemoglobin level of 8 g/dL. Chest radiography
shows diffuse interstitial infiltrates.

Which of the following is the hematologist most likely to recommend for symptom control?
A. Leukapheresis
B. Exchange transfusion
C. Combination chemotherapy
D. Bone marrow biopsy
E. Emergent whole-body radiation
28. A 65-year-old woman presents with dyspnea and facial fullness. The dyspnea has worsened significantly on the day of admission. On examination,
she has plethora, mild right arm swelling, and prominent veins on the right side of her chest. BP is 130/85 mm Hg, pulse rate is 120/min, RR is 24
/min, temperature is 37.3C (99F), and pulse oximetry is 87% on room air. Chest radiography shows a right upper lobe density.

Which of the following is likely to provide the quickest relief and is the best next option?
A. Warfarin
B. Diuretics
C. Steroids
D. Intravascular stenting
E. Radiation therapy

29. A 70-year-old woman is brought to the emergency department because of confusion and somnolence at home. On examination, she appears thin. Her
vital signs are normal and she is not orthostatic. Her mucous membranes are moist and her skin turgor is normal. During the examination, she has a
witnessed seizure. Laboratory evaluation reveals sodium level of 120 mEq/L; blood urea nitrogen and creatinine levels are normal. Complete blood
count shows a WBC count of 8,500/L, hemoglobin level of 9 g/dL, and platelet count of 200,000/L. Chest radiography shows a large hilar
shadow and chest CT confirms a pulmonary mass.

Which of the following best explains the pathophysiology of this condition?


A. Polydipsia that induces a hypervolemic state
B. Volume depletion due to poor oral intake
C. Inability to excrete dilute urine and retention of water due to excessive release of antidiuretic hormone
D. Reduced cardiac output leading to the release of antidiuretic hormone via the J receptors
E. Impaired renal absorption of sodium in the proximal renal tubule
PART 5: Hepatic, Gastrointestinal, Hematologic and Oncologic Disease in
the ICU

ANSWERS:

1C; 2C; 3A; 4D; 5D; 6E; 7A; 8C; 9B; 10D; 11C; 12D; 13A; 14D; 15E; 16C; 17C; 18D; 19C; 20E; 21C; 22A; 23C; 24A;
25E; 26E; 27A; 28D; 29C

RATIONALE (1) Answer: C

The risk of developing stress-related gastrointestinal bleeding varies in critically ill patients. It is important to recognize patients with increased risk
factors in order to develop cost-effective prophylactic regimens. In a landmark study on risk factors, Cook et al1 found that the strongest independent risk
factors for the development of stress-related bleeding in ICU patients were mechanical ventilation for more than 48 hours (RR 16/min, P < 0.01) and
coagulopathy (RR 4.3/min, P < 0.01). Coagulopathy was defined as a platelet count less than 50,000/mL, prothrombin time greater than 1.5 times control
value, or a partial thromboplastin time greater than twice the control value. Hypotension was found to have an increased odds ratio (3.7), but did not
achieve statistical significance. Renal failure, sepsis, liver failure, enteral feeding, and glucocorticoids did not achieve statistical significance. In this
study, the incidence of clinically significant stress-related bleeding was 3.7% in patients with risk factors compared to 0.1% in patients without risk
factors.

REFERENCES (1)

1. Cook DJ, Fuller HD, Guyatt GH, et al. Risk factors for gastrointestinal bleeding in critically ill patients: Canadian Critical Care Trials Group. N
Engl J Med 1994;330:377381.
2. Stainberg K. Stress-related mucosal disease in the critically ill patients: risk factors and strategies to prevent stress-related bleeding in the intensive
care unit. Crit Care Med 2002;30:S362S365.

RATIONALE (2) Answer: C

The patient's presentation is consistent with acute colonic pseudo-obstruction, also known as Ogilvie syndrome. Ogilvie syndrome is characterized by
gross dilation of the colon, usually the cecum, in the absence of an anatomic obstructing lesion. The small intestine might also be dilated in some cases.
This syndrome can be seen in critically ill patients admitted to the ICU or long-term acute care centers.

Risk factors include electrolyte abnormalities and use of antimotility agents and narcotics. The exact mechanism that produces colonic atony remains
unclear. However, autonomic nervous dysfunction is thought to play an important role. Clinical presentation usually includes abdominal distension,
tympany, diffuse tenderness, and reduced bowel sounds. Plain radiographs will reveal a dilated colon, most often cecum. Complications such as
ischemia and perforation are uncommon with cecal diameters of less than 12 cm. The rate of development and the duration of dilation probably relate
better to the rate of complications. Management involves supportive measures, with treatment of the underlying disease and the correction of any
electrolyte abnormalities. Gastrointestinal decompression with nasogastric and rectal tubes can be used.

Studies have demonstrated the effectiveness of IV neostigmine, 2 mg, in producing rapid decompression and preventing recurrence. Neostigmine is well
tolerated in general but may produce bradycardia, which is responsive to atropine. Cases refractory to conservative and medical treatment may require
colonoscopy for decompression. Colonoscopy in the ICU patient with an unprepared bowel has a high complication rate and should be avoided, if
possible. Surgery is indicated in patients where other therapeutic modalities fail or in those who develop complications such as ischemia and
perforation. Initiation of central venous alimentation is not indicated in patients with prompt response to therapy.

REFERENCES (2)

1. Ponec RJ, Saunders MD, Kimmey MB. Neostigmine for the treatment of acute colonic pseudoobstruction. N Engl J Med 1999;341:137140.
2. Seth SG, LaMont JT. Gastrointestinal problems in the chronically critically ill patient. Clin ChestMed 2001;22:135147.
3. Stephenson BM, Morgan AR, Salaman JR, et al. Ogilvie's syndrome: a new approach to an old problem. Dis Colon Rectum 1995;38:422427.
4. Turegano-Fuentes F, Munoz-Jimenez F, Del Valle-Hernandez E, et al. Early resolution of Ogilvie's syndrome with intravenous neostigmine: a
simple, effective treatment. Dis Colon Rectum 1997;40:13531357.

RATIONALE (3) Answer: A

Placement of a large-bore nasogastric (NG) tube has been described as one of the most painful experiences a conscious patient may endure. There are
very few evidence-based indications for use of a large-bore NG tube. There is even data to suggest that patients do not need an NG tube after undergoing
bowel surgery. That being said, decompression of a suspected bowel obstruction can be an important therapeutic maneuver and may alleviate abdominal
distension, nausea/vomiting, and discomfort.

Although nasojejunal enteric feeds have proven benefit in patients with acute pancreatitis (option B), a large-bore tube is not necessary.

A large-bore NG tube is not necessary in diagnosing an upper gastrointestinal tract bleed (option C). In fact, it can be misleading. Aggressive
resuscitation, medical therapy, and urgent endoscopy is the standard of care for suspected upper gastrointestinal tract bleeding. Although a large-bore
NG tube may theoretically assist with clearing out large blood clots from the stomach and enabling better endoscopic viewing, adequate clearance is
rarely achieved and IV erythromycin is a less invasive choice.

A large-bore NG tube is not indicated with refractory nausea and vomiting (option D). A Dobhoff (small-bore) feeding tube may be necessary, and
antiemetic therapy with IV fluids should be the initial management during investigation of the cause of such symptoms.

A large-bore NG tube is contraindicated in cases of suspected esophageal rupture. Therefore, option E is also incorrect.

REFERENCES (3)

1. Leung FW. The venerable nasogastric tube. Gastrointest Endosc. 2004 Feb; 59:25560.
2. Brolin RE. Partial small bowel obstruction. Surgery. 1984 Feb;95:1459.

RATIONALE (4) Answer: D

The patient presents with a likely nonsteroidal antiinflammatory druginduced bleeding gastric ulcer with some high-risk features (visible vessel,
adherent clot). Medical and endoscopic therapy is applied and the patient stabilizes. Forty-eight hours later, the patient has a significant rebleed. Since
he is stabilized, a repeat esophagogastroduodenoscopy (EGD) to confirm the site of rebleeding is the best answer. If the same ulcer is rebleeding, repeat
endoscopic therapy is certainly indicated and will reduce the risk of rebleeding. The original ulcer characteristics (only 1.5 cm, unlikely to be
malignant) do not mandate surgery. There is no information given that suggests perforation; therefore, CT (option B) is not needed.

If the patient were too unstable for upper endoscopy, an attempt at angiography (option E) would be reasonable.

REFERENCE (4)

1. Marmo R, Rotondano G, Bianco MA, et al. Outcome of endoscopic treatment for peptic ulcer bleeding: is a second look necessary? A meta-analysis.
Gastrointest Endosc. 2003;57:627.

RATIONALE (5) Answer: D

Based on the clinical scenario and other laboratory abnormalities, the most likely cause of thrombocytopenia in this patient is a dilutional effect. Most of
the literature regarding dilutional coagulopathy in the setting of massive blood transfusions relates to trauma. Traditionally, transfusing more than 10
units of packed red blood cells has been considered "massive," and it is recommended to give fresh frozen plasma after approximately 6 units of packed
red blood cells have been transfused. Dilution can cause a rise in prothrombin time, partial thromboplastin time, and international normalized ratio
(INR), as well as a drop in platelet count.

Although Helicobacter pylori is associated with chronic idiopathic thrombocytopenic purpura, the acuity of the thrombocytopenia and other lab
abnormalities makes this answer (option A) incorrect.

Although disseminated intravascular coagulation is a possible cause of thrombocytopenia and elevated INR, there is no information to suggest that the
patient has acute infection or sepsis. Therefore, option B is incorrect.

There is no mention that the patient received heparin therapy (and he probably did not since he presented with a gastrointestinal bleed), and the timing of
the decrease in platelets would be a bit unusual for heparin-induced thrombocytopenia. Therefore, option C is also incorrect.

Proton pump inhibitors are not likely to cause acute liver failure (option E), and there is no clinical evidence of such a process in this case. The elevated
INR is due to a dilutional coagulopathy, and the indirect bilirubin elevation is probably due to the blood transfusions as well.

REFERENCES (5)

1. Reed RL, Ciavarella D, Heimbach DM, et al. Prophylactic platelet administration during massive transfusion. A prospective, randomized, double-
blind clinical study. Ann Surg. 1986 Jan;203:408.
2. Counts RB, Haisch C, Simon TL, et al. Hemostasis in massively transfused trauma patients. Ann Surg. 1979;190:9199.
3. Miller RD, Robbins TO, Tong MJ, et al. Coagulation defects associated with massive blood transfusions. Ann Surg. 1971;174:794801.

RATIONALE (6) Answer: E

This patient has developed isolated right-sided ischemic colitis, probably secondary to an embolic event due to atrial fibrillation. Right-sided ischemic
colitis portends the worst prognosis of all types of colonic ischemia with regard to subsequent mesenteric ischemia. This pattern is seen with superior
mesenteric artery ischemia, and most patients will go on to require surgery. Since this process rarely leads to recovery following conservative
management, supportive care and observation (option A) is not the appropriate therapy. Imaging and a surgical consultation (option E) should be
performed. Corticosteroids (option B) and antibiotics (option C) are not indicated since the etiology of the colonic inflammation is not inflammatory or
infectious. Mesenteric angiography (option D) may further evaluate her anatomy and assess for arterial stenosis, but it is invasive. Since the patient is
clinically improving, evaluation with MR angiography or CT angiography may be more appropriate.

REFERENCE (6)

1. Sotiriadis J, Brandt LJ, Behin DS, Southern WN. Ischemic colitis has a worse prognosis when isolated to the right side of the colon. Am J
Gastroenterol. 2007 Oct;102:224752.

RATIONALE (7) Answer: A

The clinical presentation suggests hepatopulmonary syndrome (HPS), which is characterized by intrapulmonary shuntinginduced hypoxemia due to
pathologic pulmonary vascular dilation, and consequent diffusion-impeded transfer of oxygen from the alveolus to the pulmonary arteriole. The clinical
presentation is characterized by platypnea (subjective shortness of breath when moved from a supine to a sitting position) and orthodeoxia (desaturation
when moved from a supine to a sitting position). The proposed explanation for the exacerbation of the hypoxemia in the sitting position is that more
blood flow is directed to the basilar West zone 3 lung segments, where the pathologic pulmonary vasodilation is more pronounced. The test of choice to
support the diagnosis of HPS is a transthoracic echocardiogram with a bubble study, which will demonstrate delayed appearance of bubbles in the left
heart in 4 to 6 cardiac cycles. The other choices do not provide diagnostic information regarding HPS.

REFERENCE (7)

1. Rodriguez-Roisin R, Krowka MJ. Hepatopulmonary syndromea liver induced lung vascular disorder. N Engl J Med 2008;358:237887.

RATIONALE (8) Answer: C

In the setting of hepatic encephalopathy (HE) associated with acute-on-chronic liver failure, enteral administration of rifaximin has been shown to
reverse HE. The mechanism of action of rifaximin is the reduction of gastrointestinal ammonia production by the elimination of gut bacterial flora. In
contrast to HE associated with acute liver failure, the HE of acute-on-chronic liver failure is typically not associated with intracranial hypertension and
cerebral edema. Therefore, there is no role for empiric intracranial pressure monitoring, osmotherapy, or hypothermia in HE associated with acute-on-
chronic liver failure. An additional therapeutic option in treating the HE of acute-on-chronic liver failure is enteral or rectal lactulose.

REFERENCES (8)

1. Bass NM, Mullen KD, Sanyal A, et al. Rifaximin treatment in hepatic encephalopathy. N Engl J Med 2010;362:107181.
2. Frontera JA, Kalb T. Neurologic management of fulminant hepatic failure. Neurocrit Care 2011;14:31827.

RATIONALE (9) Answer: B

The presence of a hepatic hydrothorax increases the risk of spontaneous bacterial empyema (SBEM); therefore, pleural fluid analysis is indicated to
evaluate for this infectious process. The diagnostic criteria for SBEM are similar to those for spontaneous bacterial peritonitis (SBP), and the pleural
fluid analysis will demonstrate a transudate with an absolute neutrophil count greater than 250/mL, with or without a Gram stain and culture positive for
a specific microbial process. The etiology of SBEM involves the development of an infection in a preexisting hepatic hydrothorax, which occurs due to
the passage of ascites from the peritoneal to the thoracic cavity across diaphragmatic defects. The treatment is similar to SBP, and involves a 5-day
course of broad-spectrum antibacterial therapy. There is no role for large-bore chest tube drainage in the treatment of SBEM, and in fact, this
intervention could lead to hypovolemic shock and reexpansion pulmonary edema.

REFERENCE (9)

1. Alonso JC. Pleural effusion in liver disease. Semin Respir Crit Care Med 2010;31:698705.

RATIONALE (10) Answer: D

In the absence of bleeding, there is no indication to reverse the coagulopathy associated with acute liver failure. Instead, since the international
normalized ratio is the best measure of hepatic synthetic function, it should be used as a biochemical marker to predict recovery of intrinsic hepatic
function or the need for liver transplantation. The risk of bleeding in the setting of coagulopathy associated with acute liver failure has been reported to
be extremely low in multiple observational studies. A potential explanation for these observations is the loss of both procoagulant and anticoagulant
factors in the setting of acute liver failure, resulting in a normal hemostatic equilibrium.

REFERENCE (10)

1. De Gasperi A, Corti A, Mazza E, et al. Acute liver failure: managing coagulopathy and the bleeding diathesis. Transplant Proc 2009;41:12569.

RATIONALE (11) Answer: C

Hemobilia is a rare cause of upper gastrointestingal tract bleeding that has increased in incidence due to more frequent hepatobiliary procedures such as
percutaneous liver biopsy, percutaneous transhepatic cholangiography, endoscopic retrograde cholangiopancreatography (ERCP) with
biopsy/brushing/stenting, and interventional angiography. Hemobilia is diagnosed by upper endoscopy, when blood is seen flowing from the ampulla of
Vater. Hemobilia is often treated with angiography with embolization.

Option A is incorrect since the patient is actively bleeding and is presenting with hemodynamic instability. Option B is incorrect as octreotide has no
role in treating hemobilia. Octreotide is indicated in cases of portal hypertensive bleeding, including varices. Option C is the correct answer. Option D
is incorrect, and there is no role for ERCP unless a biliary obstruction develops. Option E is not correctinterventional radiology should be called (not
surgery). If an intraperitoneal bleed is seen, surgery should be consulted.

REFERENCES (11)

1. Chin MW, Enns R. Hemobilia. Curr Gastroenterol Rep. 2010;12:1219.


2. Yoshida J, Donahue PE, Nyhus LM. Hemobilia: a review of recent experience with a worldwide problem. Am J Gastroenterol 1987;82:44853.

RATIONALE (12) Answer: D

This is a case of abdominal compartment syndrome (ACS) due to excessive ascites accumulation in a cirrhotic patient following aggressive
resuscitation for shock.

Option A is incorrect; an MRI may provide additional information regarding abdominal pathology, but is not the next required step in this critically ill
patient. Option B is incorrect; although antifungal therapy could be considered, it does not address the immediate concern for ACS. Option C is
incorrect; a repeat echocardiography is not indicated. Option D is correct; this will diagnose ACS, and a paracentesis will decompress the peritoneum
and relieve inferior vena cava compression, thereby improving venous return and mean arterial pressure. Option E is incorrect; though the oliguria may
progress to acute kidney injury, the best next step is acute decompression of the abdomen.

REFERENCES (12)

1. Mohmand H, Goldfarb S. Renal dysfunction associated with intra-abdominal hypertension and the abdominal compartment syndrome. J Am Soc
Nephrol. 2011;22: 61521.
2. Daugherty EL, Hongyan L, Taichman D, et al. Abdominal compartment syndrome is common in medical intensive care unit patients receiving large-
volume resuscitation. J Intensive Care Med. 2007;22:2949.
3. Nagappan R, Ernest D, Whitfield A. Recognition and management of intra-abdominal hypertension and abdominal compartment syndrome. Crit Care
Resusc. 2005;7:298302.
RATIONALE (13) Answer: A

The patient has hepatorenal syndrome, a disease that can be rapidly fatal. Criteria for diagnosis have been recently revised and highlight the importance
of infusion of albumin, 1 g/kg daily for 2 days prior to making the diagnosis.

Several therapies have been studied and failed to show any survival advantage. The combination of octreotide and midrodrine has been shown to
improve survival in such patients (option A is correct).

Dopamine administration has no known impact on hepatorenal syndrome, so option D is incorrect. Treating hepatitis C with antivirals in this patient is
contraindicated. Thus, option C is incorrect. Transjugular intrahepatic portosystemic shunt treatment can help to improve short-term outcomes in patients
awaiting liver transplant, but since it can be associated with complications, it should be considered as a last-resort therapy after all others have failed.
Therefore, option B is incorrect.

REFERENCES (13)

1. Esrailian E, Pantangco ER, Kyulo NL, et al. Octreotide/midodrine therapy significantly improves renal function and 30-day survival in patients with
type 1 hepatorenal syndrome. Dig Dis Sci. 2007;52:742.
2. Salerno F, Gerbes A, Gins P, et al. Diagnosis, prevention and treatment of hepatorenal syndrome in cirrhosis. Gut. 2007;56:1310.
3. Brensing KA, Textor J, Strunk H, et al. Transjugular intrahepatic portosystemic stent-shunt for hepatorenal syndrome. Lancet. 1997;349:697.

RATIONALE (14) Answer: D

One of the approved indications for use of recombinant factor VIIa is serious bleeding in hemophiliacs with inhibitors. Factor VIIa enhances thrombin
generation, leading to hemostasis. Factor VIII in any form (cryoprecipitate, plasma-derived, or recombinant) would be ineffective due to the presence of
inhibitors. Factor XI requires factor VIII as a cofactor to activate factor X. Fresh frozen plasma does not contain significant amounts of factor VII to
allow sufficient thrombin generation for effective clotting. Activated prothrombin complex concentrates are another hemostatic option for hemophiliacs
with inhibitor levels.

REFERENCES (14)

1. Abshire T, Kenet G. Recombinant factor VIIa: review of efficacy, dosing regimens and safety in patients with congenital and acquired factor VIII or
IX inhibitors. J Thromb Haemost 2004;2:899909.
2. Goodnough LT, Lublin DM, Zhang L, et al. Transfusion medicine service policies for recombinant factor VIIa administration. Transfusion
2004;44:13251331.
3. Levi M, Peters M, Buller HM. Efficacy and safety of recombinant factor VIIa for treatment of severe bleeding: a systematic review. Crit Care Med
2005;33:883890.
4. Monahan PE, Aldedort LM. Factors affecting choice of hemostatic agent for the hemophilia patient with an inhibitor antibody. Am J Hematol
2004;77:346350.

RATIONALE (15) Answer: E

Uremia is one of the most common causes of platelet dysfunction in the critically ill, and bleeding from invasive procedures and surgery may be difficult
to control. Several options exist to correct platelet dysfunction. Hemodialysis to lower the blood urea nitrogen level would be optimal, but requires
considerable time to institute at night in many situations. Hemodialysis may also contribute to bleeding by continuous platelet activation, which is
induced by the interaction between blood and artificial surfaces and results in platelet exhaustion. Thus, option B is incorrect. Continuous renal
replacement techniques would not be expected to improve the uremia rapidly.

IV desmopressin acetate (DDAVP), 0.3 g/kg, has a rapid effect on improving platelet function. The onset of action is within 1 hour and the effects last
48 hours. Subcutaneous injection and nasal spray are alternative routes of administration. DDAVP results in a 2- to 6-fold increase in both factor VIII
and von Willebrand factor, which increases platelet adhesiveness. Unfortunately, this effect is short-lived and patients rapidly develop tachyphylaxis.
DDAVP is most helpful in controlling emergent bleeding while other measures are being instituted.

Platelet transfusions (option A) would be of no benefit since transfused platelets are exposed to the same uremic environment.

Conjugated estrogens (option C) are of value in improving platelet dysfunction, but the effect takes several days. Therefore, they are not an option for
emergent interventions. Erythropoietin is another long-term treatment option that improves platelet function.

Cryoprecipitate (option D) has been used in the past, but is no longer recommended because it is not effective in many patients. It also exposes the
patient to transfusion-related complications. Keep in mind that the use of antibiotics such as penicillins and certain cephalosporins may also potentiate
platelet dysfunction.

REFERENCES (15)

1. Boccardo P, Remuzzi G, Galbusera M. Platelet dysfunction in renal failure. Semin Thromb Hemostasis. 2004;30:579589.
2. Carvalho AC. Acquired platelet dysfunction in patients with uremia. Heme Oncol Clin N Amer. 1990;4:129143.
3. DeLoughery TG. Management of bleeding with uremia and liver disease. Curr Opin Hematol. 1999;6:329633.

RATIONALE (16) Answer: C

This patient has thrombotic thrombocytopenic purpura (TTP), most likely as a result of treatment with ticlopidine after his cardiac catheterization. TTP
is characterized by the pentad of thrombocytopenia, microangiopathic hemolytic anemia, fever, neurologic findings, and renal dysfunction. These clinical
features rarely present simultaneously. The peripheral smear that is shown has fragmented red cells and schistocytes, which are typical findings for
microangiopathic hemolytic anemia. In the context of the clinical presentation, TTP is the most likely diagnosis. Patients do not require all elements of
the pentad to be fulfilled for diagnosis of TTP, and a high level of vigilance must be maintained to identify these patients in a timely fashion and institute
appropriate treatment. TTP is associated with invasive enteric infections, chemotherapeutic agents such as mitomycin, cancer, HIV infection, and
antiplatelet agents (ticlopidine and clopidogrel). There is also an idiopathic form of TTP. TTP seems to be initiated by endothelial damage that leads to
the accumulation of large von Willebrand factor multimers that lead to platelet aggregation and thrombin formation in affected organ systems.

The treatment of TTP, regardless of the cause, is the discontinuation of the offending agent followed by daily plasma exchange (plasmapheresis plus
infusion of fresh frozen plasma). Prompt therapy with plasma exchange decreases the mortality of TTP to 10%, and permanent organ dysfunction is
uncommon. In severe cases that are refractory to conventional therapy, glucocorticoid therapy, and also splenectomy, have been reported to be useful
adjunctive measures.

Patients with TTP in general should not be given platelet transfusions (unless there is life-threatening or intracranial bleeding). Argatroban is reserved
for patients who develop heparin-induced thrombocytopenia and require systemic anticoagulation. There has been no proven benefit or role for systemic
corticosteroids in the initial treatment of TTP. Low-molecular-weight heparin and immunoglobulin are not indicated in TTP.

REFERENCES (16)

1. George JN. Thrombotic thrombocytopenic purpura. N Engl J Med. 2006;354:19271935.


2. Guidelines on the diagnosis and management of the thrombotic microangiopathic haemolytic anaemias. Br J Haematology. 2003;120:556573.
3. Kwaan HC, Boggio LN. The clinical spectrum of thrombotic thrombocytopenic purpura. Semin Thromb Hemostasis. 2005;31:673680.
4. Moake JL. Thrombotic microangiopathies. N Engl J Med. 2002;347:589600.
5. Medina PJ, Sipols JM, George JM. Drug-associated thrombotic thrombocytopenic purpura-hemolytic uremic syndrome. Curr Opin Hematol. 2001;
8:286293.
6. Rock G. The management of thrombotic thrombocytopenic purpura in 2005. Semin Thromb Hemostasis. 2005;31:709716.

RATIONALE (17) Answer: C

Patients with immune thrombocytopenic purpura (ITP) usually do not have significant bleeding because the few platelets present are young and active in
coagulation. This patient has a comorbidity that makes urgent surgical intervention necessary. Although steroids are the first-line treatment, platelet
counts may not increase for several days to weeks. IV immunoglobulin would be the best intervention to accomplish a transient increase in platelets
during the operative intervention. It is administered at 0.51 g/kg/day, along with methylprednisolone, 1 g/day. Depending on the response, platelet
transfusions may be needed at the time of surgery.

Another option is the use of anti-D immunoglobulin, but this is an option only in patients who are Rh Dpositive and have a spleen. The hemolysis that
can occur with anti-D immunoglobulin would also preclude its use in a patient undergoing surgery.

Platelet transfusions alone are unlikely to benefit this patient, since they would be subject to the same antibody destruction as endogenous platelets.
Splenectomy would not be warranted; rituximab and danazol are options for chronic ITP and would be unlikely to have rapid effects on the platelet
count.

REFERENCES (17)

1. Cines DB, Blanchette VS. Immune thrombocytopenic purpura. N Engl J Med. 2002;346:9951008.
2. Stasi R, Provan D. Management of immune thrombocytopenic purpura in adults. Mayo Clin Proc. 2004;79:504522.
3. Prvan D, Stasi R, Neweland AC, et al. International consensus report on the investigation and management of primary immune thrombocytopenia.
Blood 2010;115:16886.
4. Cines DB, Bussel JB, Liebman HA, et al. The ITP syndrome: pathogenic and clinical diversity. Blood 2009;113:65116522.

RATIONALE (18) Answer: D

This patient has developed heparin-induced thrombocytopenia (HIT) type II, with arterial thrombosis of the right lower extremity. There are 2 types of
heparin-associated thrombocytopenia. Type I occurs in 10%20% of patients receiving unfractionated heparin. Nonimmune factors result in mild
decreases in platelet counts after 1 to 4 days of therapy with heparin. Most patients will normalize the platelet count in a few days despite heparin
continuation. Type II is more severe and is associated with thrombotic complications in 30%80% of cases. Development of HIT type II is related to the
formation of heparin-platelet factor 4 complexes and the induction of immunoglobulin G antiheparin-PF4 antibodies. HIT type II usually develops after 5
days of heparin exposure. However, in patients with prior exposure to heparin it may appear earlier. HIT may also occur after heparin is discontinued.

Cardiac and orthopedic surgery patients and ICU patients are at higher risk of developing HIT. Once HIT type II is suspected, all sources of heparin,
including low-molecular-weight heparins, should be discontinued. Patients with thrombotic complications (venous or arterial) from HIT, or other
indications for anticoagulation should be treated with antithrombin agents such as lepirudin or argatroban, or danaparoid (not available in the United
States).

However, lepirudin is renally eliminated and needs dose adjustment for renal dysfunction. Therefore, in patients with renal insufficiency, argatroban
would be the preferred choice since it is hepatically metabolized.

Warfarin anticoagulation can exacerbate the prothrombotic state of HIT type II prior to achieving full anticoagulation with an international normalized
ratio greater than 2.0. As a consequence, warfarin should be withheld until there is evidence of improvement and the patient is fully anticoagulated with
a parenteral antithrombin agent.

REFERENCES (18)

1. Arepally GM, Ortel TL. Heparin-induced thrombocytopenia. N Engl J Med. 2006;355:809817.


2. Greinacher A, Farner B, Kroll H, et al. Clinical features of heparin-induced thrombocytopenia including risk factors for thrombosis. Thromb
Haemost. 2005;94:132135.
3. Warkentin TE. Heparin-induced thrombocytopenia: pathogenesis and management. Br J Haema. 2003;121:535555.
4. Warkentin TE. Management of heparin-induced thrombocytopenia: a critical comparison of lepirudin and argatroban. Thromb Res. 2003:110:7382.
5. Shantsila E, Lip GY, Chong BH. Heparin-induced thrombocytopenia. A contemporary clinical approach to diagnosis and management. Chest
2009;135:165164.

RATIONALE (19) Answer: C

Von Willebrand disease is the most common inherited coagulation disorder. The most common type, type I, is due to decreased release or production of
von Willebrand factor (VWF). Other types are due to abnormal VWF polymers or decreased levels of VWF and factor VIII activity. Bleeding is usually
only noted with trauma, surgery, and other invasive procedures. Impaired platelet adhesion at the vascular site of injury results in bleeding. Coagulation
studies reveal only a prolonged partial thromboplastin time and increased bleeding time. Treatment of von Willebrand disease includes desmopressin
acetate (DDAVP), factor VIII concentrate, and cryoprecipitate. DDAVP is effective in type I disease only. Factor VIII concentrate is preferred over
cryoprecipitate because of lower infection risk.

Acquired von Willebrand disease has also been described. Associated conditions include lymphoproliferative and myeloproliferative disorders and
cardiovascular defects such as endocarditis and septal defects.

Salicylates (option D) would not prolong the partial thromboplastin time, and citrate toxicity (option E) is unlikely since only 4 units of red blood cells
were transfused. Hypothermia (option B) postoperatively can result in platelet dysfunction, but is unlikely to result in an isolated prolonged partial
thromboplastin time. Likewise, the normal platelet count and normal prothrombin time make disseminated intravascular coagulation (option A) an
unlikely cause of the oozing.

REFERENCES (19)

1. Kumar S, Pruthi RK, Nichols WL. Acquired von Willebrand disease. Mayo Clin Proc. 2002; 77:181187.
2. Mannucci PM. Treatment of von Willebrand's disease. N Engl J Med. 2004;351:683694.

RATIONALE (20) Answer: E


In this stable patient recovering from septic shock without evidence of bleeding, no blood products are needed at this time. According to the Canadian
Critical Care Trials Group prospective study of transfusion thresholds2, transfusion of red blood cells is recommended only when hemoglobin level
decreases below 7 g/dL, and should target a hemoglobin level of 7 to 9 g/dL. The effect of red blood cell transfusion in patients with sepsis has been
evaluated in several studies and shows an increase in oxygen delivery but no increase in oxygen consumption. The transfusion threshold recommended in
a stable patient is distinct from the target hematocrit of 30% in patients with low central venous oxygen saturation during the first 6 hours of resuscitation
of septic shock.

No clinical trials have addressed the transfusion of platelets and fresh-frozen plasma (FFP) in patients with sepsis, and recommendations are based on
consensus opinion and experience in other patient groups. In patients with severe sepsis, platelets should be administered when counts are less than or
equal to 5,000/L, regardless of apparent bleeding. Platelet transfusion can be considered when counts are 5,000 to 30,000/L and there is a significant
risk for bleeding. Higher platelet counts may be required if surgery or invasive procedures are planned.

In the presence of active bleeding or prior to surgical or invasive procedures, FFP is indicated for coagulopathy due to a documented sufficiency of
coagulation factors. In the absence of bleeding, routine use of FFP to correct laboratory abnormalities is not recommended.

REFERENCES (20)

1. College of American Pathologists. Practice parameter for the use of fresh-frozen plasma, cryoprecipitate, and platelets. JAMA. 1994;271;777781.
2. Hbert PC, Wells G, Blajchman MA, et al. A multicenter, randomized, controlled clinical trial of transfusion requirements in critical care. N Engl J
Med. 1999;340:409417.
3. Zimmerman JL. Use of blood products in sepsis: an evidence-based review. Crit Care Med. 2004;32(suppl):S542S547.

RATIONALE (21) Answer: C

The clinical findings in association with the large lung mass are suggestive of hypercalcemia. The main clinical manifestations of hypercalcemia are
gastrointestinal (anorexia, nausea, vomiting, abdominal pain, constipation), cardiovascular (hypertension, prolonged PR and QRS intervals, shortened
QT interval, bradyarrhythmias), renal (polyuria, nephrocalcinosis), neurologic (apathy, lethargy, coma), and skeletal (bone pain).

Hypercalcemia due to lung cancer is usually associated with nonsmall cell tumors, and the most common mechanism is parathyroid hormonerelated
peptide. Other mechanisms of hypercalcemia associated with malignancy include ectopic production of vitamin D and bone metastasis. The initial step
in management of severe hypercalcemia involves replacing intravascular volume with isotonic saline solution to increase renal blood flow and enhance
calciuresis. After volume expansion is achieved, administration of a loop diuretic to increase renal excretion of calcium may help. Potassium and
magnesium will usually require replacement during diuresis.

Additional specific therapy is usually required with extreme elevations of calcium, and includes bisphosphonates and calcitonin. Glucocorticoids may
be helpful in hypercalcemia associated with excess vitamin D, such as granulomatous diseases or in patients with hematologic malignancy, such as
lymphoma or multiple myeloma.

Although phosphate is effective in lowering the serum calcium concentration, this therapy may lead to the precipitation of calcium-phosphate complexes,
and is not recommended as initial therapy in hypercalcemia.

REFERENCES (21)

1. Body JJ, Bouillon R. Emergencies of calcium homeostasis. Rev Endocr Metab Dis. 2003;4:167175.
2. Grill V, Martin TJ. Hypercalcemia of malignancy. Rev Endocr Metab Dis. 2000;1:253263.
3. Krimsky WS, Behrens RJ, Kerkvliet GJ. Oncologic emergencies for the internist. Cleve Clin J Med. 2002;69:209217.
4. Nelson KA, Walsh D, Abdullah O, et al. Common complications of advanced cancer. Semin Oncol. 2000;27:3444.
5. Stewart AF. Hypercalcemia associated with cancer. N Engl J Med. 2005;352:373379.

RATIONALE (22) Answer: A

This patient's clinical history is highly suggestive of acute chest syndrome associated with sickle cell disease. Adult patients typically develop the
syndrome 2 to 3 days after hospitalization for a vaso-occlusive crisis. The most common clinical manifestations are fever, cough, tachypnea, chest pain,
and a decrease in oxygen saturation. The etiology of the syndrome may be infection (most commonly Chlamydia and Mycoplasma), fat embolism, or
atelectasis due to respiratory splinting.

Patients should receive oxygen therapy as needed, and antibiotics to cover for typical pathogens (a cephalosporin and macrolide or quinolone).
Exchange transfusion would be preferred in this patient over simple transfusion because of his higher hemoglobin level. Simple and exchange
transfusions are capable of improving oxygenation. Exchange transfusion aims to decrease hemoglobin-S to less than 30%50%. The hemoglobin should
be maintained at 11 g/dL to avoid hyperviscosity, which could result in additional complications such as stroke.

Exchange transfusion is often preferred in patients who have more severe diseases, multilobe lung involvement, persistent or worsening hypoxemia,
neurologic abnormalities, or multiorgan failure. Leukocyte-poor blood should be used in these patients.

Although dexamethasone, 0.3 mg/kg, was shown to reduce the length of hospitalization in children with acute chest syndrome, a rebound effect resulted
in increased readmission in the treatment group. No studies are available in adults, and use of dexamethasone should be considered experimental. Thus,
option C is incorrect.

Bronchoalveolar lavage (option D) should be reserved for those patients who fail to respond to initial therapy.

Although there are anecdotal reports of use of inhaled nitric oxide (option E) in mechanically ventilated patients, this application is also experimental
and should be reserved for patients with the most severe diseases that are unresponsive to other mechanical ventilation techniques.

REFERENCES (22)

1. Ballas SK. Sickle cell anemia: progress in pathogenesis and treatment. Drugs. 2002; 62:11431172.
2. Danielson CSM. The role of red blood cell exchange transfusion in the treatment and prevention of complications of sickle cell disease. Ther Apher.
2002;6:2431.
3. Stuart MJ, Setty BNY. Acute chest syndrome of sickle cell disease: new light on an old problem. Curr Opin Hematol. 2001;8:111122.
4. Vichinsky EP, Neumayr LD, Earles AN, et al. Causes and the outcomes of the acute chest syndrome and sickle cell disease. N Engl J Med.
2000;342:18551865.
5. Vij R, Machado RF. Pulmonary complications of hemoglobinopathies. Chest 2010;138:97383.

RATIONALE (23) Answer: C

Spinal cord compression is a growing problem among cancer patients, partly because of improvements in local control and increasing life expectancy. It
affects 5%10% of all cancer patients and approximately 20,000 cases per year are diagnosed in the United States. Incidence is highest among patients
with breast, lung, and prostate cancer. It generally carries a poor prognosis, with the exception of hematological malignancies, which are exquisitely
chemosensitive and radiosensitive. Pain is the most common symptom at presentation and can rapidly progress to weakness, sensory changes, and bowel
or bladder dysfunction. Therefore, early diagnosis and intervention are key to regaining or preserving function. MRI is the diagnostic test of choice. It is
important to image the entire spine because up to 38% of patients have multiple lesions. The mainstays of treatment are steroids, radiation, and surgical
decompression. Dexamethasone is the preferred corticosteroid because of its central nervous system penetration and ability to provide immediate
palliation. However, steroids alone are not sufficient for long-term therapy. One randomized trial showed that immediate surgical decompression
followed by radiation therapy was superior to radiation therapy alone. Compared to the radiation-only group, the surgery group retained the ability to
walk in more cases (84% versus 57% of the time, P=0.001) and for a longer duration (median 122 days versus 13 days, P=0.003). Median survival was
also significantly higher in the surgery group compared to the radiation-alone group (126 days versus 100 days, P= 0.0330). Strikingly, in the surgery
group, 10 of the 16 paraplegic patients regained the ability to walk, compared to 3 of the 16 paraplegic patients in the radiation-only arm
(P=0.012).There was no difference in days of hospitalization, and 30-day morbidity was higher among patients treated with radiation alone. The
National Comprehensive Cancer Network recommends decompressive surgery and adjuvant radiation therapy in the setting of spinal instability and no
contraindication to surgery.

Chemotherapy may be an option for adjuvant therapy but would not produce the rapid effect expected with surgical decompression, with the exception of
leukemias or certain lymphomas.

REFERENCES (23)

1. Patchell RA, Tibbs PA, Regine WF, et al. Direct decompressive surgical resection in the treatment of spinal cord compression caused by metastatic
cancer: a randomised trial. Lancet. 2005;366:64348
2. Sun H, Nemecek AN. Optimal management of malignant epidural spinal cord compression. Emerg Med Clin North Am. 2009; 27:195208.
3. Schmidt MH, Klimo P Jr, Vrionis FD. Metastatic spinal cord compression. J Natl Compr Canc Netw. 2005;3:7119.

RATIONALE (24) Answer: A

Disseminated intravascular coagulation (DIC) is common in patients with hematological malignancies. It contributes to significant morbidity and
mortality predominantly due to bleeding, but also due to thrombotic complications, which are more common in patients with solid tumors.
Approximately 15% of patients with acute leukemia present with overt DIC, with the most striking presentation and harmful impact in patients with acute
promyelocytic leukemia (APL), the most curable of the acute myeloid leukemias. Approximately 5%10% of patients with newly diagnosed APL die
from bleeding complications before initiation of chemotherapy. DIC is thought to result from a combination of endothelial damage, expression of
procoagulant factors by malignant cells, and cytokine release. There is generation of thrombin mediated by tissue factor, and a hyperfibrinolytic state
which is thought to predominate in APL. The patient in question presented with a probable acute leukemia and the data suggests the diagnosis of DIC.
However, a decreased fibrinogen level would further support the diagnosis and trigger therapy with cryoprecipitate (in addition to fresh frozen plasma
and platelets), which is the optimal repletion therapy for hypofibrinogenemia. D-dimer tests assist in the diagnosis, but do not guide repletion therapy.
Platelet function assay is unreliable in the setting of thrombocytopenia, so option C is incorrect. Prothrombin time and partial thromboplastin time mixing
studies are helpful when there are no other clues to the diagnosis. In this case, the circulating immature cells and the thrombocytopenia make acute
leukemia with DIC the most likely diagnosis. Therefore, option B is incorrect. Bone marrow biopsy and review of the peripheral blood smear will
confirm the diagnosis of acute leukemia and will guide chemotherapy choice but will not contribute to the immediate stabilization of the patient with
intracranial hemorrhage and DIC. Thus, options D and E are incorrect.

REFERENCES (24)

1. Franchini M, Di Minno MND, Coppola A. Disseminated intravascular coagulation in hematologic malignancies. Semin Thromb Hemost.
2010;364:388403.
2. Sanz MA, Tallman MS, Lo-Coco F. Tricks of the trade for the appropriate management of newly diagnosed acute promyelocytic leukemia. Blood.
2005;105:30193025.

RATIONALE (25) Answer: E

Tumor lysis syndrome results from the abrupt release of cellular components into the blood after rapid lysis of malignant cells. It occurs more frequently
in patients with a large tumor burden, tumors that are very sensitive to chemotherapy, and rapidly proliferating tumors, such as Burkitt lymphoma and
acute leukemias. The hallmark of tumor lysis syndrome is the combination of elevated levels of uric acid, potassium, and phosphorus and decreased
calcium levels. The clinical consequences include cardiac arrhythmias; paresthesias; muscle cramps, tetany, and seizures from electrolyte abnormalities;
and renal failure from uric acid nephropathy and precipitation of calcium phosphate complexes due to the rapid increase in phosphorus. Purine nucleic
acids are catabolized to hypoxanthine and xanthine by the enzyme xanthine oxidase, which can be inhibited by allopurinol. These intermediaries are
further converted to uric acid, which is relatively insoluble in water and at the lower pH in the kidneys. Therefore, in hyperuricemic conditions, as the
uric acid concentration increases, the likelihood of crystal formation and deposition in the tubules also increases and this can lead to renal failure. In
most nonhuman mammals, the enzyme urate oxidase further converts uric acid into allantoin, which is 510 times more soluble in urine than uric acid.
Rasburicase is a recombinant form of urate oxidase and was found to be effective at reducing uric acid levels and improving renal parameters in patients
with hematological malignancies considered to be at high risk for tumor lysis syndrome. Rasburicase is contraindicated in patients with glucose-6-
phosphate dehydrogenase deficiency.

Allopurinol (option A) is a reasonable alternative, but in the case of renal failure and hyperuricemia, the breakdown of uric acid is more likely to
improve renal function and lower uric acid levels.

Immediate hemodialysis (option B) is not indicated unless medical measures have failed to correct the metabolic derangements.

Urine alkalization can actually exacerbate the condition by promoting calcium-phosphate crystal formation. Therefore, option C is incorrect.

Diuretics such as option D may exacerbate the renal failure by promoting intravascular volume depletion.

REFERENCES (25)

1. Coiffier B, Altman A, Pui C-H, et al. Guidelines for the management of pediatric and adult tumor lysis syndrome: an evidence-based review. J Clin
Oncol. 2008;26:27672778.
2. Cairo MS, Coiffier B, Reiter A, et al; TLS Expert Panel. Recommendations for the evaluation of risk and prophylaxis of tumour lysis syndrome
(TLS) in adults and children with malignant diseases: an expert TLS panel consensus. Br J Haematol. 2010;149: 57886. Epub 2010 Mar 16.

RATIONALE (26) Answer: E

Febrile neutropenia is defined as a single oral temperature of at least 38.3C (100.9F), or a sustained temperature of 38C (100.3F) over 1 hour with
less than 500 neutrophils per microliter, or less than 1,000 neutrophils per microliter and a predicted decline to less than 500 neutrophils per microliter
over the next 48 hours. Febrile neutropenia carries a high mortality when treatment is delayed; therefore, all patients with febrile neutropenia should
receive empiric antimicrobial therapy. Initial antibiotic selection should take into consideration potential site(s) of infection, known colonization with
resistant organisms and local resistance patterns, clinical instability, recent antibiotic use (including prophylaxis), and drug allergies. Monotherapy is
recommended for the stable, nontoxic patient. The routine use of empiric vancomycin is not recommended, unless there is suspected catheter infection
based on examination findings, mucosal damage, known methicillin-resistant Staphylococcus aureus (MRSA) colonization, or shock. In that event, the
continued use of vancomycin should be reassessed within 23 days to prevent the increase in vancomycin-resistant strains. Institution of early antifungal
therapy is not routinely recommended, although many patients may be already taking fungal prophylaxis based on their disease and risk factors for fungal
infection. Antifungal therapy is recommended in the event of persistent febrile neutropenia (4 days) despite appropriate antibiotic coverage.

The addition of an aminoglycoside is also recommended as initial therapy for the unstable febrile neutropenia patient.

REFERENCES (26)

1. Hughes WT, Armstrong D, Bodey GP, et al. 2002 guidelines for the use of antimicrobial agents in neutropenic patients with cancer. Clin Infect Dis
2002;34:730751.
2. Paul M, Yahav D, Fraser A, et al. Empirical antibiotic monotherapy for febrile neutropenia: systematic review and meta-analysis of randomized
controlled trials. J Antimicrob Chemother 2006;57:17689.

RATIONALE (27) Answer: A

Leukostasis tends to occur at WBC counts above 100,000/L (hyperleukocytosis), but there is no clear cutoff for the diagnosis or initiation of therapy.
The clinical scenario should dictate the most appropriate management strategy. Leukostasis is uncommon in indolent or chronic leukemias and is most
common in those presenting with acute myeloid leukemia. Abnormal expression of cell adhesion molecules by leukemic blasts in addition to the
elevation in the white blood cell count with increased viscosity and sludging in the microvasculature contribute to the complications of leukostasis.
Hyperleukocytosis was associated with increased 30-day mortality in prospective and retrospective studies. Signs and symptoms of leukostasis include
respiratory distress due to leukemic infiltrates in the pulmonary and cardiac microvasculature, and central nervous system alternations, including
hemorrhage that can present with headache, visual disturbance, tinnitus, or overt neurologic deterioration. It is therefore important to perform a detailed
physical examination, including a funduscopic examination for signs of ongoing or impending clinical leukostasis. The most rapid way to reduce the
blast count is with mechanical leukapheresis, and hydroxyurea is used concomitantly for further cytoreduction. Hydroxyurea alone is appropriate in
hyperleukocytosis without signs or symptoms of leukostasis.

Bone marrow biopsy will confirm the diagnosis, but does not help with the immediate need for cytoreduction.

Combination chemotherapy will be definitive therapy, but is associated with increased morbidity and mortality in the setting of leukostasis. There is no
role for radiation therapy in this setting.

In addition to leukapheresis, it is important to institute hydration and tumor lysis prophylaxis in patients presenting with malignant hyperleukocytosis.

REFERENCES (27)

1. Marbello L, Ricci F, Nosari AM, et al. Outcome of hyperleukocytic adult acute myeloid leukaemia: A single-center retrospective study and review
of literature. Leukemia Research 2008;32:12211227.
2. Bug G, Anargyrou K, Tonn T, et al. Impact of leukapheresis on early death rate in adult acute myeloid leukemia presenting with hyperleukocytosis.
Transfusion 2007; 47:184350.
3. Thibaut A, Thomas X, Belhabri A, et al. Impact of pre-induction therapy leukapheresis on treatment outcome in adult acute myelogenous leukemia
presenting with hyperleukocytosis. Ann Hematol. 2000;79:5016.
4. Inaba H, Fan Y, Pounds S, et al. Clinical and biologic features and treatment outcome of children with newly diagnosed acute myeloid leukemia and
hyperleukocytosis. Cancer 2008;113:5229.

RATIONALE (28) Answer: D

Superior vena cava (SVC) syndrome results from occlusion of the SVC due to tumor, fibrosis, or thrombosis. Physical examination reveals distended
neck veins and plethora and may reveal adenopathy. Chest radiography may reveal a widened mediastinum or a hilar mass, and Doppler ultrasonography
or contrast CT may guide diagnosis. SVC syndrome is generally not considered an emergency but can progress to the point of airway compromise.
Immediate symptomatic management is therefore indicated. Anticoagulation (option A) would be appropriate if thrombosis were documented. Radiation
therapy (option E) requires a definitive diagnosis. Initial management includes supportive care, such as providing supplemental oxygen, elevating the
head of the bed, and intravascular stenting to relieve the obstruction (option D). Intravascular stenting allows for prompt symptomatic relief and allows
time to secure a diagnosis. Steroids and diuretics (options C and B, respectively) are often used, but efficacy data are lacking.

REFERENCES (28)

1. Cheng S. Superior vena cava syndrome: a contemporary review of a historic disease. Cardiology in Review. 2009;17:1623.
2. Wilson LD, Detterbeck FC, Yahalom J. Superior vena cava syndrome with malignant causes. N Engl J Med. 2007;356:18621869.
3. Tanigawa N, Sawada S, Mishima K, et al. Clinical outcome of stenting in superior vena cava syndrome associated with malignant tumors:
comparison with conventional treatment. Acta Radiol. 1998;39:669674.
RATIONALE (29) Answer: C

The syndrome of inappropriate antidiuretic hormone (SIADH) is a frequent cause of hyponatremia. It is historically associated with bronchogenic
carcinoma, but can occur as a result of certain medications, extreme pain, pulmonary and central nervous system disorders, and malignancy. The
diagnosis requires a low sodium level in the setting of euvolemia, concentrated urine, hypo-osmolar plasma, and normal adrenal and thyroid function.
SIADH is thought to result from failure of the negative feedback mechanisms that regulate release and inhibition of antidiuretic hormone (ADH) or from
ectopic production of ADH by certain tumors. ADH excess induces water retention by the kidney despite low serum osmolality and expanded
intracellular volume. The consequences of hyponatremia depend on the severity and rapidity of the disorder, and include somnolence, confusion,
seizures, and coma. Treatment depends on the chronicity of the syndrome and the clinical condition of the patient and ranges from water restriction to
hypertonic saline to slowly increase the sodium level.

The clinical scenario points to a malignant cause for hyponatremia and makes option A unlikely. The physical examination argues against volume
depletion (option B). Reduced cardiac output (option D) can lead to release of ADH, but via the carotid sinus baroreceptors. The hyponatremia in
SIADH is not related to renal sodium losses, so option E is incorrect.

REFERENCES (29)

1. Batcheller J. Disorders of antidiuretic hormone secretion. AACN Clinical Issues in Critical Care Nursing. 1992;3:3708.
2. Freda BJ, Davidson MB, Hall PM. Evaluation of hyponatremia: a little physiology goes a long way. Cleve Clin J Med. 2004;71:63950.
3. Cogan E, Debive M-F, Pepersack T, et al. Plasma levels of atrial natriuretic factor in SIADH. N Engl J Med. 1986;314:12581259.
PART 6: Renal and Metabolic Disorders in the ICU

Instructions: For each question, select the most correct answer.

1. A 67-year-old alcoholic is admitted to the ICU with severe intoxication (ethanol level 350 mg/dL) and requires intubation for airway protection. BP
is 102/58 mm Hg, HR is 101/min, RR is 14/min, and temperature is 37.1C (98.7F). Laboratory results include the following: sodium, 118 mEq/L;
potassium, 3.1 mEq/L; chloride, 88 mEq/L; bicarbonate, 25 mEq/L; glucose, 120 mg/dL; blood urea nitrogen, 24 mg/dL; creatinine, 1.3 mg/dL;
urine osmolarity, 380 mOsm/kg; and urine sodium, 5 mEq/L.

Which of the following interventions is most appropriate to treat the hyponatremia?


A. Normal saline solution
B. 3% saline solution
C. Water restriction
D. Conivaptan
E. Furosemide

2. Which of the following is an advantage of using continuous venovenous hemofiltration rather than intermittent hemodialysis in a critically ill patient
with acute renal failure?

A. Lower cost
B. Improved mortality
C. Reliable antibiotic dosing
D. Hemodynamic tolerance
E. Effective solute removal

3. A 54-year-old man found under a bridge is admitted to the ICU with altered mental status. He is intubated for airway protection. His BP is 100/56
mm Hg (after 1 L normal saline), HR is 108/min, RR is 18/min, and temperature is 37.9C (100.2F). Laboratory data are as follows: sodium, 142
mEq/L; potassium, 5.3 mEq/L; chloride, 102 mEq/L; bicarbonate, 22 mEq/L; blood urea nitrogen, 50 mg/dL; and creatinine, 4.8 mg/dL. Urine
studies show specific gravity of 1.014, pH of 5.6, no white blood cells, rare red blood cells, negative blood on dipstick, 1+ protein on dipstick,
moderate hyaline casts, urine sodium level of 54 mEq/L, and urine creatinine level of 60 mg/dL. Which of the following is the most likely cause of
the acute kidney injury?

A. Hypovolemia
B. Nephrotoxin
C. Urinary retention
D. Glomerulonephritis

4. A 70-year-old woman with chronic kidney disease, hypertension, and diabetes is admitted with nausea, fever, and hypotension. Her BP is 88/50
mm Hg, HR is 110/min, RR is 20/min, and temperature is 38.4C (101F). Laboratory results are as follows: pH, 7.35; PaCO2, 34 mm Hg; PaO2, 68
mm Hg (room air); sodium, 132 mEq/L; potassium, 4.0 mEq/L; chloride, 103 mEq/L; bicarbonate, 18 mEq/L; albumin, 2.1 g/dL; blood urea
nitrogen, 28 mg/dL; and creatinine, 1.8 mg/dL. Which of the following acid-base disorders is present?

A. Anion gap metabolic acidosis alone


B. Nonanion gap metabolic acidosis alone
C. Nonanion gap metabolic acidosis and respiratory alkalosis
D. Anion gap and nonanion gap metabolic acidosis

5. A 55-year-old man is admitted to the ICU with fever and hypotension. His medical history includes well-controlled hypertension and a 2-month
history of fatigue and weight loss. On the day of admission, he had some dysuria and had syncope when getting up to urinate. On admission, his BP
is 70/40 mm Hg, pulse rate is 110/min, temperature is 39C (102.1F), and RR is 18/min. The physical examination reveals a thin man in mild
distress. Laboratory results include the following: sodium, 128 mEq/L; potassium, 5.6 mEq/L; chloride, 102 mEq/L; bicarbonate, 16 mEq/L; blood
urea nitrogen, 28 mg/dL; creatinine, 1.5 mg/dL; and glucose, 60 mg/dL. WBC count is 12,000/L. Cardiac enzyme results are negative. Urinalysis
shows 50 WBCs and moderate bacteria. After normal saline, 3 L over 2 hours, the patient is alert but his BP is 80/50 mm Hg. Norepinephrine is
initated and titrated to 0.1 g/kg/min to maintain BP of 90/56 mm Hg. Broad-spectrum IV antibiotics are started.

Which of the following options is most appropriate at this time?


A. Increase norepinephrine infusion.
B. Administer IV dexamethasone, 4 mg.
C. Infuse hetastarch, 500 mL.
D. Administer fludrocortisone, 50 g.

6. A 65-year-old man with a heart transplant and chronic kidney disease is admitted to the ICU with nausea, diarrhea, and respiratory distress. BP is
100/62 mm Hg, HR is 115/min, RR is 26/min, and temperature is 37.3C (99F). Arterial blood gas results include pH of 7.23, PaCO2 of 17 mm Hg,
and PaO2 of 235 mm Hg (50% ventimask). Laboratory results are as follows: sodium, 123 mEq/L; potassium, 3.9 mEq/L; chloride, 97 mEq/L;
bicarbonate, 7 mEq/L; blood urea nitrogen, 119 mg/dL; and creatinine, 5.1 mg/dL (baseline 2.1 mg/dL). Which of the following acid-base disorders
is present?

A. Anion gap metabolic acidosis alone


B. Anion gap metabolic acidosis and respiratory alkalosis
C. Nonanion gap metabolic acidosis alone
D. Anion gap metabolic acidosis and nonanion gap metabolic acidosis

7. The rhythms shown in the Figure were noted on the monitor of a 28-year-old man admitted with injuries after being hit on his bicycle by an
automobile. He suffered bilateral femur fractures and a crush injury of the left ankle. Which of the following is the most appropriate initial
intervention?
First

Second
Third

Fourth

A. Epinephrine
B. Calcium chloride
C. Sodium bicarbonate
D. Sodium polystyrene sulfonate

8. A 25-year-old man with insulin-dependent diabetes mellitus is admitted to the ICU with uncontrolled diabetes. He has a 1-day history of nausea,
vomiting, and diarrhea, and he withheld insulin because of poor oral intake. His BP is 100/60 mm Hg, HR is 114/min, RR is 28/min, and
temperature is 37.3C (99F). Physical examination reveals dry mucous membranes and mild diffuse abdominal tenderness. Laboratory
examinations reveal the following: sodium, 135 mEq/L; potassium, 3.5 mEq/L; chloride, 110 mEq/L; carbon dioxide, 8 mEq/L; blood urea nitrogen,
22 mg/dL; creatinine, 1.2 mg/dL; albumin, 4.1 g/dL; and glucose, 350 mg/dL. Arterial blood gas measurements on FIO2 of 0.21 are pH, 7.25; PaCO2,
20 mm Hg; and PaO2, 95 mm Hg; serum ketones are present at a 1:8 dilution.

Which of the following is the best explanation for the acid-base abnormality in this patient?
A. Diabetic ketoacidosis alone
B. Diabetic ketoacidosis with hyperventilation
C. Diabetic ketoacidosis and diarrhea
D. Diabetic ketoacidosis and vomiting

9. A 65-year-old man weighing 70 kg (154 lbs) presents to the emergency department with headache and lethargy. CT of the head shows a brain lesion
with mass effect and midline shift. High-dose dexamethasone is started and the patient is transferred to the ICU for close neuromonitoring. On the
second hospital day, he has worsening headache and increased lethargy. He is intubated, and a repeat CT reveals worsening cerebral edema in the
area surrounding the mass, with increased midline shift. At this time, blood chemistry is significant for sodium level of 115 mEq/L.
Which of the following is the best next step in the management of this patient?
A. Give 3% sodium chloride solution at 100 mL/h to increase sodium level to 125 mEq/L over 12 hours.
B. Give 3% sodium chloride solution at 50 mL/h to increase sodium level to 127 mEq/L over 24 hours.
C. Give 3% sodium chloride solution in repeat 100-mL boluses to increase sodium level to 120 mEq/L within 1 hour.
D. Give 3% sodium chloride solution at 125 mL/h to increase sodium level to 127 mEq/L over 8 hours.

10. A 75-year-old woman presents to the hospital with dizziness and lethargy. Her past medical history is significant for hypertension, for which she
takes lisinopril, and for chronic back pain treated with acetaminophen and ibuprofen. She has had no previous abdominal surgeries. On physical
examination, she appears chronically ill with BP of 152/78 mm Hg and HR of 74/min. She is lethargic but arousable, with no focal neurologic
signs.

Laboratory data include the following: complete blood count, normal; sodium, 138 mEq/L; potassium, 3.6 mEq/L; chloride, 98 mEq/L; bicarbonate,
9 mEq/L; glucose, 130 mg/dL; calcium, 8.9 mg/dL; lactic acid, 1.2 mmol/L; ammonia, normal; aspartate aminotransferase, 65 U/L; alanine
aminotransferase, 70 U/L; international normalized ratio, 1.4; blood urea nitrogen, 30 mg/dL; and creatinine, 1.4 mg/dL (baseline 1.2 mg/dL).
Testing is negative for salicylates, urine drug screen results are negative, acetaminophen level is less than 10 g/mL, and ethanol level is negative.
Findings of chest radiography and urinalysis are normal and blood cultures are obtained.

IV fluids are administered; 48 hours later, the anion gap is normal, creatinine level is 1.2 mg/dL, and mental status is improved. She had a similar
episode 6 months ago, which also resolved within 23 days after hospitalization.

Which of the following is the most likely diagnosis?


A. Laboratory measurement error
B. Acute renal failure
C. Adverse drug effect
D. Early sepsis

11. Induction chemotherapy was initiated in a 45-year-old man with acute lymphoblastic leukemia 2 days ago. He now has diffuse weakness and
hyperkalemia. Upon evaluation the patient is also noted to be short of breath. He has been receiving IV normal saline at 100 mL/h since initiation of
chemotherapy, in addition to oral allopurinol.

On physical examination, his BP is 177/85 mm Hg, HR is 62/min, RR is 28/min, SpO2 is 94% on 2 L/min of oxygen via nasal cannula. He has
decreased breath sounds bilaterally and crackles in up to one-third of lung fields. Additionally, there is 2+ bilateral lower extremity edema. His
urine output has declined over the past 8 hours to less than 50 mL/h.

Laboratory work shows the following: sodium, 147 mEq/L; potassium, 7.2 mEq/L; chloride, 108 mEq/L; bicarbonate, 10 mEq/L; phosphorus, 13
mg/dL; calcium, 5.7 mg/dL; blood urea nitrogen, 68 mg/dL; and creatinine, 3.0 mg/dL (baseline 1.6 mg/dL).

Which of the following is the most appropriate next step in management of this patient?
A. Change IV fluids to 5% dextrose with sodium bicarbonate, 150 mEq/L.
B. Initiate rasburicase.
C. Increase the allopurinol dose.
D. Initiate hemodialysis.

12. An 80-year-old woman presents to the hospital with shortness of breath and hemoptysis. She has been feeling sick over the last few days, and this
morning she started coughing bright red blood. Her past medical history is significant for hypertension and diabetes. On physical examination she
has BP of 120/65 mm Hg, HR of 95/min, RR of 32/min, and SpO2 of 90% on nonrebreather mask. The rest of the physical examination is
unremarkable.
Her laboratory results are as follows: blood urea nitrogen, 98 mg/dL; creatinine, 7 mg/dL; sodium, 138 mEq/L; potassium, 4.5 mEq/L; chloride, 96
mEq/L; bicarbonate, 19 mEq/L; WBCs, 12,000/L; hemoglobin, 11.5 g/dL; platelets, 140,000/L; and lactic acid, normal. Urinalysis shows
specific gravity of 1.010, no WBCs, leukocyte esterase negative, and 3050 RBCs/hpf with dysmorphic features. Antinuclear antibody testing is
negative, perinuclear antineutrophil cytoplasmic antibody testing is positive, anti-glomerular basement membrane testing is negative, and
complement levels are normal.

Chest radiograph reveals bilateral alveolar opacities. After intubation due to progressive hypoxemia and increased work of breathing, a noncontrast
CT of the chest suggesting alveolar hemorrhage is obtained.

Which of the following is the most appropriate next step in this patients management?
A. Oral prednisone and cyclophosphamide with initiation of plasmapheresis
B. High-dose IV methylprednisolone daily for 3 days with oral cyclophosphamide
C. Oral cyclophosphamide and mycophenolate for 7 days
D. Hemodialysis, oral prednisone, and azathioprine

13. Which of the following is required before initiation of enteral nutrition in the critically ill patient?

A. Presence of bowel sounds


B. Bowel movements
C. Hemodynamic stability
D. Placement of a jejunal feeding tube

14. Which of the following patients would most likely have an improved outcome from enteral nutrition with an immune-modulating formula?

A. Mechanically ventilated patient with myasthenia gravis


B. Mechanically ventilated patient after emergent hemicolectomy for bowel perforation
C. Mechanically ventilated patient with pulmonary edema secondary to systolic heart failure
D. Acute stroke patient with dysphagia and hemiparesis who is not mechanically ventilated

15. A 32-year-old patient with type 1 diabetes mellitus and multiple admissions for diabetic ketoacidosis is admitted to the ICU with a 2-day history of
weakness, nausea, and vomiting. BP is 104/70 mm Hg, HR is 118/min, RR is 20/min, and temperature is 37.4C (99.3F). Laboratory data include
the following: arterial blood gas pH, 6.84; PaCO2, 10.5 mm Hg; PaO2, 114 mm Hg; sodium, 139 mEq/L; chloride, 105 mEq/L; potassium, 2.5 mEq/L;
bicarbonate, 5 mEq/L; and glucose, 568 mg/dL. After a 1-L bolus of normal saline is administered, which of the following is the most appropriate
intervention?

A. IV regular insulin as a bolus


B. IV sodium bicarbonate as a bolus
C. IV and oral potassium
D. IV regular insulin as a bolus and IV potassium

16. A 62-year-old woman with hypertension and hyperlipidemia was diagnosed with Graves disease 1 year ago. She presents with palpitations,
dizziness, shortness of breath, and intermittent chest pain for the last 2 days. She admits to having run out of her propylthiouracil 2 weeks ago. On
examination, she is confused and diaphoretic, with irregular HR of 132/min, BP of 138/82 mm Hg, RR of 22/min, temperature of 38C (100.3F)
and SpO2 of 92% on room air. Physical examination is remarkable for restlessness, tremor of both hands, an irregular heart rhythm with a grade II/VI
systolic murmur, bibasilar rales, and warm skin. She is admitted to the ICU. Which of the following is the most appropriate first intervention?

A. Oral propylthiouracil
B. Oral saturated solution of iodine
C. Oral aspirin
D. IV diltiazem
PART 6: Renal and Metabolic Disorders in the ICU

ANSWERS:

1A; 2D; 3B; 4A; 5B; 6D; 7B; 8C; 9C; 10C; 11D; 12A; 13C; 14B; 15C; 16A

RATIONALE (1) Answer: A

This patients laboratory results are most consistent with hypovolemic hyponatremia. Assessment of volume status is usually the first step in evaluating
hyponatremia, but determination of status may be difficult. In this patient, there are subtle suggestions of hypovolemia with a slightly elevated heart rate
and a blood urea nitrogen and creatinine level that are probably high for a patient with chronic alcohol dependence. The urine studies are helpful in
assessing the cause of the hyponatremia. The calculated serum osmolarity is 250 mOsm/kg (2 sodium + glucose 18 + blood urea nitrogen 2.8). The
serum osmolarity is higher, which could be consistent with euvolemic hyponatremia (syndrome of inappropriate antidiuretic hormone secretion
[SIADH]) or hypovolemic hyponatremia. The information that suggests hypovolemic hyponatremia is the low urine sodium level. Urine sodium level is
usually greater than 40 mEq/L in SIADH and other etiologies of euvolemic hyponatremia and less than 20 mEq/L in hypovolemic states. A high urine
specific gravity would also suggest hypovolemic hyponatremia. Secretion of antidiuretic hormone is appropriate in patients with hypovolemia. The
intervention of choice is the correction of the hypovolemia rather than restriction of fluids or administration of hypertonic saline. Vasopressin
antagonists such as conivaptan should not be used in hypovolemic hyponatremia because they can cause hypotension and worsening hypovolemia.

REFERENCES (1)

1. Milionis HJ, Liamis GL, Elisaf MS. The hyponatremic patient: a systematic approach to laboratory diagnosis. CMAJ. 2002;166:105662.
2. Lien Y-Hh, Shapiro JI. Hyponatremia: clinical diagnosis and management. Am J Med. 2007; 120:65358.
3. Vaidya C, Ho W, Freda BJ. Management of hyponatremia: providing treatement and avoiding harm. Cleveland Clin J Med. 2010;77:71526.

RATIONALE (2) Answer: D

Acute renal failure worsens mortality in critically ill patients. Renal replacement therapy (RRT) is frequently implemented to control complications.
There is ongoing debate over the most effective type of RRT, but there are distinct advantages and disadvantages of intermittent hemodialysis (IHD) and
continuous venovenous hemofiltration (CVVH). The major advantage of CVVH is hemodynamic tolerance due to less fluctuation of volume status.
CVVH is commonly preferred over IHD in hypotensive patients. The need for substitution fluids, anticoagulants, and multiple filters make CVVH more
costly than IHD despite the fact that specialized personnel are not required for implementation. Studies thus far have failed to show a mortality
advantage for either IHD or CVVH. Pharmacodynamic data exist for the dosing of most antibiotics with IHD but not for CVVH. Underdosing of
antibiotics is a major concern, especially with high ultrafiltration rates. Solute removal is more effective with IHD, which is the reason CVVH must be
used continuously.

REFERENCES (2)

1. John S, Eckardt K-U. Renal replacement strategies in the ICU. Chest. 2007;132:137988.
2. Pannu N, Klarenbach S, Wiebe N, et al. Renal replacement therapy in patients with acute renal failure: a systematic review. JAMA. 2008;299:793
805.
3. VA/NIH Acute Renal Failure Trial Network. Intensity of renal support in critically ill patients with acute kidney injury. N Engl J Med. 2008;359:7
20.

RATIONALE (3) Answer: B

The etiologies of acute renal failure are usually categorized as prerenal, renal (intrinsic), and postrenal. The most common cause of acute renal failure in
ICU patients is acute tubular necrosis (ATN) secondary to ischemia/hypoperfusion injury or nephrotoxin or a combination of these factors. The
laboratory and urine findings in this patient suggest ATN due to the following findings: blood urea nitrogen to creatinine ratio of approximately 10:1,
fractional excretion of sodium of 3%, bland sediment, and urinary sodium concentration greater than 40 mEq/L. The formula for calculating the
fractional excretion of sodium is

FENa = ([urine sodium serum sodium] [urine creatinine serum creatinine]) 100.

A nephrotoxin would be consistent with these findings. Hypotension could also produce ATN but is not offered as an option. Hypovolemia is a prerenal
cause of acute renal failure with findings of blood urea nitrogen to creatinine ratio greater than 20, FENa less than 1%, and urinary sodium level less
than 20 mEq/L. The urine specific gravity would also be higher. Urinary retention is an example of postrenal etiology of acute renal failure and would
also result in a blood urea nitrogen to creatinine ratio greater than 20 and variable FENa and urine sodium concentration. The hallmark of
glomerulonephritis is an active urinary sediment with red blood cells and red blood cell casts.

REFERENCES (3)

1. Dennon P, Douglas IS, Anderson R. Acute kidney injury in the intensive care unit: an update and primer for the intensivist. Crit Care Med.
2010;38:26175.
2. Esson ML, Schrier RW. Diagnosis and treatment of acute tubular necrosis. Ann Intern Med. 2002;137:74452.
3. Singri N, Ahya SN, Levin ML. Acute renal failure. JAMA. 2003;289:74751.
4. Weisbord S, Palevsky PM. Acute renal failure in the intensive care unit. Semin Crit Care Med. 2006;27:26273.

RATIONALE (4) Answer: A

In this patient, the pH of 7.35 suggests an acidosis, and the low bicarbonate level along with decreased PaCO2 suggests a metabolic process. The
respiratory compensation is appropriate using the formula

PaCO2 = 1.5 [bicarbonate] + 8 2.

The anion gap is usually calculated as

sodium (chloride + bicarbonate).

which in this patient yields an anion gap of 11 mEq/L. The normal anion gap is usually 1012 mEq/L, depending on the laboratory normal range. The
anion gap of 11 mEq/L might be considered normal for this patient except that the albumin concentration must be taken into account. The normal range
for anion gap is decreased in hypoalbuminemic patients. For every decrease of 1 g/dL in albumin, a decrease of 2.5 to 3 mEq/L in anion gap occurs. The
following formula can be used to estimate the true anion gap (AG):

AGcorrected = AGobserved + 2.5 [normal albumin measured albumin (g/dL)]

In this patient, the anion gap is closer to 16 mEq/L, indicating an anion gap acidosis. Correction of the anion gap for phosphate concentration has been
suggested, but it will not usually impact interpretation of the acid-base disorder. The clinical scenario in this case suggests infection and/or hypotension
as the cause of the acidosis. Options B and C are incorrect since an anion gap metabolic acidosis exists and the PaCO2 is appropriate compensation for
the acidosis rather than a primary disorder. Option D is incorrect since the decrease in serum bicarbonate is similar to the increase in anion gap
(referred to as the delta gap).

REFERENCES (4)

1. Fencl V, Jabor A, Kazda A, Figge J. Diagnosis of metabolic acid-base disturbances in critically ill patients. Am J Respir Crit Care Med. 2000;
162:224651.
2. Figge J, Jabor A, Fencl V. Anion gap and hypoalbuminemia. Crit Care Med. 1998; 26:180710.
3. Morganroth ML. An analytical approach to diagnosing acid-base disorders. J Crit Illness. 1990;5:138150.
4. Paulson WD. How to interpret the anion gap. J Crit Illness. 1997;12:9699.
5. Wrenn K. The delta (delta) gap: An approach to mixed acid-base disorders. Ann Emerg Med. 1990;19:131013.

RATIONALE (5) Answer: B

The clinical findings in this patient are consistent with acute decompensation secondary to adrenal insufficiency. Clinical findings may include
hypotension, orthostasis, fatigue, fever, and weight loss. In addition, patients may have nausea, vomiting, weight loss, and abdominal pain. Diagnosis of
adrenal insufficiency is often delayed because of the nonspecific symptoms. Laboratory findings that support a diagnosis of adrenal insufficiency include
hyponatremia, hyperkalemia, prerenal azotemia, hypoglycemia, and normal anion gap acidosis. The most common causes of primary adrenal
insufficiency are autoimmune destruction, infection, and metastatic disease. Other causes include bilateral adrenal hemorrhage, particularly in the
critically ill patient with disseminated intravascular coagulation, infiltrative disorders, and some drugs. Secondary adrenal insufficiency due to failure
of the pituitary to secrete adrenocorticotropic hormone (ACTH) is most commonly due to withdrawal of corticosteroids but can also result from tumors,
head trauma, cranial radiation, and infiltrative diseases. Adrenal crisis may be confused with septic shock since the manifestations are similar. In
addition, a minor infection can lead to decompensation. Clues to the presence of adrenal insufficiency include the lack of predisposing conditions for
infection, the lack of an obvious source, presence of a mild infection, and poor response to IV fluids and vasopressor agents. The treatment of adrenal
crisis includes volume resuscitation with glucose-containing fluids and immediate administration of corticosteroids.

The most appropriate management of this patient includes immediate administration of dexamethasone that will not interfere with determination of
plasma cortisol and performance of a short ACTH stimulation test. Administration of colloids or a change in vasopressor agent is unlikely to have any
significant effect in this patient. The addition of fludrocortisone is usually not required in adrenal crisis. Once an ACTH test has been performed,
hydrocortisone, 200300 mg/day in divided doses, can be used, which has some mineralocorticoid activity.

REFERENCES (5)

1. Coursin DB, Wood KE. Corticosteroid supplementation for adrenal insufficiency. JAMA. 2002;287:23640.
2. Salvatori R. Adrenal insufficiency. JAMA. 2005;294:248188.
3. Shenker Y, Skatrud JB. Adrenal insufficiency in critically ill patients. Am J Respir Crit Care Med. 2001;163:152023.
4. Paulson WD. How to interpret the anion gap. J Crit Illness. 1997;12:9699.
5. Zaloga GP, Marik P. Hypothalamic-pituitary-adrenal insufficiency. Crit Care Clin. 2001; 17:2541.

RATIONALE (6) Answer: D

The low pH indicates acidemia, and the low bicarbonate level and decreased PaCO2 suggest a metabolic process. The respiratory compensation appears
appropriate: (1.5 7) + 8 = 18. The anion gap is calculated as 19 mEq/L, which is increased and defines the presence of an anion gap metabolic
acidosis. This is consistent with the patients worsened renal function. In the presence of an anion gap acidosis, the delta gap should be calculated. In
this case, the difference between the measured and normal anion gap is 7 and the bicarbonate value would be expected to decrease to 17 mEq/L (24
minus 7 mEq/L). The very low bicarbonate value means that more bicarbonate has been lost than can be accounted for by compensation for the increase
in hydrogenion. Therefore, a nonanion gap metabolic acidosis is also present that accounts for the very low bicarbonate value. The clinical scenario
suggests loss of bicarbonate through diarrhea, but this patient was also receiving cyclosporine and may have tubular dysfunction as well. The tachypnea
in this patient is compensation for the acidoses rather than a sign of pulmonary disease.

REFERENCES (6)

1. Fencl V, Jabor A, Kazda A, Figge J. Diagnosis of metabolic acid-base disturbances in critically ill patients. Am J Respir Crit Care Med. 2000;
162:224651.
2. Morganroth ML. An analytical approach to diagnosing acid-base disorders. J Crit Illness. 1990; 5:138150.
3. Paulson WD. How to interpret the anion gap. J Crit Illness. 1997;12:9699.
4. Wrenn K. The delta (delta) gap: An approach to mixed acid-base disorders. Ann Emerg Med. 1990;19:131013.

RATIONALE (7) Answer: B

These rhythm strips show a classic progression of severe, life-threatening hyperkalemia that is probably due to rhabdomyolysis from the traumatic
injuries. The first sign of increased serum potassium on the ECG is often peaked T waves. Other changes associated with increases in serum potassium
concentration include:

Widening of the QRS complex;


Progressive atrioventricular conduction blocks;
Slow idioventricular rhythm;
An ECG tracing resembling a sine wave; and finally,
Either ventricular fibrillation, or as in this case, asystole.

The relationship of the serum potassium concentration and ECG changes is variable. Treatment of severe hyperkalemia with ECG changes includes the
initial administration of calcium as calcium chloride or gluconate to antagonize the effects of the hyperkalemia on the myocardial membranes. The
effects of calcium last about 30 minutes and allow time for other interventions. Potassium can be redistributed from the extracellular space to the
intracellular space by administration of insulin, along with concentrated glucose to prevent hypoglycemia. Use of sodium bicarbonate to redistribute
potassium is less effective. Albuterol, 1020 mg in 4 mL saline by inhalation over 10 minutes, can also lower the serum potassium concentration
significantly. Definitive removal of potassium from the body can be accomplished with loop diuretics, sodium polystyrene sulfonate, or dialysis.
Epinephrine would increase the serum potassium concentration further by redistribution to the extracellular space.

REFERENCES (7)

1. Alfonzo AV, Isles C, Geddes C, et al. Potassium disorders: clinical spectrum and emergency management. Resuscitation. 2006;70:1025.
2. Evans KJ, Greenberg A. Hyperkalemia: a review. J Intensive Care Med. 2005;20:272290.
3. Kraft MD, Btaiche IF, Sacks GS, Kudsk KA. Treatment of electrolyte disorders in adult patients in the intensive care unit. Am J Health Syst Pharm.
2005;62:16631682.
4. Weisberg LS. Management of severe hyperkalemia. Crit Care Med. 2008;36;324651.
RATIONALE (8) Answer: C

The laboratory measurements reveal acidemia with decreased bicarbonate concentration, indicative of a metabolic acidosis. The degree of respiratory
compensation is adequate (expected Pco2 in a simple metabolic acidosis = [1.5 bicarbonate] + 8 2), thereby excluding primary hyperventilation
related to anxiety. The anion gap is increased, and in view of the clinical history, hyperglycemia, and ketonemia, the patient has diabetic ketoacidosis.
However, the anion gap is 17 mmol/L, and this is 5 mmol/L higher than the upper limit of normal of 12 mmol/L. In a pure anion gap metabolic
acidosis, the decrease in serum bicarbonate should approximate the increase in anion gap. Thus, the expected bicarbonate level in this patient is 19
mEq/L (normal bicarbonate level of 24 mEq/L, minus the change in anion gap of 5 mEq/L). However, the actual bicarbonate level of 8 mEq/L is
considerably lower than anticipated, reflective of a change in bicarbonate concentration of 16 mmol/L. The difference between the change in
bicarbonate concentration and the change in anion gap is known as the delta gap and normal values are 0 6. Because the serum bicarbonate level is
much lower than expected for a simple anion gap metabolic acidosis, the acid-base status of this patient is best described as anion gap metabolic
acidosis (diabetic ketoacidosis) with a nonanion gap (hyperchloremic) metabolic acidosis. From the history, diarrhea is the most likely explanation for
the hyperchloremic metabolic acidosis. The serum bicarbonate concentration is consistent with the pH and PaCO2 (based on the Henderson-Hasselbach
equation); therefore, a laboratory error does not account for the acid-base abnormalities.

REFERENCES (8)

1. Kellum JA. Disorders of acid base balance. Crit Care Med 2007;35:263036.
2. Morris CG, Low J. Metabolic acidosis in the critically ill; Part 2. Causes and treatment. Anaesthesia 2008;63:396411.
3. Paulsen WD, Gadallah MF. Diagnosis of mixed acid-base disorders in diabetic ketoacidosis. Am J Med Sci 1993;305:295300.
4. Wrenn K. The delta (D) gap: an approach to mixed acid-base disorders. Ann Emerg Med 1990; 19:1310.

RATIONALE (9) Answer: C

Hyponatremia with acute neurologic symptoms is considered a medical emergency that should be treated promptly with administration of 3% sodium
chloride solution. A rapid but safe correction must be done in order to prevent brain herniation and osmotic demyelinization. Important steps in
management are setting the goal range for correction, calculating the rate at which to administer the solution, monitoring changes in sodium
concentration, and establishing a diagnosis for the etiology of hyponatremia.

Current recommendations limit the increase in sodium to approximately 812 mEq/L over 24 hours or 18 mEq/L in 48 hours. The appropriate rate for
infusion of 3% sodium chloride solution remains unclear as recent studies have shown that the most commonly used formula (Adrogue-Madias) for
calculating appropriate sodium correction tends to overestimate the correction in most cases. A retrospective study of hyponatremia cases showed that
among patients with serum sodium levels less than 120 mEq/L, the observed rates of correction were higher than predicted by the Adrogue-Madias
formula in almost 75% of patients. Moreover, in that study, the lower the serum sodium concentration of the patients, the greater was the risk of
overshooting.

In patients with underlying neurologic diseases such as in this case, 2 factors that were independently associated with reversal of impending
transtentorial herniation were an increase of sodium level of at least 5 mEq/L within 1 hour or a serum sodium level of at least 145 mEq/L (option C).

Correcting the hyponatremia over 24 hours would be appropriate for patients with neurologic symptoms, but not for patients with underlying pathology
causing impending herniation. In this case, with CT showing significant risk of brain herniation, the goal is to increase serum sodium level by
approximately 5 mEq/L within 1 hour (option C). The other options do not address this immediate need and are therefore incorrect.

REFERENCES (9)

1. Adrogue HJ, Madias NE. Hyponatremia. N Engl J Med 2000;342:1581898.


2. Koenig MA, Bryan M, Lewin JL, et al. Reversal of transtentorial herniation with hypertonic saline. Neurology 2008:70:10239.
3. Verbalis JG, Goldsmith SR, Greenberg A, et al. Hyponatremia treatment guidelines 2007: expert panel recommendations. Am J Med 2007;120(
Suppl 1):S121.
4. Mohmand HK, Issa D, Ahmad Z, et al. Hypertonic saline for hyponatremia: risk of inadvertent overcorrection. Clin J Am Soc Nephrol 2007;2:1110
7.

RATIONALE (10) Answer: C

The finding of metabolic acidosis with increased anion gap is of particular concern in the seriously ill patient as it can indicate potentially fatal
etiologies such as lactic acidosis from hypoperfusion. A broad differential diagnosis must be entertained, including lactic acidosis, ketoacidosis,
uremia, and drug intoxications (methanol, ethylene glycol, iron, etc). Recently, a new entity has become increasingly recognized: pyroglutamic acidosis
(also called 5-oxoprolinuria).
The condition occurs with chronic depletion of glutathione stores, which results in less inhibition of glutamate metabolism and accumulation of 5-
oxoproline. It is associated more with females, sepsis, chronic illness, malnutrition, and chronic acetaminophen ingestion. The prevalence of this entity
is unclear.

Laboratory data show an increased anion gap acidosis with low acetaminophen level and no other obvious cause of metabolic acidosis. The diagnosis
would be made with a urinary organic acid level, or with an increased urinary osmolar and urinary anion gap in the right clinical setting and without any
other causes for such abnormalities. In this patient, the abnormal transaminase levels result from chronic acetaminophen ingestion and the high
international normalized ratio from malnutrition.

When pyroglutamic acidosis is suspected, acetaminophen should be stopped. Administration of N-acetylcysteine has been suggested to replenish
glutathione stores, but benefit is unclear.

Option C is therefore correct (adverse effect of acetaminophen). The patient does not have acute renal failure (option B is wrong) as the increase in
creatinine is less than 0.3 mg/dL above baseline and there are no signs of sepsis (option D is wrong).

REFERENCES (10)

1. Fenves AZ, Kirkpatrick HM 3rd, Patel VV, et al. Increased anion gap metabolic acidosis as a result of 5-oxoproline (pyroglutamic acid): a role for
acetaminophen. Clin J Am Soc Nephrol. 2006;1:441447.
2. Rolleman EJ, Hoorn EJ, Didden P, et al. Guilty as charged: unmeasured urinary anions in a case of pyroglutamic acidosis. Neth J Med 2008;66:351
353.
3. Martensson J, Gustafsson J, Larsson A. A therapeutic trial with N-acetylcysteine in subjects with hereditary glutathione synthetase deficiency (5-
oxoprolinuria). J Inherit Metab Dis 1989;12:120130.

RATIONALE (11) Answer: D

The patient has tumor lysis syndrome occurring in the setting of chemotherapy for acute lymphoblastic leukemia. It is characterized by hyperuricemia,
hyperkalemia, hyperphosphatemia, and hypocalcemia. Risk factors include large tumor burden, chronic kidney disease, and volume depletion.
Preventive measures are essential in decreasing the incidence of tumor lysis syndrome and include aggressive fluid hydration and hypouricemic agents
such as allopurinol or rasburicase.

Alkalization of urine with sodium bicarbonate solution has been advocated, but may be associated with increased calcium phosphate deposition. There
is no benefit of a sodium bicarbonate solution over normal saline. Rasburicase is currently approved for prevention and treatment of tumor lysis
syndrome and rapidly lowers uric acid levels by enhancing degradation to allantoin. Allopurinol slowly lowers uric acid levels by decreasing its
formation. Neither agent would treat the hyperkalemia and other metabolic abnormalities in this patient. In the setting of oliguria, volume overload, and
life-threatening hyperkalemia, hemodialysis should be initiated. (option D).

REFERENCES (11)

1. Coiffier B, Altman A, Pui CH, et al. Guidelines for the management of pediatric and adult tumor lysis syndrome: an evidence-based review. J Clin
Oncol 2008;26:2767.
2. Conger JD, Falk SA. Intrarenal dynamics in the pathogenesis and prevention of acuteurate nephropathy. J Clin Invest 1977;59:786.
3. Davidson MB, Thakkar S, Hix JK, et al. Pathophysiology, clinical consequences, and treatment of tumor lysis syndrome. Am J Med 2004;116:546
54.
4. Coiffier B, Mounier N, Bologna S, et al. Efficacy and safety of rasburicase (recombinant urate oxidase) for the prevention and treatment of
hyperuricemia during induction chemotherapy of aggressive non-Hodgkins lymphoma: results of the GRAAL1 (Groupe dEtude des Lymphomes de
lAdulte Trial on Rasburicase Activity in Adult Lymphoma) study. J Clin Oncol 2003;21:4402.

RATIONALE (12) Answer: A

The finding of acute renal failure, with active urinary sediment and dysmorphic red cells indicates glomerulonephritis. Rapid loss of kidney function
(over a few days to a few weeks) in this setting is called rapidly progressive glomerulonephritis (RPGN) and should be treated aggressively. Several
etiologies can cause RPGN including lupus nephritis, Goodpasture syndrome, and vasculitis related to antineutrophil cytoplasmic antibodies (ANCAs,
which the patient has).

In the setting of severe renal failure, or severe pulmonary hemorrhage, plasmapheresis should be initiated. Recently, a multicenter trial randomized
patients with creatinine levels greater than 5.5 mg/ dL and ANCA-positive vasculitis to pulse-dose methylprednisolone or plasmapheresis along with
oral prednisone and cyclophosphamide. The group randomized to plasmapheresis had better survival and dialysis independence.
REFERENCES (12)

1. Jayne DR, Gaskin G, Rasmussen N, et al. European Vasculitis Study Group randomized trial of plasma exchange or high-dosage methylprednisolone
as adjunctive therapy for severe renal vasculitis. J Am Soc Nephrol 2007;18:2180.
2. Adu D, Pall A, Luqmani RA, et al. Controlled trial of pulse versus continuous prednisolone and cyclophosphamide in the treatment of systemic
vasculitis. QJM. 1997;90:401.
3. Hoffman GS, Kerr GS, Leavitt RY, et al. Wegener granulomatosis: an analysis of 158 patients. Ann Intern Med. 1992;116:488.

RATIONALE (13) Answer: C

Early enteral nutrition in the critically ill is associated with a decrease in infectious complications and decrease in length of stay. Enteral nutrition
should be provided to hemodynamically stable patients and those requiring stable low doses of pressors. Hypotensive patients (mean arterial pressure
<60 mm Hg) or those requiring escalating or high doses of vasopressors should be stabilized before initiating enteral feeds. Hemodynamically unstable
patients are at risk for ischemia involving the intestinal microcirculation. The presence of bowel sounds does not correlate well with absorptive
capacity and the absence of bowel movements does not predict the ability to tolerate enteral feeding. Several studies and meta-analyses have shown no
difference in pneumonia or mortality between gastric and postpyloric feeding.

REFERENCES (13)

1. Martindale RG, McClave SA, Vanek VW, et al. Guidelines for the provision and assessment of nutrition support therapy in the adult critically ill
patient: Society of Critical Care Medicine and American Society for Parenteral and Enteral Nutrition. Crit Care Med 2009;37:130.
2. McClave SA, Chang WK. Feeding the hypotensive patient: Does enteral feeding precipitate or protect against ischemic bowel? Nutr Clin Pract
2003; 18:279284.
3. Ho KM, Dobb GJ, Webb SA. A comparison of early gastric and post-pyloric feeding in critically ill patients: A meta-analysis. Intensive Care Med
2006; 32:639649.
4. Marik PE, Zaloga GP. Gastric versus postpyloric feeding: a systematic review. Crit Care 2003; 7:R46R51.
5. Heyland DK, Drover JW, Dhaliwal R, et al. Optimizing the benefits and minimizing the risks of enteral nutrition in the critically ill: role of small
bowel feeding. JPEN J Parenter Enteral Nutr 2002;26(6 Suppl):S51S55.
6. Marik PE, Zaloga GP. Early enteral nutrition in acutely ill patients: a systematic review. Crit Care Med 2001;29:22642270.

RATIONALE (14) Answer: B

Current guidelines recommend use of enteral immune-modulating formulas in patients undergoing major surgery, trauma patients, burn patients (>30%
body surface area involvement), head and neck cancer patients, and critically ill patients on mechanical ventilation. These formulas may contain various
combinations of arginine, omega-3 fatty acids, fish oil, glutamine, selenium, vitamin C, or vitamin E. The se formulas are associated with reduction in
duration of mechanical ventilation, infection and hospital length of stay, but not with a reduction in mortality. The benefit is more uniform in patients
undergoing major surgery than critically ill patients on mechanical ventilation and in surgical patients more than in medical patients. Caution is
warranted in use of immune-modulating formulas containing arginine in patients with severe sepsis as some studies have found a greater mortality with
these formulas. Options A, C, and D are medical patients less likely to benefit from immune-modulating formulas compared to the surgical patient
presented in option B.

REFERENCES (14)

1. Martindale RG, McClave SA, Vanek VW, et al. Guidelines for the provision and assessment of nutrition support therapy in the adult critically ill
patient: Society of Critical Care Medicine and American Society for Parenteral and Enteral Nutrition. Crit Care Med 2009;37:130.
2. Moore FA, Moore EE, Kudsk KA, et al: Consensus recommendations from the US summit on immune-enhancing enteral therapy. JPEN J Parenter
Enteral Nutr 2001; 25:S61S62.
3. Heys SD, Walker LG, Smith I, et al. Enteral nutritional supplementation with key nutrients in patients with critical illness and cancer: a meta-analysis
of randomized controlled clinical trials. Ann Surg 1999; 229:467477.
4. Heyland DK, Novak F, Drover JW, et al. Should immunonutrition become routine in critically ill patients? A systematic review of the evidence.
JAMA 2001;286:944953.
5. Montejo JC, Zarazaga A, Lopez-Martinez J, et al. Immunonutrition in the intensive care unit. A systematic review and consensus statement. Clin Nutr
2003;22:221233.
6. Bower RH, Cerra FB, Bershadsky B, et al. Early enteral administration of a formula (Impact) supplemented with arginine, nucleotides, and fish oil
in intensive care unit patients: Results of a multicenter, prospective, randomized, clinical trial. Crit Care Med 1995; 23:436449.

RATIONALE (15) Answer: C


The approach to managing diabetic ketoacidosis (DKA) includes fluids, insulin, and electrolyte replacement. As the first step, fluid administration is
targeted to correct volume deficits and ensure organ perfusion. Insulin therapy is usually initiated with a bolus of 5 to 10 units of regular insulin
followed by an infusion of regular insulin, 0.1 units/kg/h. This patient presents with severe hypokalemia that mandates the delay of insulin administration
until the potassium concentration is greater than 3.3 mEq/L. Administration of insulin before correction of potassium concentration would further lower
the potassium concentration and potentially precipitate respiratory muscle weakness or arrhythmias. Thus, options A and D are incorrect. In general,
bicarbonate therapy is not indicated in the management of DKA and pH usually improves quickly with fluid therapy. Bicarbonate administration has
failed to show benefit in several studies, but no studies have addressed the use of bicarbonate in patients with DKA and pH less than 6.9. If considered
for use, bicarbonate should be administered as an infusion rather than a bolus. In this patient, a bolus of bicarbonate would result in a shift of potassium
to the intracellular space and further lower the serum potassium concentration. This intervention could result in the same consequences as administering
insulin with severe hypokalemia.

REFERENCES (15)

1. Kitabchi AE, Umpierrez GE, Miles JM, et al. Hyperglycemic crises in adult patients with diabetes. Diabetes Care 2003;32:133543.
2. Chiasson J-L, Aris-Jilwan N, Belanger R, et al. Diagnosis and treatment of diabetic ketoacidosis and the hyperglycemic hyperosmolar state. CMAJ
2003;168:85966.
3. Viallon A, Zeni F, Lafond P, et al. Does bicarbonate therapy improve the management of severe diabetic ketoacidosis? Crit Care Med
1999;27:269093.

RATIONALE (16) Answer: A

Thyroid storm represents the extreme manifestations of hyperthyroidism. The distinction between severe thyrotoxicosis and thyroid storm is often
subjective, but severe clinical manifestations should be managed similarly. This patient has significant multiorgan signs and symptoms related to
hyperthyroidism and should be managed in an ICU.

The order of treatment in thyroid storm is very important. Propylthiouracil or methimazole should be given as the first intervention to inhibit new
hormone production (thyroxine [T4] synthesis). Iodine in the form of saturated solution of iodine or Lugol solution must be administered at least 1 hour
after propylthiouracil or methimazole. If iodine is administered as the first agent, it can cause release of preformed thyroid hormone and worsen
symptoms. Thus, option B is incorrect. Iodine administered after propylthiouracil or methimazole inhibits release of preformed T4. Salicylates are
contraindicated in thyrotoxicosis because they can decrease thyroid protein binding, causing an increase in free thyroid hormone levels. Thus, option C
is incorrect. Atrial fibrillation is a frequent manifestation of hyperthyroidism but is best managed with control of the hyperthyroidism and administration
of beta-blockers. Beta-blockers control the peripheral effects of thyroid hormone and can be administered intravenously. Diltiazem would not be the
best agent for control of atrial fibrillation in this clinical scenario. Hydrocortisone is usually also administered in thyroid storm to inhibit peripheral
conversion of T4 to triiodothyronine (T3) and to treat possible adrenal insufficiency.

REFERENCES (16)

1. Cooper DS. Hyperthyroidism. Lancet 2003;362:45968.


2. Nayak B, Burman K. Thyrotoxicosis and thyroid storm. Endocrinol Metab Clin N Am 2006;35:66386.
PART 7: Environmental and Toxicologic Injury

Instructions: For each question, select the most correct answer.

1. A 35-year-old male window washer was moving a metal ladder when the ladder came in contact with a 220-V electrical line. He was holding onto
the ladder with both hands at the time of contact , was knocked clear of the ladder, and was unconscious for approximately 1 minute. Upon
awakening he was confused and reported bilateral arm and hand pain and lower extremity pain. He was brought to the burn center for further care.
Physical examination reveals a 5% total body surface burn to his bilateral arms and hands, with burns to his right groin and left foot. His pulse rate
is 102, BP is 138/78 mm Hg, and RR is 24/min. His oxygen saturation is 100% on nasal cannula at 4 L/min, and Glasgow Coma Scale score is 15.
Examination of the hands demonstrates bilateral severe burns and contractures to the digits and at the level of the wrist, with minimal sensation on
the digits. He has a large exit wound in the right groin area and lesser burns on the left foot.

Figure 1 Entrance wound to right hand

Figure 2 Right groin area exit wound

Which of the following interventions in the ICU is most likely to improve the patients chances for limb survival and limb salvage?
A. Administration of a calcium channel blocker to improve perfusion into the injured area
B. Administration of sodium bicarbonate to alkalize the urine
C. Elevation of the right arm to decrease edema formation
D. Lowering of the right arm to increase arterial perfusion

2. A 2-year-old male is involved in a house fire. He is found unconscious by fire rescue and brought to the burn center for evaluation and treatment. On
arrival, he has partial-thickness burns to his face, upper extremities, and chest. His Glasgow Coma Scale score is 6. Pulse rate is 132/min, BP is
112/56 mm Hg, RR is 34/min, carboxyhemoglobin is 40%, and SpO2 is 98% on a 100% oxygen nonrebreather mask. He has soot in his nares and
mouth and his lips have a parched appearance.

The most immediate care of this patient should include:


A. High-flow oxygen therapy with immediate bronchoscopy
B. Intubation, bronchoscopy, and hyperbaric oxygen therapy
C. Immediate hyperbaric therapy, bronchoscopy, and intubation only if the patient does not improve
D. Immediate treatment with amyl nitrate, followed by bronchoscopy
E. Intubation followed by bronchoscopy and plasma exchange therapy

3. Oxygen saturation by pulse oximetry (SpO2) was 98% while the patient was receiving 100% FIO2 via face mask. The arterial blood gas oxygen
saturation (SaO2) was only 59%.

The most likely explanation for this includes:


A. Arteriovenous shunting
B. Increased tissue oxygen extraction due to low cardiac output
C. The presence of carboxyhemoglobin
D. Malfunction of the 2-channel oximeter
E. Error in SaO2 most likely representing a venous gas

4. A 48-year-old male was involved in a fire at his home when the propane stove exploded. He sustained 70% total body surface area burns of deep-
partial to full thickness. He is admitted to the burn center, intubated, and resuscitated. He weighs 90 kg (198 lb) and over the first 8 hours receives
12.6 L of fluid. His present pulse rate is 145/min, BP is 90/54 mm Hg, RR is 22/min on conventional mechanical ventilation with FIO2 of 50%, set
rate of 18/min tidal, volume of 700 mL, and positive end-expiratory pressure (PEEP) of 5 cm H2O. Urine output rates for the last 3 hours are 15 mL,
10 mL, and 5 mL, respectively. The ventilator is alarming that the exhaled volumes are 450 mL and peak airway pressures are 45 cm H2O. Physical
examination is seen in the figure.

Figure 4 70% total body surface area with multiple escharotomies

The best next step in the management of this patient is to:


A. Increase PEEP to 15 cm H2O.
B. Convert the patients ventilator mode to synchronized intermittent mandatory ventilation and decrease tidal volume.
C. Administer a 1-L bolus of lactated Ringer solution and increase the patients hourly fluid rate.
D. Measure the patients bladder pressure.
E. Add dopamine to augment blood pressure and renal perfusion.

5. A 68-year-old man weighing 80 kg (176 lbs) was admitted to the ICU with second-degree burns over 30% of his body from a house fire. He was
intubated in the emergency department for evidence of inhalation injury. Currently, his HR is 120/min, BP is 108/52 mm Hg, RR is 18/min (sedated
on ventilator), and temperature is 36.4C (97.5F).

Which amount and type of fluid should be infused over the first 8 hours?
A. 5% dextrose lactated Ringer solution, 4,800 mL
B. Lactated Ringer solution, 4,800 mL
C. Lactated Ringer solution, 3,200 mL
D. 0.9% saline, 3,200 mL
6. Three intubated patients are admitted to the ICU following exposure to an unknown gas in a bus station. Clinical findings include severe hypoxemia
with copious respiratory and oral secretions. Additional findings include small pupils, bradycardia, and diarrhea. One of the patients is able to
follow commands but appears to have significant diffuse weakness.

Which of the following would be the most beneficial initial intervention?


A. Pyridostigmine
B. Atropine
C. Sodium nitrite and sodium thiosulfate
D. Hyperbaric oxygen
E. Pralidoxime

7. An ICU patient is suspected to have overdosed on trazodone and fluoxetine. The patient is agitated and delirious. BP is 158/106 mm Hg, HR is
116/min, RR is 22/min, and temperature is 38.7C (101.6F). She is diaphoretic with dilated pupils. Tremors and clonus of the lower extremities
are noted.

Which of the following should be administered?


A. Dantrolene
B. Lorazepam
C. Propranolol
D. Bromocriptine
E. Neuromuscular blocker

8. A man with alcohol dependence and severe alcohol-related delirium is admitted to the ICU and a bolus of lorazepam is initiated. Due to high
requirements of lorazepam to control agitation, the patient is intubated and a lorazepam infusion for control of symptoms is initiated. After 24 hours,
he is receiving lorazepam, 12 mg/h. His baseline laboratory results show the following: sodium, 132 mEq/L; chloride, 100 mEq/L; potassium, 3.4
mEq/L; bicarbonate, 22 mEq/L; blood urea nitrogen, 22 mg/dL; creatinine, 1.5 mg/dL; and glucose, 120 mg/dL. Three days into the ICU course, the
patient has a bicarbonate level of 16 mEq/L with an anion gap of 18 mEq/L, creatinine level of 1.8 mg/dL, and lactate level of 2.0 mg/dL. He has an
osmolar gap of 24 mOsm/kg. He remains sedated, with HR of 90/min, BP of 104/74 mm Hg, RR of 12/min on the ventilator, and temperature of
37.8C (99.9 F).

Which of the following interventions is most appropriate?


A. Stop lorazepam and start midazolam infusion.
B. Stop lorazepam and start fentanyl infusion.
C. Start broad-spectrum antibiotics and continue lorazepam infusion.
D. Initiate hemodialysis.

9. A 70-year-old woman is found unresponsive in her unheated house in December by her neighbors. She is known to have systolic heart failure
(ejection fraction 40%45%), chronic kidney disease, and mild dementia. On arrival to the hospital, she is intubated and a rectal temperature of
30C (86F) is measured. HR is 44/min, BP is 88/50 mm Hg, RR is 12/min (on ventilator), and SpO2 is 94% (FIO2 of 0.5). She remains unresponsive
and laboratory tests are pending.

Which of the following combinations of rewarming techniques is most appropriate for this patient?
A. Forced-air rewarming blanket and warm IV fluids
B. Warm blanket, and gastric and bladder lavage with warm fluid
C. Heated, humidified oxygen and cardiopulmonary bypass
D. Forced-air rewarming blanket and hemodialysis

10. In a patient with significant heat exposure, which of the following findings is diagnostic for heatstroke rather than heat exhaustion?

A. Tachycardia
B. Orthostatic hypotension
C. Seizures
D. Nausea and vomiting
E. Absence of sweating

11. A 32-year-old woman ingested 50 tablets of hydrocodone/acetaminophen, 5 mg/500 mg, approximately 7 hours before her friends brought her to the
hospital. On admission to the ICU she is somnolent with a weak gag reflex. BP is 90/54 mm Hg, HR is 64/min, and RR is 8/min.

Which of the following is the most appropriate initial treatment plan?


A. Administer IV naloxone, 2 mg.
B. Intubate the patient and administer IV N-acetylcysteine, 150 mg/kg over 60 min.
C. Administer IV naloxone, 2 mg, and obtain a serum acetaminophen level.
D. Administer IV naloxone, 2 mg, and IV N-acetylcysteine, 150 mg/kg over 60 min.

12. A 62-year-old man with hypertension and hyperlipidemia has experienced depression since the recent death of his wife. His daughter finds him
unresponsive after he does not show up for work. On admission to the hospital, he has an HR of 40/min, BP of 86/40 mm Hg, RR of 12/min, and
temperature of 36C (96.7F). His home medications include sustained-release verapamil, hydrochlorothiazide, and atorvastatin. ECG shows sinus
bradycardia with prolonged PR interval. He receives IV fluids, atropine, calcium chloride, and glucagon with no improvement in heart rate or
blood pressure.

Which of the following should be considered for the next intervention to improve his hemodynamic status?
A. Milrinone
B. Transcutaneous pacing
C. Insulin and glucose infusion
D. Hemodialysis

13. A 39-year-old man presents to the hospital after developing epistaxis and difficulty swallowing. He reports that he ingested 2 boxes of rat poison
the week before in a suicide attempt but changed his mind. His physical examination is remarkable for multiple ecchymoses on his lower
extremities, mild epistaxis, and a 2- 3-cm hematoma in the left tonsillar area. His international normalized ratio is 14, hemoglobin level is 11.2
g/dL, and platelet count is normal. He is admitted to the ICU because of potential airway compromise.

Which of the following is the best treatment option for this patient?
A. Fresh frozen plasma and IV vitamin K
B. Fresh frozen plasma alone
C. Fresh frozen plasma and subcutaneous vitamin K
D. IV vitamin K alone
14. A 35-year-old man is admitted to the ICU with a reported history of ethylene glycol ingestion. A friend found him comatose after not being able to
contact him for 48 hours. An empty can of antifreeze was found in the apartment. The patient is intubated for airway protection. BP is 110/60 mm
Hg, HR is 110/min, RR is 18/min, and temperature is 36.5C (97.6F). Physical examination reveals obtundation, tachycardia, and bilateral diffuse
rales. Laboratory examination shows a room air arterial blood gas measurement with pH of 7.09, PaCO2 of 30 mm Hg, PaO2 of 65 mm Hg, anion gap
of 28 mEq/L, osmolar gap of 8 mOsm/kg, creatinine level of 2.4 mg/dL, and negative serum ethanol results. An ethylene glycol level is ordered, but
results will not be available for 72 hours.

Which of the following interventions is most likely to benefit this patient?


A. Ethanol infusion
B. IV fomepizole alone
C. Hemodialysis alone
D. Hemodialysis and IV fomepizole

15. A 68-year-old man is admitted for repair of a traumatic femur fracture. On postoperative day 1, he has hand tremors, mild tachycardia, and
diaphoresis. He is oriented and responds appropriately. On questioning, he admits to drinking 2 to 3 mixed drinks every night. While awaiting the
response of the physician, the patient suffers a tonic-clonic seizure lasting 2 minutes, which resolves before any medications can be given. He is
transferred to the ICU. HR is 110/min, BP is 160/94 mm Hg, RR is 18/min, temperature is 37.6C (99.6F), and SpO2 is 95% on oxygen, 2 L/min via
nasal cannula. Which of the following is the most appropriate intervention?

A. IV metoprolol
B. IV lorazepam
C. IV fosphenytoin
D. Observation

16. Which of the following is the recommended daily caloric requirement for patients with major burn injury?

A. 2025 kcal/kg
B. 2530 kcal/kg
C. 3040 kcal/kg
D. 4050 kcal/kg
PART 7: Environmental and Toxicologic Injury

ANSWERS:

1B; 2B; 3C; 4D; 5B; 6B; 7B; 8A; 9A; 10C; 11D; 12C; 13A; 14C; 15B; 16C

RATIONALE (1) Answer: B

The electrical current travelled from his hands and exited from his groin and left foot (vertical pathways). Any tissue between these sites may have
suffered injury. As a result, the extent of the burn, 5% total body surface area (TBSA), may be misleading. The potential for this patient to develop
myoglobinemia due to muscle necrosis is high. Fluid resuscitation should be targeted to keep the urine output brisk in order to prevent acute renal injury.
With severe muscle necrosis or a pH lower than 7.1, sodium bicarbonate (correct answer B) at 12 mEq/kg will alkalize the urine and limit the
precipitation of myoglobin in the renal tubules. Calcium channel blockers (option A) given systemically will cause hypotension. Elevation of the arm
(option C) may decrease perfusion, while lowering the arm (option D) will result in increased edema formation.

REFERENCES (1)

1. Arnoldo B, Klein M, Gilgran N. Practice Guidelines for the management of electrical injuries. J Burn Care Res. 2006;27:439447.
2. Arnolodo BD, Purdue GF, Kowalske K, et al. Electrical injuries: a 20 year review. J Burn Care Rehabil. 2004;25:47984.
3. Cancio LC, Jimenez-Reyna JF, Barillo DJ, et al. 159 cases of high voltage electric injury. J Burn Care Rehabil. 2005;26:331340.

RATIONALE (2) Answer: B

This patient has a significant smoke inhalation injury with evidence of facial burns and elevated carboxyhemoglobin level (40%). The patients inability
to protect his airway, particularly in the presence of facial burns, should be recognized in the primary survey and addressed with immediate intubation.
Once the airway is secure, further workup and therapy can begin.

Inhalation injury is composed of 3 major elements. The most common is particulate injury due to the smoke and soot that is inhaled. This may result in
reactive airway and secretion clearance problems. Therapy is directed toward symptomatic relief including supplemental humidified oxygen, beta
nebulizers, and time.

The second component of inhalation injury is thermal and is due to the direct effects of heat on the upper and lower respiratory tract. This is a very
serious component that may result in respiratory epithelial sloughing, ulceration, and mucous plugging. Severe cases may lead to tracheal stenosis due to
scar formation. Bacterial pneumonia after thermal injury is commonplace. The upper respiratory tract is well developed to dissipate both hot and cold
air as it passes through the nasal passages, hypopharynx, and supraglottic area. However, once the hot air passes below the glottis, there is little to
dissipate the heat. Lower respiratory tract thermal injury including alveolar damage is the end result.

The last form of inhalation injury is due to the toxic breakdown products of combustion. Carbon monoxide (CO), cyanide, chlorides, ammonia, hydrogen
sulfide, and various aldehydes are the main components of these toxins. Hemoglobin has a 200 times greater affinity for CO than for oxygen. Treatment
for CO includes high-concentration oxygen therapy to favor off -loading of CO from the hemoglobin molecule. Unfortunately, CO quickly dissolves into
the central nervous system and will generate the formation of reactive oxygen species and lipid peroxidases. The long-term clinical neurologic
implications of CO poisoning include cognitive impairment, memory loss, and neuropsychological dysfunction. Hyperbaric therapy is aimed at lessening
the possibility of central nervous system impairment rather than normalizing serum carboxyhemoglobin (COHb) levels. Often by the time patients begin
hyperbaric therapy, they have cleared their serum level of COHb with simple 100% oxygen therapy. Therapy ideally is started within the first 12 hours
of injury.

Amyl nitrate and sodium thiosulfate are antidotes for cyanide poisoning. Amyl nitrate converts hemoglobin to methemoglobin. The cyanide binds to the
methemoglobin because it contains ferric iron. Sodium thiosulfate increases the sulfur molecule pool, favoring the conversion of cyanide to thiocyanate,
which is excreted by the kidneys.

Bronchoscopy has a central role to play in the diagnosis and treatment of smoke inhalation injury due to particulate and thermal causes. In addition to the
visual assessment of the airway for a burn injury, aggressive pulmonary toilet can aid in the directed suctioning of mucus, soot, and debris from the large
and midsized airways. The figure demonstrates the dense soot impacted on the respiratory epithelium in a patient with inhalation injury. It may take from
48 to 72 hours to clear all of the particulate matter from the airways. Bronchoscopy may be performed after the patient is intubated awaiting hyperbaric
therapy and then on a daily basis until the patient improves.

RATIONALE (3) Answer: C


The pulse oximeter oxygen saturation (SpO2) of 98% is likely to be misleading due to the presence of carbon monoxide (CO) at 40%. Most hospital
oximeters are not able to differentiate between oxyhemoglobin, carboxyhemoglobin (COHb), and methemoglobin. Both oxyhemoglobin and
carboxyhemoglobin absorb light at 660 nm. The pulse oximeter determines SpO2 from the relationship of the pulsatile component of light refractance at 2
different wavelengths. It measures reduced hemoglobin and oxyhemoglobin. If carboxyhemoglobin is present, the 2-channel oximeter cannot distinguish
the difference, and gives an artificial reading for the SpO2. Thus the SpO2 of 98% is a measure of both the oxyhemoglobin and carboxyhemoglobin
binding. The mathematical relationship is

SaO2 = SpO2 (1-SaCO).

COHb 0%10%Usually no symptoms


COHb 10%20%Mild headache, shortness of breath
COHb 20%30%Moderate headache, impaired concentration
COHb 30%40%Severe headache, cognitive difficulties
COHb 40%50%Confusion, lethargy, syncope
COHb 50%60%Respiratory failure, seizures
COHb >60%Coma, death

Administration of 100% oxygen to this patient can lessen the half-life of carbon monoxide from 3.5 hours to 90 minutes. Utilizing 100% FIO2 until
adequate half-life degradation is achieved should be the goal of therapy.

Figure 3 Right mainstem bronchus covered with soot from a structure fire.

REFERENCES (3)

1. Mlcak RP, Suman OE, Herndon DN. Respiratory management of inhalation injury. Burns. 2007; 33:213.
2. Tibbles PM, Edelsberg JS. Hyperbaric oxygen therapy. N Engl J Med. 1996;334:16421648.
3. Weaver LK. Carbon monoxide poisoning. N Engl J Med. 2009; 360:12171225.
4. Weaver LK, Valentine KJ, Hopkins RO. Carbon monoxide poisoning risk factors for cognitive sequelae and the role of hyperbaric oxygen. Am J of
Respiratory Crit Care. 2007; 176:491.
5. Mosier MJ, Gamelli RL, Halerz MM, et al. Microbial contamination in burn patients undergoing urgent intubation as part of their early airway
management. J Burn Care Research. 2008;29:304310.

RATIONALE (4) Answer: D

This patient developed an abdominal compartment syndrome (ACS). The combination of larger fluid requirements for resuscitation and the constellation
of worsening pulmonary mechanics, tachycardia, hypotension, and progressive oliguria suggest this diagnosis. ACS represents an abnormal physiology
whereby increased intra-abdominal pressures impair normal renal, cardiovascular, and pulmonary physiology. As intra-abdominal pressure rises,
pressure on the inferior vena cava decreases right heart filling, resulting in lower blood pressure. In a compensatory manner, heart rate increases to
maintain adequate cardiac output. Pressure on the diaphragm may result in diaphragmatic elevation yielding lower tidal volumes and higher airway
pressures. Increased intra-abdominal pressure also impairs the venous outflow from the kidneys, resulting in lower renal perfusion and yielding
diminished urinary output. In patients with elevated intracranial pressure, manifestations of high intra-abdominal pressures can be reflected in rising
intracranial pressure.

The quickest method to get an accurate measurement of intra-abdominal pressure is by measuring bladder pressure. Placing 75 mL of fluid into the
bladder, clamping the Foley catheter at the hub, and then simply placing a pressure transducer into the urinary catheters aspiration port will give an
approximation of the intra-abdominal pressure. If surgical decompression is performed, postoperative management will be complicated by an open
abdomen. Temporary closure with a vacuum-assisted closure (commercial or in-house fabricated) or with a Bogota bag (sterile open IV bag sutured into
the fascia) are the standard methods to manage this problem.

Grading System of Abdominal Compartment Syndrome


Grade 1 1215 cm H2O Observe normal physiology
Grade 2 1620 cm H2O Observe with serial abdominal pressure monitoring
Grade 3 2125 cm H2O Consider decompression
Grade 4 >25 cm H2O Decompression surgically or via paracentesis

REFERENCES (4)

1. Kirkpatrick AW, Ball CG, Nickerson D, et al. Intraabdominal hypertension and the abdominal compartment syndrome in burn patients. World J Surg.
2009 Jun;33:11429.
2. Markell KW, Renz EM, White CE, et al. Abdominal complications after severe burns. J Am Coll Surg. 2009;208:940949.
3. Azzopardi EA, McWilliams B, Iyer S, et al. Fluid resuscitationin adults with severe burns at risk of secondary abdominal compartment syndrome--
an evidence based systematic review. Burns. 2009 Nov;35:91120. Epub 2009 May 27.
4. De Wolf A, Poelaert J, Herck I, et al. Surgical decompression for abdominal compartment syndrome after emergency cardiac surgery. Ann Thorac
Surg. 2008 Jun;85:21335.
5. Lui, F, Sangosanya A, Kaplan L. Abdominal compartment syndrome: clinical aspects and monitoring. Crit Care Clin. 2007;23:415433.

RATIONALE (5) Answer: B

The most commonly used formula for the amount of fluid to be given for resuscitation of patients with burns greater than 20% of total body surface area
is the Parkland formula. This formula uses 4 mL per percent of total body surface area involved with burns, per kilogram of body weight. Half of this
amount is administered in the first 8 hours and the remainder is infused over the next 16 hours. Thus, in this patient the correct amount to infuse in the
first 8 hours is 4,800 mL. This calculation makes options C and D incorrect. Lactated Ringer solution is most commonly used for burn resuscitation, but
glucose-containing fluids are avoided due to concerns of hyperglycemia and osmotic diuresis. Thus, option A is incorrect. It is important to remember
that the formula provides a starting point for fluid resuscitation, and modifications of the quantity of fluid may be needed based on the patients response.
Although the usual endpoint of resuscitation is the goal of urine output of 0.5 to 1.0 mL/kg/h, urine output may not reflect adequacy of resuscitation in
patients with renal insufficiency, morbidly obese patients, and patients with very large burns.

REFERENCES (5)

1. Lehnhardt M, Jafari HJ, Druecke D, et al. A qualitative and quantitative analysis of protein loss in human burn wounds. Burns 2005;31:159167.
2. Greenhalgh DG. Burn resuscitation: The results of the ISBI/ABA survey. Burns 2010;36:176182.
3. Baxter CR, Shires T. Physiologic response to crystalloid resuscitation of severe burns. Ann New York Acad Sci 1968;150:874894.

RATIONALE (6) Answer: B

Poisoning with nerve gases, such as sarin (O-isopropyl methylphosphonofluoridate), result in a cholinergic syndrome suggested by these clinical
findings. A cholinergic syndrome can also result from organophosphate or carbamate poisoning. The toxicity is caused by inhibition of cholinesterase,
which prevents degradation of acetylcholine at the postsynaptic receptor. Muscarinic effects include bronchorrhea, bradycardia, and a hypersecretory
syndrome (salivation, lacrimation, urination, defecation, emesis). Nicotinic effects result in muscle fasciculations and weakness, and central nervous
system effects lead to headache, confusion, and central respiratory depression. Pupils are typically miotic. The primary concern in such patients is
hypoxic respiratory failure from bronchorrhea, bronchospasm, and respiratory depression. IV atropine should be administered in doses of 2 to 4 mg
repeated every 5 minutes until tracheobronchial secretions are controlled. The most common cause of death in cholinergic poisonings is asphyxiation.
Continued administration of large doses of atropine may be required as bolus or continuous infusion. Because sarin irreversibly binds to cholinesterase,
the patient is likely to require continuous atropine infusion in an ICU setting until the agent is completely metabolized. Atropine does not reverse
nicotinic effects and pralidoxime is used to reverse muscle weakness by regeneration of acetylcholinesterase. It is administered as a loading dose (12 g
in 500 mL of normal saline solution over 30 minutes) and then as a continuous infusion at 500 mg/h.

Pyridostigmine, an anticholinesterase, may precipitate a cholinergic crisis and is used for treatment of myasthenia gravis. Hyperbaric oxygen may be
considered for patients with severe carbon monoxide poisoning, but patients would not typically have evidence of a hypersecretory syndrome. Patients
with carbon monoxide poisoning will often present comatose with an anion gap metabolic acidosis, cardiovascular instability and increased levels of
carboxyhemoglobin. Sodium nitrite and sodium thiosulfate are used as an antidote in cyanide poisoning. Cyanide poisoning usually presents with more
life-threatening manifestations, such as sudden cardiovascular collapse, seizures, hypotension, arrhythmias, and severe metabolic acidosis.

REFERENCES (6)

1. Eddleston M, Szinicz L, Eyer P, Buckley N. Oximes in acute organophosphorus pesticide poisoning: a systematic review of clinical trials. Q J Med
2002;95:275-83.
2. Lawrence DT, Kirk MA. Chemical terrorism attacks: update on antidotes. Emerg Med Clin N Am. 2007;25:5695.
3. Lee P, Tai DYH. Clinical features of patients with acute organophosphate poisoning requiring intensive care. Intensive Care Med. 2001;27:69499.
4. Leiken JB, Thomas RJ, Walter FG, et al. A review of nerve agent exposure for the critical care physician. Crit Care Med. 2002;30:23462354.
5. Peter JV, Cherrian AM. Organic insecticides. Anaes Intens Care. 2000;28:1121.
6. Peter JV, Moran JL, Graham P. Oxime therapy and outcomes in human organophosphate poisoning: an evaluation using meta-analytic techniques. Crit
Care Med. 2006;34:50210.

RATIONALE (7) Answer: B

The patient described is suffering from selective serotonin reuptake inhibitor syndrome. The syndrome may occur after a single dose, high-dose
overdose, or when combined with other serotonergic agents in therapeutic dosages. Drugs associated with serotonin syndrome include monoamine
oxidase inhibitors, other antidepressants, opiates, antiemetics, lithium, linezolid, tramadol, drugs of abuse, and others. The pathophysiology is probably
related to excessive stimulation of serotonin receptors in the brain. Clinical manifestations include altered mental status, autonomic dysfunction, and
neuromuscular abnormalities ranging from tremors to myoclonus and rigidity. Although the clinical manifestations are very similar to neuroleptic
malignant syndrome, the management differs based on the differing pathophysiologies of the 2 syndromes. Management is largely supportive. Intubation
and mechanical ventilation may be necessary for patients with significant altered mental status. Benzodiazepines are the primary therapy indicated for
control of agitation. Control of autonomic instability and external cooling for sustained hyperthermia may be needed. A neuromuscular blocker would be
contraindicated since it would preclude the evaluation of seizure activity. Dantrolene and bromocriptine are not recommended in this disorder.
Serotonin antagonists such as propranolol, cyproheptadine, and methysergide have been proposed, but evidence for benefit is lacking and routine use is
not recommended.

REFERENCES (7)

1. Birmes P, Coppin D, Schmitt L, et al. Serotonin syndrome: a brief review. CMAJ. 2003;168:14391442.
2. Boyer EW, Shannon M. The serotonin syndrome. N Engl J Med. 2005;352:11121120.
3. Gillman PK. The serotonin syndrome and its treatment. J Psycopharmacol. 1999;13:100109.
4. Isbister GK, Bowe SJ, Dawson A, et al. Relative toxicity of selective serotonin reuptake inhibitors (SSRIs) in overdose. J Toxicol Clin Toxicol.
2004;42:27785.

RATIONALE (8) Answer: A

This clinical scenario and the blood chemistry findings in this patient suggest propylene glycol toxicity. Each vial of lorazepam (2 mg/mL) contains 830
mg/mL of propylene glycol. Propylene glycol is also a solvent in IV formulations of diazepam, etomidate, phenytoin, nitroglycerin, esmolol,
phenobarbital, and pentobarbital. The most common manifestations of propylene glycol accumulation are anion gap metabolic acidosis and increased
osmolar gap. Approximately 12%45% of propylene glycol is excreted unchanged in the urine in healthy individuals, and the remainder is metabolized
by the liver to lactate and pyruvate. Accumulation may occur when doses exceed the upper recommended dose of 0.1 mg/ kg/h. Renal or hepatic
insufficiency may also play a role in accumulation. Serious reported toxicities include renal dysfunction, hemolysis, cardiac arrhythmias, seizures, and
central nervous system depression or agitation. The metabolic findings of propylene glycol toxicity may also be confused with sepsis or severe
inflammatory states.

Clinical studies suggest that the osmolar gap correlates with propylene glycol accumulation. Although toxicity is more common after long periods of
lorazepam infusion (>3 days), toxicity has occurred with short-term, high-dose use. The treatment of choice is to stop the lorazepam infusion and sedate
with an agent that does not contain propylene glycol. Midazolam (option A) does not contain propylene glycol. It would not be appropriate to change to
fentanyl alone (option B) in a patient who requires benzodiazepines for alcohol withdrawal. Hemodialysis would remove propylene glycol but is not
required unless severe renal dysfunction develops. Thus, option D is incorrect. Propylene glycol is metabolized by alcohol dehydrogenase but there are
no reports of using ethanol infusion or fomepizole for toxicity.

REFERENCES (8)

1. Arroliga AC, Shehab N, McCarthy K, et al. Relationship of continuous infusion lorazepam to serum propylene glycol concentration in critically ill
adults. Crit Care Med. 2005;32:170914.
2. Parker MG, Fraser GL, Watson DM, et al. Removal of propylene glycol and correction of increased osmolar gap by hemodialysis in a patient on
high dose lorazepam infusion therapy. Intensive Care Med. 2002;28:8184.
3. Wilson KC, Reardon C, Theodore AC, et al. Propylene glycol toxicity: a severe iatrogenic illness in ICU patients receiving IV benzodiazepines.
Chest. 2005;128:167481.
4. Yaucher NE, Fish JT, Smith HW, et al. Propylene glycol-associated renal toxicity from lorazepam infusion. Pharmacotherapy. 2003;23:109499.
5. Zar T, Graeber C, Perazella MA. Recognition, treatment, and prevention of propylene glycol toxicity. Sem Dialysis. 2007;20:21719.

RATIONALE (9) Answer: A


The choice of rewarming technique for a hypothermic patient should take into account the clinical condition of the patient (severity of hypothermia,
hemodynamic stability), presence of a perfusing or nonperfusing rhythm, availability of resources, and assessment of advantages and disadvantages of
available techniques. This patient has moderate hypothermia with a perfusing rhythm. A combination of active external rewarming and less invasive
active core rewarming techniques would be a reasonable choice for this patient. The most commonly used active external rewarming technique is a
forced-air rewarming blanket, which has a rewarming rate of 1.5C2.5C/h. Other external rewarming techniques are warm water immersion, heating
pads, and radiant heat lamps. Active core rewarming techniques include heated, humidified oxygen; warm IV fluids; thoracic lavage; peritoneal lavage;
gastric/bladder/rectal lavage; hemodialysis; cardiopulmonary bypass; and extracorporeal membrane oxygenation. The application of a warm blanket is
an example of passive external rewarming and is usually used with all other techniques but requires the addition of other measures in patients with
moderate to severe hypothermia. Gastric and bladder lavage depend on surface contact area for conductive heat transfer and have limited efficacy.
Cardiopulmonary bypass is not warranted in this patient since she has a perfusing rhythm and blood pressure. Hemodialysis is invasive and would
probably result in further hypotension which may have adverse cardiac effects.

REFERENCES (9)

1. Aslam AF, Aslam AK, Vasavada BC, Khan IA. Hypothermia: Evaluation, electrocardiographic manifestations, and management. Am J Med.
2006;119:297301.
2. Epstein E, Anna K. Accidental hypothermia. BMJ. 2006;332:7069.
3. Jurkovich GJ. Environmental cold-induced injury. Surg Clin N Am. 2007;87:24767.

RATIONALE (10) Answer: C

The presence of central nervous system (CNS) dysfunction in the setting of significant heat exposure distinguishes heatstroke from milder forms of heat
injury such as heat exhaustion. CNS dysfunction can include cerebellar abnormalities, seizures, and altered level of consciousness (including coma).
Although heatstroke is usually associated with temperatures greater than 40C (104F), the temperature measured on arrival to a hospital may not reflect
exposure because of cooling during transport. Tachycardia, orthostatic hypotension, and nausea and vomiting can occur in heatstroke and heat
exhaustion. Absence of sweating is not a reliable diagnostic finding for heatstroke as patients with exertional heat stroke may have profuse sweating.

REFERENCES (10)

1. Bouchama A, Dehbi M, Chaves-Carballo E. Cooling and hemodynamic management in heatstroke: practical recommendations. Crit Care.
2007;11:R54.
2. Bouchama A, Knochel JP. Heat stroke. N Engl J Med. 2002;346:197888.
3. Gaffin SL, Gardner JW, Flinn SD. Cooling methods for heatstroke victims. Ann Intern Med. 2000;132:678.
4. Howe AS, Boden BP. Heat-related illness in athletes. Am J Sports Med. 2007;35:138496.

RATIONALE (11) Answer: D

This patient has overdosed on 2 potentially life-threatening toxins: an opiate and acetaminophen. A narcotic overdose can usually be managed by
reversal of life-threatening effects (hypoventilation, central nervous system depression) with naloxone, a pure opioid antagonist. The goal of therapy is
to restore adequate spontaneous respirations rather than complete arousal. Given the large quantity of hydrocodone ingested, an initial dose of 2 mg is
more appropriate than 0.4 mg of naloxone. Doses of naloxone up to 20 mg may be required in patients who have taken large quantities of opioids or
synthetic opioids such as hydrocodone, oxycodone, methadone, and fentanyl. Intubation is not necessary if the patient arouses with naloxone. However,
because of the short half-life of naloxone, an infusion (one-half to two-thirds of the dose that reversed respiratory depression, administered hourly and
titrated as needed) may be needed until the respiratory depressive effects of the opioid wane.

The toxic dose of acetaminophen varies by individual but any dose greater than 140 mg/kg in an acute ingestion should be considered potentially toxic.
Although an acetaminophen level is needed to compare with the Rumack-Matthew nomogram to determine the need for antidote, the late presentation of
this patient mandates administering an initial dose of N-acetylcysteine while waiting for results. N-acetylcysteine (IV or oral) is most effective at
preventing liver toxicity when administered in the first 8 hours after ingestion.

This patient requires both naloxone and immediate administration of N-acetylcysteine, making option D the correct choice. Options A and C are
incorrect since they do not include N-acetylcysteine administration. Option B is not correct since intubation is not warranted as the first step.

REFERENCES (11)

1. Sporer KA. Acute heroin overdose. Ann Intern Med 1999;130:584.


2. Brok J, Buckley N, Gluud C. Interventions for paracetamol (acetaminophen) overdoses. Cochrane Database Syst Rev 2006;(2):CD003328.
3. Heard KJ. Acetylcysteine for acetaminophen poisoning. N Engl J Med. 2008;359:28592.
4. Schilling A, Corey R, Leonard M, Eghtesad B: Acetaminophen: old drug, new warnings. Cleveland Clin J Med 2010;77:1927.
RATIONALE (12) Answer: C

This patients clinical presentation is consistent with overdose of a calcium channel blocker. Typical findings include hypotension and bradycardia. The
hypotension in calcium-channel blocker overdose, especially verapamil, is related to negative inotropy more than bradycardia. Atropine is frequently
given in these situations but it is rarely effective. The appropriate initial intervention is the administration of IV calcium chloride (bolus and infusion)
but it is only effective in reversing toxic effects in approximately 50% of patients. After failure of calcium, glucagon is a reasonable next intervention
even though data on improving hemodynamics are better with beta-blocker overdose. Glucagon acts as an inotropic agent by increasing intracellular
cyclic adenosine monophosphate, a mechanism similar to adrenergic agents. After failure of these interventions, the use of regular insulin infusion with
glucose infusion to maintain euglycemia is most appropriate. Insulin acts as an inotropic agent but the exact mechanism is not known. Reported doses of
insulin have been variable but 0.5 to 1.0 U/kg/h have been most commonly used. Use of insulin euglycemia has been reported with beta-blocker toxicity,
but experience is limited. Transthoracic pacing may increase the heart rate but will not improve contractility or blood pressure (Option B). Milrinone
can be considered as an inotropic agent in calcium channel blocker toxicity but it would most likely worsen the hypotension (Option A). Evidence of
benefit of milrinone in this setting is limited. Calcium channel blockers are lipophilic, highly protein bound, and primarily undergo hepatic metabolism.
Thus, hemodialysis (Option D) would be unlikely to remove a significant amount of drug.

REFERENCES (12)

1. Kerns W. Management of beta-adrenergic blocker and calcium channel antagonist toxicity. Emerg Med Clin N Am. 2007;25:30931.
2. Megarbane B, Karyo S, Baud FJ. The role of insulin and glucose (hyperinsulinaemia/euglycaemia) therapy in acute calcium channel antagonist and
beta-blocker poisoning. Toxicol Rev. 2004;23:21522.
3. Shepherd G, Klein-Schwartz W: High-dose insulin therapy for calcium-channel blocker overdose. Ann Pharmacother 2005;39:92330.

RATIONALE (13) Answer: A

Rat poisons contain super warfarins that are more potent and longer acting than warfarin. The most common agent involved in poisonings is
brodifacoum. These agents inhibit the synthesis of vitamin K and thus the synthesis of clotting factors II, VII, IX, and X. The risk of bleeding increases
with a higher international normalized ratio (INR). In a patient with active bleeding, immediate factor replacement is required along with administration
of IV vitamin K. In this patient, bleeding with a pharyngeal hematoma posed a threat to airway patency and requires emergent treatment. Fresh frozen
plasma in large doses is required (15 mL/kg) and must be repeated every 6 to 8 hours to maintain sufficient factor levels. Although not approved for this
use, other options for factor replacement are prothrombin complex concentrate or activated factor VII. IV vitamin K is indicated in this situation because
it corrects the INR faster than subcutaneous administration. Thus, Option A offers the best interventions to quickly reverse the coagulopathy. Oral
vitamin K is indicated after stabilization for weeks to months due to the long half-life of the super warfarin.

REFERENCES (13)

1. Bruno GR, Howland MA, McMeeking A, et al. Long-acting anticoagulant overdose: Brodifacoum kinetics and optimal vitamin K dosing. Ann Emerg
Med 2000;36:26267.
2. Spahr JE, Maul JS, Rodgers GM: Superwarfarin poisoning: a report of two cases and review of the literature. Am J Hematol 2007;82:65660.
3. Watt BE, Proudfoot AT, Bradberry SM, et al. Anticoagulant rodenticides. Toxicol Rev 2005;24:25969.

RATIONALE (14) Answer: C

Hemodialysis is most likely to benefit this patient by removing acid metabolites of ethylene glycol. Methanol and ethylene glycol poisonings may be
characterized by the presence of an osmolar gap and anion gap metabolic acidosis. All alcohols are osmotically active substances, but only methanol
and ethylene glycol are metabolized to acids (see figure below). Isopropyl alcohol is metabolized to acetone and does not result in an anion gap
metabolic acidosis. However, patients ingesting ethylene glycol may present with a normal osmolar gap due to complete metabolism of the alcohol to
toxic metabolites (late presentation) or errors in the performance of the serum osmolarity. Similarly, patients ingesting ethylene glycol may present with
an osmolar gap and normal anion gap if concurrent ethanol ingestion prevents metabolism to acid metabolites or they present very early after ingestion.
Ethanol is preferentially metabolized by alcohol dehydrogenase. In this patient, the history is strongly suggestive of ethylene glycol ingestion, and it is
supported by the laboratory findings of anion gap acidosis, obtundation, and renal insufficiency. The low pH is a poor prognostic marker and immediate
hemodialysis should be initiated. The value of ethanol infusion or fomepizole in a delayed presentation is questionable, because ethylene glycol is likely
to be completely metabolized by alcohol dehydrogenase to toxic metabolites.

However, if there is any suspicion of the presence of ethylene glycol, oral or IV ethanol or IV fomepizole can be initiated. IV fluids are indicated for
volume resuscitation, and mechanical ventilatory support can be increased to offset the low pH. Delayed presentations usually do not have urinary
oxalate crystals that may be present transiently within 4 to 6 hours after ethylene glycol ingestion.
REFERENCES (14)

1. Barceloux DG, Bond GR, Krenzelok EP, et al. American Academy of Clinical Toxicology practice guidelines on the treatment of methanol
poisoning. J Toxicol Clin Toxicol 2002; 40:415446.
2. Barceloux DG, Krenzelok EP, Olson R, et al. American Academy of Clinical Toxicology practice guidelines on the treatment of ethylene glycol
poisoning. J Toxicol Clin Toxicol 1999;37:537560.
3. Glaser DS: Utility of the serum osmol gap in the diagnosis of methanol or ethylene glycol ingestion. Ann Emerg Med 1996;27:343346.
4. Megarbane B, Borron SW, Baud SJ. Current recommendations for treatment of severe toxic alcohol poisonings. Intensive Care Med 2005;31:189
195.

RATIONALE (15) Answer: B

Approximately 5%10% of patients with mild symptoms of alcohol withdrawal develop seizures. Most seizures are brief and self-limited but multiple
seizures (2 to 6) can occur in up to 60% of patients. Even when the seizure has terminated, IV lorazepam, 2 mg, should be administered to prevent
recurrent seizures (option B). Lorazepam would also help with preventing progression to delirium. Fosphenytoin is not effective, which makes option C
incorrect. A beta-blocker can treat minor withdrawal symptoms such as tachycardia and hypertension but will not prevent seizures or prevent
progression to delirium, making option A incorrect.

REFERENCES (15)

1. Brathen G, Ben-Menachem F, Brodtkorb E, et al. EFNS guideline on the diagnosis and management of alcohol-related seizures: report of an EFNS
task force. Eur J Neurol 2005;12:57581.
2. DOnofrio G, Rathlev NK, Ulrich AS, et al. Lorazepam for the prevention of recurrent seizures related to alcohol. N Engl J Med 1999;340:91519.
3. Kosten TR, OConnor PG. Management of drug and alcohol withdrawal. N Engl J Med 2003; 348:178695.
4. Rathlev NK, Ulrich AS, Delanty N, et al. Alcohol-related seizures. J Emerg Med 2006;31:15763.

RATIONALE (16) Answer: C

Burn patients with significant injuries have a hypermetabolic state due to release of catabolic hormones such as catecholamines, cortisol, and glucagon.
Formulas for estimating resting energy expenditure are often inaccurate in burn patients because of confounding conditions. Indirect calorimetry can be
useful in estimating caloric needs but is not available in all institutions. The current recommendation for daily caloric intake is 3040 kcal/kg/day
(option C). Higher caloric intake has been associated with decreases in infections, hospital length of stay, and mortality. Protein should make up 20%
25% of calories with an intake of 22.5 g/kg/day. Carbohydrates should make up approximately 60% of the daily calories. Enteral nutrition is preferred.

REFERENCES (16)

1. Chan MM, Chan GM. Nutritional therapy for burns in children and adults. Nutrition 2009; 25:261269.
2. Dickerson RN. Estimating energy and protein requirements of thermally injured patients: art or science? Nutrition 2002;18:43942.
3. Rimdeika R, Gudaviien D, Adamonis K, et al. The effectiveness of caloric value of enteral nutrition in patients with major burns. Burns
2006;32:8386.
PART 8: Pharmacologic Issues in the ICU

Instructions: For each question, select the most correct answer.

1. A 70-year-old man has been admitted to the ICU after a colectomy because of severe chronic obstructive pulmonary disease and pain management.
The procedure was uncomplicated, and the patient was extubated afterward. On examination, the patients RR is 16/min, and he rates his pain on a
010 scale as an 8. Arterial blood gas values are pH of 7.36, PaCO2 of 45 mm Hg, PaO2 of 75 mm Hg, and bicarbonate level of 28 mEq/L on oxygen,
4 L/min by nasal cannula.

Which of the following analgesics for postoperative pain management is most appropriate to minimize respiratory depression?
A. Morphine
B. Meperidine
C. Butorphanol
D. Hydromorphone
E. Oxycodone

2. A 45-year-old woman with chronic renal failure is admitted to the ICU with a hypertensive emergency. After 30 hours of treatment with
nitroprusside, the patient develops confusion and metabolic acidosis. Administration of which of the following is most likely to prevent/ treat these
symptoms?

A. Thiosulfate
B. Cyanocobalamin
C. Pyridoxine
D. Glucagon
E. Calcium chloride

3. A 44-year-old man with HIV is admitted to the ICU with severe community-acquired pneumonia requiring mechanical ventilation. The patient is on
an outpatient retroviral regimen that includes a protease inhibitor; lopinavir/ritonavir; and a nucleoside reverse-transcriptase inhibitor, zidovudine.
The patient has done very well with this regimen (viral load is nondetectable), and his infectious disease physician recommends that he be kept on
this regimen during his ICU hospitalization. Which of the following drugs should be avoided in this case?

A. Midazolam
B. Propofol
C. Haloperidol
D. Lorazepam
E. Morphine

4. A 47-year-old woman who is 155 cm tall (5 ft 1 in) and weighs 100 kg (222 lbs) falls down a flight of stairs. Past medical history is positive for
insulin-dependent diabetes mellitus for 20 years and chronic alcohol dependence. Her preoperative blood urea nitrogen level is 30 mg/dL, serum
creatinine level is 3.2 mg/dL, serum glucose level is 160 mg/dL, aspartate aminotransferase level is 110 U/L, and international normalized ratio is
1.4.

After open reduction and internal fixation of a left femoral neck fracture, she is admitted to the ICU because of severe alcohol withdrawal. The
following day, symptoms of withdrawal are much decreased, but she complains of a sudden onset of shortness of breath. A diagnosis of pulmonary
embolism is made by helical CT scan. Subcutaneous enoxaparin, 150 mg every 12 hours is begun.
Which of the following statements best characterizes her current enoxaparin dose?
A. She is excessively anticoagulated because dosing should be based on ideal body weight, not actual body weight.
B. She is inadequately anticoagulated because of morbid obesity and increased volume of distribution.
C. She is excessively anticoagulated because of hepatic dysfunction.
D. She is inadequately anticoagulated because of increased cytochrome P-450 activity.
E. She is excessively anticoagulated because of renal dysfunction.

5. A 54-year-old man with no medical problems is admitted with severe acute pancreatitis. He deteriorates quickly in the first 2 days of
hospitalization and requires intubation. On ICU day 3, the patient develops worsening abdominal distension, refractory hypotension, hypoxia, and
elevated peak airway pressures. His liver function test results are normal, and his glomerular filtration rate is estimated to be less than 30
mL/min/1.73 m2. Bladder pressures are consistent with diagnosis of abdominal compartment syndrome. To optimize abdominal perfusion pressure,
sedation and continuous neuromuscular blockade is considered. Which of the following is the best choice for neuromuscular blockade in this
patient?

A. Succinylcholine
B. Vecuronium
C. Pancuronium
D. Cisatracurium

6. A 55-year-old woman presenting to the hospital with subarachnoid hemorrhage receives management including nimodipine, phenytoin, and
famotidine. On hospital day 12, she has a diffuse, erythematous skin rash; confusion; hypotension; and severe distress. In the ICU, she has labored
respirations at 30/min, a systolic BP palpable at 80 mm Hg, pulse rate of 150/min, and temperature of 39.5C (103F). Periorbital edema is noted,
with a diffuse, morbilliform, maculopapular rash covering her entire body including the palms and soles. Which of the following agents is
associated with this disease?

A. Nimodipine
B. Phenytoin
C. Aminoglycosides
D. Famotidine

7. A 34-year-old man is admitted to the ICU after he is found unconscious at home. He has a history of chronic back pain, diabetes mellitus, and
depression. His current medications include the following:

Extended-release venlafaxine, 225 mg/day

Acetaminophen, 1,000 mg every 6 hours as needed for pain

Tramadol, 60 mg every 6 hours

Glipizide, 10 mg/day

Metformin, 1,000 mg every 12 hours

Blood cultures grow gram-positive cocci and chest radiography shows bilateral lower lobe infiltrates. The patient weighs 100 kg (220 lbs) and is
188 cm (6 feet 2 inches) tall. He has anaphylactic allergic reaction to penicillin. His temperature is 39.8C (104F), BP is 76/30 mm Hg, and HR is
120/min. His laboratory findings include the following: blood urea nitrogen, 37 mg/dL; serum creatinine, 2.7 mg/dL; sodium, 132 mEq/L;
potassium, 3.6 mEq/L; glucose, 287 mg/dL; lactic acid, 2.8 mg/dL; and creatine kinase, 23,000 U/L. He is diaphoretic and unresponsive.

Which of the following interventions will minimize the chances of patient mortality?
A. Start IV daptomycin, 6 mg/kg daily.
B. Start IV linezolid, 600 mg every 12 hours.
C. Give IV vancomycin, 2,500 mg, followed by IV vancomycin, 1,500 mg daily with a goal trough of 1520 g/mL.
D. Start IV tigecycline, 100 mg, followed by IV tigecycline, 50 mg every 12 hours.

8. A 72-year-old man with stage 3 chronic kidney disease had an emergent open repair of ruptured thoracic aortic aneurysm 4 days ago. He is now
oliguric with a serum creatinine level of 5.1 mg/dL. He reports incisional pain after extubation. Which of the following agents is most appropriate
for the management of his acute pain in the ICU?

A. Morphine
B. Fentanyl
C. Methadone
D. Meperidine

9. A 37-year-old man with type 1 diabetes mellitus and a history of kidney transplantation 7 years ago presents with delirium caused by diabetic
ketoacidosis. His calculated serum osmolarity peaks at 304 mOsm/L, and serum glucose level reaches 528 mg/dL. Initial serum sodium level is 132
mEq/L, which normalizes to 139 mEq/L after 18 hours. The patients mental status normalizes 2 days after the administration of insulin and
hydration. Three weeks later, he notes that his right arm and leg are fidgety, a symptom that rapidly increases in intensity. Physical examination
reveals right hemichorea with a ballistic component. MRI of the brain is shown in the Figure.

Which of the following best explains the above symptoms and radiologic findings?
A. Ischemic stroke
B. Septic emboli to the brain
C. Osmotic demyelination syndrome
D. Huntington chorea
E. Autoimmune demyelination

10. A 38-year-old, morbidly obese man is admitted to the ICU for persistent cyanosis and reported hypoxemia after undergoing bronchoscopy and
transbronchial biopsy for suspected sarcoidosis. He underwent the procedure with topical benzocaine anesthetic plus midazolam and fentanyl for
sedation. Shortly afterward, he became hypoxic, with SpO2 of 85% on pulse oximetry and cyanotic lips. His examination on arrival to the ICU
reveals equal bilateral breath sounds and bilateral rales, which were also noted on preprocedure examination. On FIO2 of 1.0 via nonrebreather
mask, his oxygen saturations fail to improve. Arterial blood gas studies on nonrebreather mask show a pH of 7.30, PaCO2 of 30 mm Hg, and PaO2 of
380 mm Hg.

Which of the following is the most likely etiology of the patients decompensation?
A. Iatrogenic pneumothorax
B. Aspiration pneumonitis
C. Intracardiac shunt
D. Topical benzocaine
E. Hypoventilation from oversedation

11. A 39-year-old woman is brought to the emergency department for suspected overdose after being found down at home by her fianc. The toxicology
screen reveals tricyclic antidepressants and benzodiazepines. Her admitting ECG is shown in the Figure.

While in the emergency department, she has worsening mental status and is intubated for airway protection after onset of seizures. Shortly after
intubation, she has an unstable, wide-complex tachycardia requiring resuscitation.

Which of the following management steps is most appropriate?


A. Amiodarone for wide-complex ventricular tachycardia
B. Phenytoin for seizures
C. Flumazenil for coingestion of benzodiazepines
D. Sodium bicarbonate infusion to maintain alkalemia with pH >7.45

12. Which of the following neuromuscular blocking agents would be most appropriate for rapid-sequence intubation in a patient with a contraindication
to succinylcholine?

A. Pancuronium
B. Vecuronium
C. Atracurium
D. Rocuronium

13. A 25-year-old man is diagnosed with spinal osteomyelitis after presenting to the hospital with back pain and fever. He has a history of IV drug use
and ethanol abuse. Cultures grow methicillin-resistant Staphylococcus aureus. The patient is discharged home after 2 weeks of vancomycin in the
hospital with linezolid, 600 mg twice daily (to complete a 6-week course), and oxycodone, 10 mg every 6 hours as needed for pain.

Blood testing is done 4 weeks later and the patient is called by his primary physician to be admitted as soon as possible to the hospital. Laboratory
data show the following: sodium, 132 mEq/L; potassium, 4.0 mEq/L; chloride, 108 mEq/L; bicarbonate, 7 mEq/L; blood urea nitrogen, 18 md/dL;
creatinine, 0.9 mg/dL; and glucose, 96 mg/dL. His WBC count is 8,000/L with no left shift, hemoglobin level is 14 g/dL, and platelet count is
98,000/L.
An intensivist is called to evaluate the patient in the emergency department for possible ICU admission. He is feeling fine, and physical examination
findings including vital signs are unremarkable except for mild back tenderness on passive and active motion.The repeat blood work is essentially
unchanged, and lactic acid level shows 6 mmol/L. Arterial blood gas results are pH of 7.28, PaCO2 of 20 mm Hg, and PaO2 of 104 mm Hg on room
air. The measured serum osmolality is 286 mOsm/kg.

Which of the following is the most likely diagnosis?


A. Early sepsis
B. Linezolid-induced lactic acidosis
C. Alcoholic ketoacidosis
D. Isopropyl alcohol intoxication

14. A 30-year-old man who was involved in a motor vehicle collision is admitted to the ICU with deep partial- and full-thickness burns to over 70% of
his body surface area. Three days after admission, he is in respiratory distress and requires intubation.

Which of the following medications is contraindicated in this patient?


A. Cisatracurium
B. Succinylcholine
C. Pancuronium
D. Vecuronium
E. Rocuronium

15. Which of the following bacteria are adequately treated with IV colistin (colistimethate)?

A. Serratia marcescens
B. Burkholderia cepacia
C. Acinetobacter baumanii
D. Methicillin-resistant Staphylococcus aureus
E. Proteus vulgaris

16. A 63-year-old woman is admitted to the ICU for status epilepticus. She is initially treated with lorazepam and a loading dose of phenytoin,
followed by a maintenance dose of 200 mg 3 times a day. Because of recurrent breakthrough seizures, infusions of midazolam, 10 mg/h, and
propofol, 15 mg/kg/h, are started. In addition, she receives cefepime and linezolid for suspected ventilator- associated pneumonia.

Five days later, she develops severe hypotension requiring norepinephrine infusion. Her temperature is 38.6C (101.5F), and HR is 120/min with
intermittent bursts of supraventricular tachycardia.

Laboratory findings include the following: serum sodium, 142 mEq/L; chloride, 100 mEq/L; bicarbonate, 12 mEq/L; potassium, 5.8 mEq/L;
creatinine, 3.4 mg/dL; creatine kinase, 6,200 U/L; blood lactate, 14 mg/dL; and hemoglobin, 8.5 g/dL. Urinalysis is positive for hemoglobin but no
red cells; chest radiograph is unremarkable except for the presence of endotracheal and nasogastric tubes. Arterial blood gas shows PaO2 of 90 mm
Hg, PaCO2 of 26 mm Hg, and pH of 7.21.

Which of the following is the most appropriate next intervention?


A. Discontinue midazolam.
B. Discontinue propofol.
C. Discontinue linezolid.
D. Transfuse packed red blood cells to increase hemoglobin level to 10 g/dL.
E. Start empiric antifungal coverage pending culture results.
17. Four days ago, a 59-year-old man had open repair of a ruptured abdominal aortic aneurysm that resulted in acute kidney injury. He is being weaned
on the ventilator but is agitated and not cooperative. His Confusion Assessment Method for the ICU (CAM-ICU) score is positive. His current
sedation is a propofol drip, 40 g/kg/min, and his current pain regimen is fentanyl, 100 g/h infusion. Dexmedetomidine is being considered for this
patient. Which of the following statements about dexmedotomidine should guide management of this patient?

A. The dose should be adjusted for the patients renal insufficiency.


B. A loading bolus can cause transient hypotension.
C. It has been associated with osmolar gap.
D. It does not cause respiratory depression.
E. Effects are not changed by propofol or opioids.
PART 8: Pharmacologic Issues in the ICU

ANSWERS:

1C; 2A; 3A; 4E; 5D; 6B; 7C; 8B; 9C; 10D; 11D; 12D; 13B; 14B; 15C; 16B; 17D

RATIONALE (1) Answer: C

Opioids have supraspinal and spinal modes of action. Opioid receptors are found in the central nervous system, especially in the midbrain and lamina I
of the dorsal horn in the spinal cord, in visceral and vascular smooth muscle, in musculoskeletal structures, and at the terminals of peripheral
sympathetic and sensory neurons. Opioids modulate pain by 1) presynaptic opioid receptor binding, inhibiting the release of excitatory
neurotransmitters; 2) decreased activity in interneurons; and 3) postsynaptic hyperpolarization. Opioid receptors are classified into groups based on
their pharmacologic effects by an agonist. The mu-receptors are subdivided into mu1 and mu2 receptors, where the mu1 receptor is responsible for
supraspinal analgesia, and the mu2 receptor is responsible for respiratory depression, bradycardia, and physical dependence. Stimulation of delta
receptors produces analgesia, mood alterations, and emesis. Kappa receptors modulate spinal analgesia and have minimal respiratory depression.
Sigma receptor activation results in excitatory symptoms, such as dysphoria, hypertonia, and tachycardia.

Morphine, meperidine, hydromorphone, and oxycodone are nonspecific mu receptor agonists, which means all will depress respiration to a similar
extent when given as equipotent doses. The mu opioid receptor mediates a dose-dependent depression of ventilation, mainly via a direct action on the
medullary respiratory center. A decreased response to hypoxic drive and a decreased response to carbon dioxide occur. Abnormal breathing can be
manifested as a shallow, rapid breathing or slow Cheyne-Stokes respiration with normal or large tidal volumes and intermittent apnea. These patients
may still be conscious and breathe when reminded.

Butorphanol is an opioid agonist-antagonist. Other agonist-antagonist opioids are nalbuphine and pentazocine. The common effect seen when giving this
group of opioids is analgesia with minimal respiratory depression. Each drug has a different effect on the other opioid receptors. Butorphanol does not
have significant mu receptor activity, while nalbuphine and pentazocine are mu receptor antagonists. Their analgesic activity is modulated by stimulation
of spinal kappa receptors. Use of this family of opioids on chronic pain patients taking mu receptor agonists, such as oxycodone or methadone, should be
avoided or carefully monitored because of the potential for causing opioid withdrawal.

REFERENCES (1)

1. Austrup ML, Korean G. Analgesic agents for the postoperative period. Surg Clin North Am.1999;79:253273.
2. Bowdle TA. Adverse effects of opioid agonists and agonist-antagonists in anaesthesia. Drug Safety.1998;19:173189.
3. Hoskin PJ, Hanks GW. Opioid agonist-antagonist drugs in acute and chronic pain states. Drugs.1991;41:326344.
4. Shook JE, Watkins WD, Camporesi EM. Differential roles of opioid receptors in respiration, respiratory disease, and opiate-induced respiratory
depression. Am Rev Respir Dis. 1990;142:895909.
5. Schuh KJ, Walsh SL, Stitzer ML. Onset, magnitude and duration of opioid blockade produced by buprenorphine and naltrexone in humans.
Psychopharmacology. 1999;145:162174.

RATIONALE (2) Answer: A

This patient is developing signs and symptoms consistent with cyanide toxicity from nitroprusside. Nitroprusside has been demonstrated to cause
toxicity through the release of cyanide and accumulation of thiocyanate. Cyanide toxicity can present with unexplained cardiac arrest and changes in
mental status, including convulsions, encephalopathy, and coma. Metabolic acidosis can also be present, although this may be a late event. Risk of
cyanide toxicity can be decreased by utilizing nitroprusside at recommended doses for a short period of time. It has also been recommended that patients
receiving high doses of nitroprusside (410 mg/kg/min) receive an infusion of thiosulfate. Furthermore, hydroxocobalamin (vitamin B12a) is safe and
effective in preventing and treating cyanide toxicity associated with use of nitroprusside. Hydroxocobalamin may be given as a continuous infusion at a
rate of 25 mg/h. It is important to note that cyanocobalamin (vitamin B12) is not effective as an antidote and is not capable of preventing cyanide toxicity.

REFERENCES (2)

1. Curry SC. Sodium nitroprusside. In: Brent H, et al. Critical Care Toxicology. Philadelphia, PA: Mosby;2005.
2. Schulz V, Gross R, Pasch T, et al. Cyanide toxicity of sodium nitroprusside in therapeutic use with and without sodium thiosulphate. Klin
Wochenschr 1982; 60:13931400.
3. Varon J, Marik PE. The diagnosis and management of hypertensive crises. Chest. 2000; 118:214227.
RATIONALE (3) Answer: A

With the widespread use of antiretroviral therapy in patients with HIV, it is important for the intensivist to recognize some of the potential drug
interactions and side effects of antiretroviral medications. Decisions regarding continuation of antiretroviral therapy in patients admitted to the ICU
should be discussed with HIV experts. In addition, a team approach with consultation with a critical care pharmacy specialist is important to avoid
complications related to drug interactions. Common ICU drugs contraindicated with the use of non-nucleoside reverse-transcriptase inhibitors include
midazolam and triazolam. Common ICU drugs, contraindicated with the use of protease inhibitors include the following: midazolam, triazolam,
amiodarone, proton pump inhibitors (with atazanavir), histamine2 blockers, propafenone, and quinidine. Recognition of potential interaction between
these medications and retrovirals is important when treating patients with HIV in the ICU. In this case, of the given list, midazolam would be
contraindicated both with the protease inhibitor and the non-nucleoside reverse-transcriptase inhibitors. In such a patient, alternatives to midazolam,
such as lorazepam or oxazepam, should be considered.

REFERENCES (3)

1. Department of Health and Human Sevices. Guidelines for the use of antiretroviral agents in HIV-1-infected adults and adolescents. Available at:
www.aidsinfo.nih.gov/Guidelines/GuidelineDetail.aspx?MenuItem=Guidelines&Search=Off &GuidelineID=7&ClassID. Accessed June 14, 2007.
2. Huang L, Quartin A, Jones D, et al. Intensive care of patients with HIV infection. N Engl J Med 2006; 355:173181.

RATIONALE (4) Answer: E

Low-molecular-weight heparins (LMWHs) do not undergo hepatic metabolism and primarily undergo renal elimination. Generally, dosing of LMWH is
based on actual body weight; however, if a patient weighs more than 130 kg (286 lbs), standard dosing may cause excessive anticoagulation. This
patient is likely to be excessively anticoagulated from LMWH accumulation due to her renal failure, not from excessive LMWH dose. The primary
advantages of LMWH, compared with unfractioned heparin (UFH), are better bioavailability and consistency of action. Bioavailability is greater than
90% for LMWHs, but only 30% for UFH. This difference is related to UFH's nonspecific binding to proteins and cells. This binding contributes not only
to its poor bioavailability but also affects anticoagulant activity and decreases plasma half-life. UFH is metabolized by a rapid, zero-order kinetic
enzyme system within the liver. The saturable hepatic kinetics results in inconsistent elimination that also increases variability in drug effect. UFH and
its metabolites undergo first-order renal clearance kinetics. In contrast, less-negative-charged LMWH molecules result in relatively little nonspecific
protein binding, and dose-independent renal clearance of LMWHs results in predictable antithrombotic activity. Typically, anticoagulation monitoring is
not needed.

Plasma half-life of LMWHs is approximately 2 to 4 times longer than that of heparin, enabling them to be administered only once or twice daily.
Subcutaneous LMWHs are well absorbed from the subcutaneous tissue, are rapidly distributed to most organs and tissues, and attain antithrombotic
levels within 30 minutes of administration. Subcutaneous absorption of heparin is more variable, however, and antithrombotic levels might not be
reached for 1 to 2 hours. Anticoagulation with LMWH can be achieved with minimal elevation of partial thromboplastin time. When monitoring is done,
synthetic substrate-based assays for measuring thrombin and factor Xa inhibition are used. The anti-Xa assays are more sensitive to the effects of
LMWHs and are used more commonly than antithrombin assays. The College of American Pathologists released a consensus report on the laboratory
monitoring of anticoagulant therapy in 1998. When monitoring LMWH, the College of American Pathologists recommends checking anti-Xa activity at
peak effect (approximately 4 hours after administration), because peak levels correlate with safety and efficacy over trough levels, obtained just prior to
administration of a dose. For the acute management of venous thromboembolic disease, the College of American Pathologists suggests a peak target
level of 0.5 to 1.1 U/mL for twice-daily recipients and 1.0 to 2.0 U/mL for once-daily patients. The College of American Pathologists recommends
monitoring for patients who have renal insufficiency, for patients who will be receiving LMWHs for prolonged periods (months), or for patients who
are at extreme body weights (130 kg [286 lbs]) or who are newborn.

REFERENCES (4)

1. Hirsh J, Warkentin TE, Shaughnessy SG, et al. Heparin and low-molecular-weight heparin: Mechanisms of action, pharmacokinetics, dosing,
monitoring, efficacy, and safety. Chest. 2001;119:64S94S.
2. Mousa SA. Comparative efficacy of different low-molecular-weight heparins (LMWHs) and drug interactions with LMWH: implications for
management of vascular disorders. Semin Thromb Hemost. 2000;26:3946
3. Samama MM, Gerotziafas GT. Comparative pharmacokinetics of LMWHs. Semin Thromb Hemost. 2000;26:3138.
4. Smith BS, Gandhi PJ. Pharmacokinetics and pharmacodynamics of low-molecular-weight heparins and glycoprotein IIb/IIIa receptor antagonists in
renal failure. J Throm Thrombolysis. 2001;11:3948.
5. Spencer FA, Ball SP, Zhang Q, et al. Enoxaparin, a low-molecular-weight heparin, inhibits platelet-dependent prothrombinase assembly and activity
by factor-Xa neutralization. J Thromb Thrombolysis. 2000;9:223228.
RATIONALE (5) Answer: D

Neuromuscular blockade is available in both depolarizing and nondepolarizing preparations.

Depolarizing agents such succinylcholine directly stimulate the acetylcholine receptors at the postsynaptic endplate. They are not suggested for long-term
neuromuscular blockage

Nondepolarizing preparations act as competitive antagonists, binding to the postsynaptic endplate acetylcholine receptors but not stimulating them.
Several agents have active metabolites, which can prolong neuromuscular blockade. Both vecuronium and pancuronium are degraded by the liver into
active metabolites of 3-hydroxypancuronium and 3-desacetylvecuronium, respectively, both of which have up to 50% of the activity of the parent drug.
These metabolites then depend on renal excretion for clearance. So in hepatic or renal insufficiency, the effect of the neuromuscular blockade can be
markedly prolonged.

Cisatracurium is susceptible to degradation by Hofmann elimination and ester hydrolysis, components of the in vivo metabolic processes. Hofmann
elimination is an organ-independent chemodegradative mechanism, so there is little or no risk from the use of cisatracurium in patients with liver or
renal disease compared to the use of other neuromuscular blocking agents.

REFERENCES (5)

1. Society of Critical Care Medicine, American Society of Health-System Pharmacists. Sedative, analgesia, and neuromuscular blockade of the
critically ill adult: revised clinical practice guidelines for 2002. Crit Care Med 2002;30:117118.
2. De Laet I Hoste E, Verholen E, et al. The effect of neuromuscular blockers in patients with intra-abdominal hypertension. Intensive Care Med. 2007;
33:18114.
3. Macalino JU, Goldman RK, Mayberry JC. Medical management of abdominal compartment syndrome: case report and a caution. Asian J Surg.
2002;25:2446.

RATIONALE (6) Answer: B

Toxic epidermal necrolysis (TEN) and Stevens-Johnson syndrome (SJS) are severe adverse cutaneous drug reactions that predominantly involve the
skin and mucous membranes. They are characterized by mucocutaneous tenderness, hemorrhagic erosions, erythema, and more or less severe epidermal
detachment presenting as blisters and areas of denuded skin. These affect approximately 1 or 2 people per million annually and are considered medical
emergencies as they are potentially fatal. The clinical diagnosis should be confirmed by a skin biopsy showing full-thickness necrosis of the epidermis.

Currently, TEN and SJS are considered to be 2 ends of a spectrum of severe epidermolytic adverse cutaneous drug reactions, differing only by their
extent of skin detachment.

Mycoplasma pneumoniae and herpes simplex virus infections are well documented causes that exist alongside rare cases in which the etiology remains
unknown. Several drugs are at risk of inducing TEN/SJS, including allopurinol, trimethoprim/sulfamethoxazole and other sulfonamide antibiotics,
aminopenicillins, cephalosporins, quinolones, carbamazepine, phenytoin, phenobarbital, and NSAIDs of the oxicam type. Because of the high risk of
mortality, management of patients with SJS/TEN requires rapid diagnosis; evaluation; identification and interruption of the culprit drug; specialized
supportive care, ideally in an ICU; and consideration of immunomodulating agents such as high-dose IV immunoglobulin therapy.

REFERENCES (6)

1. Eisen ER, Fish J, Shear NH. Management of drug-induced toxic epidermal necrolysis. J Cutan Med Surg. 2000 Apr;4:96102.
2. Halevy S. Stevens-Johnson syndrome and toxic epidermal necrolysis--updates and innovations. Harefuah. 2010 Mar;149:186-90, 193.
3. Harr T, French LE. Toxic epidermal necrolysis and Stevens-Johnson syndrome. Orphanet J Rare Dis. 2010 Dec 16;5:39.
4. Viera MH, Perez OA, Patel JK, et al. Phenytoin-associated hypersensitivity syndrome with features of DRESS and TEN/SJS. Cutis. 2010
Jun;85:3127.

RATIONALE (7) Answer: C

Option A is incorrect because, while daptomycin would be an appropriate agent for gram-positive bacteremia, it is inactivated by lung surfactant and
does not achieve therapeutic levels in the lung. Therefore, it is not indicated for pneumonia. Daptomycin at higher doses can cause elevation of creatine
kinase concentrations, and this patient has a significantly elevated baseline creatine kinase level.

Option B is incorrect because, while linezolid is an appropriate agent for gram-positive pneumonia, it is also a weak monoamine oxidase inhibitor
(MAOI). MAOIs can place the patient at risk of serotonin syndrome when used concurrently with antidepressants.

Option C is correct. An immediate 2530 mg/kg loading dose should be followed by a 1520 mg/kg dose adjusted for the patient's renal function. The
ideal antibiotic regimen must be dosed appropriately to treat the offending organism and reach the site of infection with minimal drug interactions or
toxicity. Increased morbidity and mortality has been demonstrated in the literature when the initial anti-infective is inappropriate or delayed.

Option D is incorrect because, while tigecycline is approved for community-acquired pneumonia, its use for bacteremia is controversial because of the
risk of subtherapeutic blood levels and treatment failure. Recent studies showing an increased risk of all-cause mortality for high-risk infections treated
with tigecycline led the FDA to issue a warning to consider alternative agents in high-risk infections.

REFERENCES (7)

1. Rybak M, Lomaestro B, Rotschafer JC, et al. Therapeutic monitoring of vancomcyin in adult patients: a consensus review of the American Society of
Health-System Pharmacists, the Infectious Diseases Society of America, and the Society of Infectious Diseases Pharmacists. AM J Health-Syst
Pharm. 2009;66:8298.
2. Liu C, Bayer A, Cosgrove S, et al. Clinical practice guidelines by the Infectious Diseases Society of America for the treatment of methicillin-
resistant Staphylococcus aureus infections in adults and children. CID. 2011;52:138.
3. Silverman JA, Mortin LI, Vanpraagh AD, et al. Inhibition of daptomycin by pulmonary surfactant: in vitro modeling and clinical impact. J Infect Dis.
2005 Jun;15;191: 214952.
4. Takeshita J, Litzinger MH. Serotonin syndrome associated with tramadol. J Clin Psychiatry. 2009;11:273.
5. Meagher A, Ambrose P, et al. The pharmacokinetic and pharmacodynamic profile of tigecycline. Clin Inf Dis. 2005;41 Suppl 4:S33340.

RATIONALE (8) Answer: B

Option A is incorrect because, although morphine mainly undergoes hepatic metabolism, accumulation of the metabolite morphine-6-glucuronide may
prolong the sedative effective of morphine in renal failure. Furthermore, linear accumulation of morphine in renal failure patients has been demonstrated.

Option B is correct. Fentanyl is metabolized by the liver to inactive metabolites. Although an increase in sensitivity has been ascribed to fentanyl in
renal failure patients, its short half-life, predictable kinetics, and nontoxic metabolities make it the most appropriate choice.

Option C is incorrect because, while methadone is metabolized by the liver to inactive metabolites, the long half-life and unpredictable kinetics of
methadone make it very undesirable for chronic pain management. These traits make the drug more difficult to titrate for management of acute pain.

Option D is incorrect since normeperidine, the metabolite of meperidine, can accumulate in renal failure patients, with repeated dosing resulting in
seizure, tremors, and anxiety.

REFERENCES (8)

1. Ball M, McQuay HJ, Moore RA, et al. Renal failure and the use of morphine in intensive care. Lancet. 1985;1:784786.
2. Portenoy RK, Foley KM, Stulman J, et al. Plasma morphine and morphine-6- glucuronide during chronic morphine therapy for cancer pain: plasma
profiles, steady-state concentrations and the consequences of renal failure. Pain. 1991;47:1319.
3. Toombs K, Kral L. Methadone treatment for pain states. American Family Physician. 2005; 71:135358.
4. American Pain Society. Principles of Analgesic Use in the Treatment of Acute Pain and Cancer Pain. 5th edition. Glenview, IL: American Pain
Society; 2003.
5. Kaiko RF, Foley KM, Grabinski PY. Central nervous system excitatory effects of meperidine in cancer patients. Ann Neurol. 1983;13:180185.

RATIONALE (9) Answer: C

The osmotic demyelination syndrome refers to myelinolysis, typically in the central pontine region, which occurs as a result of osmotic derangements.
Although the condition is typically associated with rapid correction of hyponatremia, diabetic ketoacidosis can also cause this syndrome as a result of
osmotic shifts generated by rapid changes in serum glucose levels. Blood glucose should be monitored hourly in patients with diabetic ketoacidosis and
the insulin infusion should be adjusted to ensure a fall in glucose level of 50 mg/dL/h. Controlling the rate of glucose correction in the pediatric
population is of utmost importance because of the development of cerebral edema in this population with overt rapid correction.

The locations of these lesions are atypical for embolic stroke or septic emboli. The patient has a very rapid progression of symptoms and no prior
history of abnormal movements, both atypical for Huntington disease. In the same way, the location of lesions and prior history makes autoimmune
demyelination unlikely.
REFERENCES (9)

1. Duker AP, Espay AJ. Hemichorea-hemiballism after diabetic ketoacidosis. N Engl J Med. 2010;363:e27.
2. Norenberg MD. Central pontine myelinolysis: historical and mechanistic considerations. Metab Brain Dis. 2010 Mar;25:97106.

RATIONALE (10) Answer: D

Local anesthetic agents such as benzocaine are the most common drugs implicated in the etiology of acquired methemoglobinemia. Methemoglobin is an
altered state of hemoglobin in which the ferrous (Fe2+) ions of heme are oxidized to the ferric (Fe3+) state. The ferric hemes of methemoglobin are unable
to bind oxygen. In addition, the oxygen affinity of any remaining ferrous hemes in the hemoglobin tetramer is increased. As a result, the oxygen
dissociation curve is "left shifted," leading to functional anemia and impaired oxygen delivery to the tissues. When methemoglobin levels rise above
10%, cyanosis frequently is present, and above this level, symptoms of severe hypoxemia including metabolic acidosis, coma, and death can occur.

Methemoglobinemia causes disparity between PaO2 on arterial blood gas studies and pulse oximetry readings. Oxyhemoglobin and deoxyhemoglobin
absorb light over a range of wavelengths. Pulse oximetry uses 2 wavelengths of light; 660 nm (red region) and 940 nm (infrared region). The ratio of the
absorbance at the two wavelengths determines the oxygen saturation. In the red region, oxyhemoglobin absorbs less light than deoxyhemoglobin, with the
reverse in the infrared region. A ratio of absorbance (660 nm divided by 940 nm) of 0.43 corresponds to 100% oxygen saturation, a ratio of 1
corresponds to a saturation of approximately 85%, and a ratio of 3.4 corresponds to 0% saturation. When methemoglobin levels are 32%35%, the
absorbance ratio reaches a plateau and the oximetry reading stabilizes at 82%85%.

Clues to the diagnosis are presence of central cyanosis with normal PaO2 on arterial blood gas analysis and discrepancy in oxygen saturation between
pulse oximetry and arterial blood gas. Recommended treatment is IV methylene blue, 12 mg/kg infused over 35 minutes.

The presence of the discrepancy between oxygen saturation in arterial blood gas and pulse oximetry rules out alternate choices.

REFERENCES (10)

1. Moore TJ, Walsh CS, Cohen MR, et al. Reported adverse event cases of methemoglobinemia associated with benzocaine products. Arch Intern Med.
2004;164:119296.
2. Umbreit J. Methemoglobinemia: it's not just bluea concise review. Am J Hematol. 2007;82:13444.

RATIONALE (11) Answer: D

Tricyclic antidepressants (TCAs) are the second most common medication in overdose after analgesic medications and have a high mortality rate. In
overdose, the effects are most pronounced as central nervous system (CNS) and cardiac toxicity. The anticholinergic effects and the inhibition of the
reuptake of norepinephrine and serotonin cause CNS toxicity. CNS symptoms include altered mental status and seizures. The anticholinergic effects,
inhibition of norepinephrine and serotonin, and peripheral alpha-adrenergenic blockade contribute to cardiac toxicity. Cardiac toxicity can lead to sinus
tachycardia with prolonged QRS, QTc, and PR intervals, leading to ventricular arrhythmias; cardiac toxicity is the primary cause of death. Right bundle-
branch block and hypotension can also occur.

Treatment of TCA overdose centers on supportive care. Gastric lavage and activated charcoal can be used within 2 hours of ingestion. Dialysis and
hemoperfusion is not effective because of high lipid solubility and protein binding. Alkalization of the urine and serum (to a pH >7.45 and until ECG
changes have resolved) is performed with sodium bicarbonate to decrease the amount of free drug. Lidocaine is the drug of choice for ventricular
arrhythmias, as most other drugs alter QRS, QTc, or PR intervals. Benzodiazepines or phenobarbital are used for the treatment of seizures, and
phenytoin is only used for refractory cases.

REFERENCES (11)

1. Mokhlesi B, Leiken JB, Murray P, et al. Adult toxicology in critical care. Chest. 2003;123:897922.
2. Alapat PM, Zimmerman JL. Toxicology in the critical care unit. Chest. 2008;133:1006-1013.

RATIONALE (12) Answer: D

Rocuronium causes neuromuscular blockade sufficient for intubation (80% neuromuscular blockade) in 60 to 90 seconds and complete neuromuscular
blockade usually occurs in 3 minutes. There is no histamine release associated with rocuronium and the half-life is 20 to 35 minutes. Elimination is both
hepatic and renal with 50% uptake in the liver and subsequent excretion in the bile. Renal elimination is less and approximately 30% is excreted
unchanged in the urine. Cirrhosis will decrease the clearance and increase the recovery time and half-life of rocuronium. There are reports of increased
pulmonary vascular resistance and this should be taken into consideration in patients with pulmonary hypertension. The usual dose for intubation is 0.6
mg/kg administered intravenously, which may be increased to a maximum of 1.2 mg/kg if indicated. The other agents listed require a longer time period,
usually over 2 minutes, to achieve conditions suitable for intubation. Atracurium may cause truncal and facial flushing due to histamine release, and
transient cardiac effects including mild tachycardia and a decrease in mean arterial pressure. Fortunately, these effects rarely last more than 5 minutes.
Vecuronium is generally well tolerated and devoid of the adverse effects common to atracurium. Vecuronium undergoes rapid hepatic uptake, resulting
in a shorter duration of action than pancuronium; however, the elimination and half-life are increased in hepatic disease. Renal disease does not interfere
with elimination unless the creatinine clearance is less than 10 mL/min. Pancuronium is an older agent with a slower onset (35 minutes) and longer
duration of action (6090 minutes). Over 80% of the dose is excreted unchanged in the urine and renal impairment may reduce the clearance by 35%55
% and increase the half-life and duration of effect.

REFERENCES (12)

1. Feldman SA, Fauvel N. Onset of neuromuscular block. In: Pollard BJ. Applied Neuromuscular Pharmacology. Oxford; New York: Oxford
University Press; 1994: 6984.
2. Murray MJ, Cowen J, DeBlock H, et al. Clinical practice guidelines for sustained neuromuscular blockade in the adult critically ill patient. Crit Care
Med. 2002;30:14256.

RATIONALE (13) Answer: B

Lactic acidosis induced by linezolid is becoming increasingly recognized. It is thought to be secondary to effects of the antibiotic on the mithochondria
and is seen more frequently in older patients and with longer duration of treatment.

This patient has an anion gap metabolic acidosis (anion gap of 17) with appropriate respiratory compensation. His osmolar gap is close to normal
(calculated osmolarity = [sodium 2 + blood urea nitrogen 2.8 + glucose 18] = 11 ). Our patient displays no signs of worsening infection and has a
normal white blood cell count, so option A is wrong. Isopropyl alcohol intoxication would not cause anion gap acidosis and would result in an osmolar
gap, so option D is wrong. The absence of nausea, vomiting, and signs of alcohol ingestion make alcoholic ketoacidosis less likely. Thus, option C is
wrong.

REFERENCES (13)

1. Apodaca AA, Rakita RM. Linezolid-induced lactic acidosis. N Engl J Med 2003;348:8687.
2. Beekmann SE, Gilbert DN, Polgreen PM. Toxicity of extended courses of linezolid: results of an Infectious Diseases Society of America Emerging
Infections Network survey. Diagn Microbiol Infect Dis 2008;62:407410.
3. Soriano A, Miro O, Mensa J. Mitochondrial toxicity associated with linezolid. N Engl J Med 2005;353:23052306.

RATIONALE (14) Answer: B

Neuromuscular blockers and sedatives are utilized in rapid-sequence intubation. The use of neuromuscular blockers improves success rates for
emergency endotracheal intubation and reduces the risk of complications.

Neuromuscular blocking agents are classified by their mechanism of action (ie, depolarizing or nondepolarizing). Succinylcholine is a depolarizing
agent, while pancuronium, vecuronium, and cisatracurium are nondepolarizing agents.

Depolarizing agents stimulate all cholinergic receptors by binding directly to the postsynaptic acetylcholine receptors, causing continuous stimulation of
these receptors followed by muscular paralysis. Nondepolarizing agents competitively inhibit the postsynaptic acetylcholine receptors of the
neuromuscular motor endplate.

Lethal hyperkalemic response to succinylcholine continues to be reported. The depolarization of the acetylcholine receptors by succinylcholine and its
metabolites leads to potassium efflux from the muscle, leading to hyperkalemia that can result in lethal arrhythmias. Succinylcholine is contraindicated in
patients with a personal or family history of malignant hyperthermia and in patients deemed to be at high risk of developing severe hyperkalemia, such as
patients with burns.

REFERENCES (14)
1. Li J, Murphy-Lavoie H, Bugas C, et al. Complications of emergency intubation with and without paralysis. Am J Emerg Med. 1999;17:1413.
2. Gronert GA. Cardiac arrest after succinylcholine: mortality greater with rhabdomyolysis than receptor upregulation. Anesthesiology. 2001;94:5239.
3. Martyn JA, Richtsfeld M. Succinylcholine-induced hyperkalemia in acquired pathologic states: etiologic factors and molecular mechanisms.
Anesthesiology. 2006;104:15869.

RATIONALE (15) Answer: C

Colistin is a bactericidal antibiotic that has both hydrophilic and lipophilic moeities that bind to the lipopolysaccharides and phospholipids on the outer
cell membrane of gram-negative bacteria, acting like a detergent. Colistin is not active against gram-positive organisms or gram-negative cocci.
Common gram-negative bacteria that are intrinsically resistant to colistin are Proteus, Providencia, Serratia, Neisseria, Moraxella, Burkholderia, and
some strains of Stenotrophomonas. Other bacteria that have variable resistance against colistin include Aeromonas, Vibrio, Prevotella, and
Fusobacterium. Colistin is used primarily for multidrug-resistant Pseudomonas, Klebsiella, and Acinetobacter. Colistin is most commonly administered
intravenously as it is not absorbed in the stomach and is eliminated by the kidneys. It can also be administered via inhalation, but can cause
bronchospasm in this route, so often pretreatment with bronchodilators is performed. The most common side effect is nephrotoxicity, which can occur in
up to 40% of patients and is more likely in patients with existing kidney dysfunction.

REFERENCES (15)

1. Li J, Nation RL, Turnidge JD, et al. Colistin: the re-emerging antibiotic for multi-drug resistant Gram negative bacterial infections. Lancet Infect Dis.
2006;6:589601.
2. Li J, Nation RL, Milne RW, et al. Evaluation of colistin as an agent against multi-resistant Gram-negative bacteria. Int J Antimicrob Agents.
2005;25:1125.

RATIONALE (16) Answer: B

Propofol-related infusion syndrome (PRIS) is a serious side effect with a high mortality rate. It is characterized by dysrhythmia (eg, bradycardia or
tachycardia), heart failure, hyperkalemia, lipemia, metabolic acidosis, and/or rhabdomyolysis or myoglobinuria with subsequent renal failure. Risk
factors include poor oxygen delivery, sepsis, serious cerebral injury, and the administration of high doses of propofol (usually doses >83 g/kg/min or
>5 mg/kg/h for >48 hours), but the syndrome has also been reported following large-dose, short-term infusions during surgical anesthesia. The onset of
the syndrome is rapid, occurring within 4 days of initiation. The mechanism of the syndrome has yet to be determined.

Initial management includes prompt discontinuation of propofol infusion. Propofol accumulates in tissue, so potential for prolonged and/or refractory
toxicity exists. Prolonged action has been noted following extended midazolam infusion, possibly because of metabolite accumulation, but there is no
evidence of severe toxicity as with propofol. Therefore, option A is incorrect. Linezolid is often associated with serotonin syndrome in the presence of
monoamine oxidase inhibitors. It is unlikely in the current scenario, so option C is incorrect. Liberal packed red blood cell transfusion practice in the
ICU leads to increased length of stay and mortality. Hemoglobin levels as low as 7 g/dL should be well tolerated, so option D is incorrect. Finally, there
is no evidence of fungal infection, so adding an antifungal agent is not indicated and option E is also incorrect.

REFERENCES (16)

1. Corbett SM, Montoya ID, Moore FA, et al. Propofol-related infusion syndrome in intensive care patients. Pharmacotherapy. 2008;28:25058.
2. Jacobi J, Fraser GL, Coursin DB, et al. Clinical practice guidelines for the sustained use of sedatives and analgesics in the critically ill adult. Crit
Care Med. 2002;30:11941.

RATIONALE (17) Answer: D

Dexmedetomidine is a sedative, analgesic, and anxiolytic medication that results in highly selective agonism of central alpha2-adrenergic receptors. It
has been used in the ICU for sedation, but is also approved by the FDA for use as a procedural sedative since it does not cause respiratory depression.
Since it has both sedative and analgesic properties, the dosage should be decreased when started in presence of other sedatives and opiods.

Dexmedetomidine has been shown to cause hypertension when given as a bolus because of peripheral stimulation of alpha2-adrenergic receptors. The
dose of the drug does not need to be adjusted for renal failure since it is metabolized mostly via glucuronidation to inactive metabolites. The diluent
does not result in an osmolal gap, as with lorazepam, regardless of the dose administered.
REFERENCES (17)

1. Karol MD, Maze M. Pharmacokinetics and interaction pharmacodynamics of dexmedetomidine in humans. Best Practice Res Clin Anesthaesiology.
2000;14:26169.
2. Cormack JR, Orme RM, Costello TG. The role of alpha2-agonists in neurosurgery. J Clin Neuroscience. 2005;12:3758.
3. Jacobi J, Fraser GL, Coursin DB, et al. Clinical practice guidelines for the sustained use of sedatives and analgesics in the critically ill adult. Crit
Care Med. 2002;30:119141.
4. Riker RR, Shehabi Y, Bokesch PM, et al. Dexmedetomidine versus midazolam for sedation of critically ill patients: a randomized trial. JAMA.
2009;301:48999.
5. Pandharipande PP, Pun BT, Herr DL, et al. Effect of sedation with dexmedetomidine versus lorazepam on acute brain dysfunction in mechanically
ventilated patients: the MENDS randomized controlled trial. JAMA 2007;298:264453.
PART 9: Surgical and Obstetrical Critical Care

Instructions: For each question, select the most correct answer.

1. A 54-year-old, white man weighing 84 kg (187 lbs) receives a double lung transplant for idiopathic pulmonary hypertension and is admitted to the
ICU immediately afterward. His donor, a 47-year-old, African American woman, died of complications of traumatic brain injury from a motor
vehicle accident. She had remained intubated for 5 days before her death and her ICU stay had been complicated by suspected aspiration.

The transplant recipient requires increasing oxygen support as his pulse oximetry is persistently dropping despite increasing FIO2. He subsequently
needs his positive end-expiratory pressure (PEEP) to be increased to 10 cm H2O to keep saturations above 90%. On day 2 after surgery, he is on a
volume-controlled mode with tidal volume set to 500 mL, RR of 18/min, FIO2 of 1.0, and PEEP of 10 cm H2O. His morning arterial blood gas
results reveal a pH of 7.34 and PaO2 of 123 mm Hg. A portable chest radiography confirms his auscultatory findings of bilateral diffuse, coarse
crackles, as it demonstrates bilateral 4-quadrant airspace processes with no evidence of significant effusions.

Which of the following is the most likely explanation of his hypoxemic respiratory failure?
A. Acute bilateral pneumonia since the donor was treated for an aspiration process
B. Worsening heart failure considering his borderline right ventricular dysfunction due to pretransplant pulmonary hypertension
C. Primary graft dysfunction
D. Transfusion-related acute lung injury

2. IV morphine, 4 mg every 3 hours, fails to relieve the leg pain of a 20-year-old man who had a gunshot wound to the leg with femoral artery and vein
injury. He had arterial repair and venous ligation. In spite of elevation of his leg, there is swelling from the thigh to the foot. Passive movement of
his foot causes profound calf pain. He has palpable pulses at the posterior tibial and dorsal interosseous muscles, and his ankle brachial index
(ankle blood pressure divided by the brachial blood pressure) is normal.

Which of the following steps is most appropriate?


A. Increase the amount of pain medicine.
B. Obtain an arteriography.
C. Perform fasciotomy.
D. Place an epidural catheter for pain control.
E. Anticoagulate the patient.

3. A 25 year-old man is admitted to the ICU for observation following a collision of his motorcycle with a tree. His Glasgow Coma Scale score is 10.
His neurologic workup is in progress.

Anticonvulsants to prevent the occurrence of early posttraumatic seizures are contraindicated in which of the following clinical situations?
A. Depressed skull fracture
B. Subdural hematoma
C. Subarachnoid hemorrhage
D. Penetrating brain injury
E. Cortical contusion

4. A 48-year-old man is admitted to the ICU for observation after he falls off a porch, striking the back of his head on the hard ground. His blood
alcohol level on arrival is 0.24%. He is drowsy but arousable and reports paresthesias in all 4 limbs. His neck is stabilized with a cervical collar
until he is more awake and cervical spine injury can be ruled out. On physical examination, he has 1/5 bilateral grip and wrist extension strength;
3/5 biceps, triceps, and deltoid strength; 4/5 bilateral plantar flexion and dorsiflexion strength; and 5/5 quadriceps, hamstring, and iliopsoas
strength.

The most likely diagnosis is:


A. Multilevel spondylosis and central cord syndrome
B. C5-C6 fracture/dislocation
C. Alcoholic polyneuropathy
D. Spinal epidural hematoma

5. A 30-year-old, female restrained driver is admitted to the ICU following a motor vehicle collision in which she was struck broadside by another
vehicle on the drivers side. She has a Glasgow Coma Scale score of 9-I, opening her eyes to voice, localizing on the right side, and having been
intubated. She has a left hemiparesis; left pneumothorax; multiple rib fractures; left femur fracture; dislocated left shoulder; and multiple bruises and
ecchymoses over her left neck, shoulder, and trunk. She has anisocoria with the right pupil 5 mm and reactive, and the left 3 mm and reactive with
left ptosis.

Pending the results of the secondary survey, the most likely diagnosis for the ocular findings is:
A. Traumatic right third nerve palsy
B. Carotid artery dissection in the neck
C. Brown-Sequard syndrome
D. Brachial plexus and lumbosacral plexus injuries
E. Incidental right-sided brain tumor

6. A 42-year-old woman is admitted with intracerebral and intraventricular hemorrhage and initially treated with an endoventricular drain. Eleven
days later, she develops fever, lethargy, and leukocytosis. In cerebrospinal fluid, glucose levels are decreased and protein levels are increased.

The most appropriate initial antimicrobial coverage in this patient is:


A. Daptomycin and metronidazole
B. Levofloxacin and clindamycin
C. Ertapenem and linezolid
D. Cefepime and vancomycin
E. Gentamicin and ampicillin

7. A 57-year-old man who had a double lung transplant for emphysema 7 months ago, with cytomegalovirus (CMV) mismatch, is transferred to the ICU
from the emergency department with a 2-day history of generalized headache. While in the emergency department, he experienced a 4-minute
generalized seizure, which partially resolved with IV benzodiazepines; shortly afterward, he experienced a second seizure and was intubated for
airway protection. Upon arrival to the emergency department, systolic BP was 209 mm Hg. Capillary blood glucose level was 154 mg/dL. The rest
of his vital signs and physical examination findings were reported as unremarkable.

Preadmission medications include prednisone, cyclosporine, azathioprine, voriconazole, valganciclovir, trimethoprim/sulfamethoxazole,


metoprolol, furosemide, and pravastatin.

CT of the head reveals symmetrical white matter edema in the posterior cerebral hemispheres.

In addition to appropriate blood pressure control with an IV antihypertensive agent with close monitoring and titration, the plan of care over the
following 24 hours should include:
A. Initiation of broad-spectrum antibiotics directed toward infectious meningitis
B. Thrombolytic therapy for suspected bilateral ischemic posterior strokes
C. Discontinuation of cyclosporine and initiation of an alternative calcineurin inhibitor for immunosupression
D. IV loading of antiepileptic for planned long-term therapy with phenytoin

8. A 34-year-old woman has orthotopic liver transplant followed by improving liver function. Four days later, she develops abdominal pain and
nausea. She is alert but tachypneic and diaphoretic. HR is 118/min, BP is 88/51 mm Hg, and temperature is 38.3C (101F). She has tenderness in
the right upper quadrant and guarding on abdominal examination. Abdominal drain contents appear yellow-green as opposed to serosanguinous
earlier. Total bilirubin level is 4.8 mg/dL (elevated from 1.1 mg/dL 24 hours ago); alanine aminotransferase (ALT) and aspartate aminotransferase
(AST) levels are 50% higher than 24 hours ago. International normalized ratio (INR) is 2.4, up from 1.3 when measured 24 hours ago. The patient
is receiving tacrolimus and mycophenolate mofetil for immune suppression.

Which of the following is the most likely etiology of this presentation?


A. Toxicity from immune suppressive therapy
B. Hepatic artery thrombosis
C. Primary graft dysfunction
D. Acute cellular rejection

9. A 48-year-old man is diagnosed with severe acute cellular rejection following liver transplantation. He is treated with two 6-day courses of high-
dose corticosteroids followed by tapering doses of corticosteroids and an increased dosage of tacrolimus. There is little improvement in liver
function or clinical status, which is marked by jaundice, elevated bilirubin level, and elevated international normalized ratio. A repeat biopsy
shows persistent rejection. He is started on antithymocyte globulin. Eight hours later he has confusion, dyspnea, and wheezing. HR is 120/min, BP is
98/60 mm Hg, RR is 28/min, and temperature is 38.9C (102F). He appears anxious, using accessory muscles of breathing. He is mildly confused
but has no focal neurologic deficit or nuchal rigidity. Bibasilar rales and prolonged expiration are present on chest auscultation.

Which of the following causes for the patients acute change in status is most likely?
A. Cytokine release and complement activation
B. Anaphylactic shock
C. Calcinuerin inhibitionmediated toxicity
D. Severe sepsis

10. Which of the following arrhythmias in the early postoperative phase of cardiac transplantation is associated with allograft rejection?

A. Ventricular premature contractions


B. Atrial flutter
C. Sinus tachycardia
D. New right bundle-branch block

11. A 40-year-old man is shot in the abdomen at point-blank range. He arrives at the local trauma center in extremis with a systolic BP of 60 mm Hg by
palpation. Ten units of packed red blood cells (PRBCs) are immediately available and transfusion is undertaken. In the operating room, he is noted
to have injuries to his aorta, superior mesenteric artery, left kidney, and colon.

What additional blood component administration, in tandem with surgical control, is appropriate to best achieve adequate hemostasis?
A. Transfusion of fresh frozen plasma (FFP) based on prothrombin time or international normalized ratio
B. Maintained 1:1:1 ratio of PRBCs to FFP to platelets throughout resuscitation
C. Single-dose factor VIIa
D. FFP and platelets to be transfused only once surgical hemostasis is achieved
12. A 58-year-old man had partial gastrectomy 20 days ago for a perforated antral gastric ulcer. His postoperative course was complicated by
anastomotic leak requiring re-exploration, ileus requiring total parenteral nutrition, and acute nonoliguric renal failure. He is currently treated with
piperacillin/tazobactam and fluconazole but still has fever and leukocytosis. The patients most recent blood culture is growing Candida species
which appear to be non-albicans. His central venous catheter is removed.

Which of the following interventions is indicated?


A. Discontinue fluconazole and start amphotericin B.
B. Continue fluconazole.
C. Discontinue fluconazole and start voriconazole.
D. Discontinue fluconazole and start polymyxin B.
E. Discontinue fluconazole and start caspofungin.

13. A 20-year-old man is admitted to the ICU after sustaining open comminuted pelvic fracture during a motorcycle accident. His initial pelvic
radiograph and CT with IV contrast only are shown in the Figure.

No other thoracoabdominal injuries are noted on CT. An external fixator is placed. His initial blood pressure upon arrival is normal but his pulse
rate remains 130/min. Over the course of several hours, he has required several fluid boluses for hypotension, which temporarily responds to the
fluid. His hematocrit remains at 28% despite 4 units of packed red blood cells. His chest examination is normal to auscultation and his abdomen is
slightly distended without guarding.

Which of the following is the most appropriate next step in his management?
A. Pelvic arteriogram and embolization
B. Emergent laparotomy
C. Transesophageal echocardiography
D. Continued supportive care

14. A 19-year-old woman is admitted to the ICU with a history of having been kicked in the upper abdomen by a horse. Her SpO2 declines from 95% to
88% on 50% mist mask. Her pulse rate is 110/min and her BP is 105/68 mm Hg. Her hematocrit is 32%. Her radiograph is shown in the Figure. She
is intbuated for airway protection.
Which of the following is the most appropriate next step in her care?
A. Emergent bronchoscopy and lavage
B. Emergent laparotomy
C. CT of chest, abdomen, and pelvis
D. Chest tube placement

15. An 82-year-old woman with hypertension and diabetes mellitus is admitted to the hospital with traumatic right subdural hematoma, rib fractures,
and second and third metatarsal fracture of her right foot. On hospital day 3, she develops worsening oxygenation and tachycardia. Her temperature
is 38.1C (100.5F). Her arterial blood gas results on oxygen, 3 L/min via nasal cannula are pH, 7.42; PaCO2 of 38 mm Hg, and PaO2 of 74 mm Hg,
with SpO2 of 94%. Her WBC count is 13,000/L with normal differential. Her chest radiograph is unremarkable and her chest angiogram is shown
in the Figure.

Which of the following is the most appropriate next step in her care?
A. Begin antibiotics for aspiration pneumonitis.
B. Increase positive end-expiratory pressure and minimize excess fluid.
C. Fully anticoagulate patient.
D. Place inferior vena cava filter.

16. A 25-year-old male is evaluated at the regional trauma center after a high-speed car crash. He is initially hypotensive but rapidly stabilizes after
several liters of saline. CT reveals multiple bilateral rib fractures, a right hemopneumothorax, and a grade 4 central liver laceration with moderate
perihepatic fluid but no active arterial extravasation. A right chest tube is placed and the patient is admitted for observation. Progressive jaundice is
noted during the next several days. Three days after admission, the patient vomits a large amount of blood with a resultant 4 g/dL drop in his
hemoglobin level.
After appropriate resuscitative measures, which of the following is the best course of management?
A. Immediate exploratory laparotomy for control of presumed liver bleeding
B. Exploratory thoracotomy for control of pulmonary hemorrhage
C. Diagnostic angiography for evaluation and embolization of hepatic arterial-biliary fistula
D. Upper GI endoscopy for diagnosis and control of peptic ulcer bleeding

17. An 18-year-old woman presents to the emergency department after being assaulted with a baseball bat to her head and neck. CT reveals minimal
subarachnoid hemorrhage along with multiple complex mid-face fractures. CT angiography reveals a focal right carotid dissection at the level of the
skull base with moderate luminal irregularity, but preserved distal flow. She is systemically stable without evidence of neurologic compromise.

Which of the following is the most appropriate treatment of the carotid dissection?
A. Direct surgical repair
B. Systemic anticoagulation
C. Angiography with stenting
D. Close observation

18. A 65-year-old man with chronic obstructive pulmonary disorder and insulin-dependent diabetes has just been extubated on postoperative day 4,
following an emergency operation for an obstructing left-sided colon cancer. He is receiving nasal oxygen and bronchodilator therapy. Following
an episode of coughing, his midline wound dressing is saturated with a large amount of salmon-colored fluid.

Which of the following is the most appropriate intervention?


A. Initiation of antibiotics for a wound infection
B. Observation for any further drainage
C. CT of the abdomen
D. Returning to the operating room
E. Open skin incision at the bedside

Both questions 19 and 20 use the following information:

A 68-year-old woman is brought to the emergency department in shock after reporting abdominal pain. A ruptured aortic aneurysm is diagnosed,
and the patient is taken for emergent surgical repair. After a prolonged operation requiring massive amounts of crystalloids and blood products, the
aneurysm is repaired. The patient is transferred to the ICU, intubated, and receiving vasopressor support.

On arrival, her temperature is 36.6C (97.8F), BP is 110/58 mm Hg, HR is 92/min, and RR is 20/min, with conventional mechanical ventilation
14, tidal volume of 450 mL, positive end-expiratory pressure (PEEP) of +5 cm H2O, and FIO2 of 100%. Urine output is 75 mL/h, and the patient
continues to receive 0.9% normal saline solution at 100 mL/h. Laboratory data include the following: hemoglobin, 9.4 g/dL; normal platelet count;
normal electrolytes; blood urea nitrogen, 25 mg/dL; and creatinine, 1.6 mg/dL.

Ten hours after arrival in the ICU, the BP drops to 80/36 mm Hg, and the central venous pressure is 18 mm Hg. Urine output has decreased to less
than 50 mL/h over the last 2 hours, despite increased IV fluids. Peak airway pressure increases from 20 cm H2O to 56 cm H2O, and the patient
develops hypoxemia that does not respond to an increase in FIO2 to 100% and PEEP of 12 cm H2O. Hemoglobin obtained from an arterial blood gas
measurement is 9.2 g/dL.

19. Which of the following diagnostic tests should be done first?

A. Transesophageal echocardiography
B. CT of abdomen and pelvis
C. Measurement of bladder pressure
D. Abdominal ultrasonography
E. Placement of pulmonary artery catheter
20. Following the test above, the patient continues to deteriorate. Which of the following interventions should be done next?

A. Aggressive resuscitation with packed red blood cells, fresh frozen plasma, and crystalloids
B. Emergent dialysis
C. Pericardiocentesis at the bedside
D. Emergent laparotomy
E. Change of ventilator to pressure-control mode

21. A 32-year-old woman, gravida 3, para 3, is admitted to the obstetrics and gynecology department following spontaneous vaginal delivery under
epidural anesthesia. The next morning she reports severe back pain, urinary incontinence, and lower extremity weakness.

Which of the following immediate steps is most appropriate?


A. Radiography of the spine
B. MRI of the thoracic and lumbar spine
C. CT of the head
D. Electromyography
E. Ultrasonography of the lower extremity

22. A 93-year-old woman with a history of chronic obstructive pulmonary disease, atrial fibrillation, and malnutrition is admitted to the surgical ICU
after a fall down a flight of stairs. Her workup is significant for an epidural bleed, small right pulmonary contusions, and multiple right-sided rib
fractures (ribs 58 and 11).

Which of the following is the optimal pain management for this patient?
A. Lidocaine patch to the affected area
B. Thoracic epidural anesthesia
C. Intercostal blocks
D. Paravertebral blocks
E. Morphine patient-controlled analgesia

23. A 35-year-old, African American woman in her third pregnancy is admitted after spontaneous rupture of membranes at 37 weeks gestation with
twins. She has had no serious concerns during this pregnancy except increasing fatigue and pedal edema during the past few weeks. She believes
her symptoms have been primarily due to her excessive weight gain during the pregnancy. She has an uncomplicated vaginal delivery.
Approximately 30 minutes after delivery, she reports shortness of breath. On examination, BP is 150/80 mm Hg and RR is 35/min with accessory
muscle use. Chest examination reveals an S3 gallop, and bilateral inspiratory rales are auscultated in both lung fields.

Which of the following is appropriate in the initial management of this patient?


A. Diuresis with furosemide
B. Blood pressure control with labetalol
C. Inotropic support with dobutamine
D. Afterload reduction with hydralazine
24. A 32-year-old woman is brought to the hospital for evaluation of nausea, vomiting, headache, and confusion during the past 5 days. She recently
delivered a healthy baby, and her pregnancy was characterized by hypertension and mild proteinuria. On admission, her BP is 212/128 mm Hg, HR
is 102/min, RR is 1822/min, temperature is 37.7C (99.8F), and oxygen saturation is 98% on 2 L of oxygen via nasal cannula. On neurological
examination, her Glasgow Coma Scale score is 14, and she is drowsy but arousable, oriented to self but not to place and date, and follows simple
commands. Cranial nerve examination findings are normal, but fundoscopy suggests papilledema. Motor findings are normal. Reflexes are 3+
throughout, toes are downgoing bilaterally, and no meningeal signs are noted. MRI in the emergency department at the request of the consulting
neurologist is shown in the Figure.

T1-weighted MRI with gadolinium

Diffusion-weighted MRI

The most appropriate intervention for this patient is to start:


A. IV nicardipine to lower mean arterial pressure 15%
B. IV phenytoin
C. IV methylprednisolone.
D. IV acyclovir
E. Hyperosmolar therapy with 3% saline solution

25. A 23-year-old woman is seen in the emergency department for fatigue and mild dyspnea on exertion and 2 days of low-grade fever. She had mild
intermittent asthma during adolescence but has not had recent exacerbations and is taking no medications. The emergency department physician
discharges her with a diagnosis of upper respiratory tract infection. He prescribes albuterol by metered dose inhaler as needed and recommends
follow-up with her obstetrician in a few days. Two days after discharge, she returns with a temperature of 39C (102.2F), confusion, nausea and
vomiting, and dyspnea. BP is 138/92mm Hg, HR is 112/min, RR is 25/min , and oxygen saturation is 91%. Arterial blood gas results show pH of
7.24, PaO2 of 62 mm Hg, PaCO2 of 59 mm Hg, and oxygen saturation of 89%. Chest radiograph reveals bilateral, scattered pulmonary infiltrates.

Which of the following interventions is most appropriate?


A. Aggressive diuresis and inotropic support
B. Obtain an arteriography.
C. Noninvasive ventilation
D. Mechanical ventilation
E. Additional bronchodilator therapy
PART 9: Surgical and Obstetrical Critical Care

ANSWERS:

1C; 2C; 3C; 4A; 5B; 6D; 7C; 8B; 9A; 10B; 11B; 12B; 13A; 14B; 15D; 16C; 17B; 18D; 19C; 20D; 21B; 22B; 23A; 24A;
25D

RATIONALE (1) Answer: C

Primary graft dysfunction is a severe form of ischemia reperfusion injury and is the major cause of early morbidity and mortality after lung
transplantation. Recipient, donor, and operative variables have been identified as risk factors. In this particular patient, the donor was female, African
American, older than 45 years, and had a prolonged mechanical ventilation, all of which are well-described donor characteristics related to primary
graft dysfunction. This patient's PaO2 to FIO2 ratio and radiological findings classify him as grade 3, which is associated with the highest mortality.
Treatment is mostly supportive although large centers' experience is being published, including early consideration of extracorporeal membrane
oxygenation.

Infectious processes from a donor that lead to hypoxemic respiratory failure in organs that were macroscopically secretion-free and radiologically
normal prior to transplantation are unlikely to occur within the first 5 days of transplantation. Although heart failure and fluid mismanagement must be
considered as a possible cause of airspace processes after transplantation, such a severe hypoxemia from heart failure without significant pleural
effusions is unlikely. There is no mention of massive transfusion needs in this patient.

REFERENCES (1)

1. Lee JC, Christie JD. Primary graft dysfunction. Proc Am Thorac Soc. 2009;6:3946
2. Christie JD, Kotloff RM, Pochettino A, et al. Clinical risk factors for primary graft failure following lung transplantation. Chest. 2003;124:1232
1241.
3. de Perrot M, Bonser RS, Dark J, et al. Report of the ISHLT working group on primary lung graft dysfunction: Part III. Donor-related risk factors and
markers. J Heart Lung Transplant. 2005; 24:14601467.

RATIONALE (2) Answer: C

This patient has compartment syndrome that occurs in the extremity from increased pressure in a closed fascial muscle compartment. The increased
pressure causes small outflow veins to collapse associated with increased swelling, arterial thrombosis, and nerve and muscle death. Venous
hypertension from femoral vein ligation has caused this patient's compartment syndrome.

Elevation of the involved extremity eliminates the need for fasciotomy in 80% of patients but has failed in this case. Bleeding into the compartment,
crush injury, infection, edema, snakebites, and ischemia reperfusion injury are other causes of compartment syndrome. In the awake patient, pain is the
most frequent symptom (ie, pain out of proportion to the injury). Pain serves as warning sign that the leg needs to be examined for other signs of
compartment syndrome. Both active and passive muscle activity increase pain. Paresthesias (web space between the first and second toe in lower
extremity) are often rapidly followed by paralysis. The presence or absence of the pulse does not correlate with compartment syndrome.

Compartment pressures can be measured by manometery or commercial devices. Compartment pressures greater than 30 mm Hg (exceeding capillary
perfusion pressure) must be addressed. Elevation of the extremity is the first treatment, followed quickly by fasciotomy. Early fasciotomy to decompress
the compartment will prevent neuromuscular deficits.

Adequate pain control is important but does not take the place of recognizing compartment syndrome. The ankle-brachial index (ABI) is obtained by
measuring the blood pressure in the posterior tibal artery and the brachial artery and is calculated using ankle blood pressure divided by the brachial
blood pressure. A normal ABI is 0.9 to 1.0. The further the ABI is from 1.0, the more arterial disease is present. Because a pulse with normal ABI is
present, there is no indication for an arteriogram. If an arterogram is obtained, the main arteries will be intact, and the muscle branches will taper.
Anticoagulation does not treat compartment syndrome.

REFERENCES (2)

1. Feliciano DV, Cruse PA, Spjut-Patrinely V, et al. Fasciotomy after trauma to the extremities. Am J Surg 1988;156:533536.
2. Johansen K, Lynch K, Paun M, et al. Noninvasive vascular tests reliably exclude occult arterial trauma in injured extremities. J Trauma
1991;31:515522.
3. Whitesides TE, Heckman MM. Acute compartment syndrome: update on diagnosis and treatment. J Am Acad Orthop Surg 1996;4:209218.
RATIONALE (3) Answer: C

Posttraumatic seizures are classified as early (within 7 days of trauma) or late (more than 7 days after trauma). While prevention of seizure activity is
desirable, anticonvulsants are associated with neurobehavioral and other medical side effects. Therefore, long-term prophylaxis is not recommended.
Phenytoin has been shown to be effective for the prevention of early posttraumatic seizures. Valproate has a similar effect but may be associated with
higher mortality rates.

REFERENCES (3)

1. Brain Trauma Foundation; American Association of Neurological Surgeons; Congress of Neurological Surgeons; et al. Guidelines for the
management of severe traumatic brain injury. XIII. Antiseizure prophylaxis. J Neurotrauma. 2007;24 Supplement 1:S83S86.
2. Dikmen SS, Temkin NR, Miller B, et al. Neurobehavioral effects of phenytoin prophylaxis of posttraumatic seizures. JAMA 1991;265:12711277.
3. Temkin NR, Dikmen SS, Wilensky AJ, et al. A randomized, double-blind study of phenytoin for the prevention of post-traumatic seizures. N Engl J
Med. 1990;323:497502.
4. Temkin NR, Dikmen SS, Anderson GD, et al. Valproate therapy for prevention of posttraumatic seizures: a randomized trial. J
Neurosurg.1999;91:593600.
5. Yablon SA. Posttraumatic seizures. Arch Phys Med Rehabil.1993;74:9831001.

RATIONALE (4) Answer: A

Central cord syndrome is characterized by weakness that is worse distally than proximally and worse in the upper extremities than in the lower
extremities. Fracture/dislocation at C5-C6 would more likely result in a deficit below that level, so option B is unlikely. Alcoholic polyneuropathy is
primarily sensory, so option C is incorrect. Spinal epidural hematoma would also likely result in a deficit below the level of compression; thus, option
D is incorrect. Chronic cervical stenosis secondary to degenerative joint disease can result in central spinal cord injury in the presence of an acute
flexion or hyperextension injury.

REFERENCES (4)

1. Epstein N, Epstein JA, Benjamin V, et al. Traumatic myelopathy in patients with cervical spinal stenosis without fracture or dislocation. Methods of
diagnosis, management, and prognosis. Spine. 1980;5:489496.
2. Schneider RC, Cherry G, Pante H. The syndrome of acute central cervical spinal cord injury. J Neurosurg. 1954;11:546577.

RATIONALE (5) Answer: B

The patient has significant left hemi-body trauma after being struck on that side. External signs of trauma on the left neck suggest potential for carotid
artery injury. The abnormal pupil is on the left, since it is associated with ptosis; therefore, right third nerve injury (option A) is not the diagnosis.
Carotid dissections are commonly associated with Horner syndrome on the affected side (miosis, ptosis, and anhidrosis). While Brown-Sequard
syndrome results in a hemiparesis, the side contralateral to the motor deficit would have a sensory deficit, so it is less likely to be associated with
localization to pain on the right. Therefore, option C is incorrect. Simultaneous brachial and lumbosacral plexus injuries would be unusual, and while
brachial plexus injuries may be associated with a Horner syndrome, this constellation would be very unlikely. Thus, option D is incorrect. Right-sided
brain tumor would not be expected to cause an acute deficit in the face of trauma and would not be associated with a left ptosis, so option E is incorrect.

REFERENCES (5)

1. Miller PR, Fabian TC, Bee TK, et al. Blunt cerebrovascular injuries: diagnosis and treatment. The Journal of Trauma. 2001 Aug;51:279285.
2. Miller PR, Fabian TC, Croce MA, et al. Prospective screening for blunt cerebrovascular injuries: analysis of diagnostic modalities and outcomes.
Annals of Surgery. 2002 Sep;236:386393; Discussion 393395.
3. Watridge CB, Muhlbauer MS, Lowery RD. Traumatic carotid artery dissection: diagnosis and treatment. J Neurosurg. 1989 Dec;71:8547.

RATIONALE (6) Answer: D

The patient has nosocomial meningitis related to the endoventricular drain. The rate of infection associated with external catheters is approximately 8%.
The risk of infection is reported to be increased with an increased duration of drainage, but the extent of increase per unit of time is uncertain. Although
one study showed a sharp increase in the risk of infection after 5 days of external drainage, a prospective, randomized trial showed that removing
external catheters within 5 days is unnecessary and that catheters can be left in place for longer periods. Since infection may be acquired by the
introduction of bacteria after the insertion of a new catheter, changing uninfected catheters might actually increase the risk of infection. Other risk factors
for infection are the routine sampling of cerebrospinal fluid, leakage of cerebrospinal fluid at the site, blockage of the drain, and intraventricular
hemorrhage. Therapy for patients in whom meningitis develops after neurosurgery or for patients who are hospitalized for a prolonged period after
penetrating head trauma or basilar skull fracture should cover staphylococci and facultative or aerobic gram-negative bacilli. The regimen should
consist of vancomycin in combination with cefepime, ceft azidime, or meropenem; the choice of the second agent should be based on the antimicrobial
susceptibility profiles of the local gram-negative bacilli. Meropenem is the agent of choice if one of the carbapenems is used. Ertapenem has no activity
against Pseudomonas aeruginosa. Imipenem can lower seizure threshold.

REFERENCES (6)

1. van de Beek D, Drake JM, Tunkel AR. Nosocomial bacterial meningitis. N Engl J Med. 2010 Jan 14;362:14654.
2. Lozier AP, Sciacca RR, Romagnoli MF, et al. Ventriculostomy-related infections: a critical review of the literature. Neurosurgery 2008;62:688-700.
3. Tunkel AR, Hartman BJ, Kaplan SL, et al. Practice guidelines for the management of bacterial meningitis. Clin Infect Dis 2004;39:1267-1284.

RATIONALE (7) Answer: C

Posterior reversible encephalopathy syndrome (PRES) is a clinicoradiological entity known to complicate immunosuppressive therapy, specifically
related to calcineurin inhibitors. Electrolyte disturbances and hypertension have been implicated in facilitating the development of this neurological
complication. Recognition of this potential side effect is critical to intensive care physicians as the therapy is often limited to discontinuation of the
offending agent and replacement with an alternative option.

Although immunosupressed, this patient had no other clinical signs concerning for an infectious process. Although ischemic events may present with
seizures, thrombolytic therapy wouldn't be considered in this patient with labile blood pressure control and no focal abnormalities after his seizure.

REFERENCES (7)

1. Hinchey J, Chaves C, Appignani B, et al. A reversible posterior leukoencephalopathy syndrome. N Eng J Med. 1996;334:494500.
2. Schwartz RB, Bravo SM, Klufas RA, et al. Cyclosporine neurotoxicity and its relationship to hypertensive encephalopathy: CT and MR findings in
16 cases. Am J Roentgenol. 1995;165:627.
3. Tam CS, Galanos J, Seymour JF, et al. Reversible posterior leukoencephalopathy syndrome complicating cytotoxic chemotherapy for hematologic
malignancies. Am J Hematol. 2004;77:72.

RATIONALE (8) Answer: B

Early postoperative (<1 week) complications of liver transplantation may be related to technical or anatomical causes (vascular and hepatobiliary tree),
immune suppressive therapy, graft rejection, graft nonfunction or extrahepatic organ dysfunction.

Vascular complications include hepatic artery thrombosis (HAT), portal vein thrombosis, and hepatic congestion secondary to venous outflow
obstruction. HAT remains the most common vascular complication and is manifested by elevated transaminase levels, rising bilirubin levels, sepsis
with liver abscesses, biliary anastomosis disruption, and acute hepatic failure. Doppler ultrasonography or arteriography may be required to diagnose
HAT and differentiate it from other less common vascular problems.

Primary bile duct leaks may present as excessive bilious drainage from abdominal drains and disproportionate rise in bilirubin level compared to other
liver function tests.

Acute cellular rejection may occur in the first 2 weeks and can be associated with lack of induction immunosuppressive therapy. Clinical features range
from mild nonspecific symptoms and subtle liver function abnormalities to acute liver failure. Liver biopsy is necessary to establish diagnosis,
especially if other major complications are ruled out.

Primary graft dysfunction or nonfunction (PGNF) presents in the immediate postoperative period (usually up to 48 hours) with lack of improving liver
function characterized by depressed mental status, coagulopathy, acidosis, hypoglycemia, and hypothermia, in addition to worsening liver function test
results. In this case, the stable early postsurgical course and normal mental status argue against PGNF, which is treated by supportive care and may
require retransplantation.

Infection must be ruled out as signs of sepsis are indistinguishable from the above mentioned complications.

REFERENCES (8)

1. Koffron A, Stein JA. Liver transplantation: indications, pre-transplant evaluation, surgery, and post transplant complications. Med Clin North Am
2008;92:86188.
2. Duffy JP, Hong JC, Farmer DG, et al. Vascular complications of orthotopic liver transplantation: experience in more than 4,200 patients. J Am Coll
Surg 2009;208:896903.
3. Mehrabi A, Fonouni H, Mller SA, et al. Current concepts in transplant surgery: liver transplantation today. Langenbecks Arch Surg 2008;393:245
60.

RATIONALE (9) Answer: A

Acute cellular rejection in liver transplantation is treated with high-dose corticosteroids with a good response rate. In 10% of cases or less, rejection
does not respond to steroids, necessitating the use of agents targeting T lymphocyte function, proliferation, or activation. These rescue medications
include muromonab-CD3, thymoglobulin (antithymocye globulin [ATG]), mycophenolate mofetil, anti-interleukin-2 receptor antibodies (eg,
basiliximab), calcineurin inhibitors (eg, tacrolimus), and agents that inhibit the mammalian target of rapamycin (mTOR) (eg, sirolimus).

ATG is an infusion of antiT cell antibodies derived from horse or rabbit sera. ATG is used in the prevention and treatment of cellular rejection in renal
and hepatic transplantation as well as for aplastic anemia. It has been used in steroid-resistant cases of orthotopic liver transplantation among other
agents. Agents such as muromonab and ATG can induce a syndrome of cytokine release upon administration. Cytokine spill from T cells (tumor necrosis
factor, interleukin-2, and interferon gamma) causes the cytokine release syndrome characterized by fever, chest pain, gastrointestinal symptoms,
wheezing, and flu-like features. Complement activation and neutrophil sequestration can cause acute lung injury and pulmonary edema. Hypotension may
occur. Usually seen after the first dose, the syndrome can be treated and prevented by steroids and H1-receptor blockers. The severity of symptoms
decreases with subsequent doses.

The timing of medication use in this case does not warrant consideration of severe sepsis in this patient. Calcinuerin inhibitors may cause nephrotoxicity
and neurotoxicity in the acute postinduction phase. Hypertension, renal insufficiency, and seizures are notable features.

REFERENCES (9)

1. Aydogan C, Sevmis S, Aktas S et al. Steroid-resistant acute rejections after liver transplant. Exp Clin Transplant 2010;8:1727.
2. Abramowicz D, Schandene L, Goldman M, et al. Release of tumor necrosis factor, interleukin-2, and gamma-interferon in serum after injection of
OKT3 monoclonal antibody in kidney transplant recipients. Transplantation. 1989;47:6068.

RATIONALE (10) Answer: B

Arrhythmias are common in the early postoperative phase following cardiac transplantation. Ventricular premature beats are present in almost all
patients early on with decreasing incidence later in the postoperative course. Sustained ventricular tachycardia or ventricular fibrillation can be
associated with allograft rejection. Similarly persistent or paroxysmal atrial tachyarrhythmias (atrial flutter/fibrillation) may be harbingers of allograft
rejection in the early postsurgical period. Atrial tachyarrhythmias may also be associated with higher long-term risk of mortality in these patients.

In the denervated heart, loss of negative chronotropic vagal input leads to a normal sinus resting rate greater than 80/min.

The most common conduction disturbance in the transplanted heart is a new partial or complete right bundle-branch block seen in almost 70% of
patients. Only persistent conduction disturbances may be associated with rejection.

REFERENCES (10)

1. Cui G, Tung T, Kobashigawa J, et al. Increased incidence of atrial flutter associated with the rejection of heart transplantation. Am J Cardiol
2001;88:2804.
2. Dasari TW, Pavlovic-Surjancev B, Patel N, et al. Incidence, risk factors, and clinical outcomes of atrial fibrillation and atrial flutter after heart
transplantation. Am J Cardiol 2010;106:73741.
3. Leonelli FM, Pacifico A, Young JB. Frequency and significance of conduction defects early after orthotopic heart transplantation. Am J Cardiol
1994;73:1759.

RATIONALE (11) Answer: B

Standard resuscitative strategies in the setting of trauma have historically revolved around initial crystalloid resuscitation, the institution of packed red
blood cells (PRBCs) with severe bleeding, and the correction of coagulopathy based upon empiric clotting indices. Recent studies derived from the
military experience in Iraq and Afghanistan have demonstrated an overt coagulation defect in severely injured patients upon presentation, prior to the
dilutional effect previously used to explain such pathology. Given the direct interface coagulopathy has on ongoing hemorrhage, and its contribution to
the further propagation of hypothermia and acidosis (all leading to the "bloody vicious cycle"), the direct control of coagulopathy beginning at the time
of trauma has been studied as a means to better ameliorate the effects of severe hemorrhagic shock. This methodology, along with goal-directed
treatment of hypothermia and acidosis, has been termed damage control resuscitation (DCR). Such a protocol is typically performed in tandem with
damage control laparotomy, where severe hemorrhage and contamination is quickly controlled in the operating room, with definitive organ or vascular
repair reserved till the patient has been systemically normalized.

Although the protocol has not been universally implemented as yet, multiple studies have shown a marked improvement in survival with the early
transfusion of fresh frozen plasma (FFP) in an approximate 1:1 FFP to PRBC ratio (in the setting of severe hemorrhagic shock, >10 units of PRBCs
transfused). Operative blood loss has been shown to be significantly decreased using DCR versus conventional resuscitation schemes, with additional
benefits including less hypothermia and acidosis immediately after surgery. Late effects including cardiac dysfunction, ventilator dependence, acute
respiratory distress syndrome, and secondary abdominal compartment syndrome have all been tempered, presumably because of the decreased
requirement for mass crystalloid transfusion with early hemorrhage control. Additional work has also shown a role for the early administration of
platelets, transfused in a 1:1 or 1:2 ratio relative to PRBCs. A number of civilian trauma centers have also demonstrated improved outcomes
implementing a DCR model, and have developed mass transfusion protocols to immediately mobilize the appropriate products when a severely injured
patient arrives. Factor VIIa has been extensively studied as an ancillary approach to treating severe coagulopathy, but should not be used in isolation
without standard or DCR strategies.

REFERENCES (11)

1. Beekly AC. Damage control resuscitation: a sensible approach to the exsanguinating surgical patient. Crit Care Med. 2008;36:S26774.
2. Duchesne JC, Kimonis K, Marr AB, et al. Damage control resuscitation in combination with damage control laparatomy: a survival advantage. J
Trauma, Injury, Infection, and Critical Care. 2010;69:4652.
3. Gunter OL, Au BK, Isbell JM, et al. Optimizing outcomes in damage control resuscitation: identifying blood product ratios associated with improved
survival. J Trauma, Injury, Infection, and Critical Care. 2008;65:52734.
4. Holcomb JB, Jenkins D, Rhee P, et al. Damage control resuscitation: directly addressing the early coagulopathy of trauma. J Trauma, Injury,
Infection, and Critical Care. 2007;62:30710.

RATIONALE (12) Answer: B

Although fluconazole remains one of the most commonly used antifungal agents for the treatment of Candida infections, there is an increasing emergence
of non-albicans, fluconazole-resistant Candida species as causes of serious fungal infections. Candidemia is associated with mortality of 30%40%,
increased length of hospital stay, and increased days of mechanical ventilation. In this patient, the organism is a non-albicans Candida, which has a high
incidence of fluconazole resistance.

Another drug in the azole family is voriconazole. Cross-resistance between fluconazole and voriconazole is clearly more pronounced in some species of
Candida than others, although all are affected to some degree. Innate (C krusei) or emerging (especially C glabrata and C guilliermondii) resistance to
azoles among non-albicans Candida species is an increasing problem as non-albicans species in some series represent close to half of all Candida
infections. ARTEMIS, a global antifungal surveillance study, found, for example, that voriconazole was active against some fluconazole-resistant
isolates of Candida (C krusei, C inconspicua, C norvegensis, and C intermedia), but activity was quite poor against the remaining 20 species of
fluconazole-resistant Candida (C glabrata, C tropicalis, and others).

For decades, amphotericin B deoxycholate has been the standard therapy for invasive fungal infections. Unfortunately, it is often poorly tolerated and
associated with acute infusion-related reactions and nephrotoxicity, which makes it less than an ideal choice in this patient.

Polymyxin B has gram-negative, but not antifungal activity.

Echinocandins are a newer class of parenteral antifungal agents that inhibit the synthesis of beta-(1,3)-D-glucan in the fungal cell wall. These
compounds are fungicidal in vitro against C albicans and non-albicans Candida species. No cross-resistance with azoles has yet been reported.

REFERENCES (12)

1. Pfaller MA, Diekema DJ, Gibbs DL, et al. Results from the ARTEMIS DISK Global Antifungal Surveillance study, 1997 to 2005: an 8.5-year
analysis of susceptibilities of Candida species and other yeast species to fluconazole and voriconazole determined by CLSI standardized disk
diffusion testing. J Clin Microbiol. 2007;45:173545.
2. Horn DL, Neofytos D, Anaissie EJ, et al. Epidemiology and outcomes of candidemia in 2019 patients: data from the prospective antifungal therapy
alliance registry. Clin Infect Dis. 2009;48:1695703.
3. Man M, Marchetti O, Calandra T. Bench-to-bedside review: Candida infections in the intensive care unit. Crit Care. 2008;12:204.

RATIONALE (13) Answer: A


The patient continues to be a transient responder, suggesting that there is ongoing blood loss. There is initial extravasation seen in the retroperitoneum on
initial CT.

Literature suggests that arterial extravasation seen on initial CT of pelvis predicts a need for arterial embolization to achieve hemodynamic stability.
Pelvic hemorrhage appears best managed by initial stabilization of the pelvic bones with realignment of the fracture, followed by pelvic angiography
and possible embolization if continued hypotension occurs. Indications for laparotomy are the same as for trauma patients without pelvic fractures,
namely signs of peritoneal irritation consistent with intestinal perforation or ongoing intra-abdominal hemorrhage. This patient has a relatively benign
abdomen for intestinal perforation and with recent CT, which showed only pelvic fracture, an intra-abdominal source of bleeding is not likely.

Although sinus tachycardia is the most common dysrhymia seen with blunt cardiac injury, in this patient the ongoing need for blood suggests ongoing
blood loss, not blunt cardiac injury as the cause of instability.

REFERENCES (13)

1. Stephen DJG, Kreder HJ, Day HC, et al. Early detection of arterial bleeding in acute pelvic trauma. J Trauma 1999;47:638-642.
2. Agolini SF, Shah K, Jaffe J, et al. Arterial embolization is a rapid and effective technique for controlling pelvic fracture hemorrhage. J Trauma
1997;43:395-399.
3. The EAST Practice Management Guidelines Work Group. Clinical practice guideline:practice management guidelines for hemorrhage in pelvic
fracture. Available at www.east.org/tpg.html. Accessed June 30, 2011.

RATIONALE (14) Answer: B

This patient's radiograph is consistent with traumatic rupture of the diaphragm secondary to blunt abdominal trauma. Although in this case the findings
are obvious, radiographic findings are only diagnostic in 16%30% of patients with diaphragmatic injury. Radiographic findings include a very high
diaphragmatic shadow, the presence of bowel in the chest, and nasogastric tube passing high into the left hemithorax. With blunt injury, the
diaphragmatic hernia results from a burst injury of the diaphragm secondary to a rapid increase in abdominal pressure. The cause may be motor vehicle
crash, falls, kicks, and crush injuries. Currently, motor vehicle crash from lateral impact results in this injury more frequently than frontal collisions.
With blunt injuries, most diaphragmatic hernias occur on the left side (85%), followed by the right and bilateral injuries. Stomach, colon, omentum,
spleen, and small bowel can cause herniation into the chest. Cardiopulmonary compromise may occur from the effect of the abdominal contents on the
chest cavity. Both venous return and vital capacity are reduced. Immediate surgical repair is recommended. CT is the best noninvasive diagnostic
modality, but it cannot rule out subtle injury. Additional contrast studies can be obtained to help make the diagnosis. In this case, the diagnosis is
suspected, so no further delay in surgery should occur. Failure to recognize a traumatic diaphragmatic hernia predisposes patients to the complication of
bowel obstruction or strangulation in the future. In the ventilated patient, a diaphragmatic hernia may not be recognized until the patient is extubated.

REFERENCES (14)

1. Asensio JA, Petrone P, Demetriades D. Injury to the diaphragm. In: Moore EE, Feliciano DV, Mattox KL, eds. Trauma. Fifth ed. New York:
McGraw-Hill; 2004: 613635.
2. Ilgenfritz FM, Stewart DE. Blunt trauma of the diaphragm: a 15 county private hospital experience. Am Surg 1992;58:334339.
3. Kearney PA, Rouhana SW, Burney RE. Blunt rupture of the diaphragm: mechanism, diagnosis, and treatment. Ann Emerg Med 1989;18:13261330.
4. Shapiro MJ, Heiberg E, Durham RM, et al. The unreliability of CT scans and initial chest radiographs in evaluating blunt trauma induced
diaphragmatic rupture. Clin Radiol 1996;51:2730.
5. Wiencek RG Jr., Wilson RF, Steiger Z. Acute injuries of the diaphragm: an analysis of 165 cases. J Thorac Cardiovasc Surg 1986;92:989993.
RATIONALE (15) Answer: D

This patient has bilateral pulmonary emboli. Patients with traumatic injuries are at risk for deep venous thrombosis and pulmonary embolus. Pulmonary
embolus increases mortality of the trauma patient nearly tenfold. When there is no contraindication to anticoagulation, the trauma patient should be fully
anticoagulated as other patients; however, Brathwaite et al1 studied low-risk trauma patients who were treated for pulmonary embolism with
anticoagulation. There was a 36% bleeding complication rate in this group of patients many of which were life-threatening. At particular risk were those
elderly trauma patients who received anticoagulation. In this study the authors also noted that there were 34 other patients who received inferior vena
cava filters with no related complications or deaths. These authors concluded that anticoagulation for deep venous thrombois/pulmonary embolism
should be used selectively in trauma patients and avoided in elderly patients, who should undergo early vena cava filter placement instead. Those
trauma patients who have injuries that would be exacerbated by bleeding (eg, incomplete spinal cord injuries, head injuries, ocular trauma,
nonoperatively managed solid organ injuries) should be considered for mechanical interruption of the vena cava. This patient's traumatic subdural
hematoma is a contraindication to anticoagulation so vena cava filter should be placed.

REFERENCES (15)

1. Brathwaite CE, Mure AJ, O'Malley KF, et al. Complications of anticoagulation for pulmonary embolism in low risk trauma patients. Chest
1993;104:71820.
2. Rogers FB, Cipolle MD, Velmahos G, et al. Practice management guidelines for the prevention of venous thromboembolism in trauma patients: the
EAST Practice Management Guidelines Work Group. J Trauma-Injury Infection Crit Care. 2002;53:142164.
3. Tuttle-Newhall JE, Rutledge R, Hultman CS, et al. Statewide, population-based, time-series analysis of the frequency and outcome of pulmonary
embolus in 318,554 trauma patients. J Trauma 1997;42:90.

RATIONALE (16) Answer: C

Upper gastrointestinal bleeding after severe hepatic trauma should be considered to be related to hemobilia (arterial-biliary fistula) until proven
otherwise. The classic triad of hemobilia includes colicky right upper quadrant pain, gastrointestinal hemorrhage (hematemesis or melena), and
jaundice; all three elements, however, fully present in only 22% of involved patients. Selective celiac angiography will typically demonstrate a
segmental hepatic artery pseudoaneurysm, and establish the appropriate access for embolization. Surgery should be reserved for extensive lesions not
amenable to angiographic ablation, and typically requires formal hepatic resection with significant associated morbidity.

Iatrogenic trauma has surpassed other forms of hepatic injury as the leading cause of hemobilia in recent years. Percutaneous liver biopsy and
percutaneous biliary drainage procedures are the leading culprits, although the associated bleeding is typically relatively minor and can be managed
conservatively. Calculous gallbladder disease rarely presents with hemorrhagic cholecystitis, but urgent cholecystectomy is required in such an instance.
Primary hepatic vascular malformations or neoplastic lesions have also been reported to result in severe hemobilia. Effective biliary drainage via
endoscopic retrograde cholangiopancreatography or percutaneous transhepatic cholangiography may be required in certain circumstances to allow
adequate decompression of clot-obstructed ducts.

REFERENCES (16)

1. D'Angelica M, Fong Y. The Liver. In: Townsend CM, Beauchamp RD, Evers BM, et al, eds. Sabiston Textbook of Surgery: The Biological Basis of
Modern Surgical Practice.18th edition. Philadelphia: Saunders/Elsevier; 2008: 151517.
2. Green MH, Duell RM, Johnson CD, et al. Haemobilia. British Journal of Surgery. 2001;88: 77386.
3. Pachter HL, Liang HG, Hofstetter SR. Liver and biliary tract trauma. In: Mattox KL, Feliciano DV, Moore EE, eds. Trauma. 2000:6656.

RATIONALE (17) Answer: B

Blunt cerebrovascular injury (BCVI) involving either the carotid or vertebral artery distribution is diagnosed in about 0.1% of all trauma patients treated
in the United States. Unfortunately most injures are diagnosed after the development of neurologic symptoms, with significant associated long-term
functional impairment. Several models have been developed to effectively screen those patients at greatest risk of BCVI. Typically included in such
schemes are patients with C1-3 fractures, or ligamentous injuries at any cervical level; transverse foramen fractures (specifically veretebral artery
injuries); severe midface or mandibular fractures; and basilar skull fractures involving the carotid canal. Four-vessel cerebral angiography (FVCA)
remains the gold standard for diagnosis, although recent studies have shown 16-slice CT angiography to have nearly equivalent sensitivity and
specificity. Magnetic resonance angiography and duplex ultrasonography have been studied as alternative screening modalities; however, published data
reveal significantly lower diagnosis rates than with FVCA or CT angiography.

Specific treatment is predicated on the severity of injury. A simple grading classification described by Biffl1 is shown below. For injuries of grade II
and higher, treatment with antithrombotics should be undertaken if deemed safe in light of the associated systemic and intracerebral bleeding risk.
Systemic anticoagulation remains the gold standard, but recent studies suggest that antiplatelet agents (aspirin or clopidogrel) may be equally efficacious
in stroke prevention. Mandatory short-term follow-up of BCVI 7 to 10 days after injury should be undertaken to verify lack of progression. High-grade
lesions, especially with pseudoaneurysm formation, should be repaired by either a surgical (if accessible) or endovascular approach; there is no
discrete data at present comparing the outcomes of specific interventions.

Blunt Carotid and Vertebral Arterial Injury Grading Scale


I. Luminal irregularity or dissection with <25% luminal narrowing
II. Dissection or intramural hematoma with >25% luminal narrowing, intraluminal thrombus, or raised intimal flap
III. Pseudoaneurysm
IV. Occlusion
V. Transection with free extravasation

REFERENCES (17)

1. Biffl WL, Cothren CC, Moore EE, et al. Western Trauma Association critical decisions in trauma: screening for and treatment of blunt
cerebrovascular injuries. J Trauma Injury, Infection, and Critical Care. 2009;67:115053.
2. Bromberg WJ, Collier BC, Diebel LN, et al. Blunt cerebrovascular injury practice management guidelines: the Eastern Association for the Surgery
of Trauma. J Trauma Injury, Infection, and Critical Care. 2010;68:47177.
3. Cothren CC, Biffl WL, Moore EE, et al. Treatment for blunt cerebrovascular injuries: equivalence of anticoagulation and antiplatelet agents.
Archives of Surgery. 2009;144:68590.
4. Ringer AJ, Matern E, Parikh S, Levine NB. Screening for blunt cerebrovascular injury: selection criteria for use of angiography. J Neurosurgery
2010;112:11469.

RATIONALE (18) Answer: D

This patient has wound dehiscence, which typically occurs on postoperative day 4 or 5. Wound dehiscence is characterized by profuse drainage of
salmon-colored fluid that soaks the dressing and bedding. It is a technical problem with the wound closure technique allowing the fascia to open. It can
lead to evisceration of abdominal contents. The dressing should be removed to examine the wound. If bowel is exposed, wet saline packs are placed,
and the patient returns to the operating room for wound closure. There is a role for nonoperative management with moist dressings and an abdominal
binder. Risk factors for wound dehiscence include age older than 45 years, malnutrition, morbid obesity, cancer, uremia, diabetes, coughing, and other
causes of increased intra-abdominal pressure, infection, and hemorrhage. Unless an intra-abdominal abscess is suspected, CT of the abdomen is not
indicated. Long-term outcome issues associated with wound dehiscence include incisional hernias.

REFERENCES (18)

1. Page CP, Bohnen JM, Fletcher JR, et al. Antimicrobial prophylaxis for surgical wounds: guidelines for clinical care. Arch Surg 1993;128:7988.
2. Richards PC, Balch CM, Aldrete JS. Abdominal wound closure: a randomized prospective study of 571 patients comparing continuous vs
interrupted suture techniques. Ann Surg 1983;197:238243.
3. Riou JP, Cohen JR, Johnson H Jr. Factors influencing wound dehiscence. Am J Surg 1992;163:324330.

RATIONALE (19) Answer: C

This patient has developed hypotension, high airway pressures with hypoxemia, and oliguria. These findings are consistent with abdominal compartment
syndrome (ACS). Commonly described in patients after laparotomy for blunt and penetrating abdominal trauma, ACS was initially described after repair
of ruptured abdominal aortic aneurysm. Other conditions associated with ACS are retroperitoneal hemorrhage, burns, peritonitis, liver transplantation,
pancreatitis, and ascites. Persistent increase in intra-abdominal pressure will produce multiple organ dysfunction. Hypotension results from a
combination of decreased venous return and increased intrathoracic pressure (decreases left ventricular compliance). Respiratory failure with increased
peak ventilatory pressures, hypoxemia, and eventual hypercapnia is seen with increased intra-abdominal pressures. As intra-abdominal pressure
increases, a progression from oliguria to anuria is commonly seen.

Documentation of an increased intra-abdominal pressure is key in establishing the correct diagnosis and directing therapy. Currently, urinary bladder
pressure measurement is the most simple, reliable, and accepted methodology. A Foley catheter attached to a water manometer can be used. Intra-
abdominal hypertension is seen with intra-abdominal pressures greater than or equal to 12 mm Hg. ACS warranting decompressive laparotomy is
defined by intra-abdominal pressure greater than or equal to 20 mm Hg in association with new-onset single or multiple organ failure. However, the
decision to decompress the abdomen may come at pressures less than 20 mm Hg if intra-abdominal hypertension is seen with organ failure related to this
problem.

Decisions for surgical decompression should take into account the clinical findings and the actual pressure. A transesophageal echocardiography would
be of no value in this case, because there is no evidence of cardiac or thoracic aorta pathology causing clinical findings. CT or abdominal
ultrasonography would not reveal further information. A new collection of blood, detectable by these imaging tests, is unlikely considering hemoglobin
has remained stable. Pulmonary artery catheter pressures will not aid in making a correct diagnosis.
REFERENCES (19)

1. Malbrain ML, Cheatham ML, Kirkpatrick A, et al. Results from the International Conference of experts on intra-abdominal hypertension and
abdominal compartment syndrome. I. Definitions. Intensive Care Med 2006;32:17221732.
2. Schein M, Wittmann DH, Aprahamian CC, et al. The abdominal compartment syndrome: the physiological and clinical consequences of elevated
intraabdominal pressure. J Am Coll Surg 1995; 180:745753.
3. World Society of the Abdominal Compartment Syndrome. Preliminary consensus definitions on intraabdominal hypertension (IAH) and abdominal
compartment syndrome (ACS): results from the International ACS Consensus Definitions Conference. Available at: www.wsacs.org. Accessed June
12, 2007.

RATIONALE (20) Answer: D

In this patient with significant organ dysfunction from abdominal compartment syndrome, immediate relief of increased abdominal pressure is
mandatory. Of the options provided, a bedside laparotomy is the only treatment that will achieve this goal. The patient is critically ill and even delaying
abdominal decompression to transport the patient to an operating room might be deleterious. Abdominal decompression will dramatically improve peak
ventilatory pressures, hypoxemia, hypotension, and oliguria in this patient. There is no indication of acute hemorrhage requiring transfusion of blood
products. Although fluid resuscitation can be used to treat increased intra-abdominal pressure in early abdominal compartment syndrome, it will not be
effective in patients with severe organ dysfunction. Emergent dialysis is not indicated for this patient since the oliguria is secondary to increased intra-
abdominal pressure. A pericardiocentesis is not indicated since there is no evidence of cardiac tamponade. Finally, changing the ventilator mode to
pressure control might decrease peak airway pressure but will not result in prolonged improvement in oxygenation.

REFERENCES (20)

1. Balogh Z, McKinley BA, Holcomb JB, et al. Both primary and secondary abdominal compartment syndrome can be predicted early and are
harbingers of multiple organ failure. J Trauma 2003; 54:848861.
2. Malbrain ML, Cheatham ML, Kirkpatrick A, et al. Results from the International Conference of experts on intra-abdominal hypertension and
abdominal compartment syndrome. I. Definitions. Intensive Care Med 2006;32:1722-1732.
3. McNelis J, Marini CP, McNelis J, Marini CP, Jurkiewicz A, et al. Predictive factors associated with the development of abdominal compartment
syndrome in the surgical intensive care unit. Arch Surg 2002;137:133136.
4. Meldrum DR, Moore FA, Moore EE, et al. Prospective characterization and selective management of the abdominal compartment syndrome. Am J
Surg 1997; 174:667673.
5. Morken J, West MA. Abdominal compartment syndrome in the intensive care unit. Curr Opin Crit Care 2001;7:268274.
6. Orlando R 3rd, Eddy VA, Jacobs LM Jr., et al. The abdominal compartment syndrome. Arch Surg 2004;139:415422.

RATIONALE (21) Answer: B

This postpartum patient has developed signs and symptoms consistent with an epidural hematoma. Common signs and symptoms include sharp, severe,
radicular back pain followed by neurologic symptoms of cord compression. The differential diagnosis of this disorder includes spinal or epidural
abscess, acute herniated disk, spinal cord ischemia, spinal neoplasm, spondylitis, lipoma, transverse myelitis, and dissecting aortic aneurysm. MRI
remains the gold standard for confirming the diagnosis. Common risk factors for an epidural hematoma following epidural anesthesia include age older
than 50 years, spinal stenosis, coagulopathy, platelet inhibition, arteriovenous malformation, or multiple passes with an epidural needle.

REFERENCES (21)

1. Guffey PJ, McKay WR, McKay RE. Case report: epidural hematoma nine days after removal of a labor epidural catheter. Anesthesia Analgesia.
2010;111:9925.
2. Fleager K, Lee A, Cheng I, et al. Massive spontaneous epidural hematoma in a high-level swimmer: a case report. Journal of Bone & Joint Surgery
American. 2010;92:28436.
3. Kreppel D, Antoniadis G, Seeling W. Spinal Hematoma: a literature survey with meta-analysis of 613 patients. Neurosurg Rev. 2003; 26:1-49.

RATIONALE (22) Answer: B

Of the five modalities listed, thoracic epidural with local anesthetic provides superior analgesia for her rib fractures. The choice of epidurally
administered local anesthetic or local anesthetic plus opioid should be tailored to the individual patient and left to patient decision. Although local
anesthetic opioids provide synergistic effects and improve analgesia, the addition of opioid could potentially lead to altered mental status in elderly
patients. Both intercostal and paravertebral blocks would require repeat placement for pain control greater than 48 hours.
REFERENCES (22)

1. Karmakar MK, Ho AM. Acute pain management of patients with multiple fractured ribs. Journal Trauma-Injury Infection Critical Care.
2003;54:61525.

RATIONALE (23) Answer: A

Peripartum cardiomyopathy is a dilated cardiomyopathy that develops in the last gestational month of pregnancy or in the first 5 months after delivery. It
occurs more frequently in older women, obese women, and multiparous women with twin pregnancies. The etiology of peripartum cardiomyopathy is
unknown, although many hypotheses have been suggested, such as viral myocarditis, immune-mediated injury, selenium deficiency, and the hemodynamic
stress of pregnancy.1 The presentation of peripartum cardiomyopathy is similar to other dilated cardiomyopathies and most patients present in New York
Heart Association class III or IV functional status. Patients who present early after delivery often have dramatic symptoms and signs of congestive heart
failure. The diagnosis is usually more difficult during the late stages of pregnancy because of overlap with symptoms of pregnancy. The treatment of
peripartum cardiomyopathy is similar to the treatment of acute and chronic heart failure due to other causes of left ventricular systolic dysfunction.
Patients who are congested but have adequate perfusion require initial treatment with IV diuretics alone or in combination with vasodilators such as
nitroglycerin, nitroprusside, or nesiritide.2 Patients with diminished perfusion require augmentation of their cardiac output with inotropic drugs such as
IV dobutamine or milrinone. Patients should then be started on an angiotensin-converting enzyme inhibitor if the diagnosis is made postpartum. More
than half of these patients normalize their left ventricular ejection fraction during the first 6 months of presentation. Indicators of poor prognosis in
peripartum cardiomyopathy include a decreased left ventricular ejection fraction 6 months postpartum, larger left ventricular and diastolic dimension,
clinical presentation more than 2 weeks postpartum, age older than 30 years, African American descent, and multiparity.3

REFERENCES (23)

1. Ansari AA, Fett JD, Carraway RE, et al. Autoimmune mechanisms as the basis for human peripartum cardiomyopathy. Clin Rev Allergy Immunol.
2002;23:301324.
2. Phillips SD, Warnes CA. Peripartum cardiomyopathy: current therapeutic perspectives. Curr Treat Options Cardiovasc Med. 2004; 6:481488.
3. Elkayam U, Tummala P, Rao K, et al. Maternal and fetal outcomes of subsequent pregnancies in women with peripartum cardiomyopathy. N Engl J
Med. 2001;344:15671571.

RATIONALE (24) Answer: A

Posterior reversible encephalopathy syndrome (PRES), or reversible posterior leukoencephalopathy syndrome, is a well-established neurological
disorder with characteristic MRI findings. It is associated with a multitude of diverse clinical entities, including preeclampsia and eclampsia, acute
glomerulonephritis, systemic lupus erythematosus, thrombotic thrombocytopenic purpura, hemolytic-uremic syndrome, and drug toxicity from various
immunosuppressive and chemotherapeutic agents. Clinically the syndrome is characterized by headache, confusion, seizures, and vision loss, usually in
the setting of these clinical entities. Even though PRES is considered reversible, prompt recognition and treatment is necessary to prevent cerebral
ischemia and bad outcome. Treatment includes removing precipitating factors and controlling blood pressure. Because of possible loss of
autoregulation, particularly in posterior circulation, lowering of mean arterial pressure should be done with extreme caution in order to avoid cerebral
ischemia.

REFERENCES (24)

1. Marik PE. Hypertensive disorders of pregnancy. Postgrad Med. 2009 Mar;121:6976.

RATIONALE (25) Answer: D

Respiratory failure in pregnancy is responsible for significant morbidity and mortality of both the mother and the fetus. Respiratory failure can result
from a number of pregnancy-related complications, such as pneumonia, preeclampsia, pulmonary embolism, peripartum cardiomyopathy, asthma,
aspiration, and cardiogenic and noncardiogenic pulmonary edema. Maternal physiologic adaptations to pregnancy reduce tolerance to all types of
respiratory failure. Increased oxygen consumption and decreased functional residual capacity place both the mother and the fetus at risk from
hypoventilation and apnea. The onset of pneumonia is not gestational-age dependent.

Pregnancy increases pneumonia complications such as the need for mechanical ventilation and intubation, bacteremias, and empyemas. Fetal
complications include higher rates of preterm labor, lower average birth weights, and higher neonatal mortality rates. The most common bacterial agents
causing pneumonia in pregnancy include Streptococcus pneumoniae and Haemophilus influenzae.1 Viral pneumonias such as varicella and influenza are
less common. Risk factors for developing pneumonias during pregnancy include preexisting diseases such as HIV infection, asthma, cystic fibrosis,
anemia, cocaine and alcohol abuse, and use of tocolytic agents and corticosteroids.
Guidelines developed by the British Thoracic Society for assessing the course of the pneumonia, the need for hospitalization, and mortality prediction
consider the presence of 2 of the following 4 criteria to indicate severe illness. These include RR greater than 30/min, diastolic BP less than 60 mm Hg,
blood urea nitrogen level greater than 19.1 mg/dL, and confusion.2 The combination of a macrolide and a beta-lactam antibiotic has proven effective for
treatment of bacterial pneumonias in pregnancy and has a favorable fetal safety profile.

Peripartum cardiomyopathy most often presents abruptly after delivery of the fetus with the development of pulmonary edema and evidence of acute left
heart failure that is treated with diuresis and inotropic support once the diagnosis is confirmed by echocardiography. Asthmatic symptoms may worsen,
improve, or remain unchanged during pregnancy but would be unlikely to occur de novo in late pregnancy. Caution should be exercised using
noninvasive ventilation in pregnancy because of the increased risk of aspiration, especially in advanced pregnancy, and in particular if the patient has a
recent history of emesis.3

REFERENCES (25)

1. Lim WS, Macfarlane JT, Colthorpe CL. Treatment of community-acquired lower respiratory tract infections during pregnancy. Am J Respir Med.
2003;2:22123.
2. Yost NP, Bloom SL, Richey SD, et al. An appraisal of treatment guidelines for antepartum community-acquired pneumonia. Am J Obstet Gynecol.
2000;183:13135.
3. Lapinsky SE. Cardiopulmonary complications of pregnancy. Crit Care Med. 2005;3.3:161622.
PART 10: Administrative and Ethical Issues in the Critically Ill

Instructions: For each question, select the most correct answer.

1. Which of the following components most helpful in framing quality improvement programs in the ICU?

A. Structure, process, and outcome


B. Problem, question, and hypothesis
C. Vision, objective, and mission
D. Exposure, confounder, and disease
E. Problem, analysis, and plan

2. A 55-year-old man is admitted to the ICU after sustaining a closed head traumatic injury. He has a depressed level of consciousness, with stable
vital signs. He opens his eyes to speech, localizes to pain, and moans but does not speak. There are no lateralizing signs.

What is his Glasgow Coma Scale score?


A. 15
B. 13
C. 10
D. 8
E. 6

3. A 56-year-old man is admitted to the hospital after a motor vehicle collision. His Glasgow Coma Scale score after resuscitation is 4 and he has
flexor posturing bilaterally. His pupils are asymmetric and the right is dilated to 5 mm. His temperature is 36C (96.7F), BP is 90/50 mm Hg, HR
is 110/min, RR is 18/min, and oxygen saturation is 98% (1.0). CT of the head shows bilateral frontal contusions and subarachnoid hemorrhage with
a large right subdural hematoma. The cervical spine was immobilized in the usual fashion. Focused assessment with sonography for trauma shows a
small collection at the right retrocolic recess. The neurosurgery team has evaluated the patient and is planning to take him to the operating room.

Which of the following prediction scores will best evaluate the patients operative risk?
A. American Society of Anesthesiologists Physical Status Classification System
B. World Federation of Neurological Surgeons scale
C. Injury Severity Score (ISS)
D. Sequential Organ Failure Assessment score (SOFA)
E. Multiple Organ Dysfunction Score (MODS)

4. Which of the following processes is best supported by ICU scoring systems?

A. Adjustments of medication dosing and delivery


B. Support for decision analysis and resource allocation
C. Improving testing accuracy by improving pre-test probabilities
D. Increasing diagnostic power
5. ICU scoring systems help with estimation and prediction of mortality based on statistical models and their ability to discriminate between groups.

The performance of these models and the ability to discriminate between survivors and nonsurvivors is best evaluated by which of the following
characteristics of the model?
A. Odds ratio and 95% confidence interval
B. Standardized mortality rate
C. Frequency estimate of the disease
D. Alpha of the test statistic
E. Receiver operating characteristic

6. A 32-year-old man with acute myeloid leukemia is in the ICU after being admitted to the ICU for severe sepsis. The resident discusses with the
patient the need for inserting a central line for fluid and medication administration. The patient is thankful for the residents explanation of the risk-
to-benefit ratio and alternatives. The patient agrees that he is critically ill but refuses the intervention, arguing that there is no need for it and that he
can get two peripheral IV lines for the same purpose. The resident documents his discussion and proceeds to insert 2 peripheral IV lines.

Which of the following ethical principles supports the residents response?


A. Autonomy
B. Independence
C. Utilitarianism
D. Beneficence
E. Virtue

7. An 85-year-old woman with Alzheimer dementia is admitted to the ICU for severe community-acquired pneumonia and acute hypoxemic respiratory
failure. The patient has no advance directive or additional family members, and the team realizes that she has been living with a son-in-law who did
not know her well. In the ICU she continues to deteriorate, developing acute respiratory distress syndrome and multiple organ dysfunction syndrome
requiring further renal replacement therapy. Given the physiologic deterioration refractory to aggressive medical management, the team approaches
the son-in-law and asks if he would like to continue with advanced life support in the setting of what the treating physicians consider futile care. He
says that, though he never talked about end-of-life issues with his mother-in-law, he thought that she was the type of person that would not have
liked to be kept alive artificially.

Which of the following approaches should the team take at this point?
A. Continue with current therapy as there is no advance directive and the patients family may file legal suit.
B. Get a court order to assign a representative to the patient as the son-in-law is unaware of the patients wishes.
C. Change goals of care from full medical management to comfort care and issue a do-not-resuscitate order after documenting discussion on the
chart.
D. Request an ethics consultation from the hospital.
E. Perform a tracheostomy, percutaneous endoscopic gastrostomy, and send the patient to a nursing home.

8. Which of the following principles was established by the Declaration of Helsinki?

A. The right to adequate medical services and health care systems


B. Ethical principles for medical research involving human subjects
C. The right to refuse to eat and not be fed artificially against ones will
D. The need for the development of guidelines for the ethical practice of medicine
E. The right to die with dignity
9. In the case of Karen Quinlan (In re Quinlan, 70 NJ 10 [1976]), the Supreme Court of New Jersey established that:

A. The principle of clear and convincing evidence may be used to withdraw life support
B. The physician has a duty to warn against potential harm
C. The principle of beneficence supercedes the patients wishes
D. Patients have a right to privacy and that therapeutic decisions can be based on a substituted judgment standard
E. Physicians cannot withdraw life support unless supported by a court order.

10. An ICU team was interested in knowing about patient satisfaction after the institution of a rapid response team (RRT). Every time the RRT was
activated, the investigator collected information from patients and/or family members using a sample questionnaire. The investigators asked on a
scale of 16 if the patient or family members were satisfied with the service during urgent situations and dichotomized this outcome into 13
(satisfied) and 46 (not satisfied). They further divided the cohort into patients younger and older than 50 and compared the satisfaction results.
After 6 months, the investigators collected information from 632 subjects and reported that 78% of them were satisfied with the service. Among
those older than 50, the RRT had a significant impact on patient/family satisfaction (odds ratio, 0.8; 95% confidence interval, 0.70.9).

This example illustrates an observational study with which of the following designs?
A. Prospective cohort
B. Retrospective cohort
C. Case-control
D. Case-cohort
E. Cross-sectional

11. During his 2-year ICU training, a fellow collected information on all the procedures performed and associated complications. Specifically, this
fellow inserted 45 central lines and 5 pulmonary artery catheters (PACs) during the first year, and 40 central lines and 10 PACs during the second
year. Of these insertions, 1 was associated with a pneumothorax and 4 with line-related sepsis. Among the 4 with line-related sepsis, the fellow
recorded that 2 patients had candidemia.

Which of the following best characterizes the incidence rate per year of complications for this fellow at the end of the fellowship and the
prevalence of candidemia within his patient group?
A. 1% per year and 5%
B. 5% per year and 1%
C. 2% per year and 2.5%
D. 2.5% per year and 2%
E. Cant be calculated

12. An experienced investigator is developing a new urine test for diagnosing seizures in critically ill patients. The innovative test detects specific
neuronal proteins in the urine that are spilled into the circulation in an epileptic convulsion. In the presence of urine of a patient with a seizure, the
test turns green; it remains unchanged if there has been no seizure. This could be a revolutionary test as physicians will not need to rely solely on
clinical examination and electroencephalogram, and it could be done at the bedside. The following Figure describes the results obtained with the
new test

Seizure NoSeizure Total


Positive Test Results 1,125 425 1,550
Negative Test Results 125 1,575 1,700
Total 1,250 2,000 3,250
What is the specificity of this test?
A. 425/1,550
B. 1,575/3,250
C. 1,575/2,000
D. 1,575/3,250

13. A 45-year-old woman with a history of factor V Leiden is admitted to the hospital for evaluation of pleuritic chest pain, a swollen leg, and
shortness of breath. On admission, BP was 120/85 mm Hg, HR was 122/min, RR was 24/min, and oxygen saturation was 90% on oxygen, 2 L via
nasal cannula. Arterial blood gas results showed pH of 7.47, PaO2 of 30 mm Hg, PaCO2 of 60 mm Hg, and 90% oxygen saturation. Chest radiograph
revealed normal lung fields and cardiomediastinal figure.

Sensitivity Specificity Positive predictive value Negative predictive value


Test A 70% 99% 90% 95%
Test B 80% 95% 90% 70%
Test C 95% 50% 80% 99%
Test D 75% 99% 99% 90%

Which of the following tests would be most appropriate to establish a diagnosis of pulmonary embolism in this patient?
A. Test A
B. Test B
C. Test C
D. Test D

14. A critical care fellow in the ICU was interested in whether the implementation of a daily checklist was associated with reduced ICU length of stay
(LOS). To test his hypothesis. he measured the LOS in days in 2 different groups of critical care patients. Group 1 had the checklist implemented
during working rounds and Group 2 had no checklist implemented for working rounds. Patients admitted to the ICU between 7 am and 7 pm were in
Group 1, and those admitted between 7 pm and 7 am were in Group 2. After 6 months, he collected information on 258 patients. The results showed
that the ICU LOS of Group 1 was 4.5 2.1 days and in Group 2 was 5.1 1.8.

Which of the following test statistics would this fellow use to determine a statistical difference between the mean LOSs of the 2 groups?
A. Chi-square test
B. Student t test for mean
C. Mann-Whitney test
D. Linear regression
E. Logistic regression

15. Which of the following will best determine the probability that a test will reject the null hypothesis when this hypothesis is false?

A. Alpha
B. Beta
C. 1 beta
D. 1 alpha
PART 10: Administrative and Ethical Issues in the Critically Ill

ANSWERS:

1A; 2D; 3A; 4B; 5E; 6A; 7C; 8B; 9D; 10E; 11D; 12C; 13D; 14B; 15C

RATIONALE (1) Answer: A

Classic quality-of-care components have been described by the Donabedian model. Structure represents the first component of the quality-of-care model
and can be defined as the way we organize care. Process generally refers to what we do, or fail to do, for patients and their families. Outcomes
represent the third component of the quality-of-care model and refer to the results we achieve. Critical care clinicians interested in quality improvement
should understand the structure-process-outcome model and select aspects they are both interested in and able to improve.

REFERENCES (1)

1. Curtis JR, Cook DJ, Wall RJ, et al. Intensive care unit quality improvement: a "how-to" guide for the interdisciplinary team. Crit Care Med.
2006;34:211218.
2. Donabedian A. Continuity and change in the quest for quality. Clin Perform Qual Health Care. 1993;1:916.
3. Donabedian A. Aspects of Medical Care Administration: Specifying Requirement for Health Care. Cambridge, MA: Harvard University Press;
1973.

RATIONALE (2) Answer: D

The Glasgow Coma Scale (GCS) score was developed to provide a consistent description of patients with head injury, as well as to provide a measure
of severity. Patients are scored on eye opening, best motor response, and verbal response. Points are given for each area and the sum of the points
provides the score. The best possible score is 15 and the worst score is 3.

The severity of head injury can be characterized as mild if it scores 1415 on the GCS, moderate if the score is 913, and severe if the score is 3-8.
Repeated examination and GCS scoring in response to treatment provides information for prognosis, morbidity, and mortality. A GCS score of 8 or less
describes a coma; at that point, early tracheotomy is considered. The GCS is not a substitute for a complete neurologic examination with cranial and
peripheral nerve evaluation for isolated or lateralizing signs. By adding the scores highlighted on the scale below, it is clear that the patient described in
this question has a score of 10, which indicates a moderate head injury.

Glasgow Coma Scale


Eye Opening Spontaneous 4
To speech 3
To pain 2
None 1
Best Motor Response Obeys commands 6
Localizes pain 5
Withdraws normal flexion 4
Decorticate abnormal flexion 3
Decerebrate extension 2
Flaccid no response 1
Verbal response Oriented 5
Confused conversation 4
Inappropriate words 3
Incomprehensible sounds 2
None 1

REFERENCES (2)

1. Gennarelli TA, Champion HR, Copes WS, et al. Comparison of mortality, morbidity, and severity of 59,713 head-injured patients with 114,447
patients with extracranial injuries. J Trauma. 1994; 37:962968.
2. American College of Surgeons. Committee on Trauma. Head trauma. In: Advanced Trauma Life Support Program for Doctors: ATLS. Sixth edition.
Chicago: American College of Surgeons; 1997: 181213.
3. Jennett H, Bond M. Assessment of outcome after severe brain damage: a practical scale. Lancet. 1975;1:480484.
RATIONALE (3) Answer: A

The American Society of Anesthesiologists (ASA) physical status classification system is used to predict perioperative mortality based on functional
status. In 1963, the ASA adopted the 5-category physical status classification system; a sixth category was later added. The categories are:

1. A normal healthy patient


2. A patient with mild systemic disease
3. A patient with severe systemic disease
4. A patient with severe systemic disease that is a constant threat to life
5. A moribund patient who is not expected to survive without the operation
6. A patient declared brain dead whose organs are being removed for donor purposes

The most commonly used organ dysfunction scoring systems are the Multiple Organ Dysfunction Score (MODS), the Sequential Organ Failure
Assessment (SOFA), and the Logistic Organ Dysfunction (LOD) score, but these scores are not used to predict perioperative mortality. Thus, options D
and E are incorrect.

The clinical grading system proposed by the World Federation of Neurological Surgeons (option B) is intended to be a simple, reliable, and clinically
valid way to grade a patient with subarachnoid hemorrhage (not traumatic brain injury) and does not offer any operative risk prediction utility.

The Injury Severity Score (ISS) is an anatomical scoring system that provides an overall score for patients with multiple injuries. Each injury is
assigned an abbreviated injury scale (AIS) score and is allocated to one of 6 body regions (head, face, chest, abdomen, extremities [including pelvis],
external). Only the highest AIS score in each body region is used. The three most severely injured body regions have their score squared and added
together to produce the ISS score. The ISS offers no operative risk prediction utility, so option C is incorrect.

REFERENCES (3)

1. Saklad M. Grading of patients for surgical procedures. Anesthesiology. 1941;2:2814.


2. Le Gall JR, Klar J, Lemeshow S, et al. The Logistic Organ Dysfunc-tion system. a new way to assess organ dysfunction in the intensive care unit.
ICU Scoring Group. JAMA. 1996; 276: 80210.
3. Lemeshow S, Teres D, Pastides H, et al: A method for predicting survival and mortality of ICU patients using objectively derived weights. Crit Care
Med. 1985;13:519525.
4. Lemeshow S, Klar J, Teres D, et al: Mortality probability models for patients in the intensive care unit for 48 or 72 hours: a prospective, multicenter
study. Crit Care Med. 1994; 22:13511358.
5. Vincent JL, Moreno R, Takala J, et al. The SOFA (Sepsis- related Organ Failure Assessment) score to describe organ dysfunction/failure. On behalf
of the Working Group on Sepsis-Related Problems of the European Society of Intensive Care Medicine. Intensive Care Med. 1996; 22: 70710.
6. Marshall JC, Cook DJ, Christou NV, et al Multiple organ dysfunction score: a reliable descriptor of a complex clinical outcome. Crit Care Med.
1995;23:163852.
7. Teasdale GM, Drake CG, Hunt W, et al. A universal subarachnoid hemorrhage scale: report of a committee of the World Federation of
Neurosurgical Societies. J Neurol Neurosurg Psychiatry. 1988 Nov;51:1457.
8. Baker SP, O'Neill B, Haddon W Jr, et al. The Injury Severity Score: a method for describing patients with multiple injuries and evaluating
emergency care. J Trauma. ;1974;14:187196.
9. Keegan MT, Gajic O, Afessa B. Severity of illness scoring systems in the intensive care unit. Crit Care Med. 2011;39:163169.

RATIONALE (4) Answer: B

Adult ICU prognostic models have been used for predicting patient outcome for 3 decades. Scoring systems are also used to provide information about
disease severity, aid in clinical management by supporting decisions for resource allocation, interpret data by adjusting for case-mixture and disease
severity, and may be employed for surveys or comparative audits. ICU scoring systems do not provide information to perform adjustments in medication
delivery or increase diagnostic power by helping with testing accuracy.

REFERENCES (4)

1. Keegan MT, Gajic O, Afessa B. Severity of illness scoring systems in the intensive care unit. Crit Care Med. 2011;39:163169.
2. Afessa B, Gajic O, Keegan MT. Severity of illness and organ failure assessment in adult intensive care units. Crit Care Clin. 2007;23:639658.
RATIONALE (5) Answer: E

The relationships between the predictor and outcome variables of the development model need independent validation. A mortality prognostic model
must differentiate between survivors and nonsurvivors and be well calibrated and reliable. It also has to be periodically updated to reflect the change in
medical practice and case mix over time. The performance of the ICU prognostic models is assessed for discrimination by the area under the receiver
operating characteristic (ROC) curve, and for calibration by the Hosmer-Lemeshow statistic for calibration. An ROC curve is a graphical plot of the
sensitivity or true-positive rate, versus false-positive rate (1specificity), for a binary classifier system as its discrimination threshold is varied. The
area under the ROC curve is the measure of how well a model differentiates between groups (areas under the ROC curve of 1, 0.900.99, 0.800.89,
0.700.79, 0.600.69, and 0.60 are considered to be perfect, excellent, very good, good, moderate, and poor, respectively). Calibration refers to the
correlation between the predicted and actual outcome for the entire range of risk. The calibration is considered good if the Hosmer-Lemeshow statistic P
value is less than .05.

REFERENCES (5)

1. Keegan MT, Gajic O, Afessa B. Severity of illness scoring systems in the intensive care unit. Crit Care Med. 2011;39:163169.
2. Afessa B, Gajic O, Keegan MT. Severity of illness and organ failure assessment in adult intensive care units. Crit Care Clin. 2007;23:639658.

RATIONALE (6) Answer: A

Autonomy refers to the principle of self-rule, self-determination, or self-governance. It supports the notion that rational individuals with decisional
capacity (or competency in legal terms) are uniquely qualified to decide what is best for themselves and should be allowed to do whatever they want.
This holds true even if doing so involves considerable risk or would be deemed foolish by others, providing their decision does not infringe on the
autonomy of another. In the US, the principle of autonomy is strongly guarded by the Constitution. The universal principles of autonomy, beneficence,
and justice were established by the Belmont Report in 1976. The principle of beneficence (option D) is inherent to the role of physicians and determines
the duty to prevent evil or harm by promoting good and welfare for others. The principle of justice demands that one act to promote the greatest benefit
to the greatest number of individuals while inflicting the least amount of harm. The principle of nonmaleficence was later introduced by Beauchamp and
Childress in their landmark textbook on bioethics.2 This principle establishes the duty to refrain from inflicting harm on others and is sometimes defined
by the maxim "primum non nocere" (first do no harm).

REFERENCES (6)

1. What ethical principles I need to know? In: Kummer HB and Thompson DR, eds. Critical Care Ethics. Mount Prospect, IL: Society of Critical Care
Medicine; 2009.
2. Beauchamp Tl, Childress JF. Principles of Biomedical Ethics. New York: Oxford University Press New York; 2001.

RATIONALE (7) Answer: C

The team should switch the goals of care from full medical management to "comfort care" and issue a do-not-resuscitate order after documenting
discussion on the chart based on the best-interest standard. In the case of In re Conroy, (486 A.2d 1209 NJ 1985), Claire Conroy, an elderly mentally
and physically incapacitated woman residing in a nursing home had a guardian (nephew) who did not know her explicit wishes but felt she would not
have liked to have a feeding tube. In this case, the New Jersey Supreme Court permitted the use of the best-interest standard, which is applicable in
those cases where the burden of a therapy outweighs the benefits and the pain of interventions which would make them inhumane. The standard is based
on the principle of nonmaleficence, or primum non nocere (first do no harm).

REFERENCES (7)

1. What are some important cases in bioethics? In: Kummer HB and Thompson DR, eds. Critical Care Ethics. Mount Prospect, IL: Society of Critical
Care Medicine; 2009.
2. Beauchamp Tl, Childress JF. Principles of Biomedical Ethics. New York: Oxford University Press; 2001.

RATIONALE (8) Answer: B

The Declaration of Helsinki was adopted during the 18th World Medical Association General Assembly in June 1964. It is a statement of ethical
principles for medical research involving human subjects, including research on identifiable human material and data. It follows the principles of the
Nuremberg Code developed after the Nuremberg Trials. Different countries and political systems consider the right to adequate medical services and
health care systems as a fundamental human right; however, in some countries, even developed ones, health care coverage and access to health care
systems is not possible for all individuals. Thus, option A is incorrect. The Declaration of Tokyo established "torture to be contrary to the laws of
humanity" and that it was unethical to artificially feed a prisoner who refuses to eat (option C). The right to die with dignity (option E) is derived from
the ethical principle of autonomy and is endorsed by many medical and ethical societies (In re Quinlan, 70 NJ 10 [1976]).

REFERENCE (8)

1. Beauchamp Tl, Childress JF. Principles of Biomedical Ethics. New York: Oxford University Press; 2001.

RATIONALE (9) Answer: D

Karen Quinlan was a 21-year-old woman in a persistent vegetative state (PVS) from severe anoxic brain injury secondary to drug intoxication during a
party. She was kept alive and on a ventilator for several months. Because of her lack of improvement, her parents requested that the hospital discontinue
medical care and allow her to die from natural causes, but the hospital refused. After much rhetoric and a legal battle, the New Jersey Supreme Court
determined that a guardian ad litem (appointed by court order) was not necessary to represent Karen independently in this particular case and allowed
her father to make all decisions on her behalf. The basis of the court decision is rooted in an individual's legal right to privacy and the notion that her
father could make the assertion based on the family's best judgment (substituted judgment standard). The decision included legal immunity for the
physicians and the suggestion to involve ethics committees in such cases. This particular case sparked the development of ethics committees, advance
directives, and nursing homes for palliative care. After weaning from the ventilator in 1976, Karen continued to breathe on her own until she died in
1985.

The principle of clear and convincing evidence (option A) is a legal principle used in the US legal system (2 others being the standards of reasonable
doubt and preponderance of the evidence). This principle can be used by physicians in certain states (Missouri, New York, Florida, among others) to
withdraw life support or any other intervention when there is clear and convincing evidence of a patient's previous statements and in the absence of a
declaration such as a living will, advance directive, or durable power of attorney. The decision is based on Cruzan v. Director, Missouri Department of
Health (497 US 261[1990]), in which the court endorsed the right of a competent person to refuse medical therapy even if the refusal results in the
patient's death; the decision is also based on the liberty interest set forth in the Fourth Amendment. The case of Terry Schiavo (Schindler v. Schiavo, 866
So2d 140 [Fla Dist Ct App 2004]) was ruled following the same principle and endorsing the historic Cruzan decision.

In Tarasoff v. Regents of the University of California (551 P2d 334 [1976]), the court recognized the duty of a physician to warn another person of a
dangerous patient's intent on doing harm to that person even if this meant violating the patient-physician confidentiality agreement.

The principle of beneficence is inherent to the role of physicians and determines the duty to prevent evil or harm by promoting good and welfare for
others. One's perceptions and intentions even if in the best interest should not supercede the wishes of another competent human being. Thus, option C is
incorrect.

REFERENCE (9)

1. What are some important cases in bioethics? In: Kummer HB and Thompson DR, eds. Critical Care Ethics. Mount Prospect, IL: Society of Critical
Care Medicine; 2009.

RATIONALE (10) Answer: E

A cross-sectional study is a type of observational study and involves the collection of data at a defined point in time. It is often used to test the
prevalence of acute or chronic conditions or to answer questions about the causes of disease or the results of an intervention. Cross-sectional studies are
also called prevalence studies. There are 2 primary types of nonexperimental or observational studies in epidemiology. The cohort study or incidence
study, is a direct analogue of the experiment, where different exposure groups are compared but the investigator does not assign the exposure. In this
sense, the cohort study is the gold standard of observational studies. When the outcome has not occurred and the investigator follows it up into the future,
it is called a prospective cohort. When the outcome has occurred and the investigator goes back in time to identify the exposure, it is called a
retrospective cohort. The second type of nonexperimental, observational study is the case-control study. It employs an extra step of sampling according
to the outcome of individuals in a population where the outcome is known (cases). The cases are compared to individuals without the outcome
(controls), who are sampled independent of the exposure. A case-cohort study is a variant of the cohort study where individuals are randomly sampled
from the cohort at baseline, creating a subcohort, rather than when cases occur (incidence density sampling).

REFERENCES (10)
1. Rothman K, Greenland S. Types of epidemiological studies. In: Rothman KJ, Greenland S, eds. Modern Epidemiology. Philadelphia: Lippincott
Williams & Wilkins; 1998.
2. Dawson B, Trapp RG. Study designs in medical research. In: Dawson B, Trapp RG. Basic and Clinical Biostatistics. New York: Lange Medical
Books/McGraw-Hill; 2004.

RATIONALE (11) Answer: D

In epidemiology, the incidence is a measure of the risk of developing a new condition within a specified period of time. The incidence proportion
(cumulative incidence) is the number of new cases within a specified time period divided by the size of the population at risk (ie, for complications
[5/100] 100 = 5%). The incidence rate (incidence density or person-time incidence rate), on the other hand, is expressed as the number of new cases
per population in a given time period (ie, 5/100 2 years = 2.5% per year). Similarly, the prevalence of a disease is defined as the total number of
cases of the disease in the population at a given time and is used as an estimate of how common a condition is within a population (ie, for candidemia,
[2/100] 100% = 2%).

REFERENCES (11)

1. Rothman K, Greenland S. Measures of disease Frequency. In: Rothman KJ, Greenland S, eds. Modern Epidemiology. Philadelphia: Lippincott
Williams & Wilkins; 1998.
2. Dawson B, Trapp RG. Summarizing data and presenting data in tables and graphs. In: Dawson B, Trapp RG. Basic and Clinical Biostatistics. New
York: Lange Medical Books/McGraw-Hill; 2004.

RATIONALE (12) Answer: C

The false-positive rate (FP) is the proportion of absent events that yield a positive test outcome or the conditional probability of a positive test result
given an absent disease or condition (ie, 425/2,000 = 0.2125). The specificity of a test is the proportion of negatives which are correctly identified by a
test or the conditional probability of a negative result given an absent disease or condition (ie, 1,575/2,000 = 0.7875) and is equal to

1 FP (True negatives [True negatives + FP]).

In hypothesis testing, the FP equals the significance level, which is expressed as alpha. Hence, increasing the specificity of a test will decrease the
probability of FP or type I error (the probability of rejecting a null hypothesis when it is true).

Similarly, the false-negative rate (FN) is the proportion of positive or present events that yield a negative test outcome or the conditional probability of
a negative test result given a present or positive disease or condition (ie, 125/1,250 = 0.1 ). The sensitivity of a test is the proportion of positives which
are correctly identified by a test or the conditional probability of a positive result given a present or positive disease or condition (ie, 1,125/1,250 =
0.9) and is equal to

1 FN (True positive [True positive + FN]).

In hypothesis testing, the FN equals the beta error. Hence, increasing the sensitivity of a test will decrease the probability of FN or type II error (the
probability of failing to reject a null hypothesis when it is false).

REFERENCES (12)

1. Rothman K, Greenland S. Measures of disease frequency. In: Rothman KJ, Greenland S, eds. Modern Epidemiology. Philadelphia: Lippincott
Williams & Wilkins; 1998.
2. Dawson B, Trapp RG. Methods of evidence-based medicine and decision analysis. In: Dawson B, Trapp RG. Basic and Clinical Biostatistics. New
York: Lange Medical Books/McGraw-Hill; 2004.

RATIONALE (13) Answer: D

The positive predictive value (PPV) of a test is the proportion of subjects with a positive result that are correctly identified as positive for a diseaseor
condition, or the conditional probability of the presence of a disease or condition given a positive result. The PPV is also known as the precision of the
test, and its value depends on the prevalence of the outcome of interest, also known as the pretest probability of the disease. Sensitivity and specificity
are said to be unchangeable properties of a test because they do not vary with pretest probabilities (or prevalence) of the disease. It is much more useful
to know the probability of disease in a patient who has a positive test result (PPV) and/or the probability of nondisease in a patient with a negative test
result (NPV). In other words, as the pretest probability of disease falls, the predictive value of a positive result also falls, and the predictive value of a
negative result rises. As the pretest probability of disease increases, a positive test result is more informative than a negative result.

REFERENCES (13)

1. The PIOPED Investigators. Value of the ventilation/perfusion scan in acute pulmonary embolism. Results of the prospective investigation of
pulmonary embolism diagnosis (PIOPED). JAMA. 1990 May 2330;263:27539.
2. Stein PD, Chenevert TL, Fowler SE, et al. Gadolinium-enhanced magnetic resonance angiography for pulmonary embolism: a multicenter
prospective study (PIOPED III). Ann Intern Med. 2010 Apr 6;152:43443, W1423.
3. Rathburn SW, Raskob GE, Whitsett TL. Sensitivity and specificity of helical computed tomography in the diagnosis of pulmonary embolism. A
systematic review. Ann Intern Med. 2000 Feb 1;132:22732.
4. Rathbun SW, Whitsett TL, Vesely SK, et al. Clinical utility of D-dimer in patients with suspected pulmonary embolism and nondiagnostic lung scans
or negative CT findings. Chest. 2003;125:8515.
5. Stein PD, Hull RD, Patel KC, et al. D-dimer for the exclusion of acute venous thrombosis and pulmonary embolism: a systematic review. Ann Intern
Med. 2004;140:589.
6. Dawson B, Trapp RG. Methods of evidence-based medicine and decision analysis. In: Dawson B, Trapp RG. Basic and Clinical Biostatistics. New
York: Lange Medical Books/McGraw-Hill; 2004.

RATIONALE (14) Answer: B

Hypothesis testing is a method of making decisions from data collected from controlled experiments or observational studies. The first step in hypothesis
testing is to determine a null hypothesis (H0) and the alternate hypothesis (HA). Next, the investigator needs to pick a test statistic and a significance
level at which rejection of the null hypothesis would result not by chance alone. The significance level is usually referred as alpha or P-value. The test
statistic is determined based on the number of subjects, variance, and distribution of the data. The t test follows a Student t distribution, and is commonly
applied when the test statistic would follow a normal distribution. Test statistics that follow normal distributions are considered as parametric tests.
Several tests can be employed to determine normality of the sample, but usually the t test is robust for samples of more than 30 subjects or observations.
When the data is not known to be in a normal distribution, then nonparametric tests should be applied to avoid the possibility of type I error (rejecting a
null hypothesis that is true). The Mann-Whitney or Wilcoxon rank sum test is a type of nonparametric test statistic employed for assessing whether 2
independent samples of observations have equally large values. The chi-square test statistic, in which the sampling distribution of the test statistic
follows a chi-square distribution, could be used to determine statistical differences among proportions and not means or medians. Finally, linear and
logistic regression are statistical methods employed in modeling the relationship between a scalar variable Y and one or more variables denoted as X or
covariates, and are in the domain of multivariate analysis.

REFERENCES (14)

1. Godfrey K. Comparing the means of several groups. In: Bailar JC, Mosteller F, eds. Medical Uses of Statistics. Waltham, MA: NEJM Books; 1992.
2. Moses LE, et al. Analyzing data from ordered categories. In: Bailar JC, Mosteller F, eds. Medical Uses of Statistics. Waltham, MA: NEJM Books;
1992.
3. Dawson B, Trapp RG. Research questions about two separate or independent groups. In: Dawson B, Trapp RG. Basic and Clinical Biostatistics.
New York: Lange Medical Books/McGraw-Hill; 2004.

RATIONALE (15) Answer: C

In hypothesis testing, power is the probability that a test will reject the null hypothesis when it is false or the probability of a test's not to making a type II
error. Type II error, or beta, refers to the probability of failing to reject a null hypothesis when it is false (false-negative rate). Therefore, power equals
1 beta, or the sensitivity of the test. Hence, increasing the sensitivity of a test will decrease the probability of false negatives or type II error.

On the other hand, alpha refers to the probability of a test's rejecting the null hypothesis when it is true. Alpha is known as the significance level of a test
or the false-positive rate. Hence, increasing the specificity of a test will decrease the probability of false positives or type I error.

REFERENCES (15)

1. Rothman K, Greenland S. Measures of disease frequency. In: Rothman KJ, Greenland S, eds. Modern Epidemiology. Philadelphia; Lippincott
Williams & Wilkins.; 1998.
2. Dawson B, Trapp RG. Methods of evidence-based medicine and decision analysis. In: Dawson B, Trapp RG. Basic and Clinical Biostatistics. New
York: Lange Medical Books/McGraw-Hill; 2004.

Potrebbero piacerti anche